Sie sind auf Seite 1von 477

Total PrepKit for the GMAT

Copyright © 2001-2007 by the MBA Center Publications

All rights reserved. No part of this book may be reproduced or transmitted in any form or by
any means, electronic or mechanical, including photocopying, recording, or by any information
and retrieval system, without the written consent of The MBA Center, except where permitted
by law.

© The MBA Center


CONTENTS

I. INTRODUCTION

THE MBA CENTER GMAT COURSE ...................................................7

WHAT IS THIS THING CALLED THE GMAT?.......................................11

STEP 1 THE MBA CENTER APPROACH TO THE GMAT CAT .................19

II. LESSONS

QUANTITATIVE SECTION

INTRODUCTION ............................................................................35

STEP 2 MATH REVIEW....................................................................41

STEP 3 PROBLEM SOLVING............................................................105


3.1 Lesson ..................................................................................109
3.2 Homework/Practice Test.......................................................131

STEP 4 PROBLEM SOLVING WORKSHOP ...........................................139

STEP 5 DATA SUFFICIENCY ............................................................149


5.1 Lesson ..................................................................................151
5.2 Homework/Practice Test.......................................................179

STEP 6 DATA SUFFICIENCY WORKSHOP ............................................189

VERBAL SECTION

INTRODUCTION ..........................................................................197

STEP 7 GRAMMAR AND STYLE REVIEW .............................................203


7.1 Lesson ..................................................................................205
7.2 Homework/Practice Test.......................................................251

STEP 8 GRAMMAR WORKSHOP .....................................................257

STEP 9 SENTENCE CORRECTION .....................................................267


9.1 Lesson ..................................................................................269
9.2 Homework/Practice Test.......................................................305

STEP 10 SENTENCE CORRECTION WORKSHOP ...................................317

© The MBA Center


STEP 11 CRITICAL REASONING ......................................................327
11.1 Lesson ................................................................................329
11.2 Homework/Practice Test.....................................................357

STEP 12 CRITICAL REASONING WORKSHOP ......................................369

STEP 13 ANALYTICAL WRITING ASSESSMENT .....................................379


13.1 Lesson ................................................................................381
13.2 Homework/Practice Test.....................................................401

STEP 14 ANALYTICAL WRITING ASSESSMENT WORKSHOP .....................403

STEP 15 READING COMPREHENSION ...............................................407


15.1 Lesson ................................................................................409
15.2 Homework/Practice Test.....................................................435

STEP 16 READING COMPREHENSION WORKSHOP ...............................447

III. APPENDICES
BEFORE THE GMAT ....................................................................457

MBA ADMISSIONS......................................................................463

© The MBA Center


© The MBA Center
© The MBA Center
The MBA Center
GMAT Course

© The MBA Center


© The MBA Center
The MBA Center GMAT Course

TOUR GUIDE: THE MBA CENTER GMAT COURSE

The MBA Center GMAT course is a sophisticated global package that helps our students
prepare for the GMAT Computer Adaptive Test. The package contains the present manual
with lessons, strategies, drills, and practice questions and is accompanied by online resources
on our website - www.mba-center.net: a GMAT Diagnostic Test, GMAT Study materials, and
the latest news about the GMAT and the MBA admissions process on our website
www.mba-center.net.

Point 1: Getting ahead…

This first point requires personal reading, inquiries, self-evaluation, and an introduction to Point 1 should
our general strategies – in other words, getting acquainted with the GMAT CAT. This is what not last more
we advise you do to: than 2 weeks.

Take our FREE online GMAT Diagnostic Test at www.mba-center.net. This online test is
tailored to identify your individual strengths and weaknesses on different key points tested on
the GMAT. Alternatively, you can also take the same test at any MBA Center, free of charge.

Read the chapter "What Is This Thing Called the GMAT?" You should do this before your first
lesson. It will give you information on the origin and purpose of the GMAT, its format and most
recent changes, and samples of the types of questions being tested.

Check out our website www.mba-center.net. It features the most recent information on the
GMAT CAT, as well as comments from our students who have taken our preparation course
and the test. You are also welcome to take advantage of our free online introductory lessons
to the GMAT (see the website section "Free Study Materials").

Attend your first class with the lesson "The MBA Center Approach to the GMAT CAT," where
your instructor will teach you how to use GMAT CAT to your advantage. These general
strategies are the first and most immediate way to increase your score. They also form the basis
for the more specific strategies that follow, and are further developed at each step of our
GMAT preparation.

Point 2: Getting started...

Preparing for the GMAT is a demanding process. In order to make your job easier, we
Point 2 should
provide access to the most updated, complete, and modern test prep technology, including: not last more
than 10 weeks.
The Study Book: This textbook contains 16 steps illustrated by examples and strategies. Our
lessons provide in-depth analysis of section structures and question types for all sections of the
GMAT: the Analytical Writing Assessment, Problem Solving, Data Sufficiency, Sentence
Correction, Critical Reasoning, and Reading Comprehension.

Online Forum: the MBA Center Forum, forum.mba-center.net, is a great way of getting
answers to your questions, as well as getting feedback and opinions from those who have
already taken the test.

Practice Exercises: Don't forget to apply the theories you learn from the book, the class, and
the website when working with practice tests and other study materials.

© The MBA Center 9


Total PrepKit for the GMAT® Introduction

Point 3: Winning!

After you have completed your course, the best key to improving your score is to practice with
simulated GMAT CATs. You can download additional materials from the official GMAT website:
Point 3 should www.mba.com. Also, check regularly the website of the MBA Center: www.mba-center.net,
not last more as well as your local MBA Center to find out what additional GMAT practice CATs they may
than 4 weeks. have available in their computer lab.

10 © The MBA Center


What Is This Thing
Called the GMAT?

© The MBA Center


© The MBA Center
What Is This Thing Called the GMAT?

WHAT IS THIS THING CALLED THE GMAT?

A LITTLE GMAT HISTORY


The GMAT was first administered in the United States in 1954. That first test was very For more
different from today’s GMAT. The older test was on paper, had no Analytical Writing information, please
Assessment or Critical Reasoning questions, and was based on the pre-collegiate Scholastic see the GMAC's
Aptitude Test (SAT). Since then, generations of test takers suffered taking the pencil-and- official website,
www.gmac.com, as
paper exam. In October 1997 that old familiar format was changed to the new Computer well as the official
Adaptive Test (CAT). Since the early 1990’s, psychometricians at Graduate Management website of the GMAT,
Admissions Council (GMAC) had been working to improve and modernize their standardized www.mba.com.
tests using research data and techniques gathered from years of experimentation with
artificial intelligence. Today, the GMAT, as well as the TOEFL, GRE, and other tests, have
been computerized.

WHO IS RESPONSIBLE FOR THE GMAT?


An organization known as GMAC, the Graduate Management Admission Council,
delivers the GMAT CAT. Since January 2006, the content of the GMAT has been designed
by ACT, Inc. and the test sessions are administered by Pearson VUE, subsidiary of Pearson
PLC. GMAC is involved in "developing business education" and promoting its more than
130 business school members. GMAC is not a government agency, though it is a body of
more or less academic folks whose work can have a significant influence on your life and
career. The fact is, if you want to attend an MBA program almost anywhere in the world,
you have little choice but to take the GMAT.

WHAT DOES THE GMAT MEASURE?


You might think that the GMAT measures something like your probability of success in
business school, or that it assesses your level of education or intelligence. It does neither.
The GMAT is not a success-predictor or an IQ test. It has its limitations. Research shows that
the test cannot predict with certainty the success, failure, or grades of an individual in an
MBA program. Furthermore, the GMAT, however respected, cannot and does not take into
consideration other factors critical to professional success such as creativity, business
acumen, negotiating skills, exemplary work ethic etc... What does the GMAT measure? The
GMAT is best at measuring one’s ability to take a standardized test.

WHAT IS THE CAT LIKE?

The Tutorial

The test begins with a mandatory computer tutorial. The tutorial will methodically guide
you through the (new) format of the test, including the various computer functions
necessary for the test, such as: how to select answers, advance to the next question, show
or hide the clock, access the HELP mode, and use word processing functions. One trip
through this tutorial in your practice should be enough to get you going, since GMAT CAT
commands and functions are intentionally easy to use.

© The MBA Center 13


Total PrepKit for the GMAT® Introduction

GMAT CAT – Tutorial

End
When finished
reading
tutorial
CAT TUTORIALS click on the
icon below
How to Use the Mouse
How to Answer
How to Use Testing Tools
How to Scroll Dismiss
Tutorial

Test Section Answer


Time Help Confirm Next
Time
Quit Exit

The Analytical Writing Assessment (AWA)

After you have completed the mandatory (and somewhat insulting) tutorial you’re
instructed to write two essays of 30 minutes each: one called Analysis of an Issue and one
called Analysis of an Argument. The task itself is to write a clear, concise, and cogent short
essay (3 to 5 paragraphs or so) for each analysis topic.
Of the two topics, the first is typically the Analysis of an Issue question which might ask
you to evaluate a statement (on business, government and business, or personal success
issues) or to consider a point of view, or opinion. Whether statements or opinions, business
or personal, Issues are broad, topical subjects – such as those you might find in a newspaper or
magazine. (See the Official Guide for the GMAT Review or www.mba.com for samples.)
Your Analysis of an Issue essay should be a logical, thoughtful assessment of the Issue resented
in which you might either deconstruct and analyze a statement or discuss whether or not you
agree with an opinion. Inclusion of personal experiences and examples is strongly encouraged
for essays on this question type, a luxury not allowed on the Analysis of an Argument.
The Analysis of an Argument question type also presents you with only one subject, or
argument, to analyze and write about. This time the subject is a formal argument composed
of premises and conclusions which is no different than the arguments you’ll find in the
Critical Reasoning section. You might evaluate arguments based on basic principles of
market economics, business, and entrepreneurship, or on education, lifestyle, or health
topics. Whatever the subject matter, your interpretation and your essay should stick very
closely to the logic of the argument presented to you. The Analysis of an Argument is the
more strict of the two essays, and including personal experiences in this essay is
discouraged.
The Analytical Writing Assessment primarily tests your ability to analyze an issue or an
argument and to write a logical and well-organized essay. For more information and our
strategies for this section of the test, see the Analytical Writing Assessment lesson in Step 13
of this book.

The Quantitative And Verbal Sections

The AWA is followed by the two computer-adaptive, multiple-choice test sections,


Quantitative and Verbal. We recommend you take the optional break between the two
sections on the test day.
The Quantitative Section is the mathematics section of the test and contains 37
questions to be completed in 75 minutes. Two types of questions which test fundamental
mathematical principles appear in this section, Problem Solving and Data Sufficiency. The

14 © The MBA Center


What Is This Thing Called the GMAT?

same mathematical principles (Arithmetic, Algebra, Geometry, and Word Problems) are
tested by both question types, though in two very different ways. Problem Solving and
Data Sufficiency questions are intermixed in the Quantitative section. You will get one or
two or three Problem Solving questions, followed by one or two Data Sufficiency
questions, and then back to Problem Solving. In all, of the 37 Quantitative questions,
approximately 24 will be Problem Solving and approximately 13 will be Data Sufficiency.
And here are the vital statistics for the Verbal Section: a total of 41 questions to be Not all the
completed in 75 minutes. Two of the Verbal question types — Critical Reasoning and questions count.
Sentence Correction — are in the Verbal section, much as the two Quantitative question
types are intermingled throughout, the Quantitative section. That is, you may get one,
two Sentence Correction questions, followed by one or two Critical Reasoning questions,
then perhaps another Sentence Correction item. However, questions of the third Verbal
type — Reading Comprehension — are grouped. Each Reading passage is accompanied
by a set of three to six questions in a row, all relating to that passage.
It is a fact that about 8 of the 37 Quantitative questions and about 11 of the 41 Verbal
questions will actually not count towards your score. These are called experimental
questions or equating items — they are used only for research purposes.
Unfortunately, there is no way to know which questions are unscored. You must approach
each question as if it counts.

THE SCORING SYSTEM


Clear away the haze and recall your youthful school days. The heady days of wealth
and responsibility seemed very far away. The tests were multiple choice (multiple guess),
and only the number of questions you answered correctly determined your score. The
GMAT CAT scoring philosophy is altogether different. Your score is calculated according to
your accuracy (answer right or answer wrong) and the level of difficulty of each question.
The first question of each section is of medium difficulty (score value: 500 points). If
this first question is answered correctly, the level of difficulty for the next question will
increase. The more questions you answer correctly, the more difficult the next questions
become. In this way the program will rapidly determine your approximate level and fine
tune it afterwards. Thus, the harder the question, the higher the point or score value for
that question. Likewise, the easier the question, the lower the point value assigned
that question. If you respond incorrectly to a question, the next question will be easier
and have a lower score value, and vice versa.
A high GMAT score, therefore, indicates that a given test taker has correctly answered
more questions at higher levels of difficulty than another test taker with a lower GMAT
score, and therefore has a higher GMAT level.
The same scoring operation applies to each section of the CAT. Your overall GMAT score
is worked out by combining your two section scores (Verbal and Quantitative). In order to
provide an admissions officer with a detailed indication of your testing level and abilities,
your official score report also includes scaled scores ranging from 0 to 60 for each section,
percentile rankings of your performance on each section and on the GMAT as a whole (this
percentile ranking is a comparison between you and other test takers), and finally a
separate AWA grade.
Analytical Writing Assessment scores range from 1 to 6. Your essays are graded not by
computer but by human graders – teaching assistants working for the GMAC. After finishing
your test, you’ll immediately receive an unofficial score for the computer-adaptive portions
of the test only. Your AWA score and your official GMAT CAT test score come together by
mail in about 15 days.

© The MBA Center 15


Total PrepKit for the GMAT® Introduction

FREQUENTLY ASKED QUESTIONS ABOUT THE GMAT CAT

The results of our website open forum are in: here is a transcript of the most frequently
asked questions about the GMAT CAT and our answers.

Q: Should I answer questions wrong on purpose so the test gets easier?

A: Absolutely not! If you answer a question wrong, the computer will give you an easier
question but your score will go down as a result. If you want a good score (you know you
do!) you must answer as many questions correctly as possible. The harder the question, the
higher the point value. This is crucial information! Since the CAT scoring algorithm very
rapidly determines your testing level, answering correctly without hesitation from the first
question on is extremely important so that you start off with a high score. It’s much easier to
maintain a high score than it is to raise a low score.

Q: How much does the level of difficulty change from question to question?

A: In the beginning, quite a lot. Near the end, not so much. This is an important quirk in the
system. The objective of the CAT is to quickly determine (right from the beginning) your
approximate testing level and then fine-tune it while you continue the test. To illustrate: a
right answer on the first question results in an increase of 50 points, a right answer on the
second, an increase of 40 points – on the second-to-last question a right answer nets you an
increase of only 10 points. So, if you answer the first few questions correctly, you’ll swiftly
receive more difficult (higher score value) questions and can therefore raise your score.

Q: Does my performance on one section affect the level of difficulty of the questions in the
other sections?

A: No. Each section is scored separately and begins with a question at the 500-level score
value. If, for example, the Quantitative Section is the first section of the test, and if you get all
the questions wrong, you’ll still begin with a question valued at 500 on the Verbal Section.

Q: Is the computer program that precise?

A: At The MBA Center, we don’t think so: it’s about as precise as the pencil-and-paper test.
Standardized tests, even on computer, cannot be 100 percent accurate in their estimation of
the level of difficulty of a given question. Some students consider difficult questions to be easy,
other students consider easy questions to be difficult – it varies from test to test, student to
student. However, the Adaptive Scoring System comes closer to identifying a personalized
testing level than did the pencil-and-paper test.

Q: Can I skip a question?

A: No. Unlike the old test, on the CAT you cannot skip a question and come back to it. Neither
can you change your responses. In order to advance to the next question you must enter your
answer and then confirm it. The explanation is simple: the computer selects a question for
you which depends on an algorithm that takes into consideration all of your answers thus far
– if you could go back and change an answer, the continuity of the algorithm would be
broken.

16 © The MBA Center


What Is This Thing Called the GMAT?

ADVANTAGES AND DISADVANTAGES OF THE GMAT CAT

The Computer Adaptive Test format was introduced in order to update the old format
using the capabilities of high technology and to add new sections to the GMAT. Now that
some time has passed since the introduction of the CAT, certain conclusions can be drawn
regarding the advantages and disadvantages of the new test.

Good CAT

The most important advantage of the new CAT is flexibility. For example, the CAT
version of the GMAT is offered continuously throughout the year. Unlike the pencil-and-
paper test, you do not have to wait months between test dates. Test sessions are offered
three weeks out of every month, and you can register easily by phone or by mail. This
flexibility is particularly beneficial for students who plan to prepare thoroughly for the test
and would like to make a convenient study schedule for themselves. Now, you can take
the test when you want to, would like to, or need to.
Another advantage of the new test is that you can take it as many times as you’d like.
But keep in mind that you must wait until one calendar month has passed before
repeating the test. If you took the test on March 20th, for example, you could take it again
beginning April 1st.
Door number 3....? Something else advantageous? You should be pleased to hear that
20 percent more time is allocated per question on the new test than on the old. Because
you cannot skip questions to save time on the CAT, as you could on the old test, you now
get 20 percent more time.
In the days of the pencil-and-paper test, you had to wait six weeks for your official score.
Now, the computer has an unofficial score for you immediately following the test. Since
you’re being scored by the computer every step of the way, it “knows” your score the minute
you finish. For your official score you’ve still got to wait by the mailbox. It’s mailed to you in
about 15 days along with your AWA results.

Bad CAT

The bad CAT: if you’re not computer-literate, you have a distinct disadvantage.
Fortunately, the commands required to use the CAT are not difficult to learn, and you can
practice them on preparation software available. We recommend that you spend time
practicing with a computer well before the date of your test.
A second disadvantage of the CAT is that your essays, the Analytical Writing
Assessment, must be typed on a computer. If you use computers all the time, typing your
essays on the CAT is little different from using any of the more popular word processing
programs. However, if you do not know what “word processing” is and sense anxiety
coming on as a result, then reduce stress by practicing on a computer well before the test.
(Note: The actual test is taken using an American keyboard.)
For some test takers, not being able to skip questions on the CAT as you could on the
pencil-and-paper test is a disadvantage. If you have not yet learned how to think like the
test, such as knowing when to make educated guesses to save yourself time and
frustration, you may worry about having to answer every question. Our lessons include
alternative approaches to skipping questions and we recommend that you practice them
in class and on your own.
Strategy is very important on the CAT. For instance, you’ll never know the level of
difficulty of the questions on the actual test. But, you still must learn to answer easy
questions correctly in order to increase the level of difficulty and the score value of
following questions, and therefore raise your overall score as high as possible.

© The MBA Center 17


Total PrepKit for the GMAT® Introduction

On Reading Comprehension passages in particular, new strategy must be employed. On


the old test, an excellent strategy was to make notes in the test booklet, but you cannot do
this on the CAT. Why? Well, though some have tried, you cannot write on the monitor
screen on which you view the test. Since on this new test the old ways don’t apply, new
ones must be adopted. For instance, taking full advantage of the note-taking materials
available on test day to take notes on Reading Comprehension passages while you scroll up
and down with the mouse looking for what you need.
Finally, cost is an additional disadvantage of the CAT. It’s about 50 percent more
expensive than the old test – you’ll find this out when you register.

HOW DO ADMISSIONS OFFICERS USE GMAT CAT SCORES?


Our inquiries make one thing perfectly clear: there is no universal or uniform use of
GMAT scores. For example, Rotterdam, Erasmus considers only the Verbal Section of the test
as a relevant selection tool, using it primarily to evaluate your English abilities. Schiller
International University, on the other hand, has found a high correlation between GMAT
math scores and eventual success in their MBA program and therefore concentrates on the
Quantitative score to select, or “weed out,” applicants. Further, HEC Montreal considers the
overall GMAT score to be a good indicator of future success in their MBA curriculum,
especially for applicants with little or no scientific background. You might also consider the
policy of certain schools toward applicants who have taken the GMAT several times.
Harvard Business School, for instance, does not discriminate against the applicant and
simply accepts the highest score. Wharton, however, averages the scores submitted. The
best tactic, in general, is to achieve the highest score possible the first time you take the
test.
Another important issue is the score policy of your target school(s), and how the GMAT
is treated by admissions officers in their evaluation. Studies show that a very strong
correlation exists between the average GMAT score of an MBA class in a given school and
that school’s rank. According to Business Week, the average GMAT score in the top 10
United States MBA programs ranges from 637 to 710. Although it is true that the GMAT is
just one criterion for admissions among others (personal essays, letters of recommendation,
professional and academic background, interviews...), the test remains one of the most
important criteria, if not the most important.
One other thing is certain: the higher the ranking of the school, the more closely
considered your GMAT score will be. Indeed, Business Week reports that the GMAT counts
for 20 percent of the admissions criteria at the top schools, while U.S. News and World Report
puts the figure at 30 percent. For recognition and credibility, it is to their advantage that highly-
competitive and well-known business schools require a high GMAT score. For slightly less
competitive schools, the GMAT score and evaluation policy is typically more flexible. For any
school you consider, we advise contacting the admissions office and asking for their GMAT
admissions policy. Our website, www.mba-center.net, is also a resource for information
regarding average GMAT scores, MBA program rankings, admissions criteria and more.

18 © The MBA Center


Step 1

The MBA Center


Approach to the
GMAT CAT

© The MBA Center


Total PrepKit for the GMAT® Step 1

GENERAL OUTLINE

The GMAT is said to be a test that assesses your ability to succeed in an MBA
program. GMAC claims that it is impossible to prepare for the GMAT. However, a
standardized test means standardized content and after taking one or two practice
tests, you will see that success on the GMAT very much depends on strategy,
practice, and a thorough understanding of the precise areas of math and verbal skills
tested. The GMAT is a test that lends itself to strategy because it is a test made with
strategy in mind. For this reason, a strategic overview of each section of the test is
part of each lesson. Studying for the GMAT is not like studying for a final exam for
which you have notes to memorize and that is all. The concepts and methods
necessary for a high score on the GMAT cannot be learned by rote, but they can be
mastered by practice. Improving your GMAT score requires a method, instruction,
and practice to answer every type of question, even the most obscure.
The adaptive format means strategy is important, since the difficulty level of the
questions fluctuates throughout the greater part of the test. What is more, the
change to the adaptive format means that it is essential to be informed about how
to use it, so that you anticipate, not react to, the test.

THE FOLLOWING TOPICS WILL BE COVERED IN THIS LESSON:

The CAT Principle


Using Your Time Strategically
The Wrong Answer Factory
The Process Of Error Identification
Guessing

KEY WORDS
Computer Adaptive Test. A type of test that adjusts the difficulty level of each question to
the testing capabilities of each test taker. Test questions for a computer-adaptive test are
selected from a large database and are classified by content and level of difficulty. The main
purpose of the adaptive program is to assess each test taker’s level of ability by adapting the
level of difficulty of the questions.

Process of Error Identification. This technique helps you find the “best answer” by
eliminating answer choices you are sure are wrong. The Process of Error Identification allows
you to choose the “best answer” even when you aren’t sure what it is, and should be used
systematically. Using the Process of Error Identification is the most efficient method of
identifying and eliminating wrong answer choices and should be used in combination with
your knowledge of Phineas the test writer’s techniques and the Wrong Answer Factory
principles that follow.

Phineas and the Wrong Answer Factory. It’s time you met Phineas, a test writer and part
of the Wrong Answer Factory – the name we have given to the test makers to explain how
wrong answer choices are constructed according to standardized types, a principle no
standardized test is exempt from. Phineas isn’t extraordinary. He’s smart enough guy; he
probably did well on his SATs, went to a respectable college where he enjoyed his studies,
and possibly continued them for a while afterward. Eventually, thought, Phineas realized
that academics weren’t for him, fished around for a career, and discovered that he wasn’t
really qualified for much. So he started writing tests because the salary was decent and it
beat waiting tables. The job was relatively stress-free; it left him time to relax and hang out
with friends. Phineas is not such a bad guy; really, you might even like him, if it weren’t for
one small item:

20 © The MBA Center


The MBA Center Approach

He’s out to sucker you.

“There’s a sucker born every minute,” Phineas reminds himself as he sets down each day
Meet Phineas*
to write his questions. He proceeds to attempt to sucker test takers. How does he do this?
Facing and unknown
opponent is always
Like all pathologicals, Phineas is pretty predictable. One of his basic tricks is to make very scary. Learn how
simple concepts appear very difficult, mostly by expressing them in obscure language and Phineas thinks, how he
syntax. He's generally quite successful (you would be too if this were all you had been writes questions, and
doing for years), but the keyword above is "appear." These concepts appear difficult, but what traps he lays for
the unwary test taker.
remain simple. And after you decipher Phineas' jargon and methods, the questions he
poses will no longer appear very complex.

Another of Phineas' ruses is to provide answer choices to difficult questions that leap off
* All characters in this
the page and look correct. Phineas loves the idea of sucker test takers falling for this one. book are purely
But with practice you can learn to recognize these choices and eliminate them quickly. fictitious. Any similarity
to real people, living or
dead, is purely
How do you avoid being a sucker? One way is to learn exactly the types of wrong
coincidental.
answer choices that Phineas will use, and we’ll do that shortly.

THE CAT AND YOU

Except for the Analytical Writing Assessment and the Reading Comprehension
questions, all the questions on the computer test are adaptive. A good understanding of
the principle and use of adaptivity on the updated test is highly recommended to improve
your score.

1) The CAT Principle

The adaptivity principle on the GMAT means that the more correct answers you choose
on certain questions (the first 7 or so), the higher the level of difficulty and the score value
of the following questions. If you answer certain questions correctly, then harder questions
will follow. Likewise, if you don’t answer certain questions correctly, the next questions you
receive will be easier and your score will go down. Your objective is to reach the higher
score value questions with as much certainty as possible. Two more things:

1. Higher scores come from the value, not the number, of questions you answer
correctly.

2. The test was designed to quickly determine your individual testing level, within the
first few questions of any adaptive section, and fine tune it as you go along.

© The MBA Center 21


Total PrepKit for the GMAT® Step 1

The CAT Principle


CONTINUOUS Score
SCORING
The CAT adjusts your
score after each V600
question. Note on the
600
graph that your score
V590 X590
jumps from 40 to 50 V580
points per question in
the beginning, but only X560
10 to 20 points per V550
question near the end. 550

500

*This graph is only a


rough approximation. It
does not use the actual
CAT algorithm

1 2 3 4 5 6
Questions
Comments

The graph above gives you an idea of the CAT adaptivity principle as it would look if it
were plotted out in the traditional manner. While answering the first few questions, the
computer is tracking you and raising and lowering your score by as much as 30 to 50
points. After 7 or 8 questions, the tracking system has formed its general impression of
your testing abilities and as it continues to adapt the test to you, the following questions
vary by only 10 points or so.

CAPITAL QUIZ

Directions: Time for some practice with the CAT adaptivity principle. On this quiz there
are 10 questions which ask about the capital city of a country. Select the right answer from
among the 4 answer choices. Your first right answer is worth 4 points. After that, the points
per question range from 1 to 7 according to its level of difficulty. For every right answer,
you move up to the next question on the ladder; for every wrong answer, move down one
step. Use your quantitative skills to figure out your score at the end. Go to town!
Although our
games simulate
the adaptivity Q1-What is the capital of the US? Q6-What is the capital of the United
principle of the
CAT, your experi-
Kingdom?
ence on the actual
test may be New York Manchester
slightly different. Washington D.C. Glasgow
Los Angeles London
Detroit Nottingham
Score value 4 Score value 2

Q2-What is the capital of Finland? Q7-What is the capital of Madagascar?

Stockholm Moroni
Oslo Lumbashi
Copenhagen Antanarivo
Helsinki Fianarantsoa
Score value 5 Score value 7

22 © The MBA Center


The MBA Center Approach

Q3-What is the capital of China? Q8-What is the capital of Australia?

Honk Kong Canberra


Canton Melbourne
Shanghai Sydney
Beijing Brisbane
Score value 3 Score value 5

Q4-What is the capital of California? Q9-What is the capital of France?

Los Angeles Paris


San Francisco Brussels
San Diego Rome
Sacramento Marseilles
Score value 6 Score value 3

Q5-What is the capital of Canada? Q10-What is the capital of Japan?

Ottawa Nagoya
Toronto Tokyo
Montreal Nagasaki
Vancouver Beijing
Score value 4 Score value 1

Q7
v

Q4
v x

Q2 Q8
v x v

Q1 Q5
x v x

Q3 Q9
x v
v = right answer Q6
x = wrong answer
x

Q10

7-Antananarivo; 8- Canberra; 9-Paris; 10 Tokyo.


Answers:1-Washington; 2-Helsinki; 3-Beijing; 4-Sacramento; 5-Ottawa; 6-London;

© The MBA Center 23


Total PrepKit for the GMAT® Step 1

2) Learn how to pace yourself

Since your testing level is determined during the first few questions of the test, our first
piece of advice is to spend the most time and effort on the first seven questions, and less
on the ones that follow. Adjust your strategy to the strategy of the test. A safe time to guess
is on the last question. It will hardly affect your score.

Time

Questions

5 25 37
3) Draw a grid
Instead of letters for answer choices on the computer test you’ll see small white bubbles,
perfect for pointing your mouse toward. Since you cannot write on the monitor screen, you
cannot cross out wrong answers. Our technique and our advice to make things easier is to
draw a grid like this one:

1 2 3 4 5 6 7 8 9 10 11 12 13 14 15 16
A X X
B X X X X X
C X X X X
D X X X X X X
E

On test day, use the time (as long as 30 minutes) during the mandatory CAT-tutorial
(which you will know long before the exam) and during the optional 5-minute break
between the Quantitative and Verbal sections to draw yourself one of our helpful grids.

Two advantages to the grid:

1. You can eliminate wrong answers easily and focus on remaining possible answer
choices. This is very effective for Reading Comprehension questions for which you must
scroll up and down and move back and forth between the questions and the passage. On
the test, it’s easy to forget which answer choices you’ve recognized as wrong and waste
time looking back at the screen to make sure.
2. You can cross out an answer choice, and have a record of it, as soon as you see that
it’s incorrect. This is an especially useful system on Problem Solving and Data Sufficiency
questions in which some answer choices can be eliminated immediately.

24 © The MBA Center


The MBA Center Approach

SPECIFIC STRATEGIES Lighten your test-


prep load:
Once you learn
1)The Wrong Answer Factory what traps the
Wrong Answer
The power of good guesswork is one reason why multiple-choice tests are rarely given Factory comes up
with, you can spot
in school. That it is possible to guess the right answer without knowing the right answer them and elimi-
is usually bad news in the testing business. On the GMAT CAT, you do have the power to nate wrong ans-
guess even though every effort has been made to make it difficult for you. Wrong answer wers without sol-
choices are constructed in certain ways to tempt you and to discriminate between ving the problem.
someone who knows the right answer (and strategies) from someone who doesn’t.
Phineas’ Wrong Answer Factory realizes that most people who take the test make the same
mistakes: errors in calculation, jumping to conclusions, and overcomplicating simple
problems. Knowledge is power: if you know how wrong answers are made and what
they’re made of, you’ll avoid being the victim of the tricks and traps waiting for you.
Given a little time, a little coursework, and a little practice, you’ll be eliminating wrong
answers with deadly accuracy.

Take a look at this math problem:

01:15 GMAT CAT – Section 3 : Quantitative 1 of 37


It takes Susan three hours to mow her lawn. If her friend Melinda
helps her, they can mow the same lawn in 1 hour. How long does it
take Melinda to mow the lawn alone?

! (A)
! (B)
! (C)1 1/2 hours
! (D)
! (E)

Test Section Answer


Time Help Confirm Next
Time
Quit Exit

Phineas knows the right answer first. He has four more plausible answers to write and
he knows how the average test taker thinks.

© The MBA Center 25


Total PrepKit for the GMAT® Step 1

Here is the final version:

01:15 GMAT CAT – Section 3 : Quantitative 1 of 37

It takes Susan 3 hours to mow her lawn. If her friend Melinda


helps her, they can mow the same lawn in 1 hour. How long does it
take Melinda to mow the lawn alone?

(A) 1 hour
(B) 2 hours
(C) 1 1/2 hours
(D) 3 hours
(E) 4 hours

Test Section Answer


Time Help Confirm Next
Time
Quit Exit

Phineas got wise by reading score and experimental section reports and he knows the
average test taker is attracted by solutions that can be easily found. For instance, Phineas
knows that some test takers will just subtract: 3 – 1 = 2. That’s why he created choice (B).
And Phineas knows that other test takers will just add: 3 + 1 = 4. That’s why he created
choice (E).
Weaker test takers are often attracted by a number already seen in the problem and
might choose (A) or (D). These are totally wrong, eliminate them.

26 © The MBA Center


The MBA Center Approach

In the previous example you saw that Phineas creates traps for those who oversimplify
a difficult problem. The Wrong Answer Factory principles also apply to Verbal questions:

01:15 GMAT CAT – Section 4 : Verbal 1 of 41


Trumpeting the victory of her team in the volleyball semifinals, the
coach’s attitude was one of exuberance and guarded optimism.
! (A) the coach’s attitude was one of exuberance and guarded
optimism.
! (B) the coach’s attitude reflected her exuberance and guarded
optimism.
! (C) the attitude of the coach was exuberant and guardedly
optimistic. CAT and
Process of Error
! (D) the coach displayed an attitude of exuberance and guarded
Identification
optimism. The Identification
! (E) the coach displayed an attitude of exuberance which was process will help
guardedly optimistic. you overcome


one of the great-
est challenges on
Answer
the GMAT CAT:
Test Section
time.
Time Help Confirm Next
Time
Quit Exit

If you analyze the structure of the sentence, you can determine that it begins with a
participial phrase (-ing form). Then there is a comma and underlined text. The grammar
problem is that the subject of the participle “trumpeting” is supposed to follow the
comma.

Have a look at the Wrong Answer Factory at work in this question.

Choice (A) is wrong because the coach’s attitude is supposed to be the subject of the
participle, which is impossible here. Choice (B) should be avoided because it reproduces
the same error. Choice (C), although it offers different wording, reproduces the same type
of error (illogical connection between participle, “Trumpeting,” and subject “the attitude of
the coach,” as in (A) and (B)).
Choice (E) fixes the initial mistake because it is the coach that is trumpeting the victory
and not the “coach’s attitude” or “the attitude of the coach”. However choice (E) introduces
a new mistake with “which was guardedly optimistic” and breaks the parallelism. Only
choice (D) is valid, since it correctly refers to the participle “Trumpeting” and retains the
parallelism.

2) Looking for wrong answers is as effective as looking for right answers.

On a Multiple Choice Question test the right answer is always in front of you. By
employing common sense and an understanding of how the Wrong Answer Factory
works, you can identify and eliminate wrong answers, leaving the right answer ready to
choose.

Let’s look at an example:

What is the capital of Madagascar?

(A) Los Angeles


(B) Madrid
(C) Antanarivo
(D) Bangkok

© The MBA Center 27


Total PrepKit for the GMAT® Step 1

Unless you spent time in Madagascar, or study maps in your spare time, there is little
chance that you know the answer to this question. However, you may be able to eliminate
answer choices (A) and (B) since there is a good chance that you know Los Angeles is in the
USA, Madrid is in Spain, and neither is in Madagascar. If you know a little more geography
you might know that Bangkok is in Thailand, and bingo you pick up the right answer just
by eliminating the wrong answers. On a Multiple Choice Question test looking for wrong
answers can be just as effective as looking for right answers.

Let’s take an example from the GMAT to illustrate this principle:

01:14 GMAT CAT – Section 3 : Quantitative 2 of 37


A bookstore makes a gross profit of 15% of the wholesale cost on
a certain book it carries. If each book is sold by the bookstore for a
retail price of $23, what is the wholesale price of the book?

! (A) $10.00
! (B) $11.50
! (C) $19.55
! (D) $20.00
! (E) $23.00


Test Section Answer
Time Help Confirm Next
Time
Quit Exit

Suppose you don’t know the formula to solve this problem, or you don’t know the
difference between wholesale cost and retail price and cannot come up with the answer on
your own. It’s time to start eliminating.

The Process of Error Identification should be learned and practiced. It allows you to
eliminate answer choices even when you do not know which choice is best.

28 © The MBA Center


The MBA Center Approach

3) Guessing

Good guesswork is still important no matter how much other preparation you do. MEDIUM
Sometimes the best you can do is eliminate two or three answer choices, and move on to QUESTIONS:

the next question. You should use guesswork for two reasons: first, you cannot skip a Questions of me-
dium difficulty are
question on the GMAT CAT; second, and more importantly for a high test score, your easy enough to
testing level is constantly checked and adapted. understand but
By a certain point on the test, your testing abilities have been generally determined and difficult to answer
if you’ve done well up to that point you’ll be given difficult questions with high score because the ans-
wer choices are
values. If you become stuck, especially toward the end of a section, use strategic deliberately tric-
guesswork for a correct answer and move on into higher-score territory. ky – read answer
choices comple-
For quantitative questions calculation is not always necessary to eliminate wrong tely and don’t be
hasty.
answers. You can also use common sense to eliminate answer choices that are obviously
incorrect or do not belong.

Look at this example:

01:13 GMAT CAT – Section 3 : Quantitative 3 of 37


If Lorenzo walks to work at 3 miles per hour and returns along the
same route at 2 miles per hour, what is his average speed, in miles
per hour for the round trip?

! (A) 2.0
! (B) 2.2
! (C) 2.4
! (D) 2.5
! (E) 3.0

Test Section Answer


Time Help Confirm Next
Time
Quit Exit

Suppose you do not know how to solve this problem, or that you have little time left.
Thanks to the Wrong Answer Factory we can eliminate (A) and (E), which reuse numbers
already seen in the initial statement, as well as (D), because it is an obvious average
between 2 and 3. You still have two options left, (B) and (C), and if you are still unsure of
the answer, guess and move on to the next.

© The MBA Center 29


Total PrepKit for the GMAT® Step 1

4) Easy and difficult questions test the same material.

Two questions may test the same material, but vary in level of difficulty. Sometimes, this
is because of traps within the question itself, other times it’s because the answer choices
have been altered to make them easier or harder.

Of these two sentences which is the easier to fix?

Sentence 1: Both Christina and Marion is hungry.

Sentence 2: Both Christina and Marion, one having chosen not to eat with Jonathan because
he always eats greasy fried food and the other having made the same choice because she refuses
to be seen in public with Jonathan, wants food.

Well, it should be apparent at a glance that Sentence 2 seems to have a lot of information
written in a very confusing manner, and that Sentence 1 is a very simple statement of fact with
just one small easily recognizable error. Did you spot what it was? The same grammatical rule,
subject-verb agreement, was tested in both sentences. (Subject-verb agreement is a common
testing point in the Sentence Correction section.) However, the error was much easier to spot
in Sentence 1 than in Sentence 2, which was intentionally written to confuse you. You can
quickly see in Sentence 1 that there are two actors, Christina and Marion, and therefore that
the singular verb “is” must be changed to the plural verb “are” in order to correct the subject-
verb agreement error. The second sentence, which frankly is far more Phineas-like, contains
some information we don’t care about (for example, the bit about ”greasy fried food”) and a
misleading singular noun, ”Jonathan”, which tries to mask the subject-verb agreement error
at the end. The same type of correction as in Sentence 1 needs to be made here: the singular
verb ”wants” should be changed to the plural verb ”want”. Just as important as correcting the
grammatical error is noticing how Phineas can turn an easy question into a hard one just by
adding extra clauses and information.

When you see a question that seems wordy and difficult, try to reduce it to its essential
parts. (In the sentences above these were simply the subjects, Christina and Marion, and
the respective verbs is/are and wants/want). Here are two reasons why you should reduce:

- Easy and difficult questions often test the same problems.


- You’ll be able to choose the best answer more quickly and more accurately when you do.

30 © The MBA Center


The MBA Center Approach

CAT AND LEVEL OF DIFFICULTY


Using the level of difficulty of a question to your advantage is an excellent strategy for
the GMAT. With some practice you’ll begin to recognize which questions are easy,
medium, or difficult.

Keep in mind that:

- The Computer Adaptive Test starts with medium questions.


- The difficulty level of each question varies a lot at the beginning.
- The material tested on the GMAT does not vary significantly from question to question.
- The presence or absence of a trap makes a question more or less difficult.

For instance, if you recognize a difficult question and realize a possible answer came
too easily to you, then you’ve probably made one of the three common mistakes, and a
trap was set for you by Phineas. On the other hand if a question seems easy and an answer
seems clearly right, then trust your instincts: easy questions have no real traps.

Consider these two questions:


Question 1

01:12 GMAT CAT – Section 3 : Quantitative 4 of 37


A machine manufactures 10 nails per hour for three hours. Then
the machine is sped up and produces 20 nails per hour for three
hours. The machine, over the 6 hours, produces an average of how
many nails per hour?

! (A)12
! (B)15
! (C)16
! (D)20
! (E)90

Test Section Answer


Time Help Confirm Next
Time
Quit Exit

Question 2

01:11 GMAT CAT – Section 3 : Quantitative 5 of 37


If a book that is usually sold for $23.95 is now on sale for $19.95,
then the price decrease is closest to:

! (A) 10 %
! (B) 16 %
! (C) 20 %
! (D) 25 %
! (E) 30 %

Test Section Answer


Time Help Confirm Next
Time
Quit Exit

These questions probably appear, at a glance, to have about the same level of difficulty.
An intuitive answer to question 1 would be 15, and is effectively the best answer. An
intuitive answer, to the second question would be 20%, and is wrong.

© The MBA Center 31


Total PrepKit for the GMAT® Step 1

HOW TO USE THE LEVEL OF DIFFICULTY:

Question 1 is an easy question (score value 450), and you can trust your instinct if you
chose (B). Question 2 is of medium difficulty, and if you thought 20 percent was the right
answer don’t trust your instinct (unless you score 700 on your diagnostic test). If you do not
know how to solve this question you can always use the Process of Error Identification.
Choices (D) and (E) are wrong because the percentage decrease is too high. Choice (A) is
wrong because the percentage decrease is too low. Understanding how the Wrong Answer
Factory works, you might recognize Phineas’ trap: the possible confusion between a per-
centage increase from $19.95 to $23.95, which is 20%, and a percentage decrease from
$23.95 to $19.95, which is less than 20%: (23.95 19.95)/23.95. The only possible answer is (B).

SUMMARY

The GMAT CAT is not a linear test. It adapts to the level of each test taker who receives
points according to the difficulty level of the question.

The computer adds or subtracts more points at the beginning of an adaptive section and
fewer at the end. Your effort and concentration should be greater at the beginning when
questions directly affect your final score.

Since the Process of Error Identification is a very effective method and you cannot write
on the monitor screen when eliminating wrong answers, our advice is to take the time to
draw a scoring grid.

Phineas offers you the same chances to make mistakes throughout the test. Learn how
to recognize the most common traps.

Once familiar with Phineas’ tricks, you’ll find that looking for wrong answers can be as
effective as looking for right answers.

Use the level of difficulty to your advantage. On difficult questions, Phineas always uses
traps we’ve explained by Wrong Answer Factory principles: Consider answer choices very
carefully. On easier questions the traps set by Phineas are not difficult to spot: Use your
instinct.

32 © The MBA Center


II. LESSONS

© The MBA Center


© The MBA Center
Quantitative
Section
Introduction

© The MBA Center


Total PrepKit for the GMAT® Quantitative Section

GENERAL OUTLINE

The Quantitative Section of the GMAT CAT is designed to test your


ability to think and reason quantitatively. There are two types of
questions in this section: Problem Solving and Data Sufficiency. Both
of these question types test your mastery of basic mathematics and
your ability to solve complex problems. This introduction is designed
to familiarize you with the Quantitative Section of the GMAT CAT
and the material covered. The lessons which follow this preview will
discuss Problem Solving and Data Sufficiency questions in greater
detail and will give you specific strategies to help you improve your
score on the Quantitative Section of the GMAT CAT.

THE FOLLOWING TOPICS WILL BE COVERED IN THIS LESSON:

The GMAT Quantitative Section: General


GMAT Math: An Overview
CAT Strategy
Guessing And The CAT

36 © The MBA Center


Introduction

THE GMAT QUANTITATIVE SECTION: AN OVERVIEW

The Quantitative Section of the GMAT CAT is a 75-minute section designed to test your
mastery of basic mathematical concepts and your ability to reason quantitatively. In this
section there are 37 multiple-choice questions which fall into two distinct categories:
Problem Solving and Data Sufficiency. Problem Solving questions are simply math
problems in a multiple-choice format. Data Sufficiency questions are similar to Problem
Solving questions, but you are not required to arrive at a solution. Instead, you must
determine whether the information in the question is sufficient to solve the problem.
These two types of questions are intermingled throughout the Quantitative Section and
are presented in random order. You should expect approximately 23 Problem Solving and
14 Data Sufficiency questions in this section. However, because the number of each
question type is decided by the computer, the actual allocation of questions may vary.
Although Problem Solving and Data Sufficiency questions generally test the same skills and
require the same basic knowledge of mathematics, they will be taught separately in this
course because they require very different strategies and techniques.

What The Quantitative Section Tests

The Quantitative Section of the GMAT requires knowledge of only a limited range of
fairly low-level math concepts. These concepts are the same for every GMAT and are tested
in similar ways on each test. The most commonly tested math topic on the GMAT is
arithmetic. Virtually all of the Problem Solving and more than three-quarters of the Data
Sufficiency questions require at least some knowledge of basic arithmetic. Algebra is the
second most commonly tested topic in the Quantitative Section; over half of the questions
include algebra in some form or another. Geometry is the third most commonly tested
math topic; approximately 15 percent of all GMAT math problems relate to geometry. A
few problems in the Quantitative Section relate to topics such as logic, data interpretation
(graphs), and sets. Many GMAT math problems, particularly word problems, combine two
or more math topics.

Unfortunately, many people who take the GMAT have forgotten the basic math skills
tested. You probably learned most of the math covered before you left high school, which
was a long time ago. Don’t worry though, you have time now to brush up on the necessary
math before you take the test. All of the math topics tested on the GMAT are covered in
the Comprehensive Math Review for the GMAT, which is Step 2 in this book. We
recommend that you look through the math review and become familiar with the math
covered before you begin the Problem Solving and Data Sufficiency lessons which follow.

THE CHALLENGE
The Quantitative Section tests not only your mastery of basic math, but also your ability
to reason through complex problems quickly and accurately. Generally, the difficulty in
solving GMAT math problems is not calculating the answers, but sorting through the
information given in the questions and quickly finding the best approach to solving them.
The typical question in this section combines several simple math concepts in a way
designed to confuse the test taker. Your task then is to identify the math concepts tested
and work through the problem logically, and above all, quickly. The principal challenge of
the Quantitative Section of the GMAT is time. If you were given an unlimited amount of
time, you could probably solve all or nearly all of the questions. However, the time given
is very limited and most test takers have difficulty finishing the section in the time allowed.
For this reason the MBA Center method stresses both mastery of the necessary
mathematics and employment of the time-saving strategies in the Problem Solving and
Data Sufficiency lessons which follow.

© The MBA Center 37


Total PrepKit for the GMAT® Quantitative Section

GMAT MATH

The Quantitative Section of the GMAT CAT, like the Verbal Section, is adaptive. This
means that the computer determines your ability and selects questions that match your
level.

How The CAT Works

When you begin the Quantitative Section of the GMAT CAT, the computer will give you
a question (either Data Sufficiency or Problem Solving) of medium difficulty, in other words,
a question that roughly half of all test takers get right and roughly half get wrong. If you
answer the first question correctly the computer will give you a harder question. Likewise,
if you answer the first question incorrectly the computer will give you an easier question.
This process is then repeated for the rest of the section. As you progress through the section
each successive jump gets smaller and smaller as the computer focuses in on your level. In
theory, you will eventually arrive at a point where every question will be about at your level.
The score you receive for the section depends not on the number of questions you answer
correctly, but on the level of difficulty of those questions. The idea is to award more points
for difficult questions than for easy ones. Therefore, correct answers to difficult questions
increase your score more than do correct answers to easy questions.

CAT Strategy

In order to maximize your score on the Quantitative Section of the GMAT, it is important
to keep in mind how the CAT arrives at a score. Remember, the computer tries to find your
level of ability and present you with questions accordingly. Early in the section the computer
makes large jumps in order to find your general level of ability. As the section progresses,
the jumps get smaller and smaller as the computer focuses in on your level. Because your
final score depends on the level of difficulty of the questions you answer correctly, it is
important that you answer the early questions correctly in order to get quickly to the
harder, more valuable questions.

38 © The MBA Center


Introduction

Time Management On The CAT

Because of the incredible time pressure on the CAT, it is important that you have a
strategy for managing time. Use the chart below to help pace yourself on the Quantitative
Section of the CAT. Those test takers who are weaker at math may want to spend a bit
more time on the first eight questions to make sure they are answered correctly. Likewise,
test takers who are stronger at math should move a bit more quickly through the early
questions in order to conserve time for the difficult questions later in the section.

Time

Questions
5 25 37

Out of the Blocks — Questions 1–8


The first eight questions are by far the most important for determining your score.
Because the CAT is making large jumps at this stage of the test, a wrong answer is far more
damaging to your score than is a wrong answer later on in the section. You can spend a
bit more time per question here than you can later in the test. Take your time, check your
work and avoid guessing. Try to finish the first eight questions before twenty minutes have
elapsed in the section.

Cruising — Questions 9–25


These questions are still quite important, but a wrong answer here won’t be as
damaging to your score as a wrong answer in the first eight questions. You should move
quickly and efficiently. Make an honest attempt to answer all of these questions, but if you
get stuck, go ahead and guess. You should be done with these questions before an hour
has elapsed in the section.

The Final Stretch — Questions 26–37


These last twelve questions don’t influence your score as much as the previous
questions. You should answer each of these questions quickly and don’t hesitate to guess
if you find the question too difficult. When you get the 5-minute warning, answer the rest
of the questions.

GUESSING AND THE CAT


Guessing is especially important on the CAT for two reasons:

1) You can’t skip questions on the CAT. The computer will not let you move on to
the next question until you answer the question you are currently on, so if you don’t know
the answer to a question, you must guess. Don’t worry. There isn’t a guessing penalty on
the CAT.

2) Because the computer adapts to your level, you should find that nearly every
question on the test is difficult. For this reason you will have to guess more often than you
would on the pencil-and-paper test.

© The MBA Center 39


Total PrepKit for the GMAT® Quantitative Section

Because guessing is necessary on the CAT, it is important that you become good at it.
By using the techniques and strategies described in the Problem Solving and Data
Sufficiency lessons, you will be able to eliminate several wrong answer choices and improve
your chances of guessing the correct answer.

Answer every question in the section even if you have to guess.

When time expires, every question left unanswered in the section is considered wrong.
For this reason it is important that you answer every question in the section, even if you
have to take random guesses at the end. Remember there is no penalty for getting a
question wrong, and if you take a random guess on a question you have at least a 20
percent chance of getting it right. So make sure to allow enough time at the end to guess
on the final questions. You should of course try to eliminate any wrong answers you can,
but if you can’t, just take a guess.

A FEW FINAL WORDS ON THE CAT


Taking notes is very important

Materials for taking notes are provided at the test center. You can use it in any way you
find useful, including solving equations and drawing diagrams. Be careful to transfer
information carefully and accurately from the computer screen to your notes. We
recommend that you practice using note-taking materials when you take the practice
GMAT CATs.

Calculators are not permitted on the GMAT

You are not permitted to use a calculator when taking the GMAT. Because many of the
questions require you to do calculations, you should practice calculating in your notes
before you actually take the test.

40 © The MBA Center


Step 2
Math Review

© The MBA Center


Total PrepKit for the GMAT® Step 2.1 Lesson

GENERAL OUTLINE

The Quantitative Section of the GMAT CAT requires a good


understanding of the basic concepts of arithmetic, algebra, and
geometry, and the ability to apply those concepts efficiently and
accurately. For many test takers (perhaps a majority) the math tested
is ancient history. This chapter provides a quick review of the key
math concepts frequently tested on the GMAT. We advise you to
brush up on the math covered in this lesson before you move on to
the Problem Solving and Data Sufficiency lessons which follow. This
lesson is a basic math review, not a math course; test takers who are
particularly weak in math may wish to consult a math textbook for a
more thorough discussion of the math topics discussed.

THE FOLLOWING TOPICS WILL BE COVERED IN THIS LESSON:

Number Properties
Fractions
Percents
Average, Median, And Mode
Exponents And Square Roots
Equations
Quadrilaterals
Triangles
Circles
Phineas’ Favorite Word Problems

42 © The MBA Center


Math Review

1. NUMBER PROPERTIES
Real numbers — numbers that have locations on the number line. All
numbers on the GMAT are real numbers.
Greater than — to the right on the number line (symbol: >).
Less than — to the left on the number line (symbol: <).
Positive — greater than 0 (to the right of 0 on the number line). If you know what imaginary
Negative — less than 0 (to the left of 0 on the number line). and complex numbers are, you
can forget about them while
Q: Which is greater: –8 or –15? you’re taking the GMAT.

A: –8 is to the right of –15 on the number line, and so –8 > –15.

Integers — the positive whole numbers, zero, and the negative whole
numbers — numbers that can be expressed without a decimal or fraction.
The integers: . . . –6, –5, –4, –3, –2, –1, 0, 1, 2, 3, 4, 5, 6, . . .

Digits — the symbols 0, 1, 2, 3, 4, 5, 6, 7, 8, and 9, used in combination to


represent integers greater than 9 and less than –9.
Some positive 2-digit numbers: 10, 11, 12, 13, 29, 36, 63, 77, 90
In a whole number, the rightmost digit is the units’ digit or ones’ digit. The
next digit to the left is the tens’ digit. Continuing to the left the digits are
the hundreds’, thousands’, and so on. The digits, if any, to the right of the
decimal point are the tenths’, hundredths’, thousandths’, and so on.
The 4 Basic Operations — Symbol Result
addition + sum
subtraction – difference
multiplication × or • product
division ÷ or fraction bar quotient

Ex: The sum of 2 and 7 is 9. The product of 2 and 7 is 14.

Even numbers — integers that are divisible by 2.


The even numbers: . . . –10, –8, –6, –4, –2, 0, 2, 4, 6, 8, 10,. . .
Odd numbers — integers that are not divisible by 2.
The odd numbers: . . . –9, –7, –5, –3, –1, 1, 3, 5, 7, 9, . . .
You can predict whether a sum, difference, or product will be even or odd.
EVEN ± EVEN = EVEN EVEN × EVEN = EVEN
EVEN ± ODD = ODD EVEN × ODD = EVEN
ODD ± ODD = EVEN ODD × ODD = ODD
There is no need to memorize these rules. Just remember that such rules
exist. You can use any even and odd numbers to remind yourself.

Q: Is the product of two odd numbers even or odd?


A: Think of any two odd numbers, say 3 and 5. The product of 3 and
5 is 15, which is odd, and so you can say that the product of any
pair of odd numbers is odd.
Note that you cannot predict
If integer A divided by integer B equals an integer, you can say that A is a the even/oddness of a quo-
multiple of B, and you can say that B is a factor of A. tient. When you divide an
Ex: Some multiples of 10 are: 10, 20, 350, 970, –10, –440, and 0. even by an even, for example,
the result might be even
Ex: The factors of 10 are: 10, 5, 2, 1, –1, –2, –5, and –10. (8 ÷ 2 = 4), or it might be odd
(10 ÷ 2 = 5), or it might not be
Remainder — the integer that is left over after division. If positive integer
an integer at all (10 ÷ 4 =
A is not a multiple of positive integer B, then when you divide A by B you 2.5).
will get a quotient and a remainder.

© The MBA Center 43


Total PrepKit for the GMAT® Step 2.1 Lesson

Ex: When you divide 37 by 5, the quotient is 7 with a remainder of 2.


Ex: When you divide 9 by 23, the quotient is 0 with a remainder of 9.
Sometimes the best way to do a remainder problem is to use a specific case.
Q: When positive integer A is divided by 7, the remainder
is 3. What is the remainder when 4 times A is divided
by 7?
A: Think of a positive integer that would leave a reminder of 3 when
divided by 7. One such number is 10. Multiply 10 by 4 and you get
40. Divide 40 by 7 and you get 5 with a remainder of 5. So the
answer is 5.
By definition, 1 is not a prime Prime number — a positive integer that has exactly two distinct factors, 1
number. and itself. The smallest prime number is 2 — its only factors are 1 and 2.
The next ten prime numbers are 3, 5, 7, 11, 13, 17, 19, 23, 29, and 31.
Note that 2 is not only the There is no easy way to recognize a prime number. To test a number for
smallest prime number, but primeness, you just have to keep trying to divide it by prime numbers.
also the only even prime
number. Q: Is 187 a prime number?
A: Try dividing 187 by prime numbers, starting with the smallest one,
2. Here’s where the divisibility tests come in handy. 187 is not
divisible by 2 (its last digit is not even). It’s not divisible by 3 (the
sum of its digits is 16, which is not a multiple of 3). It’s not
divisible by 5 (its last digit is not 5 or 0). It’s not divisible by 7 (187
w 7 = 26 with a remainder of 5). It is divisible by 11 (187 w11 =
17). Therefore 187 is not prime.

Note that you need to try dividing by prime numbers only.

Q: Is 191 a prime number?


A: 191 is not divisible by 2 (its last digit is not even). It’s not divisible
by 3 (the sum of its digits is 11, which is not a multiple of 3). It’s
not divisible by 5 (its last digit is not 5 or 0). It’s not divisible by 7
(191 w 7 = 27 with a remainder of 2); by 11 (191 w 11 = 17 with
a remainder of 4); by 13 (191 w 13 = 14 with a remainder of 9);
or by 17 (191 w 17 = 11 with a remainder of 4). Therefore 191 is
prime.

When can you stop dividing by prime numbers? When you get a quotient
smaller than the prime number you’re dividing by.
Prime factorization — the expression of a positive integer as a product of
prime numbers. The prime factorization of 36, for example, is 3 s 3 s 2 s 2.
To determine an integer’s prime factorization, break it down into factors bit
by bit until all the factors are prime.
Q: Find the prime factorization of 1,650.
A: Break 1,650 down into factors bit by bit. You might first break it
down this way:
1,650 = 165 s 10
Then you could reexpress 165 as 5 s 33 and 10 as 2 s 5:
165 s10 = 5 s 33 s 2 s 5
All that’s left now is to reexpress 33 as 3 s 11:
5 s 33 s 2 s 5 = 5 s 3 s 11 s 2 s 5
That’s the prime factorization. To put it in standard form, put the
prime factors in order from least to greatest:
1,650 = 2 s 3 s 5 s 5 s 11

44 © The MBA Center


Math Review

Common multiple — a number that is a multiple of two or more given


integers.
Ex: The number 12 is a common multiple of 2, 3, 4, and 6.
Least common multiple (LCM) — the least positive integer that is a
multiple of all the integers in a given set. To determine the LCM of a set of
integers, break each integer in the set down into its prime factorization.
Next, eliminate duplicate prime factors, leaving only those duplicate factors
that are raised to the highest power. Multiply the remaining factors and the
LCM is the product.
Q: What is the least common multiple of 3, 4, 9, and 27?
A: Break each number down into its prime factors: 3 is a prime
number, so its prime factorization is simply 3; 4 = 2 × 2, or 22; 9
= 3 × 3, or 32; and 27 = 3 × 3 × 3, or 33. Next, eliminate duplicate
factors, leaving only those duplicates which are raised to the
highest power. You have a 33, so you can eliminate the 3 and 32,
leaving you with 22 × 33 = 108, which is the LCM.

When you want to find the LCM of two integers, sometimes the easiest
approach is to check out successive multiples of the larger integer until you
find one that is also a multiple of the smaller integer.
Q: What is the least common multiple of 24 and 32?
A: Take the larger integer, 32, and keep multiplying it until you find
a multiple that is also a multiple of 24. 32 × 2 = 64, which is not
a multiple of 24. 32 × 3 = 96, which is a multiple of 24. The LCM
of 24 and 32 is 96.

Common factor — a number that is a factor of two or more given integers.


Ex: 4 is a common factor of 32 and 40.
Greatest common factor (GCF) — the greatest integer that divides with
no remainder into two or more given integers.
Ex: 1, 2, 4, and 8 are all the positive common factors of 32 and 40.
Of these, 8 is the greatest common factor.
The greatest common factor is equal to the overlap in the prime
factorizations.

Q: What is the greatest common factor of 48 and 84?


A: First figure out the two prime factorizations:
48 = 2 × 2 × 2 × 2 × 3 and 84 = 2 × 2 × 3 × 7
Then find the overlap. Both factorizations have at least two 2’s and
a 3, so the overlap is:
2 × 2 × 3 = 12
Thus the greatest common factor is 12.

Sometimes you’ll find two prime factorizations that have no overlap, in


which case the greatest common factor is 1.
Q: What is the greatest common factor of 48 and 49?
A: First figure out the two prime factorizations:
48 = 2 × 2 × 2 × 2 × 3 and 49 = 7 × 7
These two have no prime factors in common. Therefore the greatest
common factor is 1. You might have known
immediately that the GCF of 48
Integers with no common factor greater than 1 are called relative primes. and 49 is 1, because
consecutive integers are always
relative primes.

© The MBA Center 45


Total PrepKit for the GMAT® Step 2.1 Lesson

PRACTICE QUESTIONS 1-10 (NUMBER PROPERTIES)

1. What is the product of the greatest 2-digit multiple of 7 and the greatest
2-digit prime number?
(A) 9,312
(B) 9,408
(C) 9,506
(D) 9,603
(E) 9,702

2. If the sum of two integers A and B is even, which of the following must be
even?
I. A×B
II. A÷B
III. A–B
(A) I only
(B) II only
(C) III only
(D) I and III only
(E) I, II, and III

3. What is the smallest positive integer that is a multiple of 1, 2, 3, 4, 5, 6,


7, 8, 9, and 10?
(A) 360
(B) 630
(C) 840
(D) 1,260
(E) 2,520

4. When positive integer X is divided by 9, the remainder is 5. What is the


remainder when 4 times X is divided by 6?
(A) 1
(B) 2
(C) 3
(D) 4
(E) 5

5. Which of the following has the greatest number of distinct prime factors?
(A) 30
(B) 32
(C) 34
(D) 36
(E) 38

46 © The MBA Center


Math Review

AB
+BA
CDC
6. In the correctly worked computation above, A, B, C, and D represent
distinct nonzero digits. What is the value of A + B + C + D?
(A) 14
(B) 16
(C) 18
(D) 20
(E) It cannot be determined from the information given.

7. What is the units’ digit of (493)(915)(381)(756)(29)?


(A) 0
(B) 1
(C) 4
(D) 5
(E) 6

8. If p and q are distinct integers, and the sum of p and q is zero, which of
the following statements must be true?
(A) The product of p and q is even.
(B) The product of p and q is odd.
(C) The product of p and q is negative.
(D) The product of p and q is zero.
(E) The product of p and q is positive.

9. When positive integer X is divided by 9, the remainder is 6. When positive


integer Y is divided by 10, the remainder is 8. What is the remainder when
X × Y is divided by 6?
(A) 0
(B) 1
(C) 2
(D) 3
(E) 4

10. If the sum of two prime numbers a and b is prime, which of the following
must be true?
I. a – b is odd.
II. ab is even.
III. a ≠ b
(A) III only
(B) I and II only
(C) I and III only
(D) II and III only
(E) I, II, and III

© The MBA Center 47


Total PrepKit for the GMAT® Step 2.1 Lesson

2. FRACTIONS
A fraction is a number written in the form N in which the top number N
D
is called the numerator and the bottom number D is called the
denominator. The denominator tells you into how many equal parts a
whole is divided, and the numerator tells you how many of those parts
you’re considering.
Equivalent fractions look different, but they’re equal in value. Any fraction
can be expressed as an equivalent fraction simply by multiplying the nume-
rator and the denominator by the same number.

Ex: The fraction 2 is equivalent to 4 and 6 .


5 10 15
To reduce a fraction, divide the numerator and denominator by their
greatest common factor. When the numerator and denominator have no
common factor greater than 1, the fraction is expressed in its lowest terms.

Q: Reduce 16 .
24
A: The greatest common factor of 16 and 24 is 8. Divide both the
numerator and denominator by 8 and you end up with 2 .
3
To add or subtract fractions, convert the fractions to equivalent fractions
with the same denominator and add or subtract the numerators.

Q: 1 + 1 = ?
2 3
A: Convert each fraction to an equivalent fraction with a denominator
of 6:
1 + 1 = 3 + 2
2 3 6 6
Now add the numerators and put the sum over the 6:
3 + 2 = 3+2 = 5
6 6 6 6

Q: 3 – 1 = ?
4 6
A: Convert each fraction to an equivalent fraction with a denominator
of 12:
3 – 1 = 9 – 2
4 6 12 12
Now subtract the numerators and put the difference over the 12:
9 – 2 = 9–2 = 7
12 12 12 12

To multiply fractions, multiply the numerators and multiply the


denominators.

Q: 1 s 2 = ?
6 3
A: Multiply the numerators (1 s 2 = 2), and multiply the
denominators (6 s 3 = 18):
1 s 2= 1s2 = 2 = 1
6 3 6s3 18 9

48 © The MBA Center


Math Review

You can sometimes avoid big numbers and a lot of simplification by


canceling common factors before you multiply. Look for factors common to
the numerator of one fraction and the denominator of the other.

Q: 14 × 30 = ?
45 49
A: You could just go ahead and multiply the numerators and multiply
the denominators:
14 × 30 = 420
45 49 2205
The result 420 can be simplified. But it would have been easier
2205
to cancel out common factors before multiplying. The numerator of
14 and the denominator of 30 are both multiples of 7, so you can
45 49
cancel a factor of 7 from each:
14 × 30 = 2 × 30
45 49 45 7
Similarly, the denominator of the first fraction and the numerator
of the second are both multiples of 15, so you can cancel a factor
of 15 from each:
2 × 30 = 2 × 2 = 4
45 7 3 7 21

To divide fractions, first flip the second fraction (the one after the ÷ sign),
and then multiply.

Q: 1 ÷ 2 = ?
6 3
A: First flip the second fraction, and then multiply:
1 ÷ 2 = 1 × 3 = 1×3 = 3 = 1
6 3 6 2 6×2 12 4

To divide an integer by a fraction, or a fraction by an integer, think of the


integer as a fraction with a denominator of 1.

Q: 3 ÷ 8 = ?
4

A: Rewrite the integer 8 as the fraction 8 . Then flip this second


fraction and multiply: 1

3 ÷ 8= 3 ÷ 8 = 3 × 1 = 3
4 4 1 4 8 32

Improper fraction — a fraction with a numerator that is bigger than the


denominator. A positive improper fraction will have a value greater than 1.

Ex: 23 is an improper fraction, because the numerator 23 is greater


5
then the denominator 5.

© The MBA Center 49


Total PrepKit for the GMAT® Step 2.1 Lesson

Mixed number — a number with an integer part and a fraction part that is
less than 1. Improper fractions can be expressed as mixed numbers.

Ex: The improper fraction 23 can also be expressed as the mixed


5
3
number 4 .
5
To convert an improper fraction to a mixed number, divide the
denominator into the numerator to get an integer quotient with a
remainder. The quotient is the integer part of the mixed number, and the
remainder becomes the new numerator, which you put over the original
denominator.

Q: Express 100 as a mixed number.


3
A: Divide 3 into 100, which yields 33 with a remainder of 1. The
integer part of the answer, then, is 33, and the fraction part is the
remainder 1 over the original denominator 3:
100 = 33 1
3 3

To convert a mixed number to an improper fraction, multiply the


integer part by the denominator of the fraction part, then add the
numerator. The result is the new numerator, which you put over the
original denominator.

Q: Express 21 3 as an improper fraction.


7
A: Multiply the integer part 21 by the denominator 7, then add the
numerator 3, yielding 150 for the new numerator. Put that over the
original denominator:

21 3 = 150
7 7

Reciprocals — two numbers whose product is 1.

Ex: 3 and 4 are reciprocals because 3 × 4 = 1.


4 3 4 3
Finding the reciprocal of a fraction is easy—just flip it—that is, switch the
numerator and denominator.

Q: What is the reciprocal of 2 ?


5
A: Flip it. The reciprocal is 5 .
2

The reciprocal of a positive number is positive, and the reciprocal of a


negative number is negative.

Q: What is the reciprocal of – 3 ?


8
A: Flip the fraction and keep the minus sign. The reciprocal is – 8 .
3
To find the reciprocal of another type of number, first reexpress it as a
fraction.

50 © The MBA Center


Math Review

Q: What is the reciprocal of 13 1 ?


3
A: First turn the mixed number into a fraction. Multiply 13 and 3,
and then add 1, giving you 40 for the new numerator. Put that over
the original denominator 3 and you get 40 . Now, to get the
3
reciprocal, flip it. The answer is 3 .
40
Comparing fractions — When you have two fractions with a common
denominator, it’s easy to see which fraction is greater — it’s the one with the
larger numerator.

Q: Which is greater: 7 or 8 ?
25 25
A: The fractions have the same denominator, so the greater one is the
one with the larger numerator: 8 .
25
One way to compare fractions that do not have a common denominator is
to reexpress them so that they do.

Q: Which is greater: 4 or 5 ?
7 8
A: Reexpress both fractions with a common denominator of 56:
4 = 32 and 5 = 35
7 56 8 56
The greater fraction is the one with the larger numerator: 35 is
56
greater than 32 , and so 5 is greater than 4 .
56 8 7
Sometimes it’s easier to compare fractions by reexpressing them with a
common numerator. In such case, the one with the smaller denominator
will be the greater fraction.

Q: Which is greater: 5 or 10 ?
16 33
A: In this case it’s a lot easier to use the common-numerator approach
than the common-denominator approach. Double the numerator
and denominator of the first fraction:
5 = 10
16 32
Now the fractions have the same numerator, so the greater one is

the one with the smaller denominator: 10 is greater than 10 , and


32 33
so 5 is greater than 10 .
16 33
Another way to compare fractions is to convert them both to decimals.

Q: Which is greater: 16 or 13 ?
25 20
A: Convert both fractions to decimals:
16 = 0.64 and 13 = 0.65
25 20
It is especially easy to convert a fraction to a decimal when you use a
calculator. But of course that is not possible on the GMAT.

© The MBA Center 51


Total PrepKit for the GMAT® Step 2.1 Lesson

Sometimes when you convert a fraction to a decimal by dividing the


bottom into the top, you get a decimal that goes on forever. This kind of
endless decimal is a repeating decimal — a decimal with one digit or
sequence of digits that repeats indefinitely.

Q: Convert 7 to a decimal.
15
A: When you divide the numerator 7 by the denominator 15, you get:

0.4666666666...
)
15 7.0000000000...

Q: Convert 15 to a decimal.
7
A: When you divide the numerator 15 by the denominator 7, you get:

2.1428571428...
)
7 15.0000000000...

In the first example above, it’s a single digit that repeats. To write that more
simply, put a bar over the repeating digit:

0.4666666666... = 0.46
In the second example, it’s a the sequence of digits 142857 that repeats. To
write that more simply, put a bar over the repeating sequence:

2.1428571428... = 2.142857

To find a particular digit in a repeating decimal, note the number of digits


in the sequence that repeats. If there are 2 digits in that sequence, then
every 2nd digit is the same. If there are 3 digits in that sequence, then every
3rd digit is the same. And so on.

Q: What is the 49th digit after the decimal point in the


decimal equivalent of 5 ?
27
A: The decimal equivalent is 0.185185185.... There are 3 digits in the
sequence that repeats, so every 3rd digit is the same: the 3rd, 6th,
9th, 12th, 15th etc. digits are all 5. Thus the 48th digit is also a
5, and therefore the 49th digit is a 1.

52 © The MBA Center


Math Review

PRACTICE QUESTIONS 11-20 (FRACTIONS)

11. All of the following are equal EXCEPT:

(A) 4 (B) 8 (C) 16 (D) 32 (E) 40


5 10 25 40 50

12. 1 + 2 × 3 – 4 =
2 3 4 5

(A) 0 (B) 1 (C) 1 (D) 1 (E) 1


5 4 3 2

13. What is the product of 2 2 and 4 1 ?


3 2

(A) 7 1 (B) 8 1 (C) 10 3 (D) 12 (E) 14 2


6 3 5 5

14. What is the sum of the reciprocals of 1 1 and 1 2 ?


9 3

(A) 9 (B) 27 (C) 1 23 (D) 1 1 (E) 2 7


25 50 50 2 9

15. Which of the following is greatest?

(A) 2 (B) 3 (C) 5 (D) 6 (E) 9


3 4 7 9 14

16. Which of the following has the greatest reciprocal?

(A) – 5 (B) – 4 (C) 4 (D) 6 (E) 5


4 5 5 5 6

© The MBA Center 53


Total PrepKit for the GMAT® Step 2.1 Lesson

17. What is the 99th digit after the decimal point in the decimal equivalent
of 1 ?
11
(A) 0
(B) 1
(C) 3
(D) 6
(E) 9

18. A recipe requires 2 1 cups of flour, 2 3 cups of sugar, and 1 1 cups of milk
2 4 3
to make one cake. Victor has 15 cups of flour, 16 cups of sugar, and 8
cups of milk. What is the greatest number of cakes Victor can make using
this recipe?
(A) 5
(B) 6
(C) 7
(D) 8
(E) 9

19. Martha biked 18 2 miles in 2 hours and 40 minutes. What was her
3
average rate of speed in miles per hour?

(A) 7

(B) 7 2
3
(C) 8 1
3
(D) 9

(E) 9 1
3

20. Exactly 3 of the people in the room are under the age of 21, and exactly
7
5 of the people in the room are over the age of 65. If the total number
13
of people in the room is greater than 50 and less than 100, how many

people in the room are under the age of 21?

(A) 21
(B) 35
(C) 39
(D) 60
(E) 65

54 © The MBA Center


Math Review

3. PERCENTS
Most percent problems can be solved by plugging into one basic formula:
Percent × Whole = Part
This formula describes the relationship among three quantities. Given any
two of them, you can find the third. Whether you’re looking for the
Percent, the Whole, or the Part, use the same formula.

Q: What is 80% of 25?


A: You know the Percent (80%) and the Whole (25), and you’re
looking for the Part:
Percent × Whole = Part
0.80 × 25 = Part
Part = 20

Q: 21 is 75% of what number?


A: You know the Percent (75%) and the Part (21), and you’re looking
for the Whole:
Percent × Whole = Part
0.75 × Whole = 21
21
Whole = = 28
0.75

Q: 30 is what percent of 24?


A: You know the Part (30) and the Whole (24), and you’re looking for
the Percent:
Percent × Whole = Part
Percent × 24 = 30
30
Percent = = 1.25 = 125%
24

In the last example, the Part was bigger than the Whole. That’s why the
Percent was greater than 100%.
If the Part can be bigger than the Whole, how are you supposed to be able
to tell which is the Part and which is the Whole? You can tell by watching
for these little key words: “of” and “is”/“are.” The Whole is almost always
associated with the word “of,” and the Part is almost always associated with
the verb “is”/“are.”
Q: What is 80% of 25?
A: The word “what” comes just before the word “is,” and so it’s the
Part. The 25 comes just after the word “of,” and so it’s the Whole.

Q: 21 is 75% of what number?


A: The 21 comes just before the word “is,” and so it’s the Part. “What
number” comes just after the word “of,” and so it’s the Whole.

Q: 30 is what percent of 24?


A: The 30 comes just before the word “is,” and so it’s the Part. The
24 comes just after the word “of,” and so it’s the Whole.

© The MBA Center 55


Total PrepKit for the GMAT® Step 2.1 Lesson

To solve a percent word problem, use the key words and your
understanding of the sense of the problem to identify the Percent, the
Whole, and the Part. Plug the two numbers you know into the formula and
calculate the number you’re looking for.
Q: In a baseball game, the starting pitcher lasted 5
innings and threw 65 pitches, 39 of which were strikes.
What percent of her pitches were strikes?
A: The total number of pitches (65) is the whole. The number of
strikes (39) is the part. You’re looking for the percent:
Percent × Whole = Part
Percent × 65 = 39
39
Percent = = 0.6 = 60%
65

Q: On his history final, John answered 84 percent of the


questions correctly. If he answered 63 questions
correctly, what was the total number of questions on
the test?
A: The number of correct answers (63) is the Part. The Percent is 84,
and you’re looking for the Whole:
Percent × Whole = Part
84% × Whole = 63
63
Whole = = 75
0.84

To calculate the percent increase or decrease, divide the amount of


change by the original amount and multiply by 100%.

Amount of increase
Percent increase = × 100%
Original amount

Amount of decrease
Percent decrease = × 100%
Original amount

Q: Last month, the Dow Jones average of American stocks


went up from 9,875 to 10,033. What was the percent
increase in the Dow Jones average last month?
A: The amount of increase is 10,033 – 9,875 = 158 points. The
original amount is 9,875. Plug those two numbers into the
formula:
Amount of increase
Percent increase = × 100%
Original amount
158
= × 100%
9875
= 0.016 × 100% = 1.6%

Q: From 1990 to 2000, the population of Middletown


decreased from 73,824 to 64,596. What was the
percent decrease in the population of Middletown
from 1990 to 2000?

56 © The MBA Center


Math Review

A: The amount of decrease is 73,824 – 64,596 = 9,228. The original


amount is 73,824:
Amount of decrease
Percent decrease = × 100%
Original amount
9228
= × 100% = 12.5%
73824

To calculate the result of a percent increase or decrease when you’re given


the original amount, find that percent of the original amount and add it or
subtract it.

Q: The regular price for a box seat is $30. On Monday


nights the price is reduced by 20 percent. What is the
price of a box seat on Monday nights?
A: First find 20% of $30:
20% of $30 = 0.2 × $30 = $6
The amount of decrease is $6. Subtract that from the regular price
to get the discounted price:
Discounted price = $30 – $6 = $24

It’s a bit more complicated to calculate the original amount when given
the percent increase or decrease and the new amount. You cannot just
multiply the percent by the new amount. The thing to do instead is to add
the percent increase to 100% (or subtract the percent decrease from 100%),
then divide that into the new amount.

Q: This year, the regular price for a box seat is $30, which
is 20 percent more than last year. What was the price
of a box seat last year?
A: You cannot just figure out what’s 20% less than $30, as we did in
the previous example. Here you must first add 20% to 100%,
which gives you 120%, and then divide that into $30:

New price $30


Old price = = = $25
120% 1.2

Be careful with combined percent increases and decreases. If a quantity


increases first by x percent, and then the new quantity increases by y
percent, you cannot simply say that the combined percent increase is the
sum x + y. You cannot add percents in this case because they are percents of
different bases.
Here, for example, is a case in which 20 and 20 do not add up to 40.

Q: In the first week of a clearance sale, the original price


P of chair was reduced by 20 percent to price Q. In the
second week price Q was reduced by another 20 percent
to final price R. Final price R is what percent less than
original price P?
A: Reducing by 20% is the same as multiplying by 0.8. Q is 20% less
than P, so:
Q = 0.8P
Then, R is 20% less than Q, so:
R = 0.8Q = 0.8(0.8P) = 0.64P
So R is 64% of P. In other words, R is 36% less than P.

© The MBA Center 57


Total PrepKit for the GMAT® Step 2.1 Lesson

PRACTICE QUESTIONS 21-30 (PERCENTS)

21. If 51 of the 85 marbles in a bag are green, what percent of the marbles
are green?

(A) 57%
(B) 60%
(C) 63%
(D) 65%
(E) 67%

22. 18 is 24 percent of what number?

(A) 73
(B) 74
(C) 75
(D) 76
(E) 77

23. If 7 of the student body is male, what percent of the student body is
16
female?

(A) 37.5%
(B) 43.75%
(C) 56.25%
(D) 62.5%
(E) 68.25%

24. In addition to the price of the meal, Janet paid sales tax equal to 8.5
percent of the price of the meal, and a tip equal to 15 percent of the price
of the meal. If she paid $2.04 in sales tax, how much was the tip?

(A) $3.06
(B) $3.36
(C) $3.42
(D) $3.60
(E) $4.08

25. If the price of a painting rises from $5,000 to $100,000, what is the
percent increase?

(A) 95%
(B) 190%
(C) 950%
(D) 1,900%
(E) 9,500%

58 © The MBA Center


Math Review

26. A promotion gave Martin a 32 percent increase in his hourly wage. If his
hourly wage after the promotion was $9.90, what was it before the
promotion?

(A) $7.50
(B) $7.60
(C) $7.72
(D) $8.00
(E) $8.18

27. The price of a share of stock increased by 20 percent in one year. The
following year the price increased by 30 percent. By what percent did the
price increase over the two-year period?

(A) 44%
(B) 45%
(C) 50%
(D) 55%
(E) 56%

28. The population of Centerville increased by exactly 20 percent between the


years 1990 and 2000. If the population in the year 2000 was 15,600,
what was the population in 1990?

(A) 12,000
(B) 12,480
(C) 13,000
(D) 14,580
(E) 18,720

29. From 1990 to 1995, the population of Smallville increased by 50 percent,


and from 1995 to 2000, the population increased by 200 percent to
13,284. The population in 2000 was how much greater than the
population in 1990?

(A) 2,952
(B) 4,428
(C) 7,971
(D) 8,856
(E) 10,332

30. In the first half of the 20th century, the population of a particular country
increased by 200 percent. In the second half of the century, the
population increased by 300 percent. What was the percent increase for
the 20th century as a whole?

(A) 500%
(B) 600%
(C) 800%
(D) 1,100%
(E) 1,200%

© The MBA Center 59


Total PrepKit for the GMAT® Step 2.1 Lesson

4. AVERAGE, MEDIAN, AND MODE


Average (arithmetic mean) — the sum of a set of numbers divided by
the number of numbers in the set

Average (arithmetic mean) = Sum of the numbers


Number of numbers

Q: What is the average (arithmetic mean) of 2, 5, 8, 19,


and 56?
A: The sum of the numbers is 2 + 5 + 8 + 19 + 56 = 90. There are 5
numbers in the list, so you divide the sum by 5:

Average = 2 + 5 + 8 + 19 + 56 = 90 = 18
5 5

Averaging averages — You can average the averages of two groups only if
you know that the two groups are the same size.

Q: A science test was administered to 40 students: 20


boys and 20 girls. The boys’ average (arithmetic mean)
score was 86 and the girls’ average score was 92. What
was the average score for all 40 students?
A: The two groups are the same size, so you can just average the
averages:

Average = 86 + 92 = 178 = 89
2 2

Do not average the averages if you do not know the relative sizes of the
groups.

Q: A Spanish test was administered to a group of


students. The boys’ average (arithmetic mean) score
was 84 and the girls’ average score was 78. What was
the average score for all the students?
A: You do not have enough information. You can not assume that the
number of boys is the same as the number of girls, and so you can
not just average the averages.

Weighted average — If you have two groups of different sizes, but you
know the actual or relative sizes, then you can find a weighted average. To
find the weighted average, multiply each average by the appropriate
weighting factor before adding, and then divide by the sum of the
weighting factors. If you are given actual numbers, use them as the
weighting factors.

Q: An English test was administered to 20 boys and 10


girls. The boys’ average (arithmetic mean) score was
84 and the girls’ average score was 78. What was the
average score for all 30 students?
A: Multiply the boys’ average by 20 and the girls’ average by 10, add
the products, and divide the sum by 30:

Average = 20(84) + 10(78) = 1680 + 780 = 2460 = 82


30 30 2

60 © The MBA Center


Math Review

Using the average to find the sum — If you know the average of a list of
numbers and you know how many numbers are in the list, you can use the
following variation of the average formula to find the sum:
Sum of the numbers = (Average) × (Number of numbers)

Q: Julia paid an average (arithmetic mean) of $42 a


night for 7 nights of hotel stays. What was the total
amount she paid for the 7 nights of hotel stays?
A: Multiply the average paid per night by the number of nights to get
the total amount paid:
Sum of the numbers = $42 × 7 = $294

This variation of the average formula is also useful when you’re looking for
a missing number in an average. Just remember, whenever you know the
average and you know how many numbers there are, then you also know
what the sum is.

Q: Martin’s average (arithmetic mean) score on 4


economics tests last semester was 95. If he scored 91,
97, and 100 on the first 3 tests, what was his score on
the 4th test?
A: Multiply the average score by the number of tests to get the sum of
the test scores:
Sum of the numbers = 95 × 4 = 380
The first 3 scores add up to 91 + 97 + 100 = 288, and so the 4th
score was 380 – 288 = 92.

Consecutive integers — integers that follow one another in order


Ex: 4, 5, 6, 7, 8, 9, 10, and 11 are consecutive integers.
Ex: 24, 26, 28, 30, and 32 are consecutive even integers.
Ex: –5, –3, –1, 1, and 3 are consecutive odd integers.
Ex: –10, –5, 0, 5, 10, 15, 20, and 25 are consecutive multiples of 5.
In all of the above examples, the numbers are evenly spaced. In other
words, the difference between one number and the next is constant.
Consecutive numbers are often evenly spaced, but not always. Look at the
following examples.
Ex: 2, 3, 5, 7, 11, 13, 17, and 19 are consecutive prime numbers.
Ex: 2, 4, 8, 16, and 32 are consecutive powers of 2.

Average of evenly-spaced numbers — When the numbers in a list are


evenly spaced, it is easy to find their average (arithmetic mean). It’s the
number in the middle.

Q: What is the average (arithmetic mean) of 13, 14, 15,


16, 17, 18, and 19?
A: The numbers are evenly spaced, and so the average is the number
in the middle: 16.

If the number of numbers is even, then the average is halfway between the
two numbers in the middle.

Q: What is the average (arithmetic mean) of 22, 24, 26,


28, 30, 32, 34, and 36?
A: The numbers are evenly spaced, but this time there is an even
number (8) of them and so the average is halfway between the two
numbers in the middle, 28 and 30. The average is 29.

© The MBA Center 61


Total PrepKit for the GMAT® Step 2.1 Lesson

How to find the middle number — When a list of evenly spaced numbers
is relatively short, it’s easy enough just to see the number that’s in the
middle. For a longer list, the way to find the number in the middle is to find
the average of the first and last numbers in the list.

Q: What is the average (arithmetic mean) of all the odd


numbers from 15 through 49 inclusive?
A: The average of the whole set of numbers is the same as the average
of the first and last numbers:
Average = First + Last = 15 + 49 = 54 = 27
2 2 2
Finding the sum of evenly spaced numbers — Because it is easy to find
the average of a set of evenly spaced numbers, it is also easy to find their
sum. Remember, the sum is equal to the average multiplied by the number
of numbers.
Q: What is the sum of all the positive integers up to and
including 99?
A: First find the average by averaging the first and last:

Average = First + Last = 1 + 99 = 100 = 50


2 2 2
There are 99 integers in the list, so to find the sum, multiply the
average by 99:
Sum = Average × Number = 50 × 99 = 4,950

To count the number of consecutive integers — subtract the smallest


from the largest and then add 1.
Q: There are how many positive 3-digit integers?
A: The smallest of the positive 3-digit integers is 100, and the largest
is 999. To count them, subtract (999 – 100 = 899) and add 1 (899
+ 1 = 900). The answer is 900.

Median — the middle number. To find the median, first arrange the
numbers in order from least to greatest and then find the number that’s
right in the middle.
Q: Find the median: 12, 734, 52, –74, 345, 1, 8, –90, 23
A: Arrange the numbers by size:
–90, –74, 1, 8, 12, 23, 52, 345, 734
Now the number in the middle is 12, and so the median is 12.
If there’s an even number of numbers in the set, then there is no one
number in the middle. In that case, find the pair of numbers in the middle
and take their average (arithmetic mean).
Q: Find the median: 45, 3, 67, 2, 498, 5, 3, 10
A: Arrange the numbers by size:
2, 3, 3, 5, 10, 45, 67, 498
This time there are 8 numbers in the set, so there is no number
exactly in the middle. The middle pair of numbers is 5 and 10. The
average of those numbers is 7.5, and so the median is 7.5.

Mode — the number that appears most frequently.


Q: Find the mode: 1, 1, 2, 3, 4, 4, 7, 7, 8, 8, 8, 8, 9, 10, 10
A: The number 8 appears four times, more often than any other
number, and so the mode is 8.

62 © The MBA Center


Math Review

If two numbers appear with equal frequency, then there are two modes.
Q: Find the mode: 24, 24, 24, 25, 27, 27, 27, 28, 29, 29, 30
A: The numbers 24 and 27 both appear three times, more often than
any other number, and so there are two modes: 24 and 27.
If no number appears more often than once, then there is no mode.
Q: Find the mode: 1, 3, 5, 7, 13, 63, 198, 873
A: No number appears more than once, and so there is no mode.

Range — the positive difference between the largest and smallest values in
a set of numbers.
Q: Find the range: 31, 43, 25, –17, 19, 63, 18, –3
A: The largest number is 63 and the smallest is –17, and so the range
is 63 – (–17) = 80.

Standard deviation — a quantitative description of the dispersion, or


spread, of a set of numbers. To find the standard deviation:
1. Calculate the average (arithmetic mean).
2. Calculate the differences between the average and each number
in the set.
3. Square these differences.
4. Calculate the average of the squares.
5. Take the square root of that average.
Q: What is the standard deviation of the set {1, 3, 3, 5}?
A: You can tell by inspection that the average (arithmetic mean) is 3.
The differences from the average are –2, 0, 0, and 2. The squares
of the differences are 4, 0, 0, and 4. The average of those squares
is 2, and the square root of that average is the square root of 2. So
the standard deviation is approximately 1.4.
On the GMAT, you are not likely to be asked to calculate the standard
deviation for a large set of numbers. But you should have some feeling for
what standard deviation means.
Q: Set A = {11, 12, 13, 14, 15}. Set B = {9, 11, 13, 15, 17}.
Which set has the greater standard deviation?
A: You should be able to tell, without actually calculating, that Set B
will have the greater standard deviation. That is because the
elements in Set B are more widely dispersed — the differences
between and the average are greater. (In fact, the standard
deviation for Set A is the square root of 2 (about 1.4), and the
standard deviation for Set B is 2 times the square root of 2 (about
2.8).

Q: Set C = {11, 12, 13, 14, 15}. Set D = {81, 82, 83, 84, 85}.
Which set has the greater standard deviation?
A: You should be able to tell, without actually calculating, that these
two sets have the same standard deviation. They have the same
dispersions about their respective averages. (In fact, the standard
deviation for each set is the square root of 2 (about 1.4).

© The MBA Center 63


Total PrepKit for the GMAT® Step 2.1 Lesson

PRACTICE QUESTIONS 31-40 (AVERAGE, MEDIAN, AND MODE)


Questions 31-34 refer to the following sets:
Set A = {0, 0, 12}
Set B = {0, 0, 0, 15}
Set C = {0, 1, 7, 7}
Set D = {1, 4, 4, 6}
Set E = {1, 4, 5, 5}

31. Which set has the greatest average (arithmetic mean)?


(A) Set A
(B) Set B
(C) Set C
(D) Set D
(E) Set E

32. Which set has the greatest median?


(A) Set A
(B) Set B
(C) Set C
(D) Set D
(E) Set E

33. Which set has the greatest mode?


(A) Set A
(B) Set B
(C) Set C
(D) Set D
(E) Set E

34. Which set has the greatest standard deviation?


(A) Set A
(B) Set B
(C) Set C
(D) Set D
(E) Set E

{0, 0, 6, 6, 6, 9, 10, 10, 12, 12}


35. What is the product of the median and the mode of the above set of
numbers?

(A) 13.5
(B) 26
(C) 32.5
(D) 45
(E) 54

64 © The MBA Center


Math Review

36. After the first four tests of the semester, Claudia’s average (arithmetic
mean) score is 76. What score does Claudia need on the fifth test to bring
her average up to 80?

(A) 84
(B) 87
(C) 90
(D) 93
(E) 96

37. A chemistry test was administered to 21 boys and 24 girls. The boys’
average (arithmetic mean) score was 60 and the girls’ average score was
90. What was the average score for all 45 students?

(A) 74
(B) 75
(C) 76
(D) 77
(E) 78

38. What is the average (arithmetic mean) of all the even numbers from 186
through 540 inclusive?

(A) 363
(B) 364
(C) 365
(D) 366
(E) 367

39. What is the sum of all the positive integers up to and including 50?

(A) 1,250
(B) 1,275
(C) 2,050
(D) 2,450
(E) 2,500

40. The average (arithmetic mean) of three integers a, b, and c is exactly


twice the median. If a < b < c, and a = 0, what is the value of c ?
b
(A) 2
(B) 3
(C) 4
(D) 5
(E) cannot be determined from the information given

© The MBA Center 65


Total PrepKit for the GMAT® Step 2.1 Lesson

5. EXPONENTS AND SQUARE ROOTS


Exponents
An exponent indicates how many times that number appears as a factor in
a product.
Q: 25 = ?
A: The exponent 5 indicates that 2 appears as a factor 5 times:
25 = 2 × 2 × 2 × 2 × 2 = 32
In the expression 25, the 2 is called the base and the whole expression is
called a power. It can be read “2 to a power of 5,” or “2 to the 5th power,”
or most simply “2 to the 5th.”
An exponent of 2 is most often read “squared,” and an exponent of 3 is
most often read “cubed.”
Ex: 52 is read “5 squared” and equals 5 × 5, or 25.
Ex: 63 is read “6 cubed” and equals 6 × 6 × 6, or 216.
Parentheses are used to show that a negative number is being raised to a
power.
Ex: (–3)2 equals (–3) × (–3), or 9.
Ex: (–3)3 equals (–3) × (–3) × (–3), or –27.
Notice that a negative number raised to an even power is positive, and that
a negative number raised to an odd power is negative.
Notice also that if there are no parentheses, you are to apply the exponent
before the minus sign.
Q: –42 = ?
A: Square the 4 to get 16, then add the minus sign:
–42 = –16
When a fraction or decimal between 0 and 1 is squared or cubed or raised
to a higher power, the result is smaller than the original number.
3
 1 1 1 1 1
Ex:   = × × =
 2 2 2 2 8
Ex: (0.9)2 = 0.9 × 0.9 = 0.81

The Rules Of Exponents

To multiply powers with the same base, keep the base and
add the exponents.
Ex: 319 × 312 = 319 + 12 = 331
To divide powers with the same base, keep the base and
subtract the exponents.
5 24
Ex: = 5 24 − 6 = 518
6
5
To raise a power to a power, multiply the exponents.
2
Ex: (610) = 610 × 2 = 620
To multiply powers with the same exponent, keep the
exponent and multiply the bases.
Ex: 59 × 49 = (5 × 4)9 = 209

66 © The MBA Center


Math Review

Square Roots
The number A is a square root of B if A squared equals B.
Q: What are the square roots of 9?
A: 3 squared equals 9, so 3 is a square root of 9. –3 squared also
equals 9, so –3 is also a square root of 9.

Every positive number has two square roots, one positive and one negative.
On the other hand, as far as the GMAT is concerned, negative numbers have
no square roots.
The symbol is used to represent the nonnegative square root.

Q: 9 = ?
A: The nonnegative square root of 9 is 3, and so 9 = 3.

You should be able to recognize numbers up to 144 that have integer square
roots. These numbers are called perfect squares.
1 = 12 16 = 42 49 = 72 100 = 102
4 = 22 25 = 52 64 = 82 121 = 112
9 = 32 36 = 62 81 = 92 144 = 122
If an integer is not a perfect square, then its square root is an irrational
number — that is, a number that cannot be expressed precisely as a fraction
or decimal. An irrational number is a real number — it has a value and a
place on the number line. Two very important irrational numbers are 2
and 3 . You should have some sense of the value of these numbers:

2 ≈ 1.414

3 ≈ 1.732
The Rules Of Square Roots

The product of square roots is equal to the square root of the


product.
Ex: 2 × 3 = 2×3 = 6
The quotient of square roots is equal to the square root of the
quotient.
Ex: 10 10
= = 2
5 5
The power of square roots is equal to the square root of the
power.
3
Ex:  2  = 2 3 = 8
 
You cannot simplify the sum of square roots if they have
different numbers inside the radical.
Ex: 2 + 3 cannot be written any more simply.
To add square roots with the same number inside the radical,
use what’s called a coefficient — a number written in front of
the radical to express how many square roots have been added.
Ex: 6+ 6+ 6 =3 6
Ex: 2 6 + 3 6 = 5 6

© The MBA Center 67


Total PrepKit for the GMAT® Step 2.1 Lesson

To simplify a square root, factor out the perfect squares inside the radical,
unsquare them, and put the result in front.

Q: Simplify: 300
A: Factor out a perfect square of 100 and reexpress 300 as 100 × 3:
300 = 100 × 3
Now you can break this square root into two and unsquare the
perfect square:
100 × 3 = 100 × 3 = 10 3

If there is already a number in front of the radical, multiply.


Q: Simplify: 2 18
A: You can factor out a perfect square of 9 and reexpress 18 as 9 × 2:
2 18 = 2 9 × 2 = 2 × 9 × 2 = 2 × 3 × 2 = 6 2

Rationalizing the denominator — To simplify an expression that has a


square root in the denominator, multiply both the numerator and
denominator by that same square root. This will give you an integer instead
of a square root on the bottom. This process is called rationalizing the
denominator.

Q: Simplify: 2 5
5 2
A: Multiply both the numerator and denominator by 2:

2 5 2 5× 2 2 10 10
= = =
5 2 5 2× 2 10 5

PRACTICE QUESTIONS 41-50 (EXPONENTS AND SQUARE ROOTS)


41. –104 = ?
(A) –10,000
(B) –1,000
(C) –40
(D) 1,000
(E) 10,000

42. 510 × 510 = ?


(A) 100
(B) 520
(C) 5100
(D) 2520
(E) 25100

43. 27 × 25 × 312 = ?
(A) 612
(B) 624
(C) 1212
(D) 1224
(E) 12420

68 © The MBA Center


Math Review

44. All of the following are equal EXCEPT:


(A) 220
(B) 219 + 219
(C) 410
(D) 45 + 45
(E) 165

45. 35 + 35 + 35 = ?
(A) 93 (B) 94 (C) 95 (D) 96 (E) 915

46. 2 5 ( 3 2 + 2 2 ) = ?
(A) 5 10
(B) 10 10
(C) 10 5
(D) 20 5
(E) 30

(−4)
4
47. =?
(A) –16 (B) –4 (C) 16 (D) 64 (E) 128

48. All of the following are equal EXCEPT:

1 1 2 3 6
(A) (B) (C) (D) (E)
2 2 4 6 12

2 3 +3 2
49. =?
2
(A) 2 + 2 3
(B) 2+ 6
(C) 2 + 3 6
(D) 3 + 2 3
(E) 3 + 6

50. 50 + 200 = ?
(A) 2 15
(B) 5 5
(C) 5 10
(D) 10 5
(E) 15 2

© The MBA Center 69


Total PrepKit for the GMAT® Step 2.1 Lesson

6. EQUATIONS
To solve an equation with one variable and no exponents, the basic rule is
this: do whatever you must to both sides to get the variable all by itself on
one side, or, in other words, to isolate the variable. You can add the same
thing to both sides, subtract the same thing from both sides, multiply both
sides by the same thing, or divide both sides by the same thing. As long as
you do exactly the same thing to both sides, you will maintain the equality.
Q: Solve: –3x + 5 = 11
A: To start to isolate x, first subtract 5 from both sides:
−3 x + 5 = 11
−3 x + 5 − 5 = 11 − 5
−3 x = 6
Now, to isolate x, divide both sides by –3:
−3 x 6
=
−3 −3
x = −2

How do you know what operations to perform in what order? Usually you’ll
just know from experience. But here is a systematic procedure:
1. Eliminate all denominators by multiplying both sides of the equation
by the LCM of all denominators.
2. Eliminate all parentheses.
3. Move all terms that contain the variable to one side and all terms that
do not contain the variable to the other side.
4. Combine all terms that contain the variable into a single term.
5. Divide both sides by the coefficient of the variable.

( )
Q: Solve: 2 x − 5 = 1 x + 5
3 4
A: First eliminate the denominators. The LCM of 3 and 4 is 12, so
multiply both sides by 12:
 
12 ×
2
3
( x − 5) = 12 ×  x + 5
1
4 
8( x − 5) = 3 x + 60

Then eliminate the parentheses by distributing the 8:


8 x − 40 = 3 x + 60
Now move every term with an x to the left (by subtracting 3x from
both sides) and every term without an x to the right (by adding 40
to both sides):
8 x − 3 x = 60 + 40
5 x = 100
Now all you have left to do to isolate x is to divide both sides by 5:
5x 100
=
5 5
x = 20

70 © The MBA Center


Math Review

Solving “In Terms Of”


If an equation has more than one variable, you cannot normally find a
numerical root, but you can still solve the equation. To “solve” means to
isolate the variable.
Q: Solve for x: x + 10 = y
A: To isolate the x, subtract 10 from both sides:
x + 10 = y
x + 10 − 10 = y − 10
x = y − 10
The equation is solved for x. The solution is said to give x in terms
of y.
When solving for one variable in terms of another, the 5-step equation-
solving procedure outlined above becomes especially useful.

np
Q: Solve for n: v =
r + nt
A: First, eliminate the denominator by multiplying both sides by
r + nt:
np
v=
r + nt

(
v r + nt =) np
r + nt
(
r + nt )
( )
v r + nt = np
Then distribute to eliminate the parentheses:
rv + ntv = np
Now move every term with an n to one side and every term without
n to the other. Subtract np from both sides and subtract rv from
both sides:
rv + ntv − np − rv = np − np − rv
ntv − np = −rv
Next, rewrite the left side as one term with n. To do that, you
“factor” n out:
(tv − p)n = −rv
Lastly, divide both sides by the “coefficient” of n:

(tv − p)n = −rv


tv − p tv − p
−rv
n=
tv − p
You now have a form with n isolated. That’s the solution. It gives
n in terms of p, r, t, and v.

Solving A Pair Of Equations


If you have two distinct equations with two unknowns, you can solve for
the roots of the equations. First, pick a variable and isolate it in one of the
equations. Then substitute the equivalent expression for that variable in the
other equation, eliminating that variable from the equation and giving you
one equation with one variable, which you can solve by the usual means.

© The MBA Center 71


Total PrepKit for the GMAT® Step 2.1 Lesson

2x + 3y = 18
Q: Solve for x and y: 
3x − 5y = 8
A: You can start with either equation and either variable. Let’s start
with the first equation and solve for x. Subtract 3y from both sides,
and then divide both sides by 2:
2 x + 3y = 18
2 x = 18 − 3y
3y
x = 9−
2
Now you know that is the same as x. Take that new expression for
x and substitute it for x in the other equation:
3 x − 5y = 8
 3y 
3  9 −  − 5y = 8
 2

Now you have one equation with one variable. To solve for y, first
distribute the 3 and multiply both sides by 2:

9y
27 − − 5y = 8
2
54 − 9y − 10 y = 16
54 − 19y = 16

Then subtract 54 from both sides and divide both sides by –19:

−19y = −38
y=2

Now that you know that y = 2, plug 2 in for y in either of the


original equations to find x:
2 x + 3y = 18
()
2 x + 3 2 = 18
2 x + 6 = 18
2 x = 12
x=6

And so the solution is x = 6 and y = 2.

Note that you can find numerical roots of a pair of equations only if they
are distinct equations.

2a − 6b − 1 = 3
Q: Solve for a and b: 
5a − 3 = 15b + 7
A: Solve the first equation for a:
2a − 6 b − 1 = 3
2a = 6 b + 4
a = 3b + 2

Now substitute 3b + 2 for a in the other equation:

72 © The MBA Center


Math Review

5a − 3 = 15 b + 7
( )
5 3 b + 2 − 3 = 15 b + 7

Look at what happens when you try to solve for b:


15 b + 10 − 3 = 15 b + 7
15 b + 7 = 15 b + 7
Any number will work for b. There are infinitely many solutions.
That’s because what looked like two equations to start with were in
fact just the same equation in two different forms. The second
equation is just another way of writing the first equation. You do
not have two distinct equations, and so you are not able to find
numerical solutions.

A popular alternative method for solving a pair of equations is to “add” the


equations. Your goal is the same: you want to get rid of one variable so you
can solve for one variable at a time. To do that, you will normally need to
reexpress one or both equations so that a variable has coefficients that will
cancel when you add.
7x − 6y = −4
Q: Solve for x and y: 
4x + 3y = 17
A: If you multiply both sides of the second equation by 2, you get 8x
+ 6y = 34. Now the coefficients of y in the two equations are
opposites and the y terms will cancel when you add the two
equations:
7 x − 6y = −4
8 x + 6y = 34
15 x = 30
Divide both sides by 15 and you find that x = 2. Plug that in for x
in one of the original equations and you’ll find that y = 3.

Solving Inequalities
To solve an inequality means to isolate the variable on one side. The process
is much the same as the process for solving an equation.
Q: 5x – 13 > 3x – 5
A: To begin to isolate x, first subtract 3x from both sides and add 13
to both sides:
5 x − 13 > 3 x − 5
5 x − 13 − 3 x + 13 > 3 x − 5 − 3 x + 13
2x > 8
Then divide both sides by 2 get the solution: x > 4.
The only thing different about solving an inequality is that when you
multiply or divide both sides by a negative number, you must flip the sign.
Q: −7x + 15 ≤ 8
A: To begin to isolate x, first subtract 15 from both sides:
−7 x + 15 ≤ 8
−7 x + 15 − 15 ≤ 8 − 15
−7 x ≤ −7
All that’s left to do now is to divide both sides by –7. When you do
that, you must flip the sign, that is, change ≤ to ≥. The solution is
x ≥ 1.

© The MBA Center 73


Total PrepKit for the GMAT® Step 2.1 Lesson

Factoring Polynomials
When simplified, polynomials have no parentheses. There are times,
however, when you want parentheses. The process of reexpressing a
polynomial with parentheses, i.e., as a product, is called factoring.

Ex: To factor the polynomial 3x2 + 7x + 2 means to reexpress it as


(3x + 1)(x + 2).
To factor a polynomial, try to figure out what simpler monomials or
polynomials could be multiplied to produce the polynomial you are trying
to factor.

Q: Factor: x2 – 10x
A: This polynomial is relatively easy to factor. You can see that the
two terms each have a factor of x, so you can just “factor out” the
x:
x2 – 10x = x(x – 10)

Q: Factor: 2x2 – x – 3
A: If this polynomial is factorable, then it will probably factor into
two binomials, one beginning with 2x and the other beginning
with x:
2x2 – x – 3 = (2x )(x )
The second part of each factor will be a number, and the product
of those numbers will be –3. Thus they could be +1 and –3, or –1
and +3: Those are the only pairs of integers that have a product of
–3. But in this case the order is important. It makes a difference
which one goes in the first factor with the 2x and which one goes
in the second factor with the x. Pick a pair and try it. Say, +1 and
–3:
(2x + 1)(x – 3) = 2x2 – 5x – 3
The first and third terms are the right ones, but the middle term is
wrong. Now try the same numbers in the other order. Put the –3 in
the first factor and the +1 in the second factor:
(2x – 3)(x + 1) = 2x2 – x – 3

Remember that some polynomials are not factorable.

Q: Factor: x2 + 3x – 5
A: If this polynomial is factorable, then it will probably factor into
two binomials, each beginning with x:
x2 + 3x – 5 = (x )(x )
The second part of each factor will be a number, and the product
of those numbers will be –5. Thus they could be –1 and +5, or +1
and –5. Those are the only possibilities. Pick a pair and try it. Say,
–1 and +5:
(x – 1)(x + 5) = x2 + 4x – 5
That pair did not work, so try the other possibility, +1 and –5:
(x + 1)(x – 5) = x2 – 4x – 5
That pair did not work either, and there are no other possibilities,
so this polynomial is not factorable.

74 © The MBA Center


Math Review

Factor Patterns
With practice and experience, you will become adept at factoring. You will
begin to recognize patterns. There are three factor patterns in particular you
should know:
1. Common monomial factor
2. Difference of squares
3. Square of a binomial
Whenever you are trying to factor a polynomial, the first thing to look for
is a common monomial factor — that is, a monomial that can be factored
out of every term in the polynomial.
Q: Factor: 2x3 + 6x2 + 10x
A: In this polynomial, every term has a factor of 2x. That means you
can “factor it out.” To do so, write 2x in front of an open
parenthesis:
2x3 + 6x2 + 10x = 2x( )
Now, inside the parentheses, write term by term what’s left after
you divide by 2x:
2x3 + 6x2 + 10x = 2x(x2 + 3x + 5)

A second factor pattern you should be able to spot is what is called the
difference of squares. This is one of the easiest patterns to recognize. It
consists of two monomials squared separated by a minus sign.
A polynomial in the form a2 – b2 factors to (a – b)(a + b). So to factor the
difference of squares, “unsquare” them both and make two binomials out
of the results — one with a minus sign and one with a plus sign.
Q: Factor: 9x2 – 16
A: This is an example of a difference of squares: in front of the minus
sign you have 9x2, which is the square of 3x, and after the minus
sign you have 16, which is the square of 4. Unsquare the first
square and you get 3x. Unsquare the second square and you get 4.
Put those results into two binomials, one with a minus sign and
the other with a plus sign:
9x2 – 16 = (3x – 4)(3x + 4)

A third factor pattern you should be able to spot is the square of a


binomial. This pattern consists of three terms — a first term that’s a square,
a third term that’s a square, and a middle term that’s equal to twice the
product of the square roots of the first and third terms. This pattern is
probably easier to visualize symbolically:
a2 + 2ab + b2 = (a + b)2
a2 – 2ab + b2 = (a – b)2

Q: Factor: y2 – 8y + 16
A: The first term is the square of y, the third term is the square of –4,
and the middle term is twice the product of y and –4. So this is the
square of the binomial y – 4:
y2 – 8y + 16 = (y – 4)2

© The MBA Center 75


Total PrepKit for the GMAT® Step 2.1 Lesson

Solving Quadratic Equations By Factoring


A quadratic equation is an equation in which the unknown is squared.
Quadratic equations can have as many as two roots. To solve a quadratic
equation by factoring, first rearrange the equation so that it is in the form
“polynomial = 0.” Then factor the polynomial. When the polynomial is
factored, you have a product that’s equal to 0, which means that one of the
factors must be 0.
Q: Solve: x2 + 12 = 7x
A: First rearrange the equation into “polynomial = 0” form:
x2 – 7x + 12 = 0
Now factor the polynomial:
(x – 3)(x – 4) = 0
In this form, the equation tells you that the product of x – 3 and
x – 4 is 0. That means that either x – 3 = 0 or x – 4 = 0.
x − 3 = 0 OR x − 4 = 0
x = 3 OR x = 4
So the two roots are 3 and 4.

Q: Solve: 2x2 + 7x = –3
A: First rearrange the equation into “polynomial = 0” form:
2x2 + 7x + 3 = 0
Now factor the polynomial:
(2x + 1)(x + 3) = 0
In this form, the equation tells you that the product of 2x + 1 and
x + 3 is 0. That means that either 2x + 1 = 0 or x + 3 = 0:
2 x + 1 = 0 OR x + 3 = 0
1
x=− OR x = −3
2

1
So the two roots are − and –3.
2

If when you put the equation into “polynomial = 0” form the polynomial
is the square of a binomial, then there will be only one root.

Q: Solve: x2 + 10x = –25


A: First rearrange the equation into “polynomial = 0” form:
x2 + 10x + 25 = 0
Now factor the polynomial:
(x + 5)2 = 0
In this form, the equation tells you that the square of x + 5 is 0.
That means that x + 5 = 0. So the only root is –5.

76 © The MBA Center


Math Review

Quadratic Formula
You can always solve a quadratic equation with what is called the quadratic
formula. Once you have the equation in “polynomial = 0” form, think of it
this way:
ax 2 + bx + c = 0
If the coefficient of the x2 term is a, the coefficient of the x term is b, and
the number term is c, then the solution to the equation is:

− b ± b 2 − 4ac
x=
2a

This formula is especially useful when the polynomial is not factorable.

Q: Solve: x2 + 2x = 9
A: First rearrange the equation into “polynomial = 0” form:
x2 + 2x – 9 = 0
Now plug a = 1, b = 2, and c = –9 into the quadratic formula:

x= =
2
− b ± b 2 − 4ac −2 ± 2 − 4 1 −9 ( )( )
= −1 ± 10
2a 21 ()

But the quadratic formula can also be handy when the polynomial is
factorable, but factoring seems like too much trouble.
Q: Solve: 12x2 + 20 = 31x
A: First rearrange the equation into “polynomial = 0” form:
12x2 – 31x + 20 = 0
Now plug a = 12, b = –31, and c = 20 into the quadratic formula:

(−31) − 4(12)(20 ) = 31 ± 1 = 4
2
− b ± b 2 − 4ac 31 ± 5
x= = OR
2a 2(12) 24 3 4

© The MBA Center 77


Total PrepKit for the GMAT® Step 2.1 Lesson

PRACTICE QUESTIONS 51-60 (EQUATIONS)

51. Solve for x:


1
2
( ) 1
x + 5 = x +1
3
(A) –1
(B) –3
(C) –5
(D) –7
(E) –9

(
52. Solve for x: 5 x + 2y = −3 4y − x − 2 )
(A) –7y + 3
(B) –3y – 3
(C) –3y + 7
(D) 3y – 7
(E) 7y + 3

3x + 2y = −1
53. Solve for x: 
4 x + 3y = 1
(A) –7
(B) –5
(C) 0
(D) 5
(E) 7

7 x + 4y = 19
54. Solve for x + y: 
4 x + 7 y = 3

(A) – 5
3
(B) 2

(C) 11
3
(D) 16
3
(E) 16

55. Solve for x: x2 – 2x = 24

(A) –4 or 8
(B) –4 or 6
(C) –3 or 8
(D) 3 or –8
(E) 4 or 6

78 © The MBA Center


Math Review

x 2 + x = 2
56. Solve for x:  2
x − 4 = 0
(A) –2 only
(B) 1 only
(C) 2 only
(D) –2 or 2
(E) There is no real solution.

57. If 2 – 1 = 1 + 2 , then x =
x x

(A) 1
3
(B) 1
2
(C) 1
(D) 2
(E) 3

58. If x ≠ 3, then 3x(x – 3) – x + 3 =


x–3
(A) x+1
(B) x–3
(C) 3x + 1
(D) 3x – 1
(E) 3x + 3

59. If (x + 3)2 = 144, which of the following could be the value of x?

(A) 15
(B) 12
(C) –9
(D) –12
(E) –15

60. If the equation x2 + kx + 1 = 0 has exactly 1 real solution, which of the


following could be the value of k?

(A) –4
(B) –3
(C) –2
(D) –1
(E) 0

© The MBA Center 79


Total PrepKit for the GMAT® Step 2.1 Lesson

7. QUADRILATERALS
Rectangle — a quadrilateral with four right angles.
Opposite sides of a rectangle are parallel and equal in length.
The diagonals of a rectangle are equal, and they bisect each other. [Bisect
means to divide into two equal pieces.]
The two dimensions of a rectangle are generally called the length and
width. [There is no consensus on which is the length and which is the
width. For some people, the length is the longer dimension and the width
is the shorter dimension. For other people, the length is the vertical
dimension and the width is the horizontal dimension. On the GMAT, it
doesn’t matter.]
The perimeter of a rectangle is length plus width plus length plus width,
or, in other words, twice the length plus twice the width. If the length is l
K L and the width is w, then the formula is:
Perimeter of a rectangle = 2l + 2w
4
Q: What is the perimeter of a rectangle JKLM in Figure 1?
A: Plug 4 and 9 in for l and w in the formula:
J 9 M Perimeter = 2l + 2w = 2(4) + 2(9) = 26
Figure 1
The area of a rectangle is length times width:
Area of a rectangle = l w

Q R Q: What is the area of rectangle PQRS in Figure 2?


A: Plug 3 and 8 in for l and w in the formula:
3
Area = lw = 3 s 8 = 24
P 8 S
Square — rectangle with four equal sides.
Figure 2 As with any rectangle, the diagonals of a square are equal and bisect each
other. The diagonals of a square are also perpendicular.
Because a square is a rectangle, the perimeter of a square is twice the
length plus twice the width. But because the length and width are equal,
you can write the formula more simply. If the length of one side is s, then:
Perimeter of a square = 4s

Because a square is a rectangle, the area of a square is length times width.


But, again, because the length and width are equal, you can write the
formula more simply:
Area of a square = s2

Q: If a square has perimeter 20, what is the area?


A: The perimeter of a square is 4 times the length of one side, and so
the length of one side is one-fourth the perimeter. One-fourth of 20
is 5. If the length of a side is 5, then the area is 5 squared, or 25.

Parallelogram — a quadrilateral with two pairs of parallel sides.


Opposite sides of a parallelogram are equal. Opposite angles are equal, and
consecutive angles are supplementary. Diagonals bisect each other.

80 © The MBA Center


Math Review

The perimeter of a parallelogram is equal to the sum of the lengths of


the four sides. If you know the lengths of any two consecutive sides, then
you know all four lengths and you can calculate the perimeter.
You cannot calculate the area of a parallelogram if all you know is the
length of the sides. To find the area you need what are called the base and
the height. Any side can serve as the base. The height then is defined as the h
perpendicular distance between the base and the opposite side. (Figure 3)
When you know the base b and the height h, then you can use the formula: b
Area of a parallelogram = bh Figure 3

Q: What is the area of the parallelogram in Figure 4?


A: The base is 6 and the height is 4: 5
4
Area = bh = 6 s4 = 24

Be aware that all squares are rectangles and that all rectangles are 6
parallelograms, but that not all parallelograms are rectangles and not all Figure 4
rectangles are squares. When a GMAT math question says “rectangle,” the
figure referred to might be a square.
Here is a summary of the characteristics and formulas for squares,
rectangles, and parallelograms:

SQUARE RECTANGLE PARALLELOGRAM


sides: All equal. Opposite sides are Opposite sides are
equal. equal.
angles: All right. All right. Opposite angles
are equal.
diagonals: Equal and Equal bisectors. Bisectors.
perpendicular
bisectors.
perimeter: P = 4s P = 2l + 2w Sum of the sides.

area: A = s2 A = lw A = bh

Trapezoid — a quadrilateral with one pair of parallel sides and one pair of
nonparallel sides.
To find the area of a trapezoid, call the two parallel sides the bases b1 and
b2 and the perpendicular distance between them the height h. Then:

Area of a trapezoid = b1 b 2 h
2 8
Think of it as the height times the average of the bases.
5
Q: What is the area of the trapezoid in Figure 5?
A: The average of the bases is 9, and the height is 5, so the area is
10
9 s 5 = 45.
Figure 5

© The MBA Center 81


Total PrepKit for the GMAT® Step 2.1 Lesson

PRACTICE QUESTIONS 61-70 (QUADRILATERALS)


61. Figure 6 is which of the following?

I. quadrilateral
II. rectangle
III. parallelogram

(A) I only
Figure 6 (B) II only
(C) I and II only
(D) I and III only
(E) I, II, and III

62. If the area of the rectangle in Figure 7 is 21, what is the perimeter?
3
(A) 10
(B) 10.5
(C) 14.5
Figure 7 (D) 20
(E) 21

63. If rectangles A and B in Figure 8 have the same perimeter, what is the area
8 of rectangle B?

5 A B 6 (A) 24
(B) 26
(C) 30
Figure 8
(D) 36
(E) 42

Q R 64. The quadrilateral in Figure 9 is a parallelogram. If the degree measure of


angle P is 2x – 25, and the degree measure of angle Q is 3x + 15, what is
the value of x?

(A) 38
P (B) 51
S
(C) 64
Figure 9 (D) 76
(E) 131

5
65. If the perimeter of the trapezoid in Figure 10 is 22, what is the area?

5 (A) 20
(B) 22
(C) 26
8 (D) 32
(E) 40
Figure 10

82 © The MBA Center


Math Review

Questions 66-70 refer to quadrilaterals A, B, C, D, and E in Figure 11.

66. How many of the quadrilaterals in Figure 11 are rectangles? 5

(A) 1
5 A 5
(B) 2
(C) 3
(D) 4
(E) 5 5

5
67. How many of the quadrilaterals in Figure 11 are parallelograms?
5 B 5
(A) 1
(B) 2
88°
(C) 3
(D) 4 5
(E) 5
6

68. Which of the quadrilaterals in Figure 11 has the greatest area? 4 C 4

(A) A
(B) B 6
(C) C
(D) D 6
(E) E
4 D 4

69. Which of the quadrilaterals in Figure 11 has the least area? 88°
6
(A) A
(B) B 8
(C) C
(D) D 3 E 3
(E) E
8

70. Which of the quadrilaterals in Figure 11 has the greatest perimeter?


Figure 11
(A) A
(B) B
(C) C
(D) D
(E) E

© The MBA Center 83


Total PrepKit for the GMAT® Step 2.1 Lesson

8. TRIANGLES
Triangle — polygon with three sides.
In one sense the triangle is the simplest of polygons: it has the fewest sides.
But in many ways the triangle is the most complex. There are many facts
and formulas for triangles, and there are several special triangles with their
own special facts and formulas.
One simple trait common to all triangles is that the three interior angles
add up to 180 degrees. So whenever you know the measures of two angles,
you can easily find the measure of the third.
86°
Q: What is the value of x in Figure 12?
A: The two given angles add up to 86 + 34 = 120 degrees. That leaves
x° 180 - 120 = 60 degrees for the other angle, and so x = 60.
34°

If you extend one side of a triangle, you create what is called an exterior
Figure 12 angle. Because an exterior angle is supplementary to the adjacent interior
angle, and because the measure of that interior angle is equal to 180° minus
the other two interior angles, it follows that an exterior angle is equal to the
sum of those other two interior angles. (Those angles — the interior angles
78° other than the adjacent one — are called the remote interior angles.)

32° Q: What is the value of y in Figure 13?



A: The angle marked y° is an exterior angle. The remote interior angles
are marked 32° and 78°, and so y = 32 + 78 = 110.
Figure 13
If you extend all three sides of a triangle, the three resulting exterior angles
will add up to 360°.

Q: What is the value of s + t + u in Figure 14?
A: The angles marked s°, t°, and u° are exterior angles at the three
u° vertices, and so s + t + u = 360.

Area Of A Triangle
To find the area of a triangle you need the base and the height. Any side can
Figure 14
serve as the base. The height then is defined as the perpendicular distance
between the base and the opposite vertex. When you know the base b and
the height h, then you can use the formula:

Area of a triangle = 1 bh
2
B
Q: In Figure 15, AC = 10 and BD = 3, what is the area of
triangle ABC?
A: AC and BD are perpendicular, so you can use them as the base and
height:

A C Area = 1 bh = 1 (10)(3) = 15
D 2 2

Figure 15 If all you know is the length of two sides of a triangle, you cannot find the
area — unless those two sides happen to be perpendicular.

F Q: In Figure 16, what is the area of triangle EFG?


A: EF and EG are perpendicular, so you can use them as the base and
2
height:
G
E 12
Area = 1 bh = 1 (2)(12) = 12
Figure 16 2 2

84 © The MBA Center


Math Review

Any side of a triangle can potentially be used as the base. It does not have
to be the side on the bottom. Look for a side for which you are given a
height. H

Q: In Figure 17, what is the area of triangle HJK?


16
A: You should use HJ as the base, because that's the only side for
which you are given a height. HJ and IK are perpendicular, so you 20
can use them as the base and height. Note that HJ is equal to the I
sum of HI and IJ: 12
5
Area = 1 bh = 1 (16 + 5)(12) = 126
2 2 J 13 K

Figure 17
Side-Angle Relationships
The longest side of a triangle will always be found opposite the biggest
angle. Likewise, the shortest side will be found opposite the smallest angle.
Thus, if you know the relative measures of the three angles, then you also
know the relative lengths of the three sides. Or, if you know the relative
lengths of the sides, then you also know the relative measures of the angles.

Q: In triangle FGH, the measure of angle F is 53°, and the


measure of angle G is 62°. Arrange the sides in order
from longest to shortest.
A: When you know the measures of two angles of a triangle, you can
determine the measure of the third. The two given angles add up to
53 + 62 = 115 degrees. That leaves 180 – 115 = 65 degrees for
measure of angle H. The biggest angle is H, and so the side opposite
that, FG, is the longest. The smallest angle is F, and so the side
opposite that, GH is the shortest. The order of the sides from
longest to shortest, then, is FG, FH, GH.

If two sides of a triangle have the same length, it follows that the angles
opposite the equal sides are equal. Likewise, if two angles have the same
measure, it follows that the sides opposite the equal angles are equal.
Furthermore, if all three sides of a triangle are equal, it follows that the
angles are all equal, and likewise, if all three angles are equal, then all three
sides are equal.

Q: In triangle LMN, the length of LM is 10, the length of


MN is 10, and the length of LN is 12. Arrange the
angles in order from biggest to smallest.
A: LN is the longest side, and so the angle opposite that, M, is the
biggest. The other two sides are equal in length, so the angles
opposite them, L and N, are equal.

Triangle Inequality Theorem — The length of one side of a triangle is


greater than the positive difference between the other two side lengths and
less than the sum. Thus, if you know the lengths of two sides of a triangle,
then you know the lower and upper limits for the length of the third side.

Q: In triangle ABC, the length of AB is 3 and the length of


BC is 7. What is the range of possible values for the
length of AC ?
A: AC must be greater than the positive difference between the other
two sides, which is 7 – 3 = 4. And AC must be less than the sum
of the other two sides, which is 7 + 3 = 10. Thus 4 < AC < 10.

© The MBA Center 85


Total PrepKit for the GMAT® Step 2.1 Lesson

Special Triangles
Isosceles Triangle — a triangle with two equal sides.
The equal sides of an isosceles triangle are called the legs. Short hatch
marks are often used to indicate equal sides.
As we have seen, if two sides of a triangle are the same length, then the
angles opposite them are equal. The equal angles of an isosceles triangle are
called the base angles (even if they're not at the bottom). The other angle
is called the vertex angle.
T
When you know that a particular pair of sides are equal, you can use one
angle measure to find the other two.

Q: In triangle STU in Figure 18, ST = TU. If the measure of


angle S is 70°, what are the measures of angles T and
U?
A: Angles S and U are the base angles and so they are equal. Thus
angle U measures 70°. They add up to 140°, which leaves 180 –
70o 140 = 40 degrees for the measure of angle T.
S U
Be careful. If all you know is that you have an isosceles triangle, you cannot
Figure 18 necessarily use one angle to find the measures of the others. You need to
know whether the given angle is a base angle or the vertex angle.

8 Q: If the measure of one angle of an isosceles triangle is


70°, what are the measures of the other two angles?
70o A: You cannot answer this question because, without a diagram or
some more information, you do not know whether the 70° angle is
a base angle or the vertex angle. If it is a base angle, then the
triangle would look like triangle STU above and the other two
55o 55o angles would be 70° and 40°. But it is also possible that the 70°
angle is the vertex angle, in which case the other two angles would
Figure 19 each be 55° and the triangle would look like Figure 19.

Equilateral triangle — a triangle with three equal sides.


If all the sides are equal, it follows that all the angles are equal. Furthermore,
60o
if the three angles are equal and add up to 180°, then each angle of an
equilateral triangle measures one-third of 180°, or 60°. (Figure 20)

Right triangle — a triangle with a right angle (that is, a 90° angle).
60o 60o
If one angle measures 90°, it follows that the other two angles are acute and
Figure 20 add up to 90°. In a right triangle, the 90° angle is always the biggest angle,
and so the side opposite the 90° will always be the longest side. The longest
side is called the hypotenuse. The other two sides, the sides that are
perpendicular, are called the legs.

Pythagorean Theorem
The lengths of the sides of a right triangle are related. This relationship is
summarized in the Pythagorean theorem, which says that the sum of the
squares of the legs is equal to the square of the hypotenuse. Algebraically
speaking, if the lengths of the legs are a and b and the length of the
hypotenuse is c, then:
a2 + b2 = c2

86 © The MBA Center


Math Review

In Figure 21, for example, the legs have lengths 8 and 15, and the
hypotenuse has length 17. The squares of the legs are 82 = 64 and 152 = 225.
The sum of the squares of the legs is 64 + 225 = 289, which is indeed equal 17
8
to the square of the hypotenuse: 172 = 289.
You can use the Pythagorean theorem to find the length of the hypotenuse
when you know the lengths of the legs, or to find the length of a leg when 15
you are given the lengths of the other leg and the hypotenuse.
Figure 21
Q: The legs of the right triangle in Figure 22 have lengths
7 and 24. What is the length of the hypotenuse?
A: The squares of the legs are 72 = 49 and 242 = 576. The sum of the 7
squares of the legs is 49 + 576 = 625. That’s the square of the
hypotenuse, so to find the hypotenuse, take the square root: 24
hypotenuse = 625 = 25 Figure 22

Q: One of the legs of the right triangle in Figure 23 has


length 10 and the hypotenuse has length 15. What is
the length of the other leg? 15
A: The square of the given leg is 102 = 100 and the square of the 10
hypotenuse is 152 = 225. The difference is 225 – 100 = 125. That’s
the square of the other leg, so to find the leg, take the square root:
leg = 125
Figure 23
The square root of 125 is not an integer, though it can be
simplified:
125  25 5  5 5
To express the length precisely, just leave it in this form. (It is
approximately 11.18.)
Be careful. You can use the Pythagorean theorem to find the third side of a
right triangle only if you know whether the given sides are both legs or a
leg and hypotenuse.
Q: The lengths of two sides of a right triangle are 5 and 4.
What is the length of the third side?
A: You cannot answer this question unless you know whether the
given side lengths of 5 and 4 are the two legs or a leg and
hypotenuse. If they are the two legs, then:

hypotenuse = 5 2 4 2  25 16  41

If they are a leg and hypotenuse, then:

leg = 5 2 4 2  25 16  9  3

Pythagorean triple — A set of three integers that satisfy the Pythagorean


theorem. The simplest and most frequently encountered Pythagorean
threesome is 3-4-5 . Whenever you see a right triangle with legs 3 and 4, you
should realize immediately, without actually using the Pythagorean
theorem, that the hypotenuse is 5. And whenever you see a right triangle
with hypotenuse 5 and leg 3 or 4, you should realize immediately what the
other leg is.
3
Q: The legs of the right triangle in Figure 24 have lengths
3 and 4. What is the length of the hypotenuse?
A: There's no need to use the Pythagorean theorem. You should realize 4
right away that the hypotenuse is 5.
Figure 24

© The MBA Center 87


Total PrepKit for the GMAT® Step 2.1 Lesson

Q: One leg of the right triangle in Figure 25 has length 3


3 and the hypotenuse has length 5. What is the length of
the other leg?
A: There's no need to use the Pythagorean theorem. You should realize
5 right away that the other leg is 4.
Figure 25 The set {3, 4, 5} is not the only set of three integers that fits the Pythagorean
theorem. So too will any set of three integers in the ratio 3:4:5, such as
{6, 8, 10}, {9, 12, 15}, {30, 40, 50}, and {54, 72, 90}.
Q: The legs of the right triangle in Figure 26 have lengths
6 6 and 8. What is the length of the hypotenuse?
A: There's no need to use the Pythagorean theorem. You should realize
right away that the hypotenuse is 10.
8
Q: One leg of the right triangle in Figure 27 has length 24
Figure 26 and the hypotenuse has length 30. What is the length
of the other leg?
A: The ratio of the given leg to the hypotenuse is 4:5, so this is a
24
multiple of the 3-4-5 triangle. Each side is multiplied by 6, and the
side you’re looking for is 3 × 6 = 18.
Be careful. A right triangle with two sides in a 3-to-4 or 3-to-5 or 4-to-5 ratio
30 is not necessarily a multiple of the 3-4-5 triangle.
Figure 27 Q: The legs of a right triangle have lengths 12 and 15.
What is the length of the hypotenuse?
A: The legs are in ratio of 4:5. This is not a multiple of a 3-4-5
triangle, because in such a triangle it must be a leg and the
hypotenuse that are in a ratio of 4:5. To find the length of the
hypotenuse in this case, you must use the Pythagorean theorem:

hypotenuse = 12 2 + 15 2 = 144 + 225 = 369 = 3 41

Q: Two sides of a right triangle have lengths 30 and 40.


What is the length of the third side?
A: There is not enough information to answer this question. You are
not told whether the two sides are the two legs or a leg and
hypotenuse. It makes a difference. If they are the two legs, then this
is a multiple of the 3-4-5 triangle and the third side is 50. But if
they are a leg and hypotenuse, then you will have to use the
Pythagorean theorem:
leg = 40 2 − 30 2 = 1600 + 900 = 700 = 10 7

Besides the 3-4-5 and its multiples, there are many other Pythagorean
triples. A very popular one on the GMAT is the 5-12-13.
Q: The legs of the right triangle in Figure 28 have lengths
5
5 and 12. What is the length of the hypotenuse?
A: There's no need to use the Pythagorean theorem. You should realize
12 right away that the hypotenuse is 13.
Figure 28 Multiples of the 5-12-13 are Pythagorean triples as well.
Q: One leg of the right triangle in Figure 29 has length 24
24 and the hypotenuse has length 26. What is the length
of the other leg?
A: The ratio of the given leg to the hypotenuse is 12:13, so this is a
26 multiple of the 5-12-13 triangle. The given sides are twice 12 and
13, respectively, and so the missing side is twice 5, or 10.
Figure 29
88 © The MBA Center
Math Review

45-45-90 Triangle
All isosceles right triangles have the same side proportions. When the legs

are both 1, the hypotenuse is 2 . When the legs are both 2, the hypotenuse

is 2 2 . When the legs are both 3, the hypotenuse is 3 2 . In other words,

whatever the length of a leg, just multiply that by 2 to get the hypotenuse.
5
Q: In Figure 30, what is the length of the hypotenuse?
A: Because the legs are both 5, this is a right isosceles triangle. Just
multiply the leg length by 2 to get the hypotenuse: 5 2 .
5
To turn a leg length into the hypotenuse, you multiply by 2 . So to turn Figure 30
the hypotenuse into a leg, you do the opposite — divide by 2.

Q: In Figure 31, what are the lengths of the legs?


A: The hatch marks indicate that the legs are equal, and so this is a
right isosceles triangle. To turn the given hypotenuse length of 6 6
into a leg length, divide by 2 :
6 6 2 6 2
leg =   3 2
2 2 2 2
Figure 31
30-60-90 Triangle
All 30-60-90 triangles have the same side proportions. When the shorter leg

(opposite the 30° angle) is 1, then the longer leg (opposite the 60° angle) is

3 and the hypotenuse is 2. So when the shorter leg is 2, then the longer
leg is 2 3 and the hypotenuse is 4. And when the shorter leg is 10, then the

longer leg is 10 3 and the hypotenuse is 20.

Q: In Figure 32, what are the lengths of the longer leg and
hypotenuse? 60o
5
A: The shorter leg is 5, so the longer leg is 3 times that, or 5 3. The
30o
hypotenuse is twice the shorter leg, or 10.
If what you are given is the hypotenuse, then divide by 2 to get the shorter Figure 32

leg, and then multiply the shorter leg by 3 to get the longer leg. And if

what you are given is the longer leg, then divide by 3 to get the shorter
leg, and then multiply the shorter leg by 2 to get the hypotenuse.

Q: In Figure 33, what are the lengths of the shorter leg


and the hypotenuse? 60o

A: The longer leg is 9. Divide that by 3 to get the shorter leg:


9 9 3 9 3 30o
shorter leg = 3 3 9
3 3 3 3
Figure 33
Multiply the short leg by 2 to get the hypotenuse:
hypotenuse 2 s 3 3  6 3

© The MBA Center 89


Total PrepKit for the GMAT® Step 2.1 Lesson

PRACTICE QUESTIONS 71-80 (TRIANGLES)

71. If the measure of one angle of an isosceles triangle is 96°, what is the
degree measure of the smallest angle?

(A) 42°
(B) 48°
(C) 64°
(D) 84°
(E) Cannot be determined from the information given.

72. If the measure of one angle of an isosceles triangle is 40°, what is the
degree measure of the biggest angle?

(A) 70°
(B) 80°
(C) 100°
(D) 140°
(E) Cannot be determined from the information given.

73. If the lengths of the two legs of a right triangle are 10 and 24, what is
the length of the hypotenuse?

(A) 25
(B) 26
(C) 30
(D) 34
(E) Cannot be determined from the information given.

74. If the lengths of two sides of a right triangle are 9 and 12, what is the
length of the third side?

(A) 12
(B) 15
(C) 18
(D) 21
(E) Cannot be determined from the information given.

75. If the lengths of two sides of an isosceles triangle are 3 and 7, which of
the following could be the perimeter of the triangle?

I. 13
II. 15
III. 17

(A) I only
(B) III only
(C) I and III only
(D) II and III only
(E) I, II, and III

90 © The MBA Center


Math Review

76. What is the value of x in Figure 34?


2x°
(A) 26
(B) 39
x° (x + 78)°
(C) 51
(D) 78
(E) 102 Figure 34

77. What is the area of the triangle in Figure 35?

(A) 25 10
(B) 50
(C) 75
(D) 90 45°
(E) 100
Figure 35

78. If the length of one leg of a right triangle is 6, and if the perimeter is 18,
what is the area of the triangle?

(A) 12.5
(B) 13.5
(C) 21
(D) 25
(E) 27

79. In Figure 36, AD = 20 and DC = 36. If the area of triangle ABC is 420, B
what is the perimeter of triangle ABC?
(A) 120
(B) 132
(C) 144 A C
(D) 156 20 D 36
(E) 168 Figure 36

80. What is the area of the triangle in Figure 37?

(A) 9 + 9 2 105°
6
(B) 9 + 9 3
45°
(C) 9 2 + 9 3
Figure 37
(D) 18 + 18 2

(E) 18 + 18 3

© The MBA Center 91


Total PrepKit for the GMAT® Step 2.1 Lesson

9. CIRCLES
Circle — set of all the points in a plane that are the same given distance
away from a particular point.
D
A circle will often be designated by the name of the point in the middle,
C which is called the center. The distance from the center to any point on the
circle is called the radius (plural: radii). All radii of a circle are the same
E length.
F A line segment with endpoints on a circle is called a chord of the circle. In
the circle in Figure 38, CD and EF are chords.
Figure 38 A chord that passes through the center of a circle is called a diameter. The
same word is also used to mean the distance across the circle. The diameter
of a circle is equal to twice the radius.
R
Q: In Figure 39, Q, R, and S are points on circle P. If
11 PR = 11 what is the length of diameter QS?
A: The radius is 11, so the diameter is twice that, or 22.
Q S
P
An angle formed by two radii, with the center as the vertex, is called a
central angle. The central angles of a circle add up to 360°.

Q: In Figure 40, E, F, and G are points on circle D. If the


Figure 39
measure of angle EDF is 121° and the measure of angle
FDG is 119°, what is the degree measure of angle EDG?
A: The three central angles add up to 360 degrees. The two given
angles add up to 121 + 119 = 240 degrees, and so angle EDG
E F measures 360 – 240 = 120 degrees.
121o
D 119o Circumference — the distance around a circle.
All circles are similar and so all circles have the same ratio of circumference
to diameter. This ratio is an irrational number approximately equal to 3.14.
This number is called pi, a Greek letter, written P. If you know the diameter
G of a circle, you can get the circumference C by multiplying by P:
Figure 40
C = Pd
Since the diameter is also twice the radius, the formula can also be written
like this:
C = 2Pr

Circumference is a measure of length. It is expressed in linear units: inches,


meters, miles, etc.
It is conventional to write numbers in front of P and letters after it. Thus,
when the diameter is expressed as d, it goes after (Pd), but when you put a
number in for d, it goes in front (12P).

Q: If the circumference of circle P is 12, what is the


radius?
A: Plug C = 12 into the formula and solve for r:
12  2 Pr
6
r
P

92 © The MBA Center


Math Review

Length Of An Arc
The part of the circumference of a circle extending from one given point to A
another is called arc. For any two points on a circle, there are two arcs.
B
Unless the two points are directly opposite each other (that is, endpoints of
a diameter), one arc will be shorter than the other. The shorter arc, the one
that goes less than halfway around the circle, is called the minor arc. The
longer arc, the one that goes more than halfway around the circle, is called
the major arc. In Figure 41, the part of the circumference of the circle
below that goes from A to C through B can be called arc ABC or minor arc C
AC.
An arc has two different measures: the degree measure and the length. The Figure 41
degree measure is a description of the arc’s curvature and is simply the
measure of the central angle formed by its endpoints and the center of the
circle.
D
Q: In circle O in Figure 42 what is the degree measure of
minor arc DE? 10
A: Central angle DOE measures 54°, and so the degree measure of 54°
minor arc DE is also 54°. E
O
The length of an arc is a fraction of the circle’s circumference. To find the
length of an arc, you need to know not just the measure of the central
angle, but also something about the size of the circle, such as radius or
diameter or circumference. To calculate the length of an arc, first divide the Figure 42
degree measure of the central angle by 360. That will tell you what fraction
of the circumference is on the arc. Then find the circumference and
multiply it by the fraction.
Q: In circle O in Figure 42, what is the length of minor arc
DE?
A: First divide the degree measure of the central angle by 360:
54 3
=
360 20
Then find the circumference. Plug r = 10 into the formula:
C = 2π(10) = 20π
Now multiply the fraction by the circumference:
3
length of arc DE = ( 20 π ) = 3 π
20
The process of finding the length of arc when you know the radius r and the
degree measure of the central angle a can be summarized in one formula:
a
length of arc = ⋅ 2 πr
360

Area Of A Circle
The formula for the area A of a circle of radius r is:
A = πr2 5
Area is expressed in square units: square inches, square feet, square meters,
etc. O
Q: What is the area of circle O in Figure 43?
A: Plug r = 5 into the formula:
A = π(52) = 25π Figure 43

© The MBA Center 93


Total PrepKit for the GMAT® Step 2.1 Lesson

The formulas for circumference and area look somewhat similar. To avoid
confusing them, remember that area is a measure of square units and
therefore has the r squared.
You need the radius to find the area. To find the area when you're given the
diameter, first divide the diameter by 2 and then proceed with the above
formula.
Q: What is the area of a circle of diameter 6?
A: First divide the diameter by 2 to get r = 3, then plug that into the
formula:
A = π(32) = 9π

To find the area when you're given the circumference, use the circumference
formula to find the radius.
Q: What is the area of a circle of circumference 20π?
A: First use the circumference formula to find r:
20 π = 2 πr
r = 10
Then plug r = 10 into the area formula:
A = π(102) = 100π

Area Of A Sector
A closed figure formed by two radii and an arc is called a sector. You can
think of a sector as a fraction of the interior of a circle. You can find the area
of a sector if you know the measure of the interior angle and something
about the size of the circle, such as the radius. Use the angle measure to
determine what fraction of the circle is contained within the sector.
P Q: What is the area of the shaded portion of circle O in
Figure 44?
A: The radius is 4, and so the area of the whole circle is 16π. The
45° central angle measures 45°. Put that over 360° to see what fraction
Q
O 4 of the circle is contained within the sector:
45 1
=
360 8
Figure 44
Thus the area of the sector is 1 of the area of the circle:
8
1
area of sector = ( 16 π ) = 2 π
8

If the degree measure of the central angle is a and the radius is r, then the
formula is:
a
area of sector = ⋅ πr 2
360

Q: What is the area of the sector formed by a 20° central


angle in a circle of radius 6?
A: Plug a = 20 and r = 6 into the formula:

area of sector =
20
360
( )
π 6 2 = 2π

94 © The MBA Center


Math Review

PRACTICE QUESTIONS 81-90 (CIRCLES)

81. What is the circumference of a circle of area 16π?

(A) 4π
(B) 8π
(C) 16π
(D) 32π
(E) 64π

82. What is the area of a circle of circumference 16π?

(A) 4π
(B) 8π
(C) 16π
(D) 32π
(E) 64π

A
83. What is the length of minor arc AB in Figure 45?
5
(A) π
108°
(B) 2π O
(C) 3π
B
(D) 4π
(E) 5π

Figure 45

84. What is the area of the shaded region in Figure 46?

(A) 52π 130° 12


(B) 60π
(C) 65π O
(D) 72π
(E) 144π

Figure 46

85. In Figure 47, the angles marked x° and y° are central angles. If the ratio
of x to y is 4 to 5, what is the value of x?

(A) 130 x°
(B) 140 y°
(C) 150
(D) 160
(E) 170
Figure 47

© The MBA Center 95


Total PrepKit for the GMAT® Step 2.1 Lesson

A B 86. In Figure 48, if ABCD is a square of area 10, what is the area of the circle?

(A) 5π
(B) 10π
(C) 25π
(D) 50π
D C (E) 100π

Figure 48

87. What is the area of the shaded region in the Figure 49?
6
70° (A) 28π
(B) 29π
O (C) 30π
(D) 31π
(E) 32π

Figure 49

A 88. In circle O in Figure 50, if the length of arc ABC is 2π, what is the area of
B the shaded region?
5
(A) 4π
O C (B) 5π
(C) 8π
(D) 10π
(E) 12π
Figure 50

89. In Figure 51, O is the center of both circles. If OD = DE = 3, what is the


area of the shaded region?

E (A) 9π
O D (B) 12π
(C) 18π
(D) 24π
(E) 27π
Figure 51

A
90. In Figure 52, if the length of minor arc AB is 10, and the length of major
arc AB is 35, what is the value of x?

x° (A) 75
O B (B) 80
(C) 85
(D) 90
(E) 95
Figure 52

96 © The MBA Center


Math Review

10. PHINEAS’ FAVORITE WORD PROBLEMS


Phineas is big on word problems. Certain types of word problems come up
on the GMAT over and over again. Here are some of Phineas’ favorites.

Rate Problem With A Change In Units


A rate is a ratio of quantities with different units. Solving a rate problem
generally involves setting up a proportion.
Phineas likes to complicate rate problems by switching one of the units
between the given information and the question. The easiest way to do a
problem like this is first to find the solution in terms of the original units,
and then to convert.

Q: A machine makes copies at a rate of 2.5 copies per


second. How many minutes will it take the machine to
make 3,000 copies?
A: First set up a proportion to find the answer in seconds. 2.5 copies
in 1 second equals 3,000 copies in how many seconds?
2.5 copies 3000 copies
=
1 second x seconds
2.5 x = 3000
x = 1200
To convert seconds to minutes, multiply by (1 min/60 sec).
1 minute
( 1200 seconds ) × = 20 minutes
60 seconds

Phineas can make matters doubly complicated by switching both units:

Q: At a filling station in France, Adrian paid 171 francs


for 25 liters of gasoline. At the conversion rates of 7.2
francs per 1 US dollar, and 3.8 liters per 1 US gallon,
what was the price of the gasoline in dollars per
gallon?
A: You could start by dividing 171 francs by 25 liters to get the rate
in francs per liter, and then convert the francs to dollars and the
liters to gallons. Or you could convert first and then divide. Both
methods will work. But perhaps the most direct method is to
multiply francs over liters by dollars over francs by liters over
gallons. If you set it up properly, the units you’re not interested in
(francs and liters) will cancel and your answer will be in dollars per
gallon:
171 francs 1 dollar 3.8 liters
× × = 3.61 dollars per gallon
25 liters 7.2 francs 1 gallon

Notice that Phineas kindly gave you the francs-per-dollar and liters-per-
gallon conversion rates. Fortunately Phineas does not expect you to know a
lot of conversion rates. You do not need to remember anything like how
many ounces in a pound, inches in a foot, yards in a mile, centimeters in
an inch, liters in a gallon, or grams in a pound. You do not even need to
know how many centimeters in a meter or grams in a kilogram.
If you ever need one of these conversion rates to solve a problem, Phineas
will give it to you. That’s because none of these units are universal. Phineas
does not expect Americans to know all about meters, liters, and grams. Nor
does he expect non-Americans to know all about inches, feet, ounces,
pounds, pints, and gallons.

© The MBA Center 97


Total PrepKit for the GMAT® Step 2.1 Lesson

So don’t waste any of your precious GMAT preparation time memorizing


conversion rates. The only units for which Phineas expects you know the
conversion rates are units of time: seconds, minutes, hours, days, weeks,
months, and years.

Combined Rate
Another way Phineas likes to complicate rate problems is by asking you to
combine them.

Q: Dale can stuff 100 envelopes in 12 minutes, and Chris


can stuff 100 envelopes in 8 minutes. Working
together, how many minutes will it take them to stuff
1,000 envelopes?
A: To combine the rates, add them:
100 100 200 300 500
+ = + =
12 8 24 24 24
That fraction can be simplified, but it’s easy to use in the next step
as is. Now that you have the combined rate, you can set up a
proportion and solve:
500 1000
=
24 x
x = 48
Working together they can stuff 1,000 envelopes in 48 minutes.

The infamous “two-trains-approaching” type word problem is really just a


matter of combining rates.

Q: A train departs Altoona at 6:00 a.m. and proceeds at


a steady rate of 75 miles per hour towards Bakersville.
Another train departs Bakersville at the same time
and proceeds along the same track towards Altoona
at a steady rate of 125 miles per hour. If the track is
650 miles long, at what time will the trains meet?
A: When the trains meet, each mile of track will have been traversed
by one or the other. Between them they will have traveled the entire
distance, at a rate equal to the sum of the individual speeds:
650 miles 650 miles
= = 3.25 hours
( 75 + 125 ) miles per hour 200 miles per hour
That means the trains will meet 3 and a quarter hours after 6:00
a.m., or at 9:15 a.m.

Working Together
When the problem concerns the amount of time it takes two individuals
alone to accomplish some task, and the amount of time it takes them
working together to accomplish the task, use this formula:
t t
+ =1
a b
In this formula, a represents the time in takes one person alone, b represents
the time it takes the other person alone, and t represents the amount of
time it takes them working together. If you are given values for two of the
quantities a, b, and t, you can solve for the third.

98 © The MBA Center


Math Review

Q: Working alone, Maria can paint a room in 8 hours.


Maria’s assistant can paint the same room in 24
hours. How much time would it take Maria and
her assistant, working together, to paint the room?
A: Here a = 8 and b = 24. Plug them into the formula and solve for t:
t t
1
8 24
3t t
1
24 24
4t
1
24
4t  24
t 6

When three individuals are working together, the formula becomes:


t t t
1
a b c

Q: Machine A can produce 1,000 widgets in 5 hours.


Machine B can produce 1,000 widgets in 10 hours. If
machines A, B, and C working independently and
simultaneously can produce 1,000 widgets in 3 hours,
how many hours would it take machine C alone to
produce 1,000 widgets?
A: Here a = 5, b = 10, and t = 3. Plug them into the formula and solve
for c:
3 3 3
1
5 10 c
9 3
1
10 c
3 1

c 10
c  30

Here’s a variation on this problem type in which you are not given any
particular values:

Q: John, working alone, can do a certain job in half the


time it takes Peter working alone. John and Peter,
working together, can do the job 2 hours faster than
John working alone. How many hours would it take
John, working alone, to do the job?
A: Use the same formula. You’re looking for a, the time it takes John
alone. Express the other times in terms of a. The time it takes Peter
alone is 2a, and the time it take them together is a – 2:
a 2 a 2
1
a 2a
2a 4 a 2
1
2a 2a
3a 6
1
2a
3a 6  2a
a 6

© The MBA Center 99


Total PrepKit for the GMAT® Step 2.1 Lesson

Average Rate
The general formula is:
Total A
Average A per B 
Total B
In the case of average speed, the formula is:
Total distance
Average speed 
Total time

Q: Mary drove 660 miles to visit her grandmother. It took


her 5 hours to drive the first 360 miles, and it took her
6 hours to drive the last 300 miles. What was her
average rate of speed, in miles per hour, for the entire
trip?
A: The total distance traveled is 660 miles. The total time is 5 + 6 =
11 hours. So:
Total distance
Average speed 
Total time
660 miles
  60 miles per hour
11 hours
Watch out for questions that ask for average rate or average speed.
Sometimes the temptation will be just to average the individual rates or
speeds, but that approach almost never gets you the correct answer. In the
following question, for example, it might seem logical to some people that
the average speed for the round trip will be the average of the two one-way
speeds.
Q: Thelma ran from her front door to the mailbox at a
rate of 15 kilometers per hour, and walked back by the
same path at a rate of 5 kilometers per hour. What
was her average rate of speed, in kilometers per hour,
for the round trip?
A: To find average speed, you need the total distance and the total
time. Here, you’re not given any distances. Let’s call the one-way
distance from the door to the mailbox d kilometers. Then the total
distance for the round trip is 2d.
And what about the total time? What you need to do is find the
two one-way times in terms of d. Going from the door to the
mailbox she goes d kilometers at 15 kilometers per hour. The first
time, then, is:
d kilometers d
 hours
15 kilometers per hour 15
Returning from the mailbox to the door, she goes d kilometers at 5
kilometers per hour. The second time, then, is:
d kilometers d
 hours
5 kilometers per hour 5
And so the total number of hours is:
d d d 3d 4d
+  + 
15 5 15 15 15
Now you have expressions for the total distance and the total time.
When you plug them into the formula and divide, the d’s drop out
and you get a numerical answer:
Total distance 2d kilometers
  7.5 kilometers per hour
Total time 4d
hours
15

100 © The MBA Center


Math Review

Mixture Problems
Another classic word problem type that Phineas loves is the mixture
problem. The situation described will be something like this: a certain
amount A, of which a certain fraction or percent is something, is combined
with a certain amount B, of which a certain fraction or quantity is
something, to yield a mixture, of which a certain fraction or percent is
something. The basic set-up is this:

(fractionA)(A) + (fractionB)(B) = (fractionA+B)(A + B)


Or:
(percentA)(A) + (percentB)(B) = (percentA+B)(A + B)

Each formula relates five quantities: A, B, and the three fractions or


percents. Given any four of these quantities, you can solve for the fifth.

Q: If 5 liters of a solution that is 60 percent alcohol is


added to 20 liters of a solution that is 40 percent
alcohol, what percent of the resulting solution is
alcohol?
A: Here you are given percentA = 60, A = 5, percentB = 40, and B = 20.
You are asked to solve for fractionA+B so call that x:

(0.60)(5) + (0.40)(20) = (x)(5 + 20)

Solve this equation and you’ll find that x = 0.44, and so the percent
alcohol in the resulting solution is 44.

Phineas also likes the variation of this problem type in which a


combination of quantities with different unit prices yields a mixture with a
new unit price. The set-up is this:

(priceA)(A) + (priceB)(B) = (priceA+B)(A + B)

Q: Peanuts sell for $0.90 a pound, and cashews sell for


$3.25 a pound. How many pounds of cashews must be
added to 6 pounds of peanuts to make a mixture that
will sell for $1.84 a pound?
A: Here you are given priceA = 0.90, A = 6, priceB = 3.25, and
priceA+B = 1.84. You are asked to solve for B, so call that x:

(0.90)(6) + (3.25)(x) = (1.84)(6 + x)

Solve this equation and you’ll find that x = 4.

© The MBA Center 101


Total PrepKit for the GMAT® Step 2.1 Lesson

PRACTICE QUESTIONS 91-100 (PHINEAS’ FAVORITE WORD PROBLEMS)

91. If one person is born every 15 seconds in country X, approximately how


many people are born in 1 year?
(A) 1,600,000
(B) 2,100,000
(C) 3,200,000
(D) 21,000,000
(E) 32,000,000

92. Machine A can produce 100 widgets in 40 minutes. Machine B can


produce 100 widgets in 30 minutes. How many hours will it take the two
machines working together to produce 7,000 widgets?
(A) 8
(B) 10
(C) 16
(D) 20
(E) 24

93. Two runners start at the same time from the same point on an oval track.
Runner A runs clockwise around the track at a rate of 25 feet per second,
and runner B runs counterclockwise at a rate of 15 feet per second. The
total distance around the track is 1000 feet. When they meet, runner A
will have run how many feet more than runner B?
(A) 100
(B) 150
(C) 200
(D) 250
(E) 300

94. Hose A can fill a pool in 10 hours. Hose B can fill the pool in 15 hours.
How many hours will it take the two hoses working simultaneously to fill
the pool?
(A) 5
(B) 6
(C) 7.5
(D) 12.5
(E) 25

95. Alice can snap a pound of beans in 6 minutes. Bert can snap a pound of
beans in 4 minutes. Carmen can snap a pound of bean in 3 minutes. How
long will it take the three of them working together to snap a pound of
beans?
(A) 1 minute
(B) 1 minute, 20 seconds
(C) 1 minute, 33 seconds
(D) 1 minute, 40 seconds
(E) 2 minutes

102 © The MBA Center


Math Review

96. Robert ran for 1 hour at 15.0 kilometers per hour, and then he walked for
8 hours at 6.0 kilometers per hour. What was his average rate of speed in
kilometers per hour for the whole trip?
(A) 7.0
(B) 7.5
(C) 8.0
(D) 9.0
(E) 10.5

97. Lydia drove half of the distance from A to B at 120 kilometers per hour,
and the other half of the distance at 80 kilometers per hour. What was
her average rate of speed, in kilometers per hour, for the whole trip?
(A) 96
(B) 98
(C) 100
(D) 102
(E) 104

98. An opera house has 2,200 seats, some in the orchestra and the rest in the
balcony. Orchestra seats cost $50 each, and balcony seats cost $30 each.
When every seat is sold, the total receipts are $82,000. How many
orchestra seats are there?
(A) 750
(B) 800
(C) 850
(D) 900
(E) 950

99. Professor Adams has a total of 54 students in two classes, A and B.


Exactly one fifth of the students in class A are seniors, and exactly one half
of the students in class B are seniors. If exactly one third of all of his
students are seniors, how many seniors are in class A?
(A) 6
(B) 8
(C) 10
(D) 12
(E) 16

100. Elizabeth put a total of $5,000 into two investments one year ago. One
investment earned 8 percent interest and the other investment earned 12
percent interest. If the combined interest earned was $462, how much
was invested at 8 percent?
(A) $1,550
(B) $1,875
(C) $2,250
(D) $3,125
(E) $3,450

© The MBA Center 103


Total PrepKit for the GMAT® Step 2.1 Lesson

ANSWER KEY TO PRACTICE QUESTIONS

1. C 51. E
2. C 52. A
3. E 53. B
4. B 54. B
5. A 55. B
6. A 56. A
7. A 57. E
8. C 58. D
9. A 59. E
10. E 60. C

11. C 61. E
12. B 62. D
13. D 63. E
14. D 64. A
15. B 65. C
16. C 66. C
17. A 67. E
18. A 68. A
19. A 69. D
20. C 70. E

21. B 71. A
22. C 72. E
23. C 73. B
24. D 74. E
25. D 75. B
26. A 76. B
27. E 77. A
28. C 78. B
29. E 79. A
30. D 80. B

31. A 81. B
32. E 82. E
33. C 83. C
34. B 84. A
35. D 85. D
36. E 86. A
37. C 87. B
38. A 88. B
39. B 89. E
40. D 90. B

41. A 91. B
42. B 92. D
43. A 93. D
44. D 94. B
45. A 95. B
46. B 96. A
47. C 97. A
48. C 98. B
49. E 99. A
50. E 100. E

104 © The MBA Center


Step 3
Problem Solving

© The MBA Center


Total PrepKit for the GMAT® Step 3.1 Lesson

GENERAL OUTLINE

Almost two-thirds of the questions in the Quantitative Section of


the GMAT CAT are Problem Solving questions, so your final score for
the section will depend to a large extent on your ability to solve
Problem Solving questions quickly and accurately. This lesson
introduces Problem Solving questions, provides a systematic method
for solving them, and gives several specific strategies designed to help
you save time and eliminate wrong answers.

THE FOLLOWING TOPICS WILL BE COVERED IN THIS LESSON:

The Problem Solving Directions


The Challenge Of These Questions
The Wrong Answer Factory
The MBA Center Approach To Problem Solving Questions
Specific MBA Center Strategies For Problem Solving Questions

KEY TERMS
The Wrong Answer Factory. The Wrong Answer Factory is the name we
have given to the people who create wrong answers designed to
attract the unwary test taker.

Inserting Numbers. Inserting numbers is a strategy that allows the test


taker to plug numbers into the problem in order to simplify calculations.

Backsolving. Backsolving is a technique, especially helpful for difficult


word problems, that consists of inserting the answer choices back into the
problem in order to determine the correct answer.

Process of Error Identification. Process of Error Identification is a


strategy by which you eliminate wrong answers to get to the right answer.

106 © The MBA Center


Problem Solving

READY…

Problem Solving questions are combined with Data Sufficiency questions


in the Quantitative Section of the GMAT CAT. Although the ultimate
decision is up to the computer, you should expect that approximately 23 of
the 37 questions in the Quantitative Section will be Problem Solving
questions.

Problem Solving questions are simply math questions in a multiple-


choice format. A Problem Solving question consists of a math problem in
question form followed by five answer choices, only one of which is correct.
The wrong answer choices are designed to tempt test takers who either
make common mistakes or don’t understand the problem. How the test
makers produce wrong answer and how you can avoid them are both discussed
later in this lesson.

THE DIRECTIONS

The directions for the Problem Solving questions are similar to the
following:

GMAT CAT – Section 3 : Quantitative Directions

End
When finished
Directions: Solve the following problems reading
and indicate the best answer from the choices directions
given. click on the
Numbers: All numbers used are real icon below
numbers.
Figures: Figures accompanying prob-
lems are intended to provide information
useful in solving the problem. Except when
otherwise specifically stated, figures are Dismiss
drawn as accurately as possible. All figures Directions
lie in a plane unless otherwise stated. Straight
lines may appear jagged on the computer
screen.

Test Section Answer


Time Help Confirm Next
Time
Quit Exit

WHAT THE DIRECTIONS MEAN

The directions first state that you should select the “best” answer. What
this means, of course, is that you should choose the correct answer. These
are math problems, so there should be only one correct answer. If you find
that more than one answer is possible, it is likely that you have
misunderstood the problem. The directions also indicate that all numbers
used are real numbers. This simply means that all of the numbers used in
the test have a place on the number line. None of the numbers are
imaginary or complex numbers. Finally, the directions state that, unless

© The MBA Center 107


Total PrepKit for the GMAT® Step 3.1 Lesson

otherwise indicated, figures are meant to provide useful information and


are drawn as accurately as possible. This means that diagrams are generally
drawn to scale, so you can use the diagrams to help you guess at the answers
if you get stuck.

Here is a typical Problem Solving question:


Score Value: 530

01:15 GMAT CAT– Section 3 : Quantitative 1 of 37

Note: On the actual GMAT


CAT the answer choices will
not be lettered as they are in If the area of a square is 80, then what is the area of the largest
this diagram. Instead, each circle that could fit within the square?
answer choice will correspond
to an oval on the screen. To
indicate your answer, you (A) 10P
simply use the mouse to click (B) 20P
on the appropriate oval. (C) 25P
(D) 40P
(E) 80P

Test Section Answer


Time Help Confirm Next
Time
Quit Exit

The correct answer to this question is (B) or 20P. Answers (A) and
(D) are meant to lure test takers who make simple multiplication
errors and choices (C) and (E) are designed to trick test takers who
don’t understand the problem.

Authors Note: On the GMAT CAT, the answer choices will be preceded by
hollow ovals. You indicate your choice by clicking on the appropriate oval using
the mouse. In this chapter and in the rest of this book, however, the answer choices
will be designated by the letters A, B, C, D, and E for ease of reference.

WHAT PROBLEM SOLVING QUESTIONS TEST


Problem Solving questions test your mastery of basic mathematical
concepts and your ability to solve complex problems accurately, and above
all, to do it quickly. The difficulty with solving Problem Solving questions
is generally not in calculating an answer, but sorting through the infor-
mation given and quickly finding the best approach to determine the
correct answer. Because you must arrive at a solution, Problem Solving ques-
tions also test your ability to work with numbers carefully and accurately.

THE CHALLENGE
The principal challenge of the Quantitative Section of the GMAT is time.
If you were given an unlimited amount of time, you could probably answer
most of the questions in this section. Unfortunately, time is very limited.
Most test takers find it difficult to finish all 37 questions in the Quantitative
Section before time has expired. Time is a particular problem with respect
to Problem Solving questions because you not only have to determine how
to solve the problems, but you must also do the math necessary to arrive at
a solution. For this reason the MBA Center method stresses both mastery of
the necessary mathematics and employment of the time-saving strategies in
this lesson.
108 © The MBA Center
Problem Solving

AIM…

THE RANGE OF DIFFICULTY OF PROBLEM SOLVING QUESTIONS

On the GMAT CAT, Problem Solving questions are chosen from a very
large database of questions categorized by content and difficulty. The
difficulty level of a question is based on how previous test takers have
responded to it. The CAT tries to determine your level of ability and give
you questions that are appropriate. In theory, you will eventually reach a
level of difficulty where you will get roughly half of the questions right and
roughly half wrong. Below are three questions representing three different
levels of difficulty:

AN EASY QUESTION
Score Value: 350

01:13 GMAT CAT– Section 3 : Quantitative 2 of 37

If x – 5 = 15, then what is the value of 2x ?

(A) 20
(B) 40
(C) 100
(D) 200
(E) 400

Test Section Answer


Time Help Confirm Next
Time
Quit Exit

A MEDIUM QUESTION
Score Value: 500

01:11 GMAT CAT– Section 3 : Quantitative 3 of 37

If x and y are prime numbers, and the average (arithmetic mean)


of x and y is 14, then x could be which of the following?

(A) 2
(B) 3
(C) 7
(D) 11
(E) 13

Test Section Answer


Time Help Confirm Next
Time
Quit Exit

© The MBA Center 109


Total PrepKit for the GMAT® Step 3.1 Lesson

A DIFFICULT QUESTION
Score Value: 680

01:09 GMAT CAT– Section 3 : Quantitative 4 of 37


Series T is a sequence of numbers where each term after the first
term is x greater than the term that precedes it. If the sum of the first
and last terms of series T is 14, then what is the sum of the first three
terms of series T and the last three terms of series T?
! (A) –7
! (B) 7
! (C) 14
! (D) 42
! (E) 84


Test Section Answer
Time Help Confirm Next
Time
Quit Exit

PHINEAS AND THE WRONG ANSWER FACTORY

For every Problem Solving question there are five answer choices, only
one of which is correct. The other four answers are wrong, but there’s more
to it than that. After writing a question and determining the correct answer,
the test writer (whom we’ve nicknamed Phineas) sets about to devise
incorrect answers that will attract test takers who have made a common
mistake, who only partially understand the problem, or who haven’t any
idea what the question is asking. This is really what makes Problem Solving
questions difficult. Take a look at the previous example of a difficult
question with a different set of answers.

Revisited Score Value: 680

01:09 GMAT CAT– Section 3 : Quantitative 4 of 37


Series T is a sequence of numbers where each term after the first


term is x greater than the term that precedes it. If the sum of the first
and last terms of series T is 14, then what is the sum of the first three
terms of series T and the last three terms of series T?
! (A) Catch-22
! (B) John F. Kennedy
! (C) July 14, 1789
! (D) 42
! (E) London

Test Section Answer


Time Help Confirm Next
Time
Quit Exit

Not quite a 680-level question now, is it? So you can see that the
wrong answers are very important in making Problem Solving
questions difficult.

110 © The MBA Center


Problem Solving

HOW PHINEAS DEVISES WRONG ANSWERS

Learning how Phineas creates wrong answers will help you to eliminate
incorrect answer choices and improve your chances on the Problem Solving
questions. In general, there are three different types of wrong answers: those
designed to trick people who have made a common mistake (adding two
numbers instead of subtracting, for example); those designed to tempt
people who only partially understand the problem; and those designed to
fool people who have no idea how to solve the problem. The following
example illustrates how Phineas devises the wrong answers for the Problem
Solving questions:
Score Value: 500

01:07 GMAT CAT– Section 3 : Quantitative 5 of 37


Jack bought a packet of pens in a store. If Patti went to another
store and bought 10 percent more pens than Jack and paid 10 percent
more per pen than Jack, what percent more did Patti spend on pens
than Jack?
! (A)
! (B)
! (C)
! (D)
! (E) 21%

Test Section Answer


Time Help Confirm Next
Time
Quit Exit

Phineas starts with the correct answer, in this case 21, but his work is far
from finished. He still must think of four plausible wrong answer choices.

Now, Phineas devises incorrect answers designed to attract test takers


who make common mistakes or who don’t understand the problem. From
years of experience designing and administering the GMAT, Phineas knows
that the most common mistakes are:

● Simple arithmetic mistakes


(adding instead of subtracting, for example)
● Omitting a step of a multiple-step problem
● Confusing units of measure or time
(forgetting to convert from hours to minutes, for example)

In addition, many test takers who don’t understand a problem are


attracted by answer choices that repeat numbers that appear in the problem
or answer choices that “look” different from the other answer choices (the
only answer choice with an exponent, for example). These are generally
wrong.

© The MBA Center 111


Total PrepKit for the GMAT® Step 3.1 Lesson

LET’S TAKE A LOOK AT PHINEAS’ FINAL PRODUCT:

Reprise Score Value: 500

01:07 GMAT CAT– Section 3 : Quantitative 5 of 37


Jack bought a packet of pens in a store. If Patti went to another
store and bought 10 percent more pens than Jack bought and paid
10 percent more for each pen than did Jack, what percent more did
Patti spend on pens than Jack?

! (A) 10%
! (B) 11%
! (C) 18.5%
! (D) 20%
! (E) 21%


Test Section Answer
Time Help Confirm Next
Time
Quit Exit

Answer choice (A) simply repeats the number 10, which appears in
the question. This kind of answer choice is very popular among test
takers who have no idea what to do.

Answer choice (B) is 10 percent more than 10 percent, an overly


simplistic solution to a medium to difficult problem.

Answer choice (C) is structurally different from the rest. It is the


only one with a decimal. This answer, like the first answer choice
attracts test takers who don’t understand the problem.

Answer choice (D) simply adds the two 10’s in the question, a one-
step solution to a multi-step question.

112 © The MBA Center


Problem Solving

THE MBA CENTER METHOD FOR PROBLEM SOLVING QUESTIONS

The MBA Center Method for Problem Solving questions emphasizes a


systematic and organized approach for each and every question. The
method is designed to help you solve the Problem Solving questions quickly
and accurately, without wasting time and without making careless errors.
You should practice using this method when you work through the
examples in this lesson and when you do the Problem Solving practice
exercises in this book.

Step 1: Read and understand the question.

In order to answer a Problem Solving question you must first understand


what information you are given and exactly what the question asks you to
find. Take the time necessary to read carefully through the problem in order
to ensure that you don’t make a careless mistake or fall into one of Phineas’
traps. Above all, make sure you understand exactly what the question asks
you to find.

Take a look at the following example:

Score Value: 550


A

01:05 GMAT CAT– Section 3 : Quantitative 6 of 37


In 1955, 300 of a company’s 750 employees were women. In


1987, the company had 900 employees, 430 of whom were women.
The number of female employees increased by what percent from
1955 to 1987?

Test Section Answer


Time Help Confirm Next
Time
Quit Exit

Or...

B Score Value: 550

01:05 GMAT CAT– Section 3 : Quantitative 6 of 37


In 1955, 300 of a company’s 750 employees were women. In


1987, the company had 900 employees, 430 of whom were women.
The number of female employees in 1987 was what percent of the
number of female employees in 1955?

Test Section Answer


Time Help Confirm Next
Time
Quit Exit

© The MBA Center 113


Total PrepKit for the GMAT® Step 3.1 Lesson

Notice the difference between the two questions. The first asks for
the percent increase and the second asks for the percent of. Two very
different questions. Note also that the number of male employees is
irrelevant in both questions. If you misunderstood the question — for
example, if you thought the second example was asking for the
percent increase — Phineas will have a wrong answer waiting for you.

Step 2: Determine the best way to solve the question.

Generally, there is more than one way to solve a Problem Solving


question. The method you should use is the one that will get you to the
correct answer in the least amount of time. You can always solve Problem
Solving questions by simply working through them mathematically, but
there are often quicker, easier ways to arrive at the correct answer.

In this lesson you will learn strategies designed to help you save time on
Problem Solving questions. When you read Problem Solving questions, try
to determine early on which time-saving strategies and shortcuts you can
use.

Step 3: Decide how much time to spend on the question.

Time is precious on the GMAT CAT, so you can’t afford to spend too
much time pondering any one question. If you’ve been through steps one
and two of the MBA Center Method and you’re stuck, you have to
determine how much time and effort to put in before you guess. Remember,
the early questions are a lot more valuable than the later questions, so you
should try to avoid guessing until later in the section. But if you get a
problem that you really don’t think you will be able to solve, you shouldn’t
waste time staring at the question. If you think you are going to have to
guess on a problem, you should try to eliminate as many answers as you can
and guess quickly rather than wasting a lot of time and energy trying to
solve the problem. Take a look back at Time Management on the CAT in the
previous lesson to get an idea of how much time you should spend on the
questions in the various stages of the section.

Step 4: If you’re stumped, guess.

Remember that the CAT determines your level of ability and gives you
Summary of the MBA Center
Method for Problem Solving questions that are appropriate, so you will find that most of the questions
on the CAT are difficult for you. Therefore, you are going to have to guess
Read and understand the on some of the questions. That’s unavoidable. But you can improve your
question. chances of guessing correctly by eliminating wrong answer choices. In this
Determine the best way to lesson you will learn how to use the Process of Error Identification to
solve the question. narrow down the answer choices. You should practice using this strategy in
order to improve your chances of guessing the correct answer.
Decide how much time to
spend on the question.

If stumped, guess.

114 © The MBA Center


Problem Solving

FIRE!

In this section of the lesson we are going to show you how to apply
specific techniques to solve the problems more easily and more accurately.

INSERTING NUMBERS
Many Problem Solving questions require you to work with algebraic
expressions (a formula or equation with variables such as x and y). Some
questions give you an algebraic expression directly while some, particularly
word problems, require you to set up an algebraic expression on your own.
Often, the best way to solve these problems is to substitute numbers for the
variables in the problem.

Here is how the strategy of inserting numbers works:

Identify the unknowns or variables in the problem.

Sometimes the problem will explicitly state what the variables are (a
carpet has dimensions x by y, for example). Often, particularly in word Make life easy for yourself.
Pick low, easy numbers to
problems, you will have to spot the variables yourself (a farmer sells 2 of insert in the problem.
5
his goods, for example).
The first step in using this strategy is determining what the variables are.

Determine the equations you need to solve the problem.

Again, you are sometimes given the equations in the problem itself or in
the answer choices. These are easy to spot. In word problems, however, you
are often required to set up the equations yourself. Look for words that
express a mathematical relationship such as “doubled” or “increased by x
percent” to help you set up equations contained in the problem.

Pick numbers to stand in for the variables.

Pick numbers that will make calculations simple. Generally, you should
pick low integers in order to make your work easy. In percent
increase/decrease problems, pick numbers that will give you 100 as a
starting value. For problems involving fractions, pick numbers that are
multiples of the denominators of the fractions so that the calculations will
work out to integers.

Insert the numbers and solve the equations.

Insert the numbers you picked above and solve the equations. For many
problems, this will lead you directly to the correct answer choice. For some
questions, particularly number property questions (“Which of the following
is a prime number?” for example) and questions with variables in the answer
choices, more than one answer choice may work using the numbers you
picked. If this happens, eliminate the answer choices you can and pick a
fresh set of numbers. The correct answer choice will work for any set of
numbers picked.

© The MBA Center 115


Total PrepKit for the GMAT® Step 3.1 Lesson

Here’s an example of a Problem Solving question and how you can


use the strategy of inserting numbers to solve it.
Score Value: 560

01:03 GMAT CAT– Section 3 : Quantitative 7 of 37


A jar contains a number of beads and a second jar contains three
times as many beads as the first. If 60 percent of the beads in the first
jar are red and 20 percent of the beads in the second jar are red, then
what percent of the total beads in both jars are red?

! (A) 25%
! (B) 30%
! (C) 33.3%
! (D) 40%
! (E) 50%


Test Section Answer
Time Help Confirm Next
Time
Quit Exit

First, you must determine what the unknowns or variables are in


the problem. In this case, you know the percentage of red beads in
both jars, but you don’t know just how many beads there are in either
jar. These are your unknowns.

Now you need to pick a total number of beads for each of the jars.
You know that all of the information necessary to determine the
answer is somewhere in the problem. Because the total number of
beads is never mentioned, you know that the total number of beads
in the jars can’t make a difference. That means that you can pick any
numbers that will make your calculations easy. Since this is a percent
problem, you want to pick numbers that will give you 100 as a
starting point. Pick 25 for the first jar. The question tells you there are
three times as many beads in the second jar as the first, so the total
number of beads in the second jar has to be 75. If there are 25 beads
in the first jar and 75 beads in the second jar, then the total number
of beads in both jars is 100, which is just what we wanted.

You’re now ready to solve the problem. If there are 25 beads in the
first jar, and 60% of them are red, then the number of red beads in
the first jar is 60% of 25, or 0.6 × 25 = 15. Likewise, there are 75 beads
in the second jar, 20% of which are red, so the number of red beads
in the second jar is 0.2 × 75 = 15. If there are 15 red beads in the first
jar and 15 red beads in the second jar, then the total number of red
beads in both jars is 30. Remember, you’re asked to determine what
percent of the total beads in both jars are red. You know that the total
number of beads in both jars is 100. The question then is this: 30 is
what percent of 100? That’s easy. It’s 30%, which is the correct answer.

116 © The MBA Center


Problem Solving

The types of questions that can usually be solved by inserting numbers:

Percent increase/decrease problems without values in the question

Score Value: 590

01:01 GMAT CAT– Section 3 : Quantitative 8 of 37 On percent problems, pick


numbers that give you 100 as
a starting value.
If the time it takes a company to build a dam is determined by the


2
formula d nw , where d is the depth of the dam in meters, w is the

width in meters, and n is the number of employees involved, and the


plans for a certain dam are changed so that the depth increases by
20 percent, the width decreases by 30 percent, and the number of
employees increases by 40 percent, what will be the percent change
in the time required to build the dam?

! (A) 40% decrease


! (B) 28% decrease
! (C) 15% increase
! (D) 30% increase
! (E) 88% increase


Test Section Answer
Time Help Confirm Next
Time
Quit Exit

Score Value: 600

00:59 GMAT CAT– Section 3 : Quantitative 9 of 37


Last year, a certain store made a 10 percent profit on all sales.


This year, sales are 20 percent higher than last year, but the store’s
profits are only 5 percent of sales. This year’s profits are what percent
of last year’s profits?

! (A) 60%
! (B) 80%
! (C) 100%
! (D) 120%
! (E) 140%

Test Section Answer


Time Help Confirm Next
Time
Quit Exit

© The MBA Center 117


Total PrepKit for the GMAT® Step 3.1 Lesson

Problems involving fractions without totals:


Score Value: 620

00:57 GMAT CAT– Section 3 : Quantitative 10 of 37


2 1
If a vendor sells
5 of his goods in the morning and 3 of the
remaining goods in the afternoon, what fraction of his goods did he

sell in the morning and afternoon combined?

! (A) 3
8
! (B) 2
5
! (C) 3
5
! (D) 2
3
! (E) 11
15


Test Section Answer
Time Help Confirm Next
Time
Quit Exit

Score Value: 570

00:55 GMAT CAT– Section 3 : Quantitative 11 of 37


A certain drug is used to fight a deadly microbe. It is known that


when the drug is given to a patient it kills 1 of the microbes in the
3
first 24 hours. During each successive 8-hour period, the drug kills
1 of the microbes present at the beginning of that period. After
2
2 days, what fraction of the microbes initially present will remain?

! (A) 1 ! (D) 1
48 8
! (B) 1 ! (E) 1
24 6
! (C) 1
12

Test Section Answer


Time Help Confirm Next
Time
Quit Exit

118 © The MBA Center


Problem Solving

Questions involving number properties:


(odd or even, prime or not prime, integer or not integer, etc.)
Score Value: 630

00:53 GMAT CAT– Section 3 : Quantitative 12 of 37


If a, b, and c are consecutive positive integers and a < b < c,
which of the following must be true?

I. a + b + c is an even integer.
II. ab is an even integer.
a+ b
III. =c
2

! (A) I only
! (B) II only
! (C) I and II only
! (D) II and III only
! (E) I, II, and III

Test Section Answer ➩


Time Help Confirm Next
Time
Quit Exit

Score Value: 640

00:51 GMAT CAT– Section 3 : Quantitative 13 of 37


If x and y are distinct prime numbers, then which of the following


is a prime number?

! (A) xy

! (B) 3xy2

x2– y2
! (C) x – y – y

2x2– 2y2 – 2x
! (D) x–y
2 2
! (E) x – y + 3y – x
x–y

Test Section Answer


Time Help Confirm Next
Time
Quit Exit

© The MBA Center 119


Total PrepKit for the GMAT® Step 3.1 Lesson

Questions with variables in the answer choices:


Score Value: 650

00:49 GMAT CAT– Section 3 : Quantitative 14 of 37

If n identical pipes can fill an x-gallon pool in t hours, then at the


same rate how long will it take one such pipe to fill a y-gallon pool?

(A) ty
xn
(B) nty
x
(C) n
xyt
(D) y
xnt
(E) xy
nt

Test Section Answer


Time Help Confirm Next
Time
Quit Exit

Score Value: 460

00:47 GMAT CAT– Section 3 : Quantitative 15 of 37

B
INSERTING NUMBERS
OUTLINE
A C
Identify the unknowns or
variables in the problem
Determine the equations D
you need to solve the
problem
Pick numbers to stand in for
Square ABCD above is partially within a circle with center A. Each
the variables of sides AB and AD of the square is a radius of the circle. If the area
Insert the numbers and of square ABCD is x, then what is the area of the circle?
solve the equations

Questions that can usually be x2 2


solved by inserting numbers: (A) (D) P x
3P
Percent increase/decrease
problems without values in
the question x x 3
(B) (E)
Problems involving fractions 4P 4P
without totals
Questions involving number (C) P x
properties (odd or even,
prime or not prime, integer or
not integer, etc.)
Test Section Answer
Questions with variables in
Time Help Confirm Next
Time
Quit Exit
the answer choices

120 © The MBA Center


Problem Solving

BACKSOLVING

Backsolving is similar to inserting numbers, but instead of picking num-


bers yourself you simply use the answer choices given to you. Remember that
there is only one correct answer among the answer choices and it is the only
answer that will agree with the information in the problem. Therefore, you
can plug each answer choice into the equation or equations in the problem.
The one that works is the correct answer. Backsolving is particularly helpful
when you get stuck on a difficult or complex problem. Generally, you should
only attempt to backsolve problems with integers as answer choices.

Here’s how backsolving works:

Start with the middle answer choice (generally).

Because the answer choices are arranged in order from least to greatest or
greatest to least, you should generally start with the middle answer choice.
That way, if the answer choice is too large or too small, you will have
eliminated the answer choice you picked and the two above or below it.
However, if it appears that using the middle answer choice will make
calculations difficult, pick a different answer choice.

Work through the problem using the answer choice you picked.

Substitute the answer choice into the equation or equations in the


problem. If all of the equations work, then you’ve found the correct answer.
If any of the equations do not work out using the answer choice you picked,
then you should eliminate it and pick another answer choice.

If necessary, check the next answer choice.

If you backsolve an answer choice and it doesn’t work, you can eliminate
it. If the answer choice is too large, you can eliminate both of the answer
choices that are larger than it. Likewise, if the answer choice is too small,
you can eliminate both of the answer choices that are smaller than it. Now
you can pick from the two remaining answer choices and try again.

Here’s an example of a Problem Solving question and how you can


use the strategy of backsolving to solve it.
Score Value: 530

00:45 GMAT CAT– Section 3 : Quantitative 16 of 37

Four friends live together in an apartment and split the monthly


rent equally. When one of the friends moves out, the remaining three
split the rent equally and each pays $50 per month more than before.
How much is the total monthly rent for the apartment?

(A) $240
(B) $360
(C) $480
(D) $600
(E) $800

Test Section Answer


Time Help Confirm Next
Time
Quit Exit

© The MBA Center 121


Total PrepKit for the GMAT® Step 3.1 Lesson

Start with the middle answer choice, in this case $480, and work
through the problem. If the total rent for the apartment is $480, then
each of the four friends initially pays $120 per month (480 ÷ 4 = 120).
The question then tells you that after one of the friends leaves, each
of the remaining three pays $50 more than before. If each person was
paying $120 before, they would now be paying $170 each per month
(120 + 50 = 170), but if that’s true then the total monthly rent would
be $510 (170 × 3 = 510), which is higher than $480, so we know that
the total monthly rent has to be higher than $480. Eliminate the
middle answer choice and the two that are lower than it.
Now we move to the next highest answer choice, in this case $600,
and work through the problem the same way. If the total rent for the
apartment is $600, then each of the four friends initially pays $150
per month (600 ÷ 4 = 150). The question then tells you that after one
of the friends leaves, each of the remaining three pays $50 more than
before. If each person was paying $150 before, they would now be
paying $200 each per month (150 + 50 = 200). If each person pays
$200 per month, then the total monthly rent is $600 (200 × 3 = 600),
which is correct. So $600 is the best answer.

The types of questions that can usually be backsolved:

Word problems with integers as answer choices:


Score Value: 580

00:43 GMAT CAT– Section 3 : Quantitative 17 of 37


A salesman is paid $5.00 per hour for every hour he works in the
office and $8.00 for every hour he spends on the road. If the
salesman earned $360 during a certain week, and if he worked twice
as many hours in the office as he spent on the road, then how many
hours did the salesman work in the office during the week?

! (A) 20
! (B) 40
! (C) 50
! (D) 60
! (E) 70

Test Section Answer


Time Help Confirm Next
Time
Quit Exit

122 © The MBA Center


Problem Solving

Score Value: 590

00:41 GMAT CAT– Section 3 : Quantitative 18 of 37


A car dealer had a sale in order to sell off excess stock. Before
the sale, the dealer had three times as many cars as he had trucks.
During the sale, he sold 100 cars and 25 trucks. If after the sale the
dealer has twice as many cars as he has trucks, then how many cars
did the dealer have prior to the sale?

! (A) 60
! (B) 90
! (C) 120
! (D) 150
! (E) 180


Test Section Answer
Time Help Confirm Next
Time
Quit Exit

Score Value: 620

00:40 GMAT CAT– Section 3 : Quantitative 19 of 37


The ratio of flowering to nonflowering plants on an acre of land is


3 to 2. After 140 nonflowering plants are removed from the property,
the new ratio of flowering to nonflowering plants is 4 to 1. How many
plants were there on the acre of land before the 140 nonflowering
plants were removed?
! (A) 224
! (B) 252
! (C) 280
! (D) 560
! (E) 700

Test Section Answer


Time Help Confirm Next
Time
Quit Exit

© The MBA Center 123


Total PrepKit for the GMAT® Step 3.1 Lesson

Problems with complex algebraic expressions and integers as answer


choices:

Score Value: 650

00:38 GMAT CAT– Section 3 : Quantitative 20 of 37


If (x – 1)(x – 2)(x – 3)(x – 4) = 24, then x could be which of the
following?

! (A) 4
! (B) 5
! (C) 6
! (D) 7
! (E) 8


Test Section Answer
Time Help Confirm Next
Time
Quit Exit

Score Value: 580

00:36 GMAT CAT– Section 3 : Quantitative 21 of 37


If (x – 1)(y – 2) + (x – 3)(y – 4) + (x – 5)(y – 6) = 8, and x + y = 9,
then x could be which of the following?

! (A) 0
! (B) 1
! (C) 2
! (D) 3
! (E) 7

Test Section Answer


Time Help Confirm Next
Time
Quit Exit

BACKSOLVING OUTLINE
● Start with the middle
answer choice (generally)
● Work through the problem
using the answer choice you
picked
● If necessary, check the next
answer choice

Questions that can usually be


backsolved:
● Word problems with inte-
ger answer choices
● Problems with complex al-
gebraic expressions and in-
tegers as answer choices

124 © The MBA Center


Problem Solving

PROCESS OF ERROR IDENTIFICATION


Remember that for each question there is only one right answer and the
other four are incorrect, so there are really two paths to the correct answer
choice: 1) You can solve the problem or 2) you can eliminate all of the
wrong answers, leaving only the best answer. Even if you are able to
eliminate only two or three answer choices, your chances of getting the
question right increase tremendously. You should therefore practice using
the Process of Error Identification to zero in on the right answer. This
strategy works especially well when a question has you completely stumped.

Here’s how The Process of Error Identification works:

Try to get a general idea what the question is asking.

In order to identify and eliminate answer choices that don’t make sense
in the context of the question, you must first understand what the question
asks you to find. Many times wrong answer choices are designed to fool the
test taker who doesn’t read the question carefully enough. Take a look at the
following example:
Score Value: 590
A

00:34 GMAT CAT– Section 3 : Quantitative 22 of 37



Regular hexagon ABCDEF is F A
inscribed in a circle with area 9π.
What is the perimeter of hexagon
ABCDEF?
E B

D C

Test Section Answer


Time Help Confirm Next
Time
Quit Exit

Or...
Score Value: 590
B

00:34 GMAT CAT– Section 3 : Quantitative 22 of 37


Regular hexagon ABCDEF is inscribed in a circle with area 9π.


What is the area of hexagon ABCDEF?

Test Section Answer


Time Help Confirm Next
Time
Quit Exit

© The MBA Center 125


Total PrepKit for the GMAT® Step 3.1 Lesson

Did you notice the difference between the two questions? The first
asks for the perimeter of the hexagon and the second asks for the area.
Two very different questions.

Eliminate answer choices that seem unreasonable.

Remember that the answer choices are designed to attract test takers who
have made a common mistake (adding instead of subtracting, for example)
or who have misunderstood the problem. Many times you will be able to
see that several of these answer choices are unreasonable if you simply take
a step back and look at the problem using your common sense.

Try these examples:


Score Value: 560

00:32 GMAT CAT– Section 3 : Quantitative 23 of 37


F A

E B

D C
Regular hexagon ABCDEF is inscribed in a circle with area 16π.
What is the area of hexagon ABCDEF?

! (A) 4 ! (D) 24 3

! (B) 6 3 ! (E) 48 2

! (C) 12 2

Test Section Answer


Time Help Confirm Next
Time
Quit Exit

First, note what the question asks you to find. In this case it is the
area of the hexagon inscribed in the circle. If you look at the figure,
you can see that the hexagon is smaller than the circle, but not that
much smaller. π is about 3.14, so you can estimate that the area of the
circle is about 48. Now, you can safely eliminate any answer choice
that is larger than 48 or significantly smaller than 48.

126 © The MBA Center


Problem Solving

Score Value: 610

00:30 GMAT CAT– Section 3 : Quantitative 24 of 37


David can sail m miles in 13 hours. At that constant rate, how many
hours will it take David to sail n miles?

mn 13m m
! (A) ! (C) ! (E)
13 n 13n

13 13n
! (B) ! (D)
mn m


Test Section Answer
Time Help Confirm Next
Time
Quit Exit

In this question you are asked to determine how long it will take
David to sail n miles. You should first notice that as n increases the
amount of time it will take David to sail n miles has to increase as
well. For example, it would certainly take David longer to sail 1,000
miles than it would to sail 10 miles. So you can eliminate any answer
choices where n appears in the denominator of the fraction.
Remember that as the denominator of a fraction increases the value
of the fraction decreases. Therefore, answer choices (B), (C) and (E) are
gone. If you guess at this point, you have a fifty-fifty shot.

Now you should note that as m increases, David is sailing faster.


For example, if David sails 1,000 miles in 13 hours he is certainly
moving faster than if he sails only 10 miles in 13 hours. You know
that the faster David sails, the less time it will take him to sail n miles,
so you can eliminate any answer choice where m appears in the
numerator of the fraction. Therefore, you can eliminate answer
choices (A), (C), and (E). Since you’ve already eliminated answer
13n
choice (B), the only answer choice remaining (D), or , which is
the correct answer. m

© The MBA Center 127


Total PrepKit for the GMAT® Step 3.1 Lesson

Try working with only the last Now try the Process of Error Identification on the following problem:
digits. In this example you
know that the last digit of the
Score Value: 610
total price of the discs is 5 and
the last digit of the total price
of the tapes is 4. If you add 00:30 GMAT CAT– Section 3 : Quantitative 25 of 37
the two together, you know
that the last digit of the total
cost of the discs and the tapes


is 9. Because we know that If John had three times the amount money he has now, he would
John has one third the have exactly the amount necessary to purchase three compact discs
amount necessary to purchase at $9.95 each and two tapes at $7.92 each. How much money does
the discs and the tapes, we John have?
know that the last digit of the
amount he actually has must ! (A) $12.32
end in a 3. ! (B) $13.55
! (C) $15.23
! (D) $16.81
! (E) $17.79


Test Section Answer
Time Help Confirm Next
Time
Quit Exit

128 © The MBA Center


Problem Solving

SUMMARY

Problem Solving questions are simply math questions in a multiple-


choice format. They are designed to test your mastery of basic math
concepts and your ability to reason quantitatively.

The main challenge of the Quantitative Section is time. The time you
are allowed is very limited and must be managed well in order to make sure
you can answer all the questions before time expires. The MBA Center
Method employs specific strategies designed to help you save time when
answering Problem Solving and Data Sufficiency questions.

For every question there is only one correct answer. The wrong answers
are designed by Phineas to attract test takers who make common mistakes
or who don’t completely understand the question. Understanding how
Phineas devises wrong answers will help you to avoid traps.

The MBA Center Method for Problem Solving questions emphasizes a


systematic and effective approach for every question. The method is
designed to help you solve the Problem Solving questions quickly and
accurately, without wasting time and without making careless errors.

The MBA Center strategies of inserting numbers, backsolving, and


eliminating incorrect answer choices are time-saving strategies that will
enable you to answer many Problem Solving questions quickly and
correctly.

PROBLEM SOLVING LESSON


ANSWER KEY

1. B 14. B
2. B 15. C
3. D 16. D
4. D 17. B
5. E 18. D
6.a 43 1 % 19. D
3
20. B
6.b 143 1 % 21. D
3
22.a 18
7. B
27 3
8. B 22.b
9. A 2
10. C 23. D
11. C 24. D
12. B 25. C
13. C

© The MBA Center 129


© The MBA Center
Problem Solving

HOMEWORK/PRACTICE TEST
PROBLEM SOLVING
ANSWER GRID

A B C D E
1 ! ! ! ! !
2 ! ! ! ! !
3 ! ! ! ! !
4 ! ! ! ! !
5 ! ! ! ! !
6 ! ! ! ! !
7 ! ! ! ! !
8 ! ! ! ! !
9 ! ! ! ! !
10 ! ! ! ! !
11 ! ! ! ! !
12 ! ! ! ! !
13 ! ! ! ! !
14 ! ! ! ! !
15 ! ! ! ! !
16 ! ! ! ! !

© The MBA Center 131


Total PrepKit for the GMAT® Step 3.2 Homework

Problem Solving – Homework Directions

End
When finished
TIME – 25 MINUTES reading
16 QUESTIONS directions
click on the
Directions: Solve the following problems icon below
and indicate the best answer from the choices
given.
Numbers: All numbers used are real
numbers.
Figures: Figures accompanying prob- Dismiss
lems are intended to provide information Directions
useful in solving the problem. Except when
otherwise specifically stated, figures are
drawn as accurately as possible. All figures lie
in a plane unless otherwise indicated. Straight
lines may appear jagged on the computer
screen.

Test Section Answer


Quit Exit Time Help Confirm Next

132 © The MBA Center


Problem Solving

1 – 1 7. If the perimeter of a rectangle is 32 feet and its


2 3
1. = area is 60 square feet, what is the length, in feet, of
1
4 each of the longer sides?

(A) 5 (B) 6 (C) 10 (D) 12 (E) 15


1 5 2 4 10
(A) (B) (C) (D) (E)
24 12 3 5 3

8. What is the least positive integer divisible by


2. Working at the same constant rate, 4 proof- each of the integers 9 through 12, inclusive?
readers can proofread 180 pages per hour. How many
pages could 15 proofreaders, all working at this same (A) 1,980 (B) 3,960 (C) 5,940
rate, proofread in 2 hours?
(D) 10,890 (E) 11,880
(A) 216 (B) 1,350 (C) 1,800 (D) 2,160 (E) 5,400

9. How many integers between 150 and 200,


3. A company’s profits for 1996 equaled 7 percent inclusive, can be evenly divided by both 3 and 5 ?
of its revenues. If its profits in 1996 were 27 million
dollars, approximately what were the company’s (A) 3 (B) 4 (C) 5 (D) 12 (E) 27
revenues, in millions of dollars, in 1996 ?

(A) 1.89 (B) 3.86 (C) 18.9 (D) 38.6 (E) 386
10. For all numbers p and q, the operation ! is
defined by p ! q = p2(q – 3). If 2 ! x = –20, what is
the value of x ?
4. How many minutes does it take to read 40 pages
reading at the constant rate of 100 pages per hour? (A) –10 (B) –8 (C) –7 (D) –5 (E) –2

(A) 5 (B) 20 (C) 15 (D) 24 (E) 40


2 3
11. In a sequence of 10 increasing consecutive
integers, the sum of the last five integers is 230. What
5. A video store charges its members a base fee of is the sum of the first 5 integers in the sequence?
$24 per year plus $3 for each 2-day video rental. If a
member was charged $42 for membership and 2-day (A) 225 (B) 220 (C) 215 (D) 210 (E) 205
video rental fees for 1990, then at the same rate how
many 2-day videos did the customer rent in 1991 if his
charges for 1991 were double his 1990 charges? 12. The fixed operating costs to a team owner of
holding a live football match total $80,000. In
(A) 6 (B) 12 (C) 14 (D) 20 (E) 28 addition, there is an average cost to the owner of $6
per spectator who attends the match. If each spectator
pays $14 for a ticket, how many spectators must
attend for the owner to make a profit of $160,000 ?
6. A company manufactures 78 faulty devices out of
every 100,000 devices it manufactures. If it (A) 10,000 (B) 20,000 (C) 30,000
manufactured a total of 3 million devices last year,
how many of them were faulty? (D) 40,000 (E) 50,000

(A) 234 (B) 260 (C) 2,340 (D) 2,600 (E) 23,400

GO ON TO THE NEXT PAGE


© The MBA Center 133
Total PrepKit for the GMAT® Step 3.2 Homework

13. If 1 < a < b < c 16. A company hires a consultant to help with the
development of a new product line. The consultant is
1
1
and if x = a and y = 1
1 and z = able to choose from two methods of payment, either a
1
b flat fee of $800 plus 6 percent of the revenues from the
1 new product, or a flat fee of $200 plus 10 percent of
c the revenues from the new product. For what amount
then which of the following must be true? of revenue would the consultant’s salary be the same
under either payment methods?
(A) z < y < x (B) x < y < z (C) x < z < y
(A) $3,750 (B) $6,250 (C) $15,000
(D) y < z < x (E) z < x < y
(D) $18,500 (E) $25,000

14. The rectangular window represented in the


figure above, of dimensions w centimeters by l
centimeters, is surrounded by a frame 4 centimeters
wide. Which of the following represents the area of the
frame, in square centimeters?

(A) 8w + 8l + 128 (B) 8w + 8l + 64

(C) 8w + 8l +16 (D) 4w + 4l + 16

(E) 4w + 4l + 4

15. If (x + 1)2 = 900, then x – 4 could be equal to


which of the following?

(A) 27 (B) 26 (C) –33 (D) –34 (E) –35

STOP
IF YOU FINISH BEFORE TIME IS EXPIRED YOU MAY
CHECK YOUR WORK
134 © The MBA Center
Problem Solving

PROBLEM SOLVING HOMEWORK/PRACTICE TEST


ANSWER KEY

1. C
2. B
3. E
4. D
5. D
6. C
7. C
8. A
9. B
10. E
11. E
12. C
13. E
14. B
15. E
16. C

© The MBA Center 135


Total PrepKit for the GMAT® Step 3.2 Homework

EXPLANATIONS TO In other words: 100 = 40


PROBLEM SOLVING 60 x

HOMEWORK/PRACTICE TEST Remember, always look to reduce on the GMAT


before multiplying out. Here, 100 = 10 = 5
Now we have 5 = 40 . 60 6 3
3 x
1. (C) First you should find a common deno- When you cross-multiply, you get 5x = 120.
minator, in this case 12. So x = 24, or (D).
You can then multiply both the top and the
bottom by 12.
Thus:
5. (D) The math in this question is not hard, but
it is somewhat tricky.
# 1 1&
12% " ( First, the charges were doubled in 1991. In 1990
$ 2 3' (6 " 4) 2
= = or ( C) the charges were $42, so in '91 the charges must be
# 1& 3 3 $84. The question is how many 2-day rentals the
12 % (
$4' customer was charged for. So we have to subtract the
$24 member fee. Now we have $60. Since each 2-day
rental was $3, we divide 60 by 3 and get 20.
2. (B) This is essentially a ratio question. Be careful not to fall for the trap of just doubling
Eliminate (A) because it's just too small. To solve, the number of 2-day rentals from 1990.
find out how many pages one proofreader does in one
hour: 180 = 45.
4
Multiply this rate by 15 proofreaders to get 675. 6. (C) Again, keep an eye on the answer choices.
Then multiply by 2 because they work for 2 hours. Notice that the choices are quite spread out.
Thus 1,350, or (B), is the answer. This indicates that it is important to get the right
number of digits.
You can set the problem up as a ratio:
78 = x
3. (E) This is a great question to backsolve.
100,000 3,000,000
Rephrase the question this way:
27 million dollars is 7% of which answer choice? Then you isolate x and get
Notice also that the numbers in the choices are
3,000,000(78) = x
pretty far apart. This means you can estimate.
100,000
Instead of 7%, try 10%. Try (C). 10% of 18.9
million, is 1.89 million. You want 27 million, so (C) is too x = 2,340
small.
So, then, are (A) and (B). Eliminate them.
Try (D). 10% of 38.6 million is 3.86 million. Still 7. (C) This is another great problem to solve by
too small. backsolving.
Try (E). 10% of 386 million is 38.6 million. Close As usual, begin with (C).
enough to 27 million. Go with (E). To actually solve Assume the length is 10. Since the area according
the question, you could have set up an equation: to the problem is 60, the width must be 6.
7% x = 27,000,000 Now check that a rectangle with length 10 and
Then divide 27,000,000 by .07 and you get width 6 actually has perimeter 32.
approximately 386 million. This is correct, as 10 + 10 + 6 + 6 = 32,
Note though, that when the choices are far apart so (C) is the right answer.
(here for example, you have 3.86, 386, and 386
million) the challenge is to get the right number of
decimal places, not the exact figure.
8. (A) This looks like a really tough problem with
really big numbers, but hold on. Relax.
Remember that the GMAT is not testing you to see
4. (D) This is a ratio question. if you are a human calculator. To find a least common
Be careful of minutes and hours. multiple, take the prime factors of each number.
100 pages: 60 minutes = 40 pages: x minutes. For 9, the prime factors are 3 and 3.
For 10, 5 and 2.

136 © The MBA Center


Problem Solving

For 11, just 11.


For 12, 2, 2, and 3.
Now, just remember not to count the prime factors Another option is to actually figure out what
that come up for more than 1 number. the numbers are and then compute the sum.
For example, 10 has a prime factor of 2, but so Since five consecutive numbers add up to 230, the
does 12. You only count the 2's from the 12. average of the 5 numbers is 230/5, or 46.
Now you multiply out all the prime factors without The last 5 numbers, then are 44, 45, 46, 47, 48.
the repeats: 3 x 3 x 5 x 2 x 11 x 2 = 1,980. The first 5 are 39, 40, 41, 42, 43, and these
numbers add up to 205.
Another method to solve this problem is to
backsolve.
Start with (C) and divide each number into 5940.
They all go evenly. (C) could be the right answer at 12. (C) This is probably easiest to solve by back-
this point. solving.
(D) and (E) are definitely wrong, since we're As usual, start with C. Suppose there are 30,000
looking for the least positive integer. spectators. Multiply by 14 to get $420,000.
Then try (A) and (B), both of which also work. Then subtract the $80,000 fixed cost and you get
(A) is the smallest so it must be the answer. $340,000.
Then subtract the cost of $6 for each spectator,
$180,000. You now have left $160,000.
This is the expected value, so C is the right answer.
9. (B) Numbers that can be divided by both 3
and 5 must be divisible by 15.
150 is divisible by 15 (and thus 3 and 5). That is
the first one. 13. (E) This is a good problem to do by subbing.
Then every 15 there is another: 165, 180, 195, Pick easy numbers for a, b, and c, such as 2, 3,
making a total of 4. and 4.
Inclusive means you include the two numbers
1 1 1 1
given, 150 and 200. Now x = a or 2 Then y = or or 3.
1 1
b 3
1
Lastly z =
10. (E) A weird symbol question. Don't be 1
intimidated by the symbols, because the definition # 1&
is given to you. % (
$c '
p ! q = p2(q – 3).
Now, since you are asked about 2 ! x, you just
substitute 2 for p and x for q. This is the same as 1 , or with our numbers, 1 .
The result is 22(x – 3) = –20 c 4
4(x – 3) = –20
1 1
4x – 12 = –20 Then put them in order: , , 3, or z < x < y.
4 2
4x = –8
x = –2

14. (B) To find the area of the frame, you can find
11. (E) You know that the integers are consecu- the area of the big rectangle and subtract the area
tive. of the smaller rectangle, leaving only the frame.
This makes our work much easier, but don't get The length of the big rectangle is l + 4 + 4, or l +
tricked! 8.
Saying that the sum is just 5 less is too easy; The width of the big rectangle is w + 4 + 4, or w + 8.
instead, think about the fact that the 1st integer is 5 The area of the big rectangle,
less than the 6th, the 2nd is 5 less than the 7th, the then, is (l + 8)(w + 8) = lw + 8w + 8l + 64.
3rd is 5 less than the 8th, etc. Now subtract the area of the smaller rectangle,
In other words, each number on the list of the first lw, and you have left 8w + 8l + 64, or B.
5 is 5 less than a corresponding number on the list of Note that a common error on this problem is to
the last 5. consider the length of the big rectangle l + 4, instead
The total, then is 25 less than the list of the last 5, of l + 8. You have to count the 4 centimeters on both
making the sum 205. sides of the frame.

© The MBA Center 137


Total PrepKit for the GMAT® Step 3.2 Homework

15. (E) Remember that with squares, you have


to consider a negative and a positive possibility.

Since (x + 1) 2 = 900, (x + 1) = +30 or –30.


If x + 1 = 30, x = 29 and x – 4 = 25,
but this is not in the answer choices.
Therefore try (x + 1) = –30. Then x = –31.
Then x – 4 = –35.
You could also backsolve, testing each answer
choice against the equation.

16. (C) This problem could also be solved either


algebraically or by backsolving.
To do it algebraically:
800 + 0.06x = 200 + 0.10x
600 = 0.04x
15,000 = x

138 © The MBA Center


Step 4
Problem Solving
Workshop

© The MBA Center


Total PrepKit for the GMAT® Step 4 Workshops

Problem Solving – Workshop Directions

End
When finished
TIME – 60 MINUTES reading
32 QUESTIONS directions
click on the
Directions: Solve the following problems icon below
and indicate the best answer from the choices
given.
Numbers: All numbers used are real
numbers.
Figures: Figures accompanying prob- Dismiss
lems are intended to provide information use- Directions
ful in solving the problem. Except when other-
wise specifically stated, figures are drawn as
accurately as possible. All figures lie in a
plane unless otherwise stated. Straight lines
may appear jagged on the computer screen.

Test Section Answer

Quit Exit Time Help Confirm Next

140 © The MBA Center


Problem Solving

EASY
5. If 3 – x3 = 2 + x2 , what is the value of x ?

(A) –1

1. A new car costs $5,250 if the buyer pays cash. (B) 2


3
On an installment plan, the buyer pays 10 percent
down and 12 monthly payments of $450 each. How (C) 1
much greater would the expense be paying with the
installment plan than paying cash? (D) 3
2
(A) $150 (E) 5
(B) $225
(C) $600
(D) $675
(E) $750 6. Which of the following is equivalent to the two
inequalities x + 5 > 12 and x – 4 < 9 ?

2. There are 8 married couples at a dinner party. If (A) 7 < x < 13


5 individuals leave, what is the greatest number of (B) 7 < x < 17
married couples that can remain? (C) 5 < x < 17
(D) 5 < x < 13
(A) 6 (E) 5 < x < 7
(B) 5
(C) 4
(D) 3
(E) 2
(6,8)

3. A publishing company publishes three types of C


books—fiction, biography, and history—in the ratio
2:5:8, respectively. If the company publishes an
average of 40 fiction books per month, how many
books does it publish per month?

(A) 60 7. What is the area of the circular region with


(B) 150 center C shown above?
(C) 260
(D) 300 (A) 20
(E) 600 (B) 36
(C) 64
(D) 100
4. Brian read 108 pages of a novel on Tuesday. On (E) 198
Wednesday he read the remaining 192 pages, reading
at the same average rate he read on Tuesday. If his
reading on Wednesday took 3 hours longer than his
reading on Tuesday, what is his average reading rate,
in pages per hour?

(A) 28
(B) 36
(C) 42
(D) 64
(E) 100

GO ON TO THE NEXT PAGE


© The MBA Center 141
Total PrepKit for the GMAT® Step 4 Workshops

A
8. If (4x – 1) 2 = 0, then x =
B

60°
C
(A) x = – 1 O
2
(B) x = – 1
4
(C) x = 0 12. If the circle above with center O has an area of
16), then what is the perimeter of ABCO?
(D) x = 1
4 4
(A) 8 + 3
(E) x = 1
2 8
(B) 16 + 3

9. If a > b, c > d, e > f, and f > c, then which of the (C) 16 + 4


following must be true? (D) 16 + 8
(E) 32 + 8
I. a>e
II. e>d
III. e>c

(A) I only
(B) II only
(C) III only
(D) I and III
(E) II and III

10. A store decides to decrease the price of all its


items by 15 percent. After 30 days in order to sell more
items the store again decreases the price of its items by
an additional 15 percent. What was the total decrease
compared to the original price?

(A) 15%
(B) 27.75%
(C) 30%
(D) 70%
(E) 85%

11. A company had profits in 1992 equal to 11


percent of its revenues. If the profits amounted to 22
million dollars, approximately what were the
company’s revenues, in millions of dollars, in 1992?

(A) 100
(B) 122
(C) 150
(D) 200
(E) 220

GO ON TO THE NEXT PAGE


142 © The MBA Center
Problem Solving

MEDIUM
17. Of the following, the closest approximation to
(20 - 90)(201)
is:
10 - 45
13. Forty percent of the pets in a certain apartment
building are dogs. Of the pets which are not dogs, 24
are cats and 18 are birds. If cats, dogs, and birds are (A) 20
the only types of pets in the building, how many pets (B) 40
are in the building? (C) 60
(D) 80
(A) 70 (E) 100
(B) 90
(C) 110
(D) 130 18. A car dealer has a large inventory of cars of
(E) 150 which 25 percent are red, 15 percent are blue, 30
percent are black and the remainder are other colors.
For each color of car, 70 percent are equipped with
14. If (8 + 4 ) (a + 3) = 0, and a * –3, then a = automatic transmission and of these, 20 percent have
a
air-conditioning. What is the probability that a car
(A) –8 chosen at random will not be black and will have both
(B) –2 automatic transmission and air-conditioning?

(C) – 1 (A) 9.8%


2
(B) 14%
(D) 1 (C) 30%
2
(D) 70%
(E) 2 (E) 90.2%

15. The average (arithmetic mean) of the five A B


numbers x, 91, 120, 134, and 220 is what per cent of
the sum of the five numbers?
D C
(A) 20%
(B) 22%
(C) 25%
(D) 30% 19. For the figure above, what is the ratio of the
(E) It cannot be determined from the information area of the larger square to the area of the smaller
given. square?

2
16. If x is the product of three consecutive positive (A) 4
integers, which of the following must be true?
2
I. x is a multiple of 3 (B) 2
II. x is a multiple of 4
III. x is a multiple of 5 (C) 1

(A) I only (D) 2


(B) II only
(C) I and II only (E) 2
(D) I and III only
(E) I, II, and III

GO ON TO THE NEXT PAGE


© The MBA Center 143
Total PrepKit for the GMAT® Step 4 Workshops

20. If x and y are prime numbers, then the product


of x and y could be any of the following EXCEPT

(A) an odd integer


(B) an even integer O
(C) a divisor of 45
(D) equal to 36
(E) less than 5

21. The value of x for which x2 + 6 = 5x 23. If circle O above has area x, what is the length
and x2 + 12 = 7x is, of its radius in terms of x ?

(A) –3 (A) x
(B) –2
(C) 2 x
(D) 3 (B)
(E) 4
x
(C)
22. A chemist starts with a solution that is 10
percent alcohol. After the chemist adds a certain x
amount of pure alcohol, the resulting solution is 25 (D) 2
percent alcohol. By what percent was the volume
increased? x
(E)
(A) 15%
(B) 20%
(C) 50%
(D)120%
(E) 150%

GO ON TO THE NEXT PAGE


144 © The MBA Center
Problem Solving

DIFFICULT
27. If N = 1000x + 100y + 10z, where x, y, and z
are different positive integers less than 4, the
remainder when N is divided by 9 is
24. A swimming pool is in the shape of a
rectangular solid, and its width is 25 meters, its length (A) 2
is 50 meters and its depth is 4 meters. If the pool is half (B) 4
full, approximately how much water, in cubic meters, (C) 6
(D) 8
is needed to make it 7 full? (E) 9
8
(A) 312
(B) 625 28. If x and y are positive integers such that
(C) 1,875
1 > 1 1 2
(D)2,188
3 x + 3 > 9 and 2 < y < 20, then which of
(E) 2,500
the following CANNOT be true?

25. In a certain high school, the ratio of teachers to (A) x+y=3


students is 5 to 108. If 12 new students entered the (B) x+y=4
school, the ratio would change to 5 to 112. How many (C) x–y=3
teachers does the high school have? (D) x–y=4
(E) x2 + y2 = 25
(A) 5
(B) 10
(C) 15
(D) 20
(E) 25

a° b°
26. A store sold a total of 416 toy robots during an
8-day sale and a total of 275 toy trains during an 11-
a
day sale. The average (arithmetic mean) number of toy 29. In the figure above, if a+b = 3 , then a =
4
robots sold per day during the 8-day sale was what
percent greater than the average number of toy trains (A) 45
sold per day during the 11-day sale? (B) 75
(C) 105
(A) 28 (D) 115
(B) 47 (E) 135
(C) 67
(D) 108
(E) 127

GO ON TO THE NEXT PAGE


© The MBA Center 145
Total PrepKit for the GMAT® Step 4 Workshops

30. At a certain store, 1 of the sodas sold in one 32. A bookstore has a shelf that contains
8
biographies which normally sell for $20 each and
week were Pepsis and 1 of the remaining sodas sold mysteries that normally sell for $12 each. During a
3
sale, the biographies and mysteries are discounted at
were Coca Colas. If x of the sodas sold were Coca different rates so that a customer saves a total of $19
from the normal price by buying 5 discounted
Colas, how many were Pepsis? biographies and 3 discounted mysteries. If the sum of
the discount rates for the two types of books is 35
percent, what is the discount rate on mysteries?
3
(A) 8 x
(A) 9
(B) 3 x (B) 10
7
(C) 13
(C) 7 x (D) 22
16
(E) 25
7
(D) 8 x

(E) x

31. If a, b, and c are positive integers and


3a = 3b + 3c, which of the following must be true?

I. a = b
II. b < c
III. a > c

(A) I only
(B) II only
(C) III only
(D) I and III
(E) II and III

STOP
IF YOU FINISH BEFORE TIME IS EXPIRED
YOU MAY CHECK YOUR WORK

146 © The MBA Center


© The MBA Center
Total PrepKit for the GMAT® Step 4 Workshops

PROBLEM SOLVING WORKSHOP


ANSWER KEY

1. D
2. B
3. D
4. A
5. E
6. A
7. D
8. D
9. E
10. B
11. D
12. A
13. A
14. C
15. A
16. A
17. A
18. A
19. E
20. D
21. D
22. B
23. B
24. C
25. C
26. D
27. C
28. D
29. E
30. B
31. C
32. E

148 © The MBA Center


Step 5
Data Sufficiency

© The MBA Center


Total PrepKit for the GMAT® Step 5.1 Lesson

GENERAL OUTLINE

Approximately one-third of the questions in the Quantitative


Section of the GMAT CAT are Data Sufficiency questions. These
questions test your ability to think quantitatively, distinguish
between relevant and irrelevant data, and determine whether or not
you have sufficient information to answer a given question. This
lesson introduces Data Sufficiency questions and provides a syste-
matic method for solving them.

THE FOLLOWING TOPICS WILL BE COVERED IN THIS LESSON:

Introduction To Data Sufficiency


The Two Types Of Data Sufficiency Questions
The MBA Center Approach To Data Sufficiency Questions
MBA Center Strategies For Data Sufficiency Questions
Common Data Sufficiency Traps

KEY TERMS

Question: The heart of every data sufficiency problem is the initial


question. You must determine whether, based on the information provided
in the statements, you have information sufficient to answer the initial
question.

Statement: Each Data Sufficiency problem includes two numbered


statements which provide information related to the initial question. The
information in the statements is sufficient only if it allows you to answer
the initial question.

Determine-the-Quantity Question: This type of Data Sufficiency


question asks you to determine the value of a quantity. The information in
the statements is sufficient to answer the question only if it allows you to
determine one discrete value for the quantity.

Yes-or-No Question: As the name implies, this type of question requires


either a yes or a no answer. The information in the statements is sufficient
only if it allows you to answer the question definitely yes or definitely no.

150 © The MBA Center


Data Sufficiency

READY…

Data Sufficiency problems are combined with Problem Solving questions


in the Quantitative Section of the GMAT CAT. You should expect that
approximately 14 of the 37 questions in the Quantitative Section will be
Data Sufficiency problems. Like Problem Solving questions, Data Sufficiency
problems test your mastery of basic math concepts and your ability to
reason quantitatively. In addition, Data Sufficiency problems test your
ability to distinguish between relevant and irrelevant data and determine
whether you have enough information to answer a given question. In
general, Data Sufficiency problems require fewer calculations than Problem
Solving questions. Therefore, the ability to work with numbers and perform
calculations is less important for Data Sufficiency problems than it is for
Problem Solving questions.

Data Sufficiency covers the same math topics as Problem Solving:

1. Arithmetic
2. Algebra
3. Geometry
4. Word Problems

Data Sufficiency problems consist of a question followed by two


statements. You must determine whether the information provided in the
statements, either individually or combined, is sufficient to answer the
question.

Here is a typical Data Sufficiency question:


Score Value: 500

01:15 GMAT CAT – Section 3 : Quantitative 1 of 37

What is the area of a rectangular lawn?

(1) 75 percent of the lawn is mowed.

(2) A rectangular region, 10 meters by 12 meters, is mowed,


and the rest of the lawn is unmowed.

(A) Statement (1) ALONE is sufficient to answer the question,


but statement (2) alone is not sufficient.
(B) Statement (2) ALONE is sufficient to answer the question,
but statement (1) alone is not sufficient.
(C) Statements (1) and (2) COMBINED are sufficient to answer
the question, but neither statement alone is sufficient.
(D) Each of statements (1) and (2) is sufficient to answer the
question.
(E) Statements (1) and (2), alone or combined, are not
sufficient to answer the question.

Test Section Answer


Quit Exit Time Help Confirm Next

© The MBA Center 151


Total PrepKit for the GMAT® Step 5.1 Lesson

This question does not require you to determine the area of the lawn.
Rather you must determine whether the information given to you in the
statements, alone or combined, is sufficient to determine the area.

Statement (1) is clearly not sufficient to answer the question.


Knowing that 75% of the lawn is mowed is not sufficient to
determine the size of the entire lawn because you don’t know the area
of the mowed region. Likewise, statement (2) is not sufficient to
determine the size of the lawn. If you know that a 10-by-12 (or 120
square meters) region of the lawn is mowed, you know that the area
Sufficient is sufficient of the lawn is greater than 120 square meters, but you can’t determine
If you determine the area of
the lawn, you’ve spent too the total area of the lawn.
much time on the question.
You only need to determine If you combine statements (1) and (2) you can determine the area
whether you have enough of the lawn, so the third answer choice is correct. Statement (1) tells
information to calculate the
area of the lawn. you that a region representing 75% of the area of the lawn is mowed
and statement (2) gives you the area (10 by 12, or 120 square meters)
of that region. If you let x equal the total area of the lawn, then you
can determine the area of the lawn from the equation 75% (or .75) x
x = 160. So the total area of the lawn is 160 square meters.

Authors Note: On the GMAT CAT, the answer choices will be preceded by
hollow ovals. You indicate your choice by clicking on the appropriate oval using
the mouse. In this chapter and in the rest of this book, however, the answer choices
will be designated by the letters A, B, C, D, and E for ease of reference.

152 © The MBA Center


Data Sufficiency

THE DIRECTIONS
The directions for the Data Sufficiency questions are similar to the
following:

GMAT CAT – Section 3 : Quantitative Directions

End


When finished
Directions: Each data sufficiency prob- reading
lem consists of a question followed by two directions
statements containing data related to the click on the
question. You must determine whether the icon below
information provided in the statements, either
individually or combined, is sufficient to
answer the question given. Using the
information in the statements, plus your
knowledge of mathematics and commonly Dismiss
known facts (such as the number of seconds Directions
in a minute), you must indicate whether

(A) Statement (1) ALONE is sufficient to


answer the question, but statement (2)
alone is not sufficient
(B) Statement (2) ALONE is sufficient to
answer the question, but statement (1)
alone is not sufficient
(C) Statements (1) and (2) COMBINED are
sufficient to answer the question, but
neither statement alone is sufficient
(D) Each of statements (1) and (2) is
sufficient to answer the question
(E) Statements (1) and (2), alone or com-
bined, are not sufficient to answer the
question

Numbers: All numbers used are real


numbers.
Figures: Figures accompanying data
sufficiency problems will accurately reflect the
information provided in the question, but will
not necessarily reflect the additional
information given in statements (1) and (2).
All figures lie in a plane unless otherwise
stated. Straight lines may appear jagged on
the computer screen.

In data sufficiency questions that ask for a


value (what is the value of x?, for example),
the information provided in the statements is
sufficient only if it allows you to determine
exactly one numerical value.

Test Section Answer


Quit Exit Time Help Confirm Next

© The MBA Center 153


Total PrepKit for the GMAT® Step 5.1 Lesson

WHAT THE DIRECTIONS MEAN

The directions first describe the form of the Data Sufficiency questions:
a question (which is often accompanied by information or data) followed
by two statements. Note that the directions do not indicate that you need
to answer the question. You don’t. You are only required to determine
whether the statements are sufficient to answer the question. The directions
also indicate that you must use your knowledge of mathematics in order to
determine if the information in the statements is sufficient. The math
required by the Data Sufficiency questions is the same basic math required
for the Problem Solving questions, all of which is covered in the
Comprehensive Math Review in this manual.

With respect to figures, the directions indicate that those accompanying


Data Sufficiency questions will reflect information given in the question,
but will not necessarily reflect the information given in the statements. This
simply means that, to the extent information provided in one or both of the
statements seems to disagree with a figure, you should use the information
in the statement and disregard the figure.

The Data Sufficiency Finally, and perhaps most importantly, the directions indicate that when
Answer Choices a problem asks for the value of a quantity (“What is the value of x?” for
Note that the Data Sufficiency
answer choices are the same example), the information in the statements is sufficient only if it allows
for every Data Sufficiency you to determine exactly one value for the quantity. For example, if one of
question. Memorize them the statements tells you that x2 = 25, this is not sufficient to determine the
now so that you will not have value of x because x could be 5 or –5.
to waste time reading them
on the actual test.

THE RELATIONSHIP BETWEEN DATA SUFFICIENCY AND PROBLEM


SOLVING QUESTIONS
Data Sufficiency questions are similar to Problem Solving questions in
several ways. Both types of questions test your mastery of basic math
concepts and your ability to think quantitatively, and both require you to
see the relationship between given data. Data Sufficiency questions are,
however, different in that you do not need to calculate an answer. Rather,
you must determine how information given in the statements is related to
the question and whether it is sufficient to answer the question. Take a look
at Example 1 in the form of a Problem Solving question.

Revisited Score Value: 500

01:15 GMAT CAT – Section 3 : Quantitative 1 of 37

If 75 percent of a rectangular lawn is mowed, and the part that


is mowed is a rectangular region of 10 meters by 12 meters, then
what is the area, in square meters, of the lawn?

(A) 90
(B) 120
(C) 160
(D) 250
(E) 300

Test Section Answer


Quit Exit Time Help Confirm Next

154 © The MBA Center


Data Sufficiency

Note that you are provided with the same information as you were when
this problem was presented as a Data Sufficiency question, but in this exam-
ple you must actually calculate an answer. Another important difference
between Problem Solving and Data Sufficiency questions is that you know
that the information provided in Problem Solving questions is sufficient to
determine the answer, but in Data Sufficiency questions it is possible that
the information provided in both statements is not sufficient.

THE FORMAT — INFORMATION PROVIDED IN THE QUESTION


In Data Sufficiency problems you are often provided with information in
the question itself. You can assume any information provided in the question
is true when you examine the statements.
Take a look at the following examples:
A Score Value: 500

01:13 GMAT CAT – Section 3 : Quantitative 2 of 37


What is the value of x ?

(1) x > 13

(2) x < 19

(A) Statement (1) ALONE is sufficient to answer the question, but statement (2) alone
is not sufficient
(B) Statement (2) ALONE is sufficient to answer the question, but statement (1) alone
is not sufficient
(C) Statements (1) and (2) COMBINED are sufficient to answer the question, but
neither statement alone is sufficient
(D) Each of statements (1) and (2) is sufficient to answer the question
(E) Statements (1) and (2), alone or combined, are not sufficient to answer the question

Test Section Answer


Quit Exit Time Help Confirm Next

B
Score Value: 500

01:13 GMAT CAT – Section 3 : Quantitative 3 of 37


What is the value of the prime number x ?

(1) x > 13

(2) x < 19

(A) Statement (1) ALONE is sufficient to answer the question, but statement (2) alone
is not sufficient
(B) Statement (2) ALONE is sufficient to answer the question, but statement (1) alone
is not sufficient
(C) Statements (1) and (2) COMBINED are sufficient to answer the question, but
neither statement alone is sufficient
(D) Each of statements (1) and (2) is sufficient to answer the question
(E) Statements (1) and (2), alone or combined, are not sufficient to answer the question

Test Section Answer


Quit Exit Time Help Confirm Next

© The MBA Center 155


Total PrepKit for the GMAT® Step 5.1 Lesson

Notice the difference? In Example 2(a) the information provided


in the statements is not sufficient, even combined, to determine the
value of x because x could be any value greater than 13 and less than
19. In Example 2(b) you are told in the question that x is a prime
number. You can therefore assume this fact when you analyze the
information provided in the statements. Statement (1) tells you that
x is greater than 13, so you know that x is a prime number greater
than 13. Likewise, statement (2) tells you that x is less than 19, so you
know that x is a prime number less than 19. Alone, neither of the
statements is sufficient to determine the value of x. But if you know
that x is a prime number between 13 and 19, you can determine that
x is in fact equal to 17. Therefore, combined, the information in the
statements is sufficient to determine the value of x.

Here are two additional examples of Data Sufficiency questions


with information provided in the question:

Score Value: 550

01:11 GMAT CAT – Section 3 : Quantitative 3 of 37


What is the value of integer x ?

(1) x2 < 13

(2) x4 > 60

(A) Statement (1) ALONE is sufficient to answer the question, but statement (2) alone
is not sufficient
(B) Statement (2) ALONE is sufficient to answer the question, but statement (1) alone
is not sufficient
(C) Statements (1) and (2) COMBINED are sufficient to answer the question, but
neither statement alone is sufficient
(D) Each of statements (1) and (2) is sufficient to answer the question
(E) Statements (1) and (2), alone or combined, are not sufficient to answer the question


Test Section Answer
Quit Exit Time Help Confirm Next

Score Value: 600

01:09 GMAT CAT – Section 3 : Quantitative 4 of 37


What is the value of positive integer x ?

(1) x2 < 13

(2) x4 > 60

Test Section Answer


Quit Exit Time Help Confirm Next

156 © The MBA Center


Data Sufficiency

THE TWO TYPES OF DATA SUFFICIENCY QUESTIONS


There are two types of Data Sufficiency Questions: (1) questions that ask
you to determine the value of a quantity and (2) yes-or-no questions. For
questions that ask the value of a quantity, the information in the statements
is sufficient only if you are able to determine one and only one value for
that quantity. For yes-or-no questions the information in the statements is
sufficient only if you are able to answer the questions definitely “yes” or
definitely “no.”

Take a look at the following examples:

Score Value: 550

01:07 GMAT CAT – Section 3 : Quantitative 5 of 37


If x and y are positive integers, what is the value of x + y ?

(1) x2 = y

(2) x > 16

(A) Statement (1) ALONE is sufficient to answer the question, but statement (2) alone
is not sufficient
(B) Statement (2) ALONE is sufficient to answer the question, but statement (1) alone
is not sufficient
(C) Statements (1) and (2) COMBINED are sufficient to answer the question, but
neither statement alone is sufficient
(D) Each of statements (1) and (2) is sufficient to answer the question
(E) Statements (1) and (2), alone or combined, are not sufficient to answer the question

Test Section Answer


Quit Exit Time Help Confirm Next

Or...
Score Value: 550

01:05 GMAT CAT – Section 3 : Quantitative 6 of 37


If x and y are positive integers, is x + y less than 20 ?

(1) x2 = y

(2) x > 16

Test Section Answer


Quit Exit Time Help Confirm Next

See the difference? Example 5 asks you to determine the value of


x + y, so you need to determine one and only one value for x + y. Alone,
statement (1) is not sufficient because x and y could be many different
values (2 and 4, or 3 and 9, for example). Statement (2) is also not

© The MBA Center 157


Total PrepKit for the GMAT® Step 5.1 Lesson

sufficient because it doesn’t give a value for y and it only indicates that
x is greater than 16. Combined, the statements are not sufficient. For
example, x and y could be, respectively, 5 and 25, or 6 and 36, giving
two different values for x + y.

Example 6 is a yes-or-no question. You need only determine


whether the answer to the question “is x + y less than 20?” is definitely
“yes” or definitely “no.” Statement (1) is not sufficient because x+y
could be less than 20, but it could also be greater than 20. For
example, if x and y are 3 and 9, then x + y is less than 20, but if x and
y are 5 and 25, then x + y is not less than 20. Statement (2) is also not
Don’t forget that for yes-or- sufficient because it doesn’t give a value for y and it only indicates
no Data Sufficiency problems that x is greater than 16. Combined, the statements are sufficient. If
a “no”answer is just as good you know that x is greater than 16, then from statement (1) you know
as a “yes” answer.
that y is greater than 256, so you know that x + y is greater than 20.
This time you can answer the question positively “no” based on
combined information.

Look at these examples of yes-or-no and quantity Data Sufficiency


questions.
Score Value: 550

01:03 GMAT CAT – Section 3 : Quantitative 7 of 37


If x, y, and z are positive integers, what is the average
(arithmetic mean) of x, y, and z ?

(1) x + y = 20

(2) y + z = 30

(A) Statement (1) ALONE is sufficient to answer the question, but statement (2) alone
is not sufficient
(B) Statement (2) ALONE is sufficient to answer the question, but statement (1) alone
is not sufficient
(C) Statements (1) and (2) COMBINED are sufficient to answer the question, but
neither statement alone is sufficient
(D) Each of statements (1) and (2) is sufficient to answer the question
(E) Statements (1) and (2), alone or combined, are not sufficient to answer the question

Test Section Answer


Quit Exit Time Help Confirm Next

Or... Score Value: 550

01:01 GMAT CAT – Section 3 : Quantitative 8 of 37

If x, y, and z are positive integers, is the average (arithmetic


mean) of x, y, and z greater than 20 ?

(1) x + y = 20
(2) y + z = 30

Test Section Answer


Quit Exit Time Help Confirm Next

158 © The MBA Center


Data Sufficiency

Score Value: 600

00:59 GMAT CAT – Section 3 : Quantitative 9 of 37


Triangle ABC is inscribed in a circle.
What is the area of triangle ABC ? C B
(1) Triangle ABC is an equilateral triangle.

(2) The perimeter of triangle ABC is 9.

(A) Statement (1) ALONE is sufficient to answer the question, but statement (2) alone
is not sufficient
(B) Statement (2) ALONE is sufficient to answer the question, but statement (1) alone
is not sufficient
(C) Statements (1) and (2) COMBINED are sufficient to answer the question, but
neither statement alone is sufficient
(D) Each of statements (1) and (2) is sufficient to answer the question
(E) Statements (1) and (2), alone or combined, are not sufficient to answer the question


Test Section Answer
Quit Exit Time Help Confirm Next

Or... Score Value: 600

00:57 GMAT CAT – Section 3 : Quantitative 10 of 37

A

Triangle ABC is inscribed in a circle.


Is the area of triangle ABC greater
than 40 percent of the area
of the circle? C B
(1) Triangle ABC is an equilateral triangle.

(2) The perimeter of triangle ABC is 9.

(A) Statement (1) ALONE is sufficient to answer the question, but statement (2) alone
is not sufficient
(B) Statement (2) ALONE is sufficient to answer the question, but statement (1) alone
is not sufficient
(C) Statements (1) and (2) COMBINED are sufficient to answer the question, but
neither statement alone is sufficient
(D) Each of statements (1) and (2) is sufficient to answer the question
(E) Statements (1) and (2), alone or combined, are not sufficient to answer the question

Test Section Answer


Quit Exit Time Help Confirm Next

© The MBA Center 159


Total PrepKit for the GMAT® Step 5.1 Lesson

THE CHALLENGE

As with Problem Solving questions, the challenge with Data Sufficiency


questions is time. In order to finish all 37 questions in the Quantitative
Section you will need to move quickly and efficiently through all of the
Data Sufficiency questions presented. Because you are not required to
calculate answers, Data Sufficiency questions should require less time than
Problem Solving questions. Even so, Data Sufficiency questions are often
complex and generally require a fair amount of thought and a bit of
intuition. For this reason the MBA Center method stresses a systematic
approach to every Data Sufficiency question.

160 © The MBA Center


Data Sufficiency

AIM…
THE DIFFICULTY WITH DATA SUFFICIENCY
The two principal mistakes test takers make on Data Sufficiency
problems are (1) failing to see relationships between the statements and the
question and (2) seeing relationships that don’t exist. From years of
experience writing Data Sufficiency problems, Phineas knows the average
test taker’s weaknesses and designs questions with traps and pitfalls that
exploit those weaknesses. Here are some of the mistakes that average test
takers make.

COMMON MISTAKE 1 — MISINTERPRETING DATA


Score Value: 620

00:55 GMAT CAT – Section 3 : Quantitative 11 of 37

What is the weight of a shipment of watermelons?

(1) There are exactly 120 watermelons in the shipment.

(2) Each watermelon weighs at least 3 pounds.

(A) Statement (1) ALONE is sufficient to answer the question, but statement (2) alone
is not sufficient
(B) Statement (2) ALONE is sufficient to answer the question, but statement (1) alone
is not sufficient
(C) Statements (1) and (2) COMBINED are sufficient to answer the question, but
neither statement alone is sufficient
(D) Each of statements (1) and (2) is sufficient to answer the question
(E) Statements (1) and (2), alone or combined, are not sufficient to answer the question

Test Section Answer


Quit Exit Time Help Confirm Next

The average test taker knows that statement (1) is not sufficient
because there is no mention of the weight of the watermelons, and he
knows that statement (2) is not sufficient because there is no mention
of how many watermelons are in the shipment. Now, the average test
taker might think, “if I multiply the quantity (120) by the minimum
weight (3) I can determine the gross weight, so the answer must be (C).”
This is wrong. To get the gross weight, you need to multiply the
quantity by the average weight, which is not given to you in the
problem.

© The MBA Center 161


Total PrepKit for the GMAT® Step 5.1 Lesson

COMMON MISTAKE 2 —
SEEING RELATIONSHIPS BETWEEN THE STATEMENTS AND THE
QUESTION THAT DON’T EXIST
Score Value: 580

00:53 GMAT CAT – Section 3 : Quantitative 12 of 37


If a machine produces 25 units per minute, how many units
did it produce in time period T?

(1) The machine was started at 6 a.m. and stopped at 11 a.m.

(2) The machine operated for 5 hours.

(A) Statement (1) ALONE is sufficient to answer the question, but statement (2) alone
is not sufficient
(B) Statement (2) ALONE is sufficient to answer the question, but statement (1) alone
is not sufficient
(C) Statements (1) and (2) COMBINED are sufficient to answer the question, but
neither statement alone is sufficient
(D) Each of statements (1) and (2) is sufficient to answer the question
(E) Statements (1) and (2), alone or combined, are not sufficient to answer the question


Test Section Answer
Quit Exit Time Help Confirm Next

The average test taker might think that the correct answer is (D)
because both statement (1) and statement (2) provide the length of
time period T and, since you know the rate at which the machine is
producing units, determining the number of units produced in 5
hours seems a matter of simple arithmetic. This is wrong. Neither of
the statements mentions time period T. Regarding statement (1) you
can’t assume that the time period given is equal to time period T. This
is also true with respect to statement (2).

162 © The MBA Center


Data Sufficiency

COMMON MISTAKE 3 — FAILING TO SEE RELATIONSHIPS THAT DO


EXIST
Score Value: 670

00:51 GMAT CAT – Section 3 : Quantitative 13 of 37

Is the odd integer x a prime number?

(1) x + 2 is a prime number.

(2) x – 3 is a prime number.

(A) Statement (1) ALONE is sufficient to answer the question, but statement (2) alone
is not sufficient
(B) Statement (2) ALONE is sufficient to answer the question, but statement (1) alone
is not sufficient
(C) Statements (1) and (2) COMBINED are sufficient to answer the question, but
neither statement alone is sufficient
(D) Each of statements (1) and (2) is sufficient to answer the question
(E) Statements (1) and (2), alone or combined, are not sufficient to answer the question

Test Section Answer


Quit Exit Time Help Confirm Next

The average test taker will probably see that statement (1) is not
sufficient on its own to answer the question. Moving on to statement
(2), the average test taker will probably think that it too is not
sufficient, failing to notice that if x is an odd integer then x – 3 must
be an even integer. Statement (2) tells us that x – 3 is prime, so x – 3
must be equal to 2 because it is the only even prime number. If we
know that x – 3 is equal to 2, then we know that x is equal to 5.

HOW TO ACE DATA SUFFICIENCY


The three examples above illustrate the most common mistakes test
takers make on Data Sufficiency problems. In order to help you avoid these
traps we have developed a systematic three-step approach that you should
use for every question.

THE MBA CENTER METHOD FOR DATA SUFFICIENCY QUESTIONS

The MBA Center method for Data Sufficiency questions emphasizes a


step-by-step approach to every question. If you work systematically through
each statement, eliminating wrong answer choices as you go, you will save
time, avoid common traps, and (if you end up having to guess) improve
your chances of guessing the correct answer. Practice using this method
when you work through the examples in this lesson and when you do the
Data Sufficiency practice exercises in this book.

© The MBA Center 163


Total PrepKit for the GMAT® Step 5.1 Lesson

Step 1: Read and understand the question before you move on to the
statements.

First read and understand the question. Before you move on to the
statements ask yourself what information you need in order to answer the
question. For example, if the question asks “What is the value of x?”, then you
need an equation or expression that will allow you to determine a single
value for x. But if the question asks “What is the value of x + y?” or “What is
the average of x and y?”, then you need the values of both x and y or some
equation or expression that will allow you to determine the sum of x and y.

Take a look at the following examples of Data Sufficiency questions


without statements.
Try to determine what information would allow you to answer the
questions.

00:49 GMAT CAT – Section 3 : Quantitative 14 of 37


If x, y, and z are consecutive positive integers and x < y < z,
what is the value of y ?


Test Section Answer
Quit Exit Time Help Confirm Next

00:47 GMAT CAT – Section 3 : Quantitative 15 of 37


If x + 2y + 3z = 50, what is the value of x ?

Test Section Answer


Quit Exit Time Help Confirm Next

Step 2: Examine the statements separately, eliminating wrong answer


choices.

After you read the question and have some idea what information would
be sufficient to answer it, move on to the statements. Pick the statement
that looks easier and start with it. If that statement alone is sufficient to
answer the question, you can eliminate answer choices (C) and (E). If the
statement is insufficient, you can eliminate answer choices (A) or (B)
(depending on which statement you started with), and (D).

Next, move on to the other statement. It is important that you disregard


all information from the previous statement. For example, if you started
with statement (1) and determined that it was insufficient, make sure that
you don’t unintentionally use information contained in statement (1) when
you examine statement (2).

164 © The MBA Center


Data Sufficiency

The following flow chart illustrates how you should eliminate


wrong answer choices when you examine the statements.

If you start with statement 1:

STATEMENT 1 STATEMENT 2

Sufficient Answer D
Sufficient Eliminate B, C, E
Insufficent Answer A

Sufficient Answer B
Insufficient Eliminate A, D
Insufficent Eliminate B

Or if you start with statement 2:

STATEMENT 2 STATEMENT 1

Sufficient Answer D
Sufficient Eliminate A, C, E
Insufficent Answer B

Sufficient Answer A
Insufficient Eliminate B, D
Insufficent Eliminate A

The above flow chart illustrates how you should eliminate incorrect
answer choices as you examine the statements. If, for example, you know
that statement (1) alone is sufficient, but you are unsure whether statement
(2) alone is sufficient, you can eliminate answer choices (B), (C), and (E), Eliminating some wrong
which gives you a fifty percent chance of guessing the correct answer. answer choices is better than
eliminating none - it also
improves your chances of
Step 3: If necessary, consider the statements together. guessing the correct answer.

You should only reach this step if you determine that each statement
alone is insufficient. If either statement alone is sufficient to answer the
question, you do not need to consider the statements together. At this
point, the only possible answer choices are (C) and (E). Consider all of the
information in the question and in the statements as one large problem and
ask yourself if you have sufficient information to answer the question. Also,
try to determine if information in one statement makes information in the
other statement more useful. Take a look at the following example.

© The MBA Center 165


Total PrepKit for the GMAT® Step 5.1 Lesson

Score Value: 650

00:41 GMAT CAT – Section 3 : Quantitative 16 of 37

If x > 0 and y > 0, what is the value of x ?


(1) xy = z –y
(2) z=9

(A) Statement (1) ALONE is sufficient to answer the question, but statement (2) alone
is not sufficient
(B) Statement (2) ALONE is sufficient to answer the question, but statement (1) alone
is not sufficient
(C) Statements (1) and (2) COMBINED are sufficient to answer the question, but
neither statement alone is sufficient
(D) Each of statements (1) and (2) is sufficient to answer the question
(E) Statements (1) and (2), alone or combined, are not sufficient to answer the question


Test Section Answer
Quit Exit Time Help Confirm Next

Note that the information contained in each of statements (1) and


(2) is insufficient alone. But from statement (2) you know that z = 9, so
you can substitute 9 for z in statement (1), which gives you the

equation xy = 9-y, or, xy = 1y which allows you to determine that x = 1 .


9 9

166 © The MBA Center


Data Sufficiency

FIRE!
DATA SUFFICIENCY STRATEGIES

INSERTING NUMBERS
Inserting numbers for unknowns or variables can be as useful for solving
Data Sufficiency questions as it is for solving Problem Solving questions.
Any number property Data Sufficiency question (is x prime?, is x an odd
integer?, is x + y an even integer?, is x a multiple of 6?, for example) is a good
candidate for inserting numbers. If substituting different numbers for the
variables in a statement gives you two different answers to the question,
then you know that the statement is not sufficient to answer the question.
Try inserting numbers for variables in the following example.
Score Value: 550

00:39 GMAT CAT – Section 3 : Quantitative 17 of 37

Is the positive integer x a multiple of 48 ?

(1) x is a multiple of 16.

(2) x is a multiple of 6.

(A) Statement (1) ALONE is sufficient to answer the question, but statement (2) alone
is not sufficient
(B) Statement (2) ALONE is sufficient to answer the question, but statement (1) alone
is not sufficient
(C) Statements (1) and (2) COMBINED are sufficient to answer the question, but
neither statement alone is sufficient
(D) Each of statements (1) and (2) is sufficient to answer the question
(E) Statements (1) and (2), alone or combined, are not sufficient to answer the question

Test Section Answer


Quit Exit Time Help Confirm Next

Start with statement (1). You can pick the value 16 for x, which is
a multiple of 16 but is not a multiple of 48. But you can also pick 48
as a value for x, which is a multiple of both 16 and 48. Because you
are able to pick two values for x which satisfy statement (1) and give
you two different answers to the question “is x a multiple of 48?” you
know that statement (1) is not sufficient. Now move on to statement
(2). It is also not sufficient because you can pick the values 6 or 48 for
x, again giving you two different answers to the question “is x a
multiple of 48?” Now you can combine the statements. The lowest
number you can pick for x that is a multiple of both 16 and 6 is 48. If
you continue to think of larger and larger values for x that are
multiples of both 16 and 6, you will see that they are all multiples of
48, so statements (1) and (2) combined are sufficient to answer the
question.

© The MBA Center 167


Total PrepKit for the GMAT® Step 5.1 Lesson

Try inserting numbers in the following example.


Score Value: 530

00:37 GMAT CAT – Section 3 : Quantitative 18 of 37


Is the integer m odd?

(1) m is a multiple of both 3 and 7.

(2) m – 7 is an odd integer.

(A) Statement (1) ALONE is sufficient to answer the question, but statement (2) alone
is not sufficient
(B) Statement (2) ALONE is sufficient to answer the question, but statement (1) alone
is not sufficient
(C) Statements (1) and (2) COMBINED are sufficient to answer the question, but
neither statement alone is sufficient
(D) Each of statements (1) and (2) is sufficient to answer the question
(E) Statements (1) and (2), alone or combined, are not sufficient to answer the question


Test Section Answer
Quit Exit Time Help Confirm Next

Try counting variables and equations.

Remember that in order to solve for variables algebraically, you need as


many equations as you have variables. For some Data Sufficiency questions
you can simply count the number of variables you have and the number of
equations you have in order to determine if there is information sufficient
to answer the question. Take a look at the following example.

Score Value: 550

00:35 GMAT CAT – Section 3 : Quantitative 19 of 37


How many coins does Kelly have?

(1) John has twice as many coins as Kelly.

(2) If Kelly had four times as many coins as she currently


has, then she would have ten more coins than John.

(A) Statement (1) ALONE is sufficient to answer the question, but statement (2) alone
is not sufficient
(B) Statement (2) ALONE is sufficient to answer the question, but statement (1) alone
is not sufficient
(C) Statements (1) and (2) COMBINED are sufficient to answer the question, but
neither statement alone is sufficient
(D) Each of statements (1) and (2) is sufficient to answer the question
(E) Statements (1) and (2), alone or combined, are not sufficient to answer the question

Test Section Answer


Quit Exit Time Help Confirm Next

168 © The MBA Center


Data Sufficiency

Make sure when you are


What are the equations contained in the statements? If you let j counting equations that you
equal the number of coins John has and k equal the number of coins actually have different equa-
Kelly has, then statement (1) yields the equation j = 2k. So you have tions. For example, if state-
two variables, but only one equation. So statement (1) alone is not ment (1) gives you the
equation x + 4y = 4 and
sufficient. Statement (2) yields the equation 4k = j + 10. Again, statement (2) gives you the
statement (2) gives you one equation and two variables, so it is not equation 2x – 8 = –8y, you
sufficient alone. If you combine statements (1) and (2), you have two can’t solve for either x or y
equations and two variables, so you can solve for either variable. because the equation in
statement (2) is the same as
the equation in statement (1),
simply written in a different
Try counting variables and equations in the following examples. form.

Score Value: 600

00:33 GMAT CAT – Section 3 : Quantitative 20 of 37


What is the length of side a ? a c

(1) a2 + b2 = 25

(2) b2 + c2 = 41
b
(A) Statement (1) ALONE is sufficient to answer the question, but statement (2) alone
is not sufficient
(B) Statement (2) ALONE is sufficient to answer the question, but statement (1) alone
is not sufficient
(C) Statements (1) and (2) COMBINED are sufficient to answer the question, but
neither statement alone is sufficient
(D) Each of statements (1) and (2) is sufficient to answer the question
(E) Statements (1) and (2), alone or combined, are not sufficient to answer the question

Test Section Answer


Quit Exit Time Help Confirm Next

Score Value: 530

00:31 GMAT CAT – Section 3 : Quantitative 21 of 37


If n + x = y, what is the value of x ? Sometimes you do not need


to solve for each variable
(1) n – x = 20 separately. For example, if a
question asks you to deter-
(2) y–x=n mine the value of 3x + 4y,

then knowing that 12x + 16y =


7 would be sufficient to
answer the question.
Test Section Answer
Quit Exit Time Help Confirm Next

© The MBA Center 169


Total PrepKit for the GMAT® Step 5.1 Lesson

DATA SUFFICIENCY TRAPS


In Problem Solving questions, Phineas tries to fool the unwary test taker
with tempting wrong answer choices. Likewise, in Data Sufficiency there are
traps you should be aware of. The following are some of the most common
traps in Data Sufficiency questions.

SOMETIMES IT IS NECESSARY TO CALCULATE


Even though Data Sufficiency questions do not usually require you to do
any calculations, occasionally it is necessary to do so. Try the following
example:

Score Value: 600

00:29 GMAT CAT – Section 3 : Quantitative 22 of 37


What is the value of x ?

(1) x2 + 25 = 10x

(2) x2 – 8x = –15

(A) Statement (1) ALONE is sufficient to answer the question, but statement (2) alone
is not sufficient
(B) Statement (2) ALONE is sufficient to answer the question, but statement (1) alone
is not sufficient
(C) Statements (1) and (2) COMBINED are sufficient to answer the question, but
neither statement alone is sufficient
(D) Each of statements (1) and (2) is sufficient to answer the question
(E) Statements (1) and (2), alone or combined, are not sufficient to answer the question

Test Section Answer



Quit Exit Time Help Confirm Next

170 © The MBA Center


Data Sufficiency

REDUNDANCY IN THE STATEMENTS

Often one of the statements will provide you with all or some of the
same information contained in the other statement. Phineas does this
because an unwary test taker may become confused and assume that because
one statement is sufficient, the other statement, which contains some of the
same information, is also sufficient. Take a look at the following example.

Score Value: 500

00:27 GMAT CAT – Section 3 : Quantitative 23 of 37

Mike leaves Town A at 8:00 a.m. and drives at a constant


speed to Town B. At what time will Mike arrive at Town B?

(1) Mike passes Town C, which is midway between


Town A and Town B, at 10:00 a.m.

(2) Mike passes Town C two hours after leaving Town A.

(A) Statement (1) ALONE is sufficient to answer the question, but statement (2) alone
is not sufficient
(B) Statement (2) ALONE is sufficient to answer the question, but statement (1) alone
is not sufficient
(C) Statements (1) and (2) COMBINED are sufficient to answer the question, but
neither statement alone is sufficient
(D) Each of statements (1) and (2) is sufficient to answer the question
(E) Statements (1) and (2), alone or combined, are not sufficient to answer the question

Test Section Answer


Quit Exit Time Help Confirm Next

© The MBA Center 171


Total PrepKit for the GMAT® Step 5.1 Lesson

ASSUMPTION TRAPS
Many Data Sufficiency questions are written in such a way that the
average test taker will assume information that isn’t provided. Here are
some of the most common assumption traps on Data Sufficiency questions.

Score Value: 550

00:23 GMAT CAT – Section 3 : Quantitative 24 of 37

From January to February the number of full-time


technicians at Company X increased by what percent?

(1) In January, Company X had 200 employees, 25 percent of whom


were full-time technicians.

(2) In February, 28 percent of the employees at Company X were full-


time technicians.

(A) Statement (1) ALONE is sufficient to answer the question, but statement (2) alone
is not sufficient
(B) Statement (2) ALONE is sufficient to answer the question, but statement (1) alone
is not sufficient
(C) Statements (1) and (2) COMBINED are sufficient to answer the question, but
neither statement alone is sufficient
(D) Each of statements (1) and (2) is sufficient to answer the question


(E) Statements (1) and (2), alone or combined, are not sufficient to answer the question

Test Section Answer


Quit Exit Time Help Confirm Next

172 © The MBA Center


Data Sufficiency

COMMON MATH TRAPS

Certain math traps appear frequently in Data Sufficiency questions. For


example, if a statement indicates that x2 = 25, don’t forget that x could be
either 5 or –5. Here is a list things you should remember in order to avoid
some of the most common GMAT math traps:

The square root of a number can be either positive or negative.


A quadratic equation can have two, one, or zero solutions.
An even power of a nonzero number is always positive. (If x is not
equal to 0, and y is an even number, then xy is a positive number
even if x is a negative number).
An odd power of a nonzero number can be either positive or
negative. If x is not equal to 0, and y is an odd number, then xy is a
positive number if x is positive and a negative number if x is
negative).
The product of an integer (odd or even) and an even integer is always
an even integer.
The sum of any two odd numbers will always be even.
The sum of any two even numbers will always be even.
The sum of an even number and an odd number will always be odd.
2 is the smallest prime number and it is the only even prime number.

Here are several examples of Data Sufficiency questions containing


common math traps:
Score Value: 550

00:21 GMAT CAT – Section 3 : Quantitative 25 of 37

Is x2 > x3 ?

(1) x xx

(2) x2 < x4

(A) Statement (1) ALONE is sufficient to answer the question, but statement (2) alone
is not sufficient
(B) Statement (2) ALONE is sufficient to answer the question, but statement (1) alone
is not sufficient
(C) Statements (1) and (2) COMBINED are sufficient to answer the question, but
neither statement alone is sufficient
(D) Each of statements (1) and (2) is sufficient to answer the question
(E) Statements (1) and (2), alone or combined, are not sufficient to answer the question

Test Section Answer


Quit Exit Time Help Confirm Next

© The MBA Center 173


Total PrepKit for the GMAT® Step 5.1 Lesson

Score Value: 550

00:19 GMAT CAT – Section 3 : Quantitative 26 of 37


If a, b, and c are all positive integers, is abc an even number?

(1) a + b is a prime number.

(2) b + c is an even number.

(A) Statement (1) ALONE is sufficient to answer the question, but statement (2) alone
is not sufficient
(B) Statement (2) ALONE is sufficient to answer the question, but statement (1) alone
is not sufficient
(C) Statements (1) and (2) COMBINED are sufficient to answer the question, but
neither statement alone is sufficient
(D) Each of statements (1) and (2) is sufficient to answer the question
(E) Statements (1) and (2), alone or combined, are not sufficient to answer the question


Test Section Answer
Quit Exit Time Help Confirm Next

Score Value: 550

00:17 GMAT CAT – Section 3 : Quantitative 27 of 37


What is the value of x ?

(1) x2 + 14x + 49 = 0

(2) x2 + 2x – 35 = 0

(A) Statement (1) ALONE is sufficient to answer the question, but statement (2) alone
is not sufficient
(B) Statement (2) ALONE is sufficient to answer the question, but statement (1) alone
is not sufficient
(C) Statements (1) and (2) COMBINED are sufficient to answer the question, but
neither statement alone is sufficient
(D) Each of statements (1) and (2) is sufficient to answer the question
(E) Statements (1) and (2), alone or combined, are not sufficient to answer the question

Test Section Answer


Quit Exit Time Help Confirm Next

174 © The MBA Center


Data Sufficiency

Score Value: 550

00:15 GMAT CAT – Section 3 : Quantitative 28 of 37


For all positive integers s and n, the function @ is defined by
s@n = sn + 2s. If s@n is an even number, is n an odd number?

(1) 3s is an odd number.

(2) s@5 is an odd number.

(A) Statement (1) ALONE is sufficient to answer the question, but statement (2) alone
is not sufficient
(B) Statement (2) ALONE is sufficient to answer the question, but statement (1) alone
is not sufficient
(C) Statements (1) and (2) COMBINED are sufficient to answer the question, but
neither statement alone is sufficient
(D) Each of statements (1) and (2) is sufficient to answer the question
(E) Statements (1) and (2), alone or combined, are not sufficient to answer the question


Test Section Answer
Quit Exit Time Help Confirm Next

© The MBA Center 175


Total PrepKit for the GMAT® Step 5.1 Lesson

SUMMARY

-Data Sufficiency questions represent one-third of all the questions in


the Quantitative Section.

-A Data Sufficiency question is composed of a question followed by two


statements.

-The Data Sufficiency answer choices are always the same. Memorize
them so that you will not waste time reading them on the actual test.
You must indicate whether:

- Statement (1) ALONE is sufficient, but statement (2) alone is not


sufficient to answer the question asked;

- Statement (2) ALONE is sufficient, but statement (1) alone is not


sufficient to answer the question asked;

- Both statements (1) and (2) TOGETHER are sufficient to answer


the question asked; but NEITHER statement ALONE is sufficient.

- EACH statement ALONE is sufficient to answer the question


asked;

- Statements (1) and (2) TOGETHER are NOT sufficient to answer


the question asked, and additional data specific to the problem are
needed.

USE THE MBA CENTER 3-STEP APPROACH TO EACH QUESTION.

- Step 1: Read and understand the question before you move on to the
statements.
- Step 2: Examine each statement separately, using the Process of Error
Identitfication to eliminate the wrong answer choices.
- Step 3: If necessary, consider the two statements together.

In addition to our general approach, the MBA Center has developed


specific tips such as:

- Guessing
- Inserting numbers
- Counting variables

On difficult problems do not hesitate to use more advanced strategies


like:

- Identifying redundant statements


- Recognizing assumption traps
- Avoiding common math traps

176 © The MBA Center


Data Sufficiency

ANSWERS TO EXAMPLES

1. C
2.a E
2.b C
3. E
4. C
5. E
6. C
7. E
8. C
9. C
10. A
11. E
12. E
13. B
14. N/A
15. N/A
16. C
17. C
18. B
19. C
20. C
21. E
22. A
23. A
24. E
25. A
26. E
27. A
28. D

© The MBA Center 177


© The MBA Center
Data Sufficiency

HOMEWORK/PRACTICE TEST
DATA SUFFICIENCY
ANSWER GRID

A B C D E
1 ! ! ! ! !
2 ! ! ! ! !
3 ! ! ! ! !
4 ! ! ! ! !
5 ! ! ! ! !
6 ! ! ! ! !
7 ! ! ! ! !
8 ! ! ! ! !
9 ! ! ! ! !
10 ! ! ! ! !
11 ! ! ! ! !
12 ! ! ! ! !
13 ! ! ! ! !
14 ! ! ! ! !
15 ! ! ! ! !
16 ! ! ! ! !
17 ! ! ! ! !
18 ! ! ! ! !
19 ! ! ! ! !
20 ! ! ! ! !

© The MBA Center 179


Total PrepKit for the GMAT® Step 5.2 Homework

Data Sufficiency – Homework Directions

End


TIME – 25 MINUTES
When finished
20 QUESTIONS reading
Directions: Each data sufficiency problem directions
consists of a question followed by two statements click on the
containing data related to the question. You must
determine whether the information provided in the
icon below
statements, either individually or combined, is
sufficient to answer the question given. Using the
information in the statements, plus your knowledge of
mathematics and commonly known facts (such as the
number of seconds in a minute), you must indicate
Dismiss
whether Directions
(A) Statement (1) ALONE is sufficient to answer
the question, but statement (2) alone is not
sufficient
(B) Statement (2) ALONE is sufficient to answer
the question, but statement (1) alone is not
sufficient
(C) Statements (1) and (2) COMBINED are
sufficient to answer the question, but neither
statement alone is sufficient
(D) Each of statements (1) and (2) is sufficient to
answer the question
(E) Statements (1) and (2), alone or combined, are
not sufficient to answer the question

Numbers: All numbers used are real numbers.


Figures: Figures accompanying data sufficiency
problems will accurately reflect the information
provided in the question, but will not necessarily reflect
the additional information given in statements (1) and
(2). All figures lie in a plane unless otherwise stated.
Straight lines may appear jagged on the computer
screen.

In data sufficiency questions that ask for a value


(what is the value of x?, for example), the information
provided in the statements is sufficient only if it allows
you to determine exactly one numerical value.

Example:
How much money did Bill spend on movie tickets
last week?
(1) Bill bought 3 movie tickets last week.
(2) Bill spent an average of $6 per movie ticket
that he purchased last week.
Explanation: Statement (1) indicates the number
of movie tickets Bill purchased last week, but not the
price of the tickets. It is therefore not sufficient to
answer the question. Statement (2) alone indicates
the average price Bill paid per movie ticket last week,
but not the number of tickets purchased. Statement (2)
is thus also not sufficient by itself to answer the
question. Taken together, the two statements do
provide enough information to answer the question.
Therefore the answer is (C).

Test Section Answer


Quit Exit Time Help Confirm Next

180 © The MBA Center


Data Sufficiency

A Statement (1) BY ITSELF is sufficient, but statement (2) by itself is not sufficient.
B Statement (2) BY ITSELF is sufficient, but statement (1) by itself is not sufficient.
C NEITHER statement BY ITSELF is sufficient, but the two statements COMBINED are sufficient.
D EACH statement BY ITSELF is sufficient.
E Statement (1) and statement (2) COMBINED are not sufficient.

1. A stadium manager expected 13,200 spectators 6. What is the value of y ?


to attend a sporting event. If everyone who bought a
ticket attended the event, how many full-price tickets (1) 8y + 16 = 0
were sold? (2) (y + 4)2 = y2

(1) The number of full-price tickets sold was equal


to 81 percent of the number of spectators 7. Rudolph and Nicholas each drove from
expected by the stadium manager. Brimsburg to Durville along the same route. If Rudolph
left Brimsburg at 8:00 a.m. and arrived in Durville at
(2) Of the 11,880 people who attended the 12:15 p.m. the same day, at what time did Nicholas
event, 90 percent had bought full-price arrive in Durville?
tickets.
1
(1) Nicholas left Brimsburg 2 hour earlier than
2. Of the positive integers a, b, c, d, and e, which is Rudolph.
the largest?
(2) The route from Brimsburg to Durville that
(1) a + b = e Rudolph and Nicholas took is 220 miles.
(2) b + c = d

8. If 12 people are waiting for an elevator, of whom


3. Is x2 an odd integer? 8 want to go down, can the 8 who want to go down
all enter the elevator without exceeding the maximum
(1) x is an odd integer. weight capacity of the elevator?
(2) x is a multiple of 3.
(1) The maximum weight capacity of the elevator
is 450 kilograms.

4. What is the value of 2mn ? (2) Ten of the people waiting for the elevator
have an average weight of 54 kilograms.
(1) m + n = 8
(2) m – n = 8


5. An arts center rents out a music hall for
performances, charging a base fee plus a commission
that is equal to a fixed percentage of the revenues
generated from ticket sales. How much did the arts x° y°
center charge for Thursday’s concert?
9. Is the the figure above an isosceles triangle?
(1) The base fee for a concert in the music hall is
$1,200. (1) z = 36
(2) The art center’s commission is 8 percent of the (2) x = 2z
concert revenues.

GO ON TO THE NEXT PAGE


© The MBA Center 181
Total PrepKit for the GMAT® Step 5.2 Homework

A Statement (1) BY ITSELF is sufficient, but statement (2) by itself is not sufficient.
B Statement (2) BY ITSELF is sufficient, but statement (1) by itself is not sufficient.
C NEITHER statement BY ITSELF is sufficient, but the two statements COMBINED are sufficient.
D EACH statement BY ITSELF is sufficient.
E Statement (1) and statement (2) COMBINED are not sufficient.

10. Is ay = 6 – ax ? 15. If a certain photograph collection consists of


some black-and-white photographs and some color
(1) 2a(x + y) = 12 photographs, how many color photographs are in the
(2) x = y = 4 and a = 0.75 collection?

(1) If 5 more photographs, all of them black-and-


11. What is the average (arithmetic mean) of a, b, white, were added to the collection, the ratio
and c ? of color photographs to black-and-white
photographs would then be 8 to 5.
(1) The average (arithmetic mean) of a and b is
24. (2) There are 35 more color photographs than
(2) –c = a + b black-and-white photographs in the collec-
tion.

12. What is the value of integer k ? 16. Is 3y greater than 50 ?

y
(1) 3 =9
(1) k is an integer.
2
1
(2) k is prime. (2) < 50
(3 )
y

13. Is x > y ? 17. What is the average (arithmetic mean)


of x and y ?
(1) x2 > y2
(2) x – y > 0 (1) x – y = 6
(2) The average (arithmetic mean) of x + 5 and
y + 7 is 17.

14. In what year did Carson and Joan get married?


18. What is the value of m + n ?
1
(1) Carson’s sister Loraine, who got married 2 2 (1) (m + n)2 = 25
years before Carson and Joan got married, (2) m and n are both positive integers.
got married in 1964.

(2) Carson and Joan’s 16th wedding anniversary


was in 1982.

GO ON TO THE NEXT PAGE

182 © The MBA Center


Data Sufficiency

A Statement (1) BY ITSELF is sufficient, but statement (2) by itself is not sufficient.
B Statement (2) BY ITSELF is sufficient, but statement (1) by itself is not sufficient.
C NEITHER statement BY ITSELF is sufficient, but the two statements COMBINED are sufficient.
D EACH statement BY ITSELF is sufficient.
E Statement (1) and statement (2) COMBINED are not sufficient.

II I

III IV

19. In the rectangular coordinate system shown


above, point (p,q) lies in which quadrant?

(1) pq < 0
(2) p + q < 0

20. Is abc < 24 ?

(1) a < 2, b < 3, and c < 4

(2) ab = 4 and c2 = 15

STOP
IF YOU FINISH BEFORE TIME IS EXPIRED
YOU MAY CHECK YOUR WORK
© The MBA Center 183
Total PrepKit for the GMAT® Step 5.2 Homework

DATA SUFFICIENCY HOMEWORK/PRACTICE TEST


ANSWER KEY

1. D
2. E
3. A
4. C
5. E
6. D
7. E
8. E
9. C
10. D
11. B
12. C
13. B
14. B
15. C
16. A
17. B
18. C
19. E
20. B

184 © The MBA Center


Data Sufficiency

EXPLANATIONS TO 4. (C)
DATA SUFFICIENCY Statement (1) is not sufficient. You could verify
HOMEWORK/PRACTICE TEST this by trying a few numbers.
For example, 2 + 6 = 8. Then, when you go back
to the question, 2(2)(6) = 24. Try another set of
numbers. 4 + 4 = 8. 2(4)(4) = 32. Since in one case
you get 24 and in another case you get 32, the
1. (D) statement is not sufficient.
Statement (1) is sufficient because the problem
states how many spectators the manager expected, so Statement (2) is also not sufficient. You could try
it is possible to compute 81% of this to find out how 10 – 2 = 8. Then, when you go back to the question,
many had full-price tickets. 2(10)(2) = 40.
Statement (2) is also sufficient because it is Try a second example. 7,000,008 – 7,000,000 = 8.
possible to compute 90% of 11,880 to get the Now, when you go back to the question,
answer. 2(7,000,008)(7,000,000) does not equal 40. (No
need to calculate here.)

When you take the 2 statements together, you have


2. (E) 2 linear equations and 2 variables, meaning you can
Statement (1) is not sufficient. Sub numbers as a solve for each variable and then plug their values into the
test. For example a = 5, b = 10, and e = 15. The other expression 2mn. You don't want to waste your time
variables, c and d could be anything. If they are both actually solving the equations.
smaller than 15, for example 11 and 12, then e is the
largest. If, however, c is 50 and d is 1,000,000, then d
is the biggest.

Statement (2) is not sufficient for the same


reason. d could be the biggest, but the statement says 5. (E)
nothing about a and e, so they could be very small or Before going to the statements, you might want to
very big. make a mental note that there are 3 pieces of missing
information: 1) the base fee, 2) the revenues generated
Combined, the choices are still not sufficient. from ticket sales, and 3) the percentage of revenues from
For example, it is possible that a = 5, b = 10, and e = ticket sales that determines the commission.
15 (from statement (1)), and b still is 10, c = 20, and
d = 30. In this case d is the biggest. But if, for example, Statement (1) is not sufficient, as it provides only
c = 1 and d = 11, e becomes the biggest. the base fee. You don't know the revenues from ticket
sales or the percent of the revenues used to determine
the commission.

3. (A) Statement (2) is also insufficient, as we don't


Before going to the statements, you might ask know the base fee or the revenues generated from
yourself when is x2 an odd integer? It turns out that ticket sales.
whenever x is an odd number, x2 is odd, and whenever x
is an even number, x2 is even. Together the statements are still not sufficient,
as we know the base fee and the percent, but not the
Statement (1) is sufficient. To verify this, try a few actual revenues from ticket sales.
examples with simple odd numbers, such as 1, 3, 5,
and 7. In each case, x2 is odd. The answer, then, based
on statement (1), is “always yes,” so the statement is
sufficient. 6. (D)
Statement (1) is sufficient because 1 linear
Statement (2) is not sufficient. Try a few equation with 1 unknown can always be solved.
multiples of 3: 3, 6, 9, 12 etc. With 3, 32 = 9 which is
odd, so the answer to the question is “yes.” With 6, 6 Statement (2) is also sufficient, though it is
2

= 36, which is even, so the answer is “no.” With 1 trickier to see why than in statement (1). An equation
“yes” and 1 “no” the answer is really “maybe,” which with a variable squared is sometimes sufficient to solve
is the same as “not sufficient.” for the variable, and it is best to actually work it out.
In this case:

© The MBA Center 185


Total PrepKit for the GMAT® Step 5.2 Homework

(y + 4)2 = y2 9. (C)
y2 + 8y + 16 = y2 Remember that isosceles means that 2 sides (and
8y + 16 = 0 therefore 2 angles) are equal.

You can stop here, as you now have the same Statement (1) alone is not sufficient. The
equation you had in statement (1). Again, the equation statement z = 36 doesn't indicate anything about x
now is linear (meaning there are no exponents) and so it and y except that they add up to 144 (because 3
can be solved for 1 variable. angles of a triangle must add up to 180°). x and y
could be the same (they could each be 72), or they
could be different.

7. (E) Statement (2) is also insufficient, as x or z remain


There is lots of missing information in this question. variables. It's possible that x = 90, z = 45, and
For example, how far apart are the towns? When did therefore y = 45 and the triangle is isosceles. It is also
Nicholas leave? How fast did Nicholas go? possible that x = 2, z = 1, and therefore y = 177 and
the triangle is not isosceles.
Statement (1) is not sufficient, as we don't know
how far apart the towns are or how fast Nicholas Taken together, the statements are sufficient.
went. We know that z = 36, x, which is double z, is then 72.
And knowing 2 angles of a triangle you can always
Statement (2) is not sufficient because we don't find the third. It is not necessary to continue, as
we don't know when Nicholas left or how fast he whether the triangle is isosceles is irrelevant, it is only
went. relevant to be able to determine if it is so. If you know
all 3 angles, then certainly you can figure out whether
Together the statements are still not sufficient, the triangle is isosceles.
as we know when Nicholas left and how far he went,
but not how fast he went.

10. (D)
Remember to look to simplify equations, as statements
8. (E) on the GMAT will often miraculously turn into exactly, or
This question is tricky, because there are a lot of almost exactly, what is necessary to solve the problems.
unknowns. What is the maximum weight capacity of the
elevator? How much do the people who are waiting Statement (1) is sufficient. Simplify!
weigh? And which 8 of the 12 people waiting will get in? 2a(x + y) = 12
And what did they eat for dinner last night? (Just a(x + y) = 6
kidding.) ax + ay = 6
ay = 6 – ax
Statement (1) is not sufficient, because although Since this is exactly the equation in the question,
the problem gives the capacity of the elevator, it does the statement is sufficient to answer the question.
not give the weight of the people.
Statement (2) is also sufficient. Statement (2)
Statement (2) is also not sufficient, the weight gives you a value for each variable. Since the values
capacity of the elevator is not indicated. are given, it is possible to verify whether the equation
works. As is it not necessary to determine whether the
The statements together still don’t supply enough answer to the question is “yes” or “no,” it is not
information. If the 8 people who actually get in each necessary to continue.
weigh 54 kg, their total weight is 432 kg which is safe
(less than the capacity of 450 kg). But if, for example,
6 people get in who each weigh 54 kg, and the other
2 who get in were the 2 not referred to in statement 11. (B)
(2), and they each weigh 200 kg (that is possible, Before going to the statements, you might want to
albeit unlikely), the 8 would certainly exceed the just jot down the average formula:
capacity. With 2 different answers (1 yes and 1 no), a + b + c = avg
the statements are not sufficient. 3
Now go to statement (1). It is not sufficient
because you don't know anything about c. c could be
2, or it could be 2,000,000,000.

186 © The MBA Center


Data Sufficiency

Statement (2) is sufficient, though it might not 14. (B)


be apparent at first glance. If you use the equation Statement (1) is not sufficient, though it may
you wrote out, appear to be at first glance. If Loraine got married in
a + b + c = avg. January 1964, then Carson and Joan got married in
3 July 1966. But if Loraine got married in December
and plug in –c for a + b 1964, then Carson and Joan got married in June 1967.
Then you get –c + c = avg Therefore you can't be sure what year Carson and
3 Joan got married.
0 = avg
Therefore, the average is 0 and the statement is Statement (2) is sufficient. No matter what
sufficient. month Carson and Joan's wedding anniversary takes
place, their marriage was exactly 16 years earlier, and
must have therefore been in 1966.

12. (C) 15. (C)


Statement (1) is not sufficient. Test out a few This question can be simplified a great deal by
thinking of it algebraically.
examples. When is k an integer? It turns out,
2
Statement (1) is not sufficient. There are 2
k
whenever k is even, is an integer. When k is unknowns: the number of color photographs (call it x)
2
and the number of black-and-white photographs (call
k it y). We have only 1 equation and therefore cannot
odd, is not an integer. Therefore statement (1)
2 solve for the 2 variables.
merely tells us that k is even. If you wanted to set up the equation, you could write:
x =8
Statement (2) by itself is definitely not y+5 5
sufficient. There are lots of prime numbers: 2, 3, 5, 7, There is no way to solve this equation.
11, 13, 17, 19…
Statement (2) is also not sufficient, as knowing
The 2 statements together are sufficient. that there are 35 more color photographs than black-
Statement (1) tells us that k is even and statement (2) and-white photographs doesn't indicate how many
tells us k is prime. There is only 1 even prime number, there actually are.
2, so we have determined the value of k.
The 2 statements combined are sufficient, as we
now have 2 equations and 2 unknowns and can
therefore solve. The second equation, by the way, is:
x – 35 = y

13. (B) 16. (A)


Questions with exponents and inequalities can be Statement (1) is sufficient. If 3 y  9 , then
difficult. Make sure to consider negatives and fractions. y  2 , and y = 4.
And remember, when statement (1) looks hard, don't If you know exactly what y is, then you can figure
give up. Go on to statement (2). out whether 3y is greater than 50. (It is not.)

Statement (2) is sufficient. When you rewrite the


equation (by adding y to both sides) you get x > y Statement (2) is not sufficient. The easiest way to
Therefore, the answer to the question is “yes” and deal with statement (2) is to try different values for y.
the statement is sufficient.
1
If  50 , then y could be 1, in which case the
Statement (1) is not sufficient, even though it 3
y

may look sufficient. When is x2 > y2 ? If x = 5 and y answer to the question would be “no.” But y could
=3, then x2 > y2. When you go back to the question, also be 100, in which case the answer to the question
the answer is “yes.” But what about trying negatives? would be “yes.” Statement (2) allows two different
Try x = –7 and y = –4. Again, x2 > y2 (49 > 16). But answers to the question and is therefore insufficient.
when you go back to the question, x is not greater
than y (–7 < –4).

© The MBA Center 187


Total PrepKit for the GMAT® Step 5.2 Homework

17. (B) which case the point is in quadrant II. It therefore is


Again, as this is an average question, you might first impossible to determine the quadrant and the
jot down the average formula: statement is not sufficient.
x + y = avg
2 Combined, the statements are also not
Now, go to the statements. sufficient. A point… such as (–8, 6) is in quadrant II,
but (9, –12) is in quadrant IV. It is impossible to be
Statement (1) is not sufficient. You could make x sure.
= 12 and y = 6, or we could make x = 106 and y = 100.
Depending on which numbers you use, you derive
different averages.

Statement (2) is sufficient. When you write out


the equation, you get
x+5+y+7
= 17
2
This then becomes
x + y + 12 = 17
2
Then, x + y + 12 = 34
Then, x + y = 22
Then, x + y = 11
2
Therefore, you know the average of x and y.

18. (C)
Statement (1) looks sufficient, but actually is
not. (m + n)2 = 25 tells us that m + n = 5 or –5. Since
there are 2 possible answers, the statement is not
sufficient.

Statement (2) is definitely not sufficient.


Knowing that m and n are both positive integers does
not tell us what their sum is.

When you combine the statements, you have enough


information, because statement (1) narrowed down the
sum to 2 possibilities and statement (2) eliminates the
negative possibility.

19. (E)
Here it is necessary to work out the possibilities.

In statement (1), when pq < 0, either p is positive


and q is negative, or vice versa. It is possible, for
example, to have the point (3, –2), which is in
quadrant IV, or (–2, 5), which is in quadrant II.
Statement (1) is therefore not sufficient.

Statement (2) tells us that the sum is negative.


Maybe both variables are negative and the
coordinates are, for example, (–5, –10), in which case
the point is in quadrant III, or maybe one variable is
negative and the other is positive, such as (–5, 2), in

188 © The MBA Center


Step 6
Data Sufficiency
Workshop

© The MBA Center


Total PrepKit for the GMAT® Step 6 Workshop

Data Sufficiency – Workshop Directions

End


When finished
TIME 60 MINUTES reading
31 QUESTIONS directions
Directions: In each of the problems below, you will find a question click on the
followed by two statements, marked (1) and (2). You must decide whether icon below
the information given in the two statements is sufficient to answer the
question, based on your understanding of the principles of mathematics and
knowledge of common facts (for example, the fact that there are one
hundred cents in a dollar or that north and east are perpendicular directions).
Dismiss
You then are to select the answer
A if statement (1) BY ITSELF is sufficient to answer the question but Directions
statement (2) by itself is not sufficient;
B if statement (2) BY ITSELF is sufficient to answer the question but
statement (1) by itself is not sufficient;
C if neither statement BY ITSELF is sufficient to answer the question,
but the two statements COMBINED are sufficient to answer the
question;
D if EACH statement BY ITSELF is sufficient to answer the question;
E if NEITHER statement (1) nor statement (2) is sufficient to answer
the question BY ITSELF or COMBINED.
Numbers: All the numbers in this section are real numbers.
Diagrams: Diagrams are drawn as accurately as possible according to
the data contained in the QUESTION, but are not necessarily drawn to scale
as far as the new information in the two STATEMENTS:
Unless stated otherwise all figures lie in a plane. All lines drawn in figures
may be assumed to be straight. All angles indicated must have a
measurement greater than 0 degrees. The relative positions of lines, angles,
points, shapes etc. may be assumed to be in the order in which they appear.
For questions that ask for the value of a variable, a statement is sufficient
only when it allows you to find exactly one value for the variable.
Example:
How much money did Bill spend on movie tickets last week?
(1) Bill bought 3 movie tickets last week.
(2) Bill spent an average of $6 per movie ticket that he purchased last
week.
Explanation: Statement (1) indicates the number of movie tickets Bill
purchased last week, but not the price of the tickets. It is therefore not
sufficient to answer the question. Statement (2) alone indicates the average
price Bill paid per movie ticket last week, but not the number of tickets
purchased. Statement (2) is thus also not sufficient by itself to answer the
question. Taken together, the two statements do provide enough information
to answer the question. Therefore the answer is (C).

Test Section Answer


Quit Exit Time Help Confirm Next

190 © The MBA Center


Data Sufficiency

A Statement (1) BY ITSELF is sufficient, but statement (2) by itself is not sufficient.
B Statement (2) BY ITSELF is sufficient, but statement (1) by itself is not sufficient.
C NEITHER statement BY ITSELF is sufficient, but the two statements COMBINED are sufficient.
D EACH statement BY ITSELF is sufficient.
E Statement (1) and statement (2) COMBINED are not sufficient.

EASY 6. A store sells paperback and hardcover copies of


a certain book. How many hardcover copies did the
store sell in May?

(1) The store sold 20 percent fewer hardcover than


1. Does a = 1 ? paperback copies of the book in May.
(2) The store sold 15 more paperback than
(1) b = 2a hardcover copies of the book in May.
(2) b = 2

7. A shelf 300 centimeters long is to be cut into 4


2. If 60 guests attended a catered party, how many pieces. If the 2 middle-sized pieces are the same size,
guests ordered both the salad and the soup? then what is the length of the longest piece?

(1) 28 guests ordered the salad. (1) The combined length of 2 of the pieces is 130
(2) 35 guests ordered the soup. centimeters.
(2) The shortest piece is exactly 60 centimeters long.

3. If there are 9 players on a baseball team, what is


the average (arithmetic mean) age of the players? 8. The value of a certain car depreciates by 10
percent each year. What was the original price of the
(1) The oldest player is 29 and the youngest is 25. car?
(2) Seven of the players are either 26 or 27.
(1) The car was purchased 2 years ago, and its
present value is $10,000.
(2) Two years after the car was purchased, its
4. If a and b are positive integers, what percent of depreciated value was 81% of its original price.
a is b ?

(1) 3a = 5b
(2) a + b = 80
a
9. If = c , then what is the value of a ?
b
(1) bc = 4
5. What is the value of y ?
1
(2) b = 8 and c =
2
(1) 4y – 2 = 2y

(2) 2 +1=3
y
10. What is the value of integer x ?

(1) x4 < 25
(2) x = x2

GO ON TO THE NEXT PAGE


© The MBA Center 191
Total PrepKit for the GMAT® Step 6 Workshop

A Statement (1) BY ITSELF is sufficient, but statement (2) by itself is not sufficient.
B Statement (2) BY ITSELF is sufficient, but statement (1) by itself is not sufficient.
C NEITHER statement BY ITSELF is sufficient, but the two statements COMBINED are sufficient.
D EACH statement BY ITSELF is sufficient.
E Statement (1) and statement (2) COMBINED are not sufficient.

MEDIUM


c 13. Is x3 > x2 ?
b

z° y° (1) x2 > 1
a (2) x > –1

11. Is the triangle above a right triangle? 14. If xy ≠ 0, is xy > 0 ?

(1) a2 + b2 = c2 (1) x3 < 0


(2) z = 2y (2) x = –y

A B
s r
15. What is the value of 2 + 2 ?
I II III
s r
(1) 3+ 3 = 6
D C
(2) r + s = 18

12. What is the area of rectangle ABCD ?

(1) The sum of the areas of rectangles I and II is 28. 16. Does c = 8 ?
(2) The sum of the areas of rectangles II and III is 24.
(1) The average of a, b, and c is 8.
(2) a – b = 0

GO ON TO THE NEXT PAGE


192 © The MBA Center
Data Sufficiency

A Statement (1) BY ITSELF is sufficient, but statement (2) by itself is not sufficient.
B Statement (2) BY ITSELF is sufficient, but statement (1) by itself is not sufficient.
C NEITHER statement BY ITSELF is sufficient, but the two statements COMBINED are sufficient.
D EACH statement BY ITSELF is sufficient.
E Statement (1) and statement (2) COMBINED are not sufficient.

A B 20. Of the positive integers v, w, x, y, and z, which


Q
is the greatest?

(1) v + w = z
P (2) w + x = y
R

S
D C 21. What is the value of a + b ?

(1) 2a + 3b = 15
(2) 6a + 9b = 45
17. The circular base of a statue is set on a
quadrilateral stand, as indicated in the figure above,
such that the circular base just reaches the edge of the
stand at points P, Q, R, and S. What is the area of the
circular base?

(1) The diagonal AC of the stand is 12.


(2) The circumference of the base is 8π.

18. If Lydia is exactly 1.5 times as old as Francis,


how old is Francis?

(1) Seven years ago Lydia was exactly twice as old


as Francis.
(2) Fourteen years from now Lydia will be exactly 25
percent older than Francis.

19. If a, b, and c are positive integers, does

a = b2 ?
c

(1) c = b
ac

b
(2) c =
a

GO ON TO THE NEXT PAGE


© The MBA Center 193
Total PrepKit for the GMAT® Step 6 Workshop

A Statement (1) BY ITSELF is sufficient, but statement (2) by itself is not sufficient.
B Statement (2) BY ITSELF is sufficient, but statement (1) by itself is not sufficient.
C NEITHER statement BY ITSELF is sufficient, but the two statements COMBINED are sufficient.
D EACH statement BY ITSELF is sufficient.
E Statement (1) and statement (2) COMBINED are not sufficient.

DIFFICULT 25. On a new manager’s first day at an express


mail delivery company, 25 percent of the first 60
packages were delivered late. After the first 60, all
other packages that day were delivered on time. What
is the total number of packages delivered on time on
the manager’s first day?
22. What is the value of ab ?
c
(1) A total of 120 packages were delivered on the
manager’s first day.
(1) a = 3 and b = 2 (2) Of the packages delivered on the manager’s first
c 4 c 3
day, 87.5 percent were delivered on time.

(2) c =6
ab
26. In a clothing store, 60 percent of the shirts are
short-sleeved and 70 percent of the shirts are cotton. If
20 percent of the cotton shirts are white, how many of
23. An omega sequence is an infinite sequence of the cotton shirts are white?
positive integers that contains an infinite number of
prime integers. If D is an infinite sequence of positive (1) There are 300 short-sleeved shirts in the store.
integers, is D an omega sequence?
(2) Of the shirts that are not cotton, 1 are white.
3
(1) D contains an infinite number of odd integers.
(2) The integer 3 is not an element of D.


x❐
24. A credit card company offered a promotion in
▲❋
1995 such that all users of credit card X would get a
travel voucher at the end of the year equal in value to
a fixed percentage of the amount of money charged on 27. In the multiplication problem shown above,
the credit card for that year. Mr. Pavlov charged d each of the symbols ✪, ❐, ▲, and ❋ represents a
dollars on credit card X and Mrs. Pavlov charged positive digit (not including 0). If ▲ > ❋ and ✪ > ❐,
$3,400 on credit card X. If Mr. Pavlov got a travel what is the value of ❐ ?
voucher equal in value to $380, what was the value of
Mrs. Pavlov’s travel voucher? (1) ✪ = 8
(2) ▲ = 3
(1) The vouchers were equal in value to $0.05 for
each dollar charged to credit card X in 1995.
(2) d = 7,600

GO ON TO THE NEXT PAGE


194 © The MBA Center
Data Sufficiency

A Statement (1) BY ITSELF is sufficient, but statement (2) by itself is not sufficient.
B Statement (2) BY ITSELF is sufficient, but statement (1) by itself is not sufficient.
C NEITHER statement BY ITSELF is sufficient, but the two statements COMBINED are sufficient.
D EACH statement BY ITSELF is sufficient.
E Statement (1) and statement (2) COMBINED are not sufficient.

28. Is abc = 1 ? 30. If n is an integer, is 3n less than 100 ?

(1) ab = 1
(2) bc = 1 (1) 3n + 1 > 100

(2) 3n – 1 = 3n – 162

29. What is the average (arithmetic mean) of a and


b?
31. Is ab < 6 ?
(1) The average (arithmetic mean) of a + 4 and b +
4 is 12. (1) a < 3 and b < 2
(2) The average (arihmetic mean) of a, b, and 10 is
(2) 1 <a< 2 and b2 < 64
26 . 2 3
3

STOP
IF YOU FINISHED BEFORE TIME IS EXPIRED
YOU MAY CHECK YOUR WORK

© The MBA Center 195


Total PrepKit for the GMAT® Step 6 Workshop

DATA SUFFICIENCY WORKSHOP


ANSWER KEY

1. C
2. E
3. E
4. A
5. D
6. C
7. E
8. A
9. D
10. E
11. D
12. E
13. C
14. C
15. D
16. E
17. B
18. D
19. D
20. E
21. E
22. B
23. E
24. D
25. D
26. A
27. D
28. E
29. D
30. B
31. B

196 © The MBA Center


Verbal Section
Introduction

© The MBA Center


Total PrepKit for the GMAT® Verbal Section

GENERAL OUTLINE

The Verbal Section of the GMAT CAT is designed to test your


abilities in three subject areas: logic, grammar, and reading. For each
subject area there is a corresponding question type: Critical
Reasoning, Sentence Correction, and Reading Comprehension. Both
Sentence Correction and Reading Comprehension questions test
traditional verbal skills. Sentence Correction questions assess your
ability to correct grammatical errors and Reading Comprehension
questions evaluate your critical reading skills. Critical Reasoning
questions, though included in the Verbal Section, test analytical skills
and assess your abilities in logic. This introduction is designed to
familiarize you with the Verbal Section of the GMAT CAT and the
material covered. The lessons that follow this preview will discuss
Critical Reasoning, Sentence Correction, and Reading
Comprehension questions in detail and will give you specific
strategies to help you improve your score on the Verbal Section of the
GMAT CAT.

THE FOLLOWING TOPICS WILL BE COVERED IN THIS LESSON:

The GMAT Verbal Section: An Overview


What the Verbal Section Tests
CAT Strategy
What About Guessing on the CAT?

198 © The MBA Center


Introduction

The GMAT Verbal Section: An Overview HINT


Note-taking materials
are very important.
The Verbal Section of the GMAT CAT is a 75-minute section designed to test your Use them for drawing
mastery of basic logic and verbal skills as well as your ability to perform basic analyses. In a scoring grid (see
this section there are 41 multiple-choice questions which fall into three categories: Critical The MBA Center
Reasoning, Sentence Correction, and Reading Comprehension. Critical Reasoning Approach to the
GMAT CAT), for
questions are analytical problems presented as formal arguments that include inferences, diagramming Rea-
premises, and a conclusion. The eight Critical Reasoning question types cover eight ways ding Comprehen-
to test logical analysis, for example making intuitive deductions. Sentence Correction questions sion passages, and
test your ability to identify and correct grammatical mistakes in sentences which are either for outlining the
premises and conc-
partially or completely underlined. Your job is to determine whether the underlined lusion of a Critical
section is right or wrong and, if wrong, to choose which of the answer choices best Reasoning argu-
corrects the error. No sentences will appear which have not been underlined. Reading ment. We recom-
Comprehension questions test your ability to read and understand what is stated or mend that you take
notes when you
implied in a text within a short amount of time. These questions always appear with a take the practice
passage of between 250 and 450 words, and you do not have the option to select from GMAT CATs.
several passages. You should expect approximately 14 Critical Reasoning questions, 14
Sentence Correction questions, and 13 Reading Comprehension questions in this section.
However, because the number of each question type is decided by the computer, the
actual allocation of questions may vary. The three question types in the Verbal Section do
not test the same skills. Therefore, each type requires different strategies and techniques.
For these reasons Critical Reasoning, Sentence Correction, and Reading Comprehension
questions will be taught separately in this course.

What the Verbal Section Tests

The Verbal Section of the GMAT requires knowledge of basic skills in logic, grammar,
and reading. These concepts are the same for every GMAT and are tested in similar ways
on each test. The two most commonly tested skills in the Verbal Section of the GMAT are
making inferences (Critical Reasoning and Reading Comprehension) and spotting subject-
verb agreement (Sentence Correction). Inferences are implied in every Critical Reasoning
argument, and typically about one-third of the Critical Reasoning questions are inference
questions. In Reading Comprehension, inference questions were once somewhat rare,
though now you are likely to see two or three inference questions for each passage.
Sentence Correction questions which test subject-verb agreement are more frequently
asked than any of the other nine question types reviewed in your Sentence Correction
lesson. Though questions are mixed on the CAT Critical Reasoning, Sentence Correction,
and Reading Comprehension, the logic, grammar, and reading tested are distinct skills,
each requiring separate mastery and strategy. HINT
The 41 questions
in the Verbal
The sections are designed to test precise concepts that contribute to a comprehensive Section include
assessment of general verbal skills. Each general verbal skill is broken down into a certain 11 experimental
number of testing points designed to measure more specific verbal skills, which we’ve questions which
are unrecogniza-
simplified into question types. For instance, in Sentence Correction there are ten testing
ble from any other
points, Critical Reasoning has eight, and Reading Comprehension contains four. The ideal question:
GMAT assessment is a thorough assessment. the experimental
questions are not
designed to assess
Many students learned grammar long ago and, as with math, need a reminder of basic new testing points,
rules. Since the grammar rules tested are basic, if unfamiliar, and count toward roughly they are designed
one-third of your score on the Verbal Section (based on 14 or so Sentence Correction to assess old tes-
questions) we recommend that you take a look at the Grammar Review in this book before ting points in new
ways.
you begin the Sentence Correction lesson in Step 7.

© The MBA Center 199


Total PrepKit for the GMAT® Verbal Section

THE CHALLENGE

The Verbal Section tests not only your mastery of certain verbal skills, but also your ability
to determine answers to difficult questions quickly and accurately. The principal challenge
of the Verbal Section of the GMAT is time. If you were given an unlimited amount of time,
you could probably answer all or nearly all of the questions correctly. However, the
testmaker designed each question in this section to confuse you, and the time allowed is
limited. As a result most test takers have difficulty finishing the section in the time allowed.
For these reasons, the MBA Center approach stresses both mastery of necessary basic verbal
skills and use of the time-saving strategies in the Critical Reasoning, Sentence Correction,
and Reading Comprehension lessons which follow.

The Verbal Section also challenges your command of standard written English. All of the
material in each of the three subject areas, Critical Reasoning, Sentence Correction, and
Reading Comprehension presumes a high level of competence in the stylistic conventions
and grammar rules of standard written English. Though the GMAT has been given in non-
native English speaking countries for many years, the test is still written in American English,
which can prove challenging for many test takers whose first language is not English. While
understanding the language in which the test is written is a significant challenge for many
who take the test, nearly all test takers are challenged by the analytical skills required by
over 60 percent of Verbal Section questions. The analytical skills tested involve your ability
to reason, to make inferences, and to understand logical relationships. While completing
the lessons which follow and practicing on your own it is important to understand how
these analytical skills are tested in the Verbal Section, since all Critical Reasoning questions
and at least two or three Reading Comprehension questions per passage are designed to
measure these skills. Though the skills tested in the Verbal Section should not be unfamiliar
to you, the challenge on the GMAT is to correctly answer questions which have been
designed to confuse you within the limited amount of time you are given.

THE GMAT VERBAL SECTION AND THE CAT


How the CAT Works

When you begin the Verbal Section of the GMAT CAT, the computer will give you a
question (either Critical Reasoning or Sentence Correction) of medium difficulty. A medium
difficulty question is one that roughly half of all test takers get right and roughly half get
wrong. If you answer the first question correctly the computer will give you a harder
question. Likewise, if you answer the first question incorrectly the computer will give you
an easier question. This process is repeated for the rest of the section. Every time you answer
a question correctly the computer gives you a more difficult question. Similarly, each time
you answer a question incorrectly, the computer gives you an easier question. As you
progress through the section each successive jump gets smaller and smaller as the
computer focuses in on your level. In theory, you will eventually arrive at a point where
every question will be at your level. The score you receive for the section depends not on
the number, but on the difficulty level of the questions you answer correctly. The idea is to
award more points for difficult questions than for easy ones. Therefore, correct answers to
difficult questions increase your score more than do correct answers to easy questions.

CAT Strategy

In order to maximize your score on the Verbal Section of the GMAT it is important to
keep in mind how the CAT arrives at a score. Remember, the computer tries to find your
level of ability and assign questions to you accordingly. Early in the section, the computer
makes large jumps in level of difficulty in order to find your general level of ability. As the
section progresses the jumps get smaller and smaller as the computer focuses in on your
level. Because your final score depends on the level of difficulty of the questions you answer

200 © The MBA Center


Introduction

correctly, it is important that you correctly answer the early questions in order to get
quickly to the harder, more valuable questions. Also worth noting is that not all questions
in the Verbal Section require the same amount of time to answer. For example, general
questions in Reading comprehension take less time to answer than do detail questions.

Time Management on the CAT

Because of the incredible time pressure on the CAT, it is important that you have a
strategy for managing time. Use the chart below to help pace yourself on the Verbal
Section of the CAT. Note that test takers who are weaker in verbal skills may want to spend
a bit more time on the first nine questions in order to be sure to answer them correctly.
Likewise, test takers who are stronger in verbal skills should move a bit more quickly
through the early questions in order to conserve time for the difficult questions later in the
section.

Time

9 27 41

Out of the Blocks - Questions 1–9


The first nine questions are by far the most important questions. Because the CAT is
making large jumps at this stage of the test, a wrong answer is far more damaging to your
score than is a wrong answer later on in the section. You can spend a bit more time per
question here than you can later in the test. Take your time, check your work, and avoid
guessing. Try to finish the first nine questions before 25 minutes have elapsed in the
section.

Cruising - Questions 10–27


These questions are still quite important, but a wrong answer here won’t be as
damaging to your score as a wrong answer in the first nine questions. You should move
through these quickly and efficiently. Make an honest attempt to answer all of these
questions, but if you get stuck, go ahead and guess. You should be done with this block
of questions before an hour has elapsed in the section.

The Final Stretch - Questions 28–41


These questions are not as important as the previous questions. You should move
quickly from question to question and don’t hesitate to guess if you find a question too
difficult. Just be sure to answer all 41 questions. If you are about to run out of time, guess
on the remaining questions.

© The MBA Center 201


Total PrepKit for the GMAT® Verbal Section

WHAT ABOUT GUESSING ON THE CAT?

Guessing is especially important on the CAT for two reasons:

1) You can’t skip questions on the CAT. Until you answer the question you are
currently on, the computer will not let you move on to the next question. So if you don’t
know the answer to a question, go ahead and guess. Don’t worry, there isn’t a guessing
penalty on the CAT.

2) Because the computer adapts to your level, you should find that nearly every
question on the test is difficult. For this reason you will have to guess more often than you
would on the pencil-and-paper test.

Because guessing is necessary on the CAT, it is important that you become good at it.
By using the techniques and strategies described in the Critical Reasoning, Sentence
Correction, and Reading Comprehension lessons, you will be able to eliminate several
wrong answer choices and improve your chances of guessing the correct answer.

Answer every question in the section even if you have to guess.

When time expires, every question left unanswered in the section is considered wrong.
For this reason you should answer every question in the section, even if you have to take
random guesses at the end. Remember: there is no penalty for getting a question wrong.
Even if you take a random guess on a question, you have at least a 20 percent chance of
getting it right. So make sure to allow enough time at the end to guess on the final
questions if you have to. You should of course try to eliminate any wrong answers you can,
but if you can’t, taking a guess is better than not answering a question at all.

202 © The MBA Center


Step 7
Grammar and Style
Review

© The MBA Center


Total PrepKit for the GMAT® Step 7.1 Lesson

GENERAL OUTLINE

If you want to be able to choose the grammatically correct answers


to multiple-choice Sentence Correction questions and to know how
to write Analytical Writing Assessment essays that conform to the
stylistic conventions of Standard Written American English, you need
to know what the rules and conventions are. This lesson presents all
the rules and conventions you need to know, in 100 bite-sized pieces,
grouped into ten top grammar and style problem areas.

THE FOLLOWING TOPICS WILL BE COVERED IN THIS LESSON:

Sentence Structure
Subject-Verb Agreement
Noun Number
Verb Tense
Pronouns
Modifiers
Comparisons
Parallelism
Idioms
Style

204 © The MBA Center


Grammar and Style Review

STANDARD WRITTEN AMERICAN ENGLISH (SWAE)


If you want to join the club, you have to learn the code. A prerequisite for
admission to the educated American elite of government, academia, or
business is mastery of Standard Written American English, or SWAE. This is
the language you are expected to use when you write an academic essay, a
journal article, a formal letter, a legal brief, a business plan, or a diplomatic
communiqué.

THE RULES OF SWAE


The rules of SWAE are not codified in any one document or managed by any
particular academy. The rules are established by precedent and formalized
by consensus.

About the great majority of rules there is no dissent. There are people who
say “I knows” or “he be,” but they generally know not to write that way.
About some rules, however, there are disputes. And these disputes can
become surprisingly heated in some circles. In one corner, there are the rigid
traditionalists who learned grammar from pedants and who cannot abide a
split infinitive or a preposition at the end of a sentence. In the other corner,
there are the radicals eager to abandon all rules and to accept the demise of
“whom” or the use of “they” as a singular pronoun.

PHINEAS AND THE RULES OF SWAE


The people who write GMAT questions and who grade GMAT essays fall
somewhere between these two extremes. To avoid controversy, they do not
test you on rules about which there is little consensus. You will not need to
worry about split infinitives or prepositions at the ends of sentences. Nor do
GMAT writers test you on rules that every literate native speaker knows. You
will not have to watch out for “I knows” or “he be.” The rules tested on the
GMAT are those that editors and scholars generally agree on but that many
educated Americans have trouble with. These are the rules presented on the
following pages.
Learning these rules is critical to success on the GMAT. In one of the
multiple-choice Verbal question types, Sentence Correction, your task is to
detect violations of the rules of SWAE and to identify the answer choices
that correct these violations. And in the Analytical Writing Assessment,
your task is to produce essays that conform to the rules of SWAE.

Learn the 100 rules on the following pages, and you will be ready to find
and correct just about any mistake in a Sentence Correction question and
to avoid the most common pitfalls in writing AWA essays.

© The MBA Center 205


Total PrepKit for the GMAT® Step 7.1 Lesson

1. SENTENCE STRUCTURE
The first issue to consider in proofing a sentence is its structure. Is it a
sentence? SWAE has quite strict notions about what constitutes a sentence.
Beware the sentence fragment — that is, an incomplete sentence, a string
of words that does not have all it takes to make a sentence. Beware also the
run-on sentence — an overcomplete sentence, a string of words that should
actually be written as two sentences.
Rule 1 Every sentence must have a subject and a conjugated verb.
In speech and casual writing it may be acceptable to produce sentences
without verbs, but it is not acceptable in SWAE.
INCORRECT:Violence on television. That is what makes today’s young
people so indifferent to human suffering.
CORRECT: Violence on television is what makes today’s young people so
indifferent to human suffering.

Rule 2 A clause beginning with a subordinating conjunction


cannot be the main clause of a sentence.
Conjunctions like “before,” “after,” “although,” “because,” “if,” “when,” or
“while” are subordinating conjunctions. A clause that begins with a
subordinating conjunction cannot stand alone. Such a clause must be
attached to another clause.
INCORRECT:Why was the move postponed? Because the new location was
not yet ready for occupancy.
CORRECT: The move was postponed because the new location was not yet
ready for occupancy.

INCORRECT:Although every attempt was made to rescue the agreement,


ranging from compromise through concession to threats.
CORRECT: Every attempt was made to rescue the agreement, ranging from
compromise through concession to threats.
The conjunctions “and,” “but,” “or,” “nor,” “for,” “so,” and “yet” are
coordinating conjunctions. A clause that begins with one of these
conjunctions can be attached to another clause, but such a clause can also
stand alone. Some traditionalists object to the practice, but in
contemporary SWAE it is acceptable to begin a sentence with “and” or
“but.”
Rule 3 A clause beginning with a “which” or “who” cannot be the
main clause of a sentence (unless the sentence is a
question).
“Which” and “who” are relative pronouns. A clause that begins with a
relative pronoun is a subordinate clause and needs another clause to attach
to.
INCORRECT:The president has a degree from Harvard, and the vice
president has a degree from the University of Virginia. Which
is the Harvard of the South.
CORRECT: The president has a degree from Harvard, and the vice
president has a degree from the University of Virginia, which
is the Harvard of the South.

Rule 4 Do not join two independent clauses with a comma.


If two clauses can each stand alone as a sentence, then they must be
separated by something more final than a comma. A period usually works

206 © The MBA Center


Grammar and Style Review

best, but a semicolon will work as well.


INCORRECT:The tomato is not a vegetable, it is a fruit.
CORRECT: The tomato is not a vegetable. It is a fruit.
CORRECT: The tomato is not a vegetable; it is a fruit.

INCORRECT: The forecasts were accurate, the market is rebounding.


CORRECT: The forecasts were accurate. The market is rebounding.
CORRECT: The forecasts were accurate; the market is rebounding.

INCORRECT:A monetary donation is not required, there are other ways to


contribute.
CORRECT: A monetary donation is not required. There are other ways to
contribute.
CORRECT: A monetary donation is not required; there are other ways to
contribute.
Be aware that American readers find the semicolon pretentious. Usually a
period is all you need. Use semicolons sparingly.
Rule 5 Do not use adverbs to join clauses.
The following words are adverbs:
thus however instead
therefore nevertheless
These words are not conjunctions and should not be used to join clauses.
Begin a new sentence, or at least use a semicolon.
INCORRECT:The quadrilateral is a parallelogram, therefore opposite angles
are equal.
CORRECT: The quadrilateral is a parallelogram. Therefore, opposite angles
are equal.
CORRECT: The quadrilateral is a parallelogram; therefore, opposite angles
are equal.
INCORRECT:No former President has ever served in the Senate, however one
did serve in the House of Representatives.
CORRECT: No former President has ever served in the Senate. However,
one did serve in the House of Representatives.
CORRECT: No former President has ever served in the Senate; however,
one did serve in the House of Representatives.

INCORRECT:Auto emissions and industrial waste are under control,


nevertheless air pollution is on the increase.
CORRECT: Auto emissions and industrial waste are under control.
Nevertheless, air pollution is on the increase.
CORRECT: Auto emissions and industrial waste are under control;
nevertheless, air pollution is on the increase.

INCORRECT:The meeting was not held in the conference room as planned,


instead it took place in the director’s office.
CORRECT: The meeting was not held in the conference room as planned.
Instead, it took place in the director’s office.
CORRECT: The meeting was not held in the conference room as planned;
instead, it took place in the director’s office.

© The MBA Center 207


Total PrepKit for the GMAT® Step 7.1 Lesson

EXERCISE 1 (SENTENCE STRUCTURE)

Directions: Rewrite the following sentences to correct any sentence structure


errors. (Note that some sentences may be correct as written.)

1. The secretary arrived too late, the meeting had already begun.

2. The dangers of secondhand smoke have been exaggerated, therefore


the government should consider repealing some of the more
restrictive antismoking laws.

3. The reasons for the price adjustment: increasing costs and decreasing
demand.

4. English is not the most widely spoken language in the world,


nevertheless it is the most practical language for international
business.

5. Martinez and Johnson were commended for the new layout. But Chung
deserves some credit as well.

6. Ask the librarian, he will know.

7. Because the head writer was in the hospital — appendicitis — and the
artist was on vacation — honeymoon — progress ceased.

8. Because it has not rained in two months — the worst drought on


record — and reservoir levels are dangerously low.

9. Call Novak, he is the only one who can help us now.

10. Test scores across the state have risen sharply under this
administration. However, there is still room for improvement.

208 © The MBA Center


Grammar and Style Review

2. SUBJECT-VERB AGREEMENT

Once you have verified that the sentence has a subject and a verb, the next
step is to make sure the subject and verb agree.

Singular subjects and plural subjects take different verb forms. No native
speaker has difficulty with agreement in the typical sentence in which the
verb immediately follows the subject. You would never mistakenly say “he
are” or “they is.” But when the sentence is more complicated — when the
subject and verb are separated, or when the subject comes after the verb —
subject-verb agreement becomes more complicated.
Rule 6 The subject and verb must agree no matter what comes
between them.
No matter how many words and what kind of words come between them,
the subject and verb must agree. Do not let extra information clauses
distract you.
INCORRECT:The rate at which falling objects accelerate are unrelated to
their mass.
CORRECT: The rate at which falling objects accelerate is unrelated to
their mass.

Rule 7 For most subjects in the form “X of Y,” the verb agrees
with X.
Usually the subject form “X of Y” consists of the subject X followed by a
prepositional phrase “of Y” that provides extra information about X. As
with any extra information clause that separates the subject and verb, the
“of Y” clause should have no effect on the agreement of the verb with the
true subject.
INCORRECT:The original intention of the tax cut measures were to jump-
start a sluggish economy.
CORRECT: The original intention of the tax cut measures was to jump-
start a sluggish economy.

Rule 8 When X in the subject form “X of Y” is a quantity word, the


verb agrees with Y.
Sometimes the subject form “X of Y” actually consists of the subject Y
preceded by a statement of quantity “X of.” This is the case when X is a
fraction, a percent, or a word such as “all,” “most,” “rest,” or “majority.”
INCORRECT: One quarter of the investments is bonds.
CORRECT: One quarter of the investments are bonds.

INCORRECT: Seventy-five percent of the portfolio are stocks.


CORRECT: Seventy-five percent of the portfolio is stocks.

INCORRECT: The majority of the House of Representatives are Republican.


CORRECT: The majority of the House of Representatives is Republican.

Rule 9 “A number of” is plural. “The number of” is singular.


“A number of” is a quantity expression, and so verb agrees with the word
that follows “of,” which will always be plural. (You cannot use “number”
with a singular.) In the phrase “the number of,” the singular noun
“number” is the subject.

© The MBA Center 209


Total PrepKit for the GMAT® Step 7.1 Lesson

INCORRECT: A number of reforms has been proposed.


CORRECT: A number of reforms have been proposed.

INCORRECT: The number of duplications are negligible.


CORRECT: The number of duplications is negligible.

Rule 10 Only the word “and” can make a plural subject out of two
or more singulars.
Such phrases as “together with,” “combined with,” “along with,” and “as
well as,” do not create plural subjects.
INCORRECT:The Senator, along with her husband and children, were
expected to attend.
CORRECT: The Senator, along with her husband and children, was
expected to attend.
INCORRECT:The Times, the Post, as well as the Daily News have called for
the judge’s resignation.
CORRECT: The Times, the Post, and the Daily News have called for the
judge’s resignation.

Rule 11 When a compound subject is formed using “or” or “nor,”


the verb agrees with the nearer subject.
If two plural nouns are combined with “or” or “nor,” the verb is plural. But
if two singular nouns are combined this way, the verb is singular.
INCORRECT:Either the Senator or her spokesperson are expected to make a
statement shortly.
CORRECT: Either the Senator or her spokesperson is expected to make a
statement shortly.

When one part of an “or” or “nor” subject is singular and the other part is
plural, the verb agrees with the part that is closer to the verb — usually the
part after “or” or “nor.”
INCORRECT:Neither her sons nor her daughter have been seen publicly in
recent weeks.
CORRECT: Neither her sons nor her daughter has been seen publicly in
recent weeks.

Rule 12 The verb “be” agrees with the subject, not the complement.
Even when the complement is plural, if the subject is singular, then the verb
must be singular.
INCORRECT: The only furniture were a lamp and a mattress.
CORRECT: The only furniture was a lamp and a mattress.

INCORRECT: But the true greatness of America are the people.


CORRECT: But the true greatness of America is the people.

Rule 13 In a sentence beginning with “there,” the subject follows


the verb.
The word “there” at the beginning of a sentence is not the subject. Do not
write “there is” or “there was” when what follows is plural.

210 © The MBA Center


Grammar and Style Review

INCORRECT: There is a number of possible explanations.


CORRECT: There are a number of possible explanations.

INCORRECT: There is not enough people in the department to get the job
done.
CORRECT: There are not enough people in the department to get the job
done.

INCORRECT: Each year there was more and more of what the
administration euphemistically termed “inoperative
statements.”
CORRECT: Each year there were more and more of what the
administration euphemistically termed “inoperative
statements.”

Rule 14 In a sentence beginning with an adverbial phrase of


location, the subject is often postponed.
Sometimes, for reasons of emphasis or variety, a writer may choose to move
an adverbial phrase of location to the front of a sentence, in which case the
usual subject-verb order is reversed. Nevertheless, even if the subject comes
after the verb, the subject and verb must agree.
INCORRECT: Between Fifth and Sixth Avenues stand a famous little church.
CORRECT: Between Fifth and Sixth Avenues stands a famous little
church.

INCORRECT: At the back of the book is to be found a glossary and an index.


CORRECT: At the back of the book are to be found a glossary and an
index.

Rule 15 In a sentence beginning with a negative adverb or adverbial


phrase, the interrogative form is used, with the subject
following the verb.
A writer may choose to accentuate the negative by moving a word or phrase
such as “hardly,” “never,” “rarely,” or “not in a long time” to the head of a
sentence. In that case, the interrogative form is used. That is, “do” or “have”
or some modal verb is needed before the subject.
INCORRECT: Never he had seen so many elephants.
CORRECT: Never had he seen so many elephants.

INCORRECT: Rarely it snows in New Orleans.


CORRECT: Rarely does it snow in New Orleans.
The verb still agrees with the subject even when the subject is postponed.
INCORRECT: Hardly does the returning veterans recognize their home town.
CORRECT: Hardly do the returning veterans recognize their home town.

INCORRECT: In no other U.S. state does more people go to college.


CORRECT: In no other U.S. state do more people go to college.

© The MBA Center 211


Total PrepKit for the GMAT® Step 7.1 Lesson

EXERCISE 2 (SUBJECT-VERB AGREEMENT)

Directions: Rewrite the following sentences to correct any subject-verb


agreement errors. (Note that some sentences may be correct as written.)

1. The only thing he values more than his career are his wife and children.

2. The majority of the people of California supports recycling.

3. In the last ten years, the number of professional sports teams have
doubled.

4. The only person who knows where the keys are is Wilson.

5. Heavy rain combined with gusty winds has caused widespread


damage.

6. Not in several days have the mother or the children been seen.

7. Nuclei that are too large or ones in which the neutron-to-proton ratio
is unfavorable are radioactive.

8. No less astonishing than the dimensions of the surviving examples are


their variety.

9. On the ground floor of the office building is a newsstand, a coffee shop,


and a restaurant.

10. Never before in the history of the state has Montana’s vote-counting
procedures received more attention.

212 © The MBA Center


Grammar and Style Review

3. NOUN NUMBER
It can be a challenge to verify that the verb agrees with the subject when it
is not clear whether the subject is singular or plural.
Rule 16 There are some plural nouns that do not end in -s.
Some words borrowed directly from Greek or Latin have plural forms that
do not end in -s.
singular plural
criterion criteria
phenomenon phenomena
datum data
medium media
curriculum curricula
memorandum memoranda
bacterium bacteria
nucleus nuclei
radius radii
stimulus stimuli
alumnus alumni
Do not use plural forms such as “criteria,” “phenomena,” “bacteria,” or
“alumni” as singular nouns.
INCORRECT: The first criteria in the selection was experience.
CORRECT: The first criterion in the selection was experience.

INCORRECT: He is an alumni of Harvard University.


CORRECT: He is an alumnus of Harvard University.
In contemporary usage, the plural forms “data” and “media” are often used
as singular nouns.
“data”
In the old days, English teachers taught that you must say “the data are,”
but today “the data is” has become so common that it cannot be considered
incorrect.
CORRECT: The data is inconclusive.
CORRECT: The data are inconclusive.

“media”
This word is still used as the plural of “medium.” Oil painting is one
“medium,” pastel is another “medium,” and oil painting and pastel are two
“media.” Do not use “media” as a singular this way:
INCORRECT: Television is the preferred media for political advertising.
CORRECT: Television is the preferred medium for political advertising.
However, “media” is today very often used as a singular noun to refer to all
the agencies of mass communication as an entity. This usage is so
widespread that it can no longer be considered incorrect.
CORRECT: Everywhere the princess goes, the media follow.
CORRECT: Everywhere the princess goes, the media follows.
Remember, however, that someone may object if you use “data” or “media”
as a singular noun. To be on the safe side, treat them as plurals, just like
“criteria” and “phenomena.”

© The MBA Center 213


Total PrepKit for the GMAT® Step 7.1 Lesson

Rule 17 Uncountable nouns have no plural form.


Nouns such as “money,” “gasoline,” “smog,” “wheat,” beauty,” “grammar,”
and “darkness” denote concepts that cannot be counted. Nouns of this type
go by various names: uncountable nouns, noncount nouns, mass nouns.
Whatever you call them, they have no plural form. Do not put -s on the end
of “information,” “advice,” “baggage,” or “luggage.”
INCORRECT: Each passenger is allowed two carry-on baggages.
CORRECT: Each passenger is allowed two pieces of carry-on
baggage.

INCORRECT: The counselor offered two advices.


CORRECT: The counselor offered two pieces of advice.

Rule 18 Some nouns have both a countable and an uncountable


meaning.

CORRECT: Business is booming.


CORRECT: It is always risky to start a new business.

CORRECT: Some doctors believe that red wine, in moderation, is good for
heart.
CORRECT: The wines of California have an excellent reputation.

Rule 19 There are some nouns that look plural but are in fact
singular.
Here are some nouns that end in -s in the singular form and are unchanged
in the plural:
means series species
INCORRECT: The two specimens look different but are actually members of
the same specie.
CORRECT: The two specimens look different but are actually members of
the same species.
Here are some nouns that look like plurals but generally act like
uncountable singulars:
news mathematics physics
measles statistics economics
INCORRECT: The news are bad.
CORRECT: The news is bad.

INCORRECT: Quantum physics were all that the guests discussed.


CORRECT: Quantum physics was all that the guests discussed.
Some of these words do have a countable, plural use:
CORRECT: Economics is a popular major.
CORRECT: The economics of the situation demand a tax cut.

CORRECT: Elementary statistics is a prerequisite for this course.


CORRECT: Statistics suggest that people drive more safely now than ten
years ago.

214 © The MBA Center


Grammar and Style Review

Rule 20 Collective nouns are usually singular.


In American usage, collective nouns such as “government,” “committee,”
“public,” “family,” “team,” “band,” “choir,” and “audience” are almost
always singular.
INCORRECT: The government were responsible.
CORRECT: The government was responsible.

INCORRECT: The Senator’s family have left town.


CORRECT: The Senator’s family has left town.

INCORRECT: The choir have arrived and are ready to sing.


CORRECT: The choir has arrived and is ready to sing.

It should be noted that, in British English, collective nouns are often treated
as plurals. The three “incorrect” versions above will sound just fine to most
British ears.

Rule 21 Collective nouns are sometimes used as expressions of


quantity, in which case they can be either singular or plural.
When designating a cohesive unit, a collective noun is singular. But when
a collective noun is used to designate individuals acting separately, it can be
plural.
CORRECT: After winning the championship, the team went their
separate ways.
This construction sounds funny to most American ears, and so the sentence
would normally be restated something like this:
CORRECT: After winning the championship, the team members went
their separate ways.
The word “group,” however, sounds fine as either a singular or a plural. You
can’t go wrong!
CORRECT: A group of people are waiting for the bus.
CORRECT: A group of people is waiting for the bus.

Rule 22 Amounts of money and units of measure are generally


singular.

INCORRECT: Two hundred dollars are not too much to spend for a business
suit.
CORRECT: Two hundred dollars is not too much to spend for a business
suit.

Rule 23 Titles and compounds that are thought of as units are


generally singular.
Sometimes the word “and” is just a connecting word in the middle of an
expression that is in reality felt to be a singular unit. Expressions such as
“bacon and eggs,” “gin and tonic,” and “pride and joy” are singular.
INCORRECT: Bacon and eggs were once considered a healthful breakfast.
CORRECT: Bacon and eggs was once considered a healthful breakfast.
Titles and names of organizations are generally singular.
INCORRECT: “Romeo and Juliet” are Shakespeare’s most popular play.
CORRECT: “Romeo and Juliet” is Shakespeare’s most popular play.

© The MBA Center 215


Total PrepKit for the GMAT® Step 7.1 Lesson

INCORRECT: The United Nations are debating the issue.


CORRECT: The United Nations is debating the issue.

Rule 24 Most indefinite pronouns are singular.


These indefinite pronouns are always singular:
each anything everything nothing
either anyone everyone no one
neither anybody everybody nobody

INCORRECT: Neither of the proposed solutions are likely to succeed.


CORRECT: Neither of the proposed solutions is likely to succeed.

Rule 25 The indefinite pronouns “none,” “all,” “any,” and “some”


can be either singular or plural.
It depends on the context.
CORRECT: The show has more than 12 original songs and all are tuneful
and engaging.
CORRECT: Most of the music is new, and all is tuneful and engaging.

Sometimes you have a choice.


CORRECT: None of the singers were especially memorable.
CORRECT: None of the singers was especially memorable.

Rule 26 Use “many” with countable nouns and “much” with


uncountable nouns.
Do not use “much” with a countable noun.
INCORRECT: The goal was realized at the cost of much sweat and tears.
CORRECT: The goal was realized at the cost of much sweat and many
tears.

Rule 27 Use “few/fewer/fewest” with countable nouns and


“little/less/least” with uncountable nouns.
Do not use “less” or “least” with a countable noun.
INCORRECT: The theater has the most empty seats on Monday nights and
the least on Saturday nights.
CORRECT: The theater has the most empty seats on Monday nights and
the fewest on Saturday nights.

Rule 28 Use “number” with countable nouns and “amount” with


uncountable nouns.
Do not use “amount” with a countable noun.
INCORRECT: Order was assured by the presence of a large amount of police.
CORRECT: Order was assured by the presence of a large number of
police.

216 © The MBA Center


Grammar and Style Review

EXERCISE 3 (NOUN NUMBER)

Directions: Rewrite the following sentences to correct any noun number errors.
(Note that some sentences may be correct as written.)

1. Anyone requiring additional informations should consult the company's website.

2. The United Nations have been debating a human rights resolution.

3. A group of people are waiting for the shuttle bus.

4. A solar eclipse is a phenomena that is worth a journey to witness.

5. The public are opposed to the proposed legislation.

6. This year’s promotional event attracted less people than last year’s did.

7. On Sundays, when admission is free, the amount of people at the


museum is astonishing.

8. Everyone sang well and were heartily applauded.

9. For the first time there were less species on the endangered list than in
the previous year.

10. None of students was able to complete the assignment in the allotted
time.

© The MBA Center 217


Total PrepKit for the GMAT® Step 7.1 Lesson

4. VERB TENSE
Rule 29 Use the past perfect (“had” + past participle) if one clause
takes place at an earlier time than another clause that is in
the past.

INCORRECT: The performance already started when they arrived.


CORRECT: The performance had already started when they arrived.

INCORRECT:The Civil War forced the United States to decide issues that the
framers of the Constitution raised but left unresolved.
CORRECT: The Civil War forced the United States to decide issues that the
framers of the Constitution had raised but left unresolved.
Do not use the past perfect unnecessarily.
INCORRECT:From 1920 to 1930, the number of automobiles in the state
had more than tripled.
CORRECT: From 1920 to 1930, the number of automobiles in the state
[...] more than tripled.

INCORRECT:In recent years, the corporate dress code had relaxed


considerably.
CORRECT: In recent years, the corporate dress code has relaxed
considerably.

Rule 30 Use the future perfect (“will have” + past participle) if one
clause takes place at an earlier time than another clause
that is in the future.

INCORRECT: By the time the lecture ends, it will stop raining.


CORRECT: By the time the lecture ends, it will have stopped raining.

Rule 31 Use the infinitive perfect (“to have” + past participle) if the
infinitive takes place at an earlier time than the rest of the
clause.

INCORRECT: The sculptor is believed to be born before 1400.


CORRECT: The sculptor is believed to have been born before 1400.

Rule 32 After “when” referring to a future event, use the present


tense.

INCORRECT: When the sun will come out, the temperature will rise.
CORRECT: When the sun comes out, the temperature will rise.

INCORRECT:When Jenkins will submit his report tomorrow morning, the


governor will have all the information needed to make a
decision.
CORRECT: When Jenkins submits his report tomorrow morning, the
governor will have all the information needed to make a
decision.

Rule 33 After “if” referring to a possible future event, use the


present tense, and in the other clause use “will” or “can.”

INCORRECT: If the sun comes out tomorrow, we go on a picnic.


CORRECT: If the sun comes out tomorrow, we will go on a picnic.

218 © The MBA Center


Grammar and Style Review

INCORRECT: If the value of the dollar be higher this summer than last, more
Americans will vacation abroad.
CORRECT: If the value of the dollar is higher this summer than last, more
Americans will vacation abroad.

INCORRECT: If our proposal will be accepted, we can get funding.


CORRECT: If our proposal is accepted, we can get funding.

Rule 34 In a sentence beginning with “if” or “when” and referring


to a scientific fact or general truth, you may use the present
tense in both clauses.

CORRECT: If a triangle has two equal sides, it also has two equal angles.
CORRECT: When water is heated to 100 degrees Celsius, it boils.
CORRECT: Whenever it rains more than an inch in one week, the river
floods.

Rule 35 To express a condition that is contrary to fact in the


present, use the past tense in the “if” clause, and use
“would” or “could” in the other clause.

(Technically, it’s not the past tense, but the contrary-to-fact conditional. But
the form is the same as the past tense for every verb except “to be.”)

INCORRECT: If the director has an assistant, the department runs more


smoothly.
CORRECT: If the director had an assistant, the department would run
more smoothly.

INCORRECT: If Young earns a bigger salary, the family would travel more.
CORRECT: If Young earned a bigger salary, the family would travel more.

Rule 36 In the case of the verb “be,” use “were” in the “if” clause
even for the singular.

INCORRECT: If this was a school with enormous enrollment, students would


have less personal contact with professors.
CORRECT: If this were a school with enormous enrollment, students
would have less personal contact with professors.
More and more often now one hears and reads “was” in this situation,
although “If I was you...” still sounds illiterate to most educated Americans.
To avoid offending the traditionalists, use “were.”

Rule 37 “Was” is correct in an “if” clause when it is expressing


uncertainty about the past, not contrary to fact.

INCORRECT: If the department head were on vacation last week, then she
will be relaxed for today’s meeting.
CORRECT: If the department head was on vacation last week, then she
will be relaxed for today’s meeting.

INCORRECT: If Martha were able to get a ticket to the opera, she probably
got home late last night.
CORRECT: If Martha was able to get a ticket to the opera, she probably
got home late last night.

© The MBA Center 219


Total PrepKit for the GMAT® Step 7.1 Lesson

Rule 38 To express a condition that is contrary to fact in the past,


use the past perfect (“had” + past participle) in the “if”
clause, and use “would have” + past participle in the other
clause.

INCORRECT: If the rain continued another hour, the river would have
overflowed its banks.
CORRECT: If the rain had continued another hour, the river would
have overflowed its banks.

Rule 39 Do not use “would” in the “if” clause.

INCORRECT: If writers would refer to the dictionary more often, there would
be fewer misspellings.
CORRECT: If writers referred to the dictionary more often, there would
be fewer misspellings.

INCORRECT: If the student would have studied harder, he would have


passed the test.
CORRECT: If the student had studied harder, he would have passed the
test.

Rule 40 In a “that” clause following a verb of recommendation,


request, or requirement, use the subjunctive.

The subjunctive form is the same for all subjects. There is no -s suffix in the
third person singular.

INCORRECT: The advisor recommended that the student repeats the test.
CORRECT: The advisor recommended that the student repeat the test.

INCORRECT: The Constitution requires that the President reports regularly


to Congress on the state of the nation.
CORRECT: The Constitution requires that the President report regularly
to Congress on the state of the nation.
The subjunctive form for the verb “be” is “be.”
INCORRECT: The management suggests that valuables are to be secured in
the safe.
CORRECT: The management suggests that valuables be secured in the
safe.

Rule 41 In a “that” clause following a noun or adjective derived


from a verb of recommendation, request, or requirement,
use the subjunctive.

INCORRECT: Next time the associates will take seriously our insistence that
everyone is on time for a meeting.
CORRECT: Next time the associates will take seriously our insistence that
everyone be on time for a meeting.

Rule 42 Do not use “should” or “must” with the subjunctive.

INCORRECT: The law requires that a motorcyclist must wear a helmet.


CORRECT: The law requires that a motorcyclist wear a helmet.

220 © The MBA Center


Grammar and Style Review

EXERCISE 4 (VERB TENSE)

Directions: Rewrite the following sentences to correct any verb tense errors.
(Note that some sentences may be correct as written.)

1. If two objects are dropped simultaneously from the same height, they
reach the ground at the same time.

2. If the stationery store was open yesterday, the office manager probably
bought some paper and pens.

3. If the team would have started the project a month earlier, they would
be finished by now.

4. If Americans would eat less red meat, they would have lower
cholesterol levels.

5. Financial experts recommend that one should diversify one’s


investments.

6. He resents the suggestion that he rewrites the paper.

7. He resents the suggestion that he plagiarized.

8. The lecturer began with a quick plot synopsis for the benefit of those
who did not read the novel.

9. The civilization is believed to flourish in the fifteenth century.

10. Once the student will pass the comprehensive oral examination, she
has satisfied all the requirements for the degree.

© The MBA Center 221


Total PrepKit for the GMAT® Step 7.1 Lesson

5. PRONOUNS
Rule 43 Pronoun reference must be logical and unambiguous.
A pronoun must generally have an antecedent.
INCORRECT:The President objected to several provisions and vetoed the
bill; however, Congress overrode it.
CORRECT: The President objected to several provisions and vetoed the
bill; however, Congress overrode the veto.
And it must be absolutely clear what the antecedent is. It is better to repeat
the noun than to make the reader figure out what you mean.
INCORRECT:It is not surprising that Jackson produced a more incisive
analysis that Jones did, because he has less firsthand
experience.
CORRECT: It is not surprising that Jackson produced a more incisive
analysis that Jones did, because Jones has less firsthand
experience.

Rule 44 Pronouns must agree with their antecedents.


Agreement is a matter of grammar, not reality. An antecedent is a noun. If
the noun is singular, any pronoun referring to it must be singular, even
when it designates more than one individual.
INCORRECT:The government should not spend money on space
exploration; they should pay more attention to citizens’ basic
needs.
CORRECT: The government should not spend money on space
exploration; it should pay more attention to citizens’ basic
needs.

Rule 45 Do not confuse subject and object pronouns.


“I,” “we,” “he,” “she,” “they,” and “who” are used as subjects. “Me,” “us,”
“him,” “her,” “them,” and “whom” are used as objects of verbs and
prepositions. Native speakers rarely have trouble with the distinction. You
know perfectly well never to say or write, “Me love she,” or, “Whom is
him?”
The distinction becomes a little more complicated when you have a
compound subject or object. Use subject pronouns for all parts of
compound subjects or complements. Use object pronouns for all parts of
compound objects. Do not let the word “and” or “or” confuse you.

INCORRECT:You teach a child to read, and he or her will be able to pass a


literacy test.
CORRECT: You teach a child to read, and he or she will be able to pass a
literacy test.

INCORRECT:Between you and I, the state of American education is


alarming.
CORRECT: Between you and me, the state of American education is
alarming.
The distinction between “who” and “whom” seems to be the most difficult
for native speakers. The incorrect versions of the following two sentences
are encountered more frequently than the correct versions.
INCORRECT: Who did the director choose?
CORRECT: Whom did the director choose?

222 © The MBA Center


Grammar and Style Review

INCORRECT: Who is the new computer for?


CORRECT: Whom is the new computer for?
“Whom” has virtually disappeared from the spoken language. As a matter
of fact, if you use “whom” in everyday speech, your listeners will think you
are showing off. In writing, however, you should use “whom” when it is
appropriate.
A good way to decide between “who” and “whom” is to try answering the
question with “he” or “him.” The answer to the question “Who/whom did
the director choose?” would be: “The director chose him.” Likewise, the
answer to the question “Who/whom is the new computer for?” would be:
“The computer is for him.” When “him” is the answer, “whom” is the
question.

Rule 46 Use subject pronouns after “it is” and “it was.”
“Be” is not a transitive verb, but a linking verb. A pronoun after “is” or
“was” is not a direct object, but a complement. Use the subject pronouns
after “is” and “was.”
INCORRECT: It is him who must decide.
CORRECT: It is he who must decide.

INCORRECT: It was them who won the competition.


CORRECT: It was they who won the competition.

Rule 47 Use the relative pronoun “which” for things only. Use “who”
or “whom” for people.
Do not use “which” for people.
INCORRECT: The film director which this reviewer most admires is
Hitchcock.
CORRECT: The film director whom this reviewer most admires is
Hitchcock.

Rule 48 In a subordinate clause, use “who” or “whom,” and


“whoever” or “whomever,” according to the pronoun’s
grammatical function in that clause.
The distinction between “who” and “whom” is even more difficult when
they are relative pronouns introducing subordinate clauses. In that case, it’s
the pronoun’s grammatical function within the subordinate clause that
determines the form to use.
INCORRECT: The protagonist, who the author calls Mr. X, is handsome and
mysterious.
In this sentence the relative pronoun refers to the subject “protagonist,” but
within the subordinate clause the pronoun is the object of the verb “calls.”
Therefore, the object pronoun “whom” is correct.
CORRECT: The protagonist, whom the author calls Mr. X, is handsome
and mysterious.
You can use the “he/him” test here, too. Take the clause that the pronoun
“who” or “whom” is to be a part of. Think of it as a question, and then
answer that question with “he” or “him.” In the above example, the relative
clause is “who/whom the author calls Mr. X.” Think of it as a question:
“Who/whom does the author call Mr. X?” The answer to that question is
“The author calls him Mr. X.” When the answer is “him,” the correct
pronoun is “whom.”

© The MBA Center 223


Total PrepKit for the GMAT® Step 7.1 Lesson

Here is another example:


INCORRECT: They will award the prize to him whom they truly believe
deserves it.
This time the relative pronoun refers to the object “him,” but within the
subordinate clause the pronoun is the subject of the verb “deserves.”
Therefore, the subject pronoun “who” is correct.
CORRECT: They will award the prize to him who they truly believe
deserves it.
Use the “he/him” test. The clause is “who/whom they truly believe deserves
it.” Think of it as a question: “Who/whom do they truly believe deserves
it?” The answer is “They truly believe he deserves it.” When the answer is
“he,” the correct pronoun is “who.”
Do not let the words “they truly believe” confuse you. That is a
grammatically irrelevant extra information clause. If you omit that phrase,
it may become clearer why “who” is correct.
CORRECT: They will award the prize to him who deserves it.

The same rule applies to the relative pronouns “whoever” and “whomever.”
It is the clauses they introduce that tell you which form to use.
INCORRECT: This letter is addressed to whoever you think it may concern.
CORRECT: This letter is addressed to whomever you think it may
concern.
Here the pronoun is the object of “may concern.” You would say “you think
it may concern him,” and so “whomever” is correct.
INCORRECT: This letter is addressed to whomever you think may be
interested.
CORRECT: This letter is addressed to whoever you think may be
interested.
This time the pronoun is the subject of “may be interested.” You would say
“you think he may be interested,” and so “whoever” is correct.

Rule 49 You may use either “which” or “that” for a restrictive


relative clause, but for a nonrestrictive relative clause you
must use “which.”
A restrictive clause is one that limits, that tells you which one. A restrictive
clause is essential to the meaning of a sentence. Some old-fashioned
grammarians insist that you must use “that” in restrictive clauses. Today it
is common and acceptable to use “which.”
CORRECT: “The Queen of Spades” is a story that had a profound
influence on all subsequent Russian prose.
CORRECT: “The Queen of Spades” is a story which had a profound
influence on all subsequent Russian prose.
A nonrestrictive clause is one that does not limit the word or phrase it refers
to. A nonrestrictive clause is not essential to the understanding of the
sentence, and is always set off from the rest of the sentence with commas.
Do not use “that” at the start of a nonrestrictive clause. You must use
“which.”
INCORRECT: David was born in the province of Saskatchewan, that is
located between Manitoba and Alberta.
CORRECT: David was born in the province of Saskatchewan, which is
located between Manitoba and Alberta.

224 © The MBA Center


Grammar and Style Review

Rule 50 Do not use “which” to refer to a whole idea.


“Which” should always have a specific antecedent.
INCORRECT: The material is expensive and the workmanship is faultless,
which is why the price is high.
CORRECT: The price is high because the material is expensive and the
workmanship is faultless.

Rule 51 If the antecedent is “everyone,” “everybody,” “anyone,” “no


one,” etc., the pronoun should be singular.
“Everyone” and “everybody” are particularly troublesome, in that they
denote more than one person but are grammatically singular.
INCORRECT: Everyone must do their part if democracy is to succeed.
CORRECT: Everyone must do his or her part if democracy is to succeed.

Rule 52 Do not use “they” or “them” to avoid specifying the sex of


a singular antecedent.
If casual conversation it is common to use “they” and “their” to refer to a
singular antecedent of unspecified sex. The intention is laudable, but the
method is ungrammatical.
INCORRECT: Someone phoned this morning, but they did not leave a
message.
CORRECT: Someone phoned this morning, but he or she did not leave a
message.
A good way to be both politically and grammatically correct is to use the
gender-neutral plural throughout the sentence.
INCORRECT: Every student is required to submit their final schedule by
October first.
CORRECT: All students are required to submit their final schedules
by October first.

Rule 53 Do not use “they” or “them” as a general person pronoun.


“They” should always have a specific antecedent.
INCORRECT: They just announced on the radio that the two biggest banks
in the state are merging.
CORRECT: It was just announced on the radio that the two biggest banks
in the state are merging.

Rule 54 Do not switch pronouns mid-sentence.


Use either “one” or “you” for the general person pronoun, but do not mix
them.
INCORRECT: If one works hard, you will succeed.
CORRECT: If one works hard, one will succeed.
CORRECT: If you work hard, you will succeed.

Rule 55 The possessive form of “one” is “one’s.”


Do not use “your,” “his,” “his or her,” or “their.”
INCORRECT: One should always remember to bring his or her calculator to
class.
CORRECT: One should always remember to bring one’s calculator to
class.

© The MBA Center 225


Total PrepKit for the GMAT® Step 7.1 Lesson

EXERCISE 5 (PRONOUNS)
Directions: Rewrite the following sentences to correct any pronoun errors. (Note
that some sentences may be correct as written.)

1. My partner invited my wife and me to join him and his wife for dinner.

2. No one will buy such an expensive product unless they really need it.

3. The directors are looking for someone who they can trust to abide by
the rules.

4. When one considers the candidate’s youth and inexperience, you have
to say that the victory was unexpected.

5. If the television viewer loses interest for just a few seconds, they will
switch channels.

6. It is sunny and hot today, but they say it will rain tomorrow.

7. If one invests early and wisely, one will be able to enjoy his or her
retirement.

8. Because the parties have been unable to resolve the dispute, a judge
will have to do it.

9. After questioning the two suspects, the investigator became convinced


that it was them who committed the crime.

10. The judges will award the scholarship to whoever they feel will benefit
the most.

226 © The MBA Center


Grammar and Style Review

6. MODIFIERS
Rule 56 Use adjectives to modify nouns, and adverbs to modify
anything else.
Adverbs are words that answer the questions “how?” “when?” and “where?”
Most adverbs of manner (adverbs that answer the question “how?”) are
formed by adding the suffix “-ly” to an adjective. There are, however, some
adverbs of manner that do not take the “-ly” suffix. The most important
ones are “fast,” “hard,” and “late.” There is no such word as “fastly,” and
the words “hardly” and “lately” have completely different meanings from
“hard” and “late.”
INCORRECT: The day went fastly.
CORRECT: The day went fast.
INCORRECT:The student worked hardly on the essay and got the good grade
she deserved.
CORRECT: The student worked hard on the essay and got the good grade
she deserved.
Do not use an adjective to modify a verb.
INCORRECT: He takes his career serious.
CORRECT: He takes his career seriously.
The adjective “good” must never be used as an adverb. The adverb is “well.”
INCORRECT: The soprano sang good.
CORRECT: The soprano sang well.

Rule 57 Use an adjective after a linking verb to modify the subject.


Use an adverb to modify the verb.
Verbs that describe a state of being (“be,” “remain,” and “become,” for
example) and verbs that relate to the five senses (“feel,” “seem,” “look,”
“sound,” “smell,” and “taste,” for example) are called linking verbs. After a
linking verb it makes sense to use an adjective because it is intended to
modify the subject.
INCORRECT: The reheated coffee smelled and tasted bitterly.
CORRECT: The reheated coffee smelled and tasted bitter.
INCORRECT: The show was canceled because the star felt badly.
CORRECT: The show was canceled because the star felt bad.
However, if the word is intended to modify the verb, use an adverb. Note
the differences in the following examples.
CORRECT: The evidence proved conclusive.
CORRECT: The evidence proved conclusively that the accused was guilty.
CORRECT: A suitcase left unattended will look suspicious.
CORRECT: Security agents will look suspiciously at a suitcase left
unattended.

Rule 58 Absolute adjectives (“unique,” “perfect,” “impossible,” etc.)


cannot be qualified with words such as “very” or “most.”
INCORRECT: Madagascar is the most unique tourist destination.
CORRECT: Madagascar is a unique tourist destination.

Rule 59 Avoid misplaced modifiers.


Modifying phrases must be placed unambiguously close to the element they
are modifying.

© The MBA Center 227


Total PrepKit for the GMAT® Step 7.1 Lesson

INCORRECT: Whether baked, fried, or mashed, everyone loves potatoes.


CORRECT: Everyone loves potatoes, whether baked, fried, or mashed.
INCORRECT: She met a man who owns three beach houses in Phoenix.
CORRECT: In Phoenix, she met a man who owns three beach houses.

Rule 60 Avoid squinting modifiers.


A modifying phrase must be placed unambiguously. A squinting modifier is
one that is situated between two possible referents. How is the reader to
decide? The following sentence, for example, can be interpreted two ways:
AMBIGUOUS: People who jog frequently develop knee problems.
It is not clear whether “frequently” modifies “jog” or “develop.” To make it
clear, rewrite the sentence.
CORRECT: Frequent joggers develop knee problems.
CORRECT: Joggers frequently develop knee problems.

Rule 61 Place limiting modifiers (“only,” “just,” “hardly,” “almost,”


etc.) immediately before the elements they modify.
Be especially careful with the placement of “only.” There is a tendency to
place it ambiguously before the verb no matter what it modifies.
AMBIGUOUS: The dean only takes appointments with undergraduates on
Mondays and Wednesdays.
Proper placement of “only” resolves the ambiguity.
CORRECT: The dean takes appointments only with undergraduates on
Mondays and Wednesdays.
CORRECT: The dean takes appointments with undergraduates only on
Mondays and Wednesdays.

Rule 62 Avoid dangling modifiers.


Modifying phrases must have something to modify. A dangling modifier is
one that has no referent in the sentence. It just hangs there.
INCORRECT:Having read the book and seen the film, there is no question
that the book is better.
CORRECT: Having read the book and seen the film, I can say with
certainty that the book is better.

Rule 63 An introductory modifying phrase must modify the subject


of the sentence.
When a modifying phrase is placed at the start of a sentence, the phrase is
felt to modify the subject of the sentence, which should generally be found
right after the comma.
INCORRECT:With a population of more than five million, some visitors
find the city overwhelming.
CORRECT: With a population of more than five million, the city
overwhelms some visitors.
An introductory modifying phrase cannot modify a noun in the possessive
(-’s) form. A modifying phrase at the beginning of a sentence is felt to
modify the subject, not the subject’s possessor.
INCORRECT:Having recently secured several lucrative contracts, Goldberg’s
reputation in the office is on the rise.
CORRECT: Having recently secured several lucrative contracts, Goldberg
enjoys his reputation in the office being on the rise.

228 © The MBA Center


Grammar and Style Review

EXERCISE 6 (MODIFIERS)
Directions: Rewrite the following sentences to correct any modifier errors. (Note
that some sentences may be correct as written.)

1. Driving quickly became his favorite pastime.

2. She tries to attract men with makeup and perfume.

3. Born and raised near the Mexican border, Spanish came easily to her.

4. There is a sculpture on the facade that represents the rise of technology.

5. He hired a crew with a truck that is accustomed to handling delicate


antiques.

6. Having decided to get the story at any cost, nothing could stop the
reporter.

7. Straight out of the oven and still piping hot, the family devoured the
whole pie.

8. Even after providing explicit instructions, the new employee did not
know what to do.

9. Dressed in a T-shirt and jeans, the supervisor ordered the assistant to


go home and change into more professional attire.

10. No longer sworn enemies, relations between the United States and
Russia have improved considerably since the end of the Cold War.

© The MBA Center 229


Total PrepKit for the GMAT® Step 7.1 Lesson

7. COMPARISONS
Rule 64 Items that are compared must be logically comparable.
A carefully written sentence will compare a quality to a quality, a quantity
to a quantity, or a thing to thing. Make sure the sentence does not, when
taken literally, actually compare a quality to a thing or a quantity to a thing.
INCORRECT: The taste of the artificial substitute is indistinguishable from
the real thing.
CORRECT: The taste of the artificial substitute is indistinguishable from
that of the real thing.

INCORRECT: The population of California is greater than Canada.


CORRECT: The population of California is greater than that of Canada.
If a sentence compares two things, make sure they are comparable.
INCORRECT: The operas of Verdi are more tuneful than Wagner.
CORRECT: The operas of Verdi are more tuneful than those of Wagner.

INCORRECT: The ink cartridges for the new printer cost twice much as the
old printer.
CORRECT: The ink cartridges for the new printer cost twice much as
those for the old printer.

Rule 65 Items that are compared must be grammatically


comparable.

INCORRECT: The mother envies her son because he has had more
opportunities than her.
CORRECT: The mother envies her son because he has had more
opportunities than she.

Rule 66 Avoid incomplete comparisons. A comparison must be clear


and unambiguous.

INCORRECT: Martin likes asparagus more than his wife.


CORRECT: Martin likes asparagus more than his wife does.
CORRECT: Martin likes asparagus more than he does his wife.

INCORRECT: Executives are more concerned about profits than employees.


CORRECT: Executives are more concerned about profits than employees
are.
CORRECT: Executives are more concerned about profits than about
employees.

Rule 67 When comparing something with other members of the


same group, use the word “other” or “else.”

INCORRECT: The population of California is greater than that of any


American state.
CORRECT: The population of California is greater than that of any other
American state.

INCORRECT: Tanya earns a bigger salary than anyone in her family does.
CORRECT: Tanya earns a bigger salary than anyone else in her family
does.

230 © The MBA Center


Grammar and Style Review

Rule 68 Use the comparative form of the adjective with two items.
Use the superlative form with three or more items.

INCORRECT:Her children are both in college, the oldest at Harvard and the
youngest at Yale.
CORRECT: Her children are both in college, the older at Harvard and the
younger at Yale.

INCORRECT:Of all the films this reviewer has seen in the past year, this one
is the more memorable.
CORRECT: Of all the films this reviewer has seen in the past year, this one
is the most memorable.

Rule 69 Use the “-er” and “-est” suffixes to form the comparative
and superlative of one-syllable adjectives and some two-
syllable adjectives. Use “more” and “most” for longer
adjectives.
For adjectives of one syllable, the comparative/superlative is formed with
the “-er/-est” suffixes.
tall taller tallest
big bigger biggest
For adjectives of three or more syllables, the comparative/superlative form
is “more/most” + adjective.
beautiful more beautiful most beautiful
expensive more expensive most expensive
Most two-syllable adjectives also take the “more/most” form, but some take
the “-er/-est” form.
cheerful more cheerful most cheerful
happy happier happiest
A few adjectives have irregular comparative and superlative forms.
good better best
bad worse worst
far farther farthest
many more most
much more most
little less least

Rule 70 Use the short comparative or superlative form if it exists.


Otherwise, use the “more” or “most.”
Do not invent “-er” and “-est” forms.
INCORRECT: The interior of the island is peacefuler than the coast.
CORRECT: The interior of the island is more peaceful than the coast.

Rule 71 Do not use “more/most” or “less/least” with the “-er/est”


form.
Use one or the other. Never use both on the same adjective.
INCORRECT:It would be more easier to do business with Johnson if he were
a more pleasanter person.
CORRECT: It would be easier to do business with Johnson if he were a
more pleasant person.

© The MBA Center 231


Total PrepKit for the GMAT® Step 7.1 Lesson

EXERCISE 7 (COMPARISONS)
Directions: Rewrite the following sentences to correct any comparison errors. (Note
that some sentences may be correct as written.)

1. Americans eat more corn than Europeans.

2. The second of the author’s two published works sold best.

3. The critic maintained that it was the worse film she ever saw.

4. Unlike Los Angeles, most New Yorkers use public transportation.

5. Every year, more foreign tourists visit Paris than any European city.

6. The United States is the most rich and wasteful country in the world.

7. Alice may be younger than her husband, but she is more experienced
than he.

8. His job title is director of marketing, but his salary is still less than a
secretary.

9. Abraham Lincoln was perhaps one of the most wisest men ever to serve
as President.

10. Choose either regular or express service: the former is least expensive
and the latter is fastest.

232 © The MBA Center


Grammar and Style Review

8. PARALLELISM
Rule 72 Items in a list must be parallel.
If the first item in a list is a noun or a noun phrase, all items must be nouns
or noun phrases.
INCORRECT: Last month’s drop in sales was due to increased competition,
poor publicity, and because many clients were away for the
holidays.
CORRECT: Last month’s drop in sales was due to increased competition,
poor publicity, and the fact that many clients were away for
the holidays.
If the first item in a list is a conjugated verb or verb phrase, all items must
be similarly conjugated verbs or verb phrases.
INCORRECT: Our team is successful because we communicate effectively,
work well together, and we respect one another.
CORRECT: Our team is successful because we communicate effectively,
work well together, and [...] respect one another.
If the first item in a list is a gerund construction, all items must be gerund
constructions.
INCORRECT: Air travel means sitting in the same position for hours,
watching a bad print of a tiresome film, eating food that has
no flavor, and you have to refrain from smoking.
CORRECT: Air travel means sitting in the same position for hours,
watching a bad print of a tiresome film, eating food that has
no flavor, and refraining from smoking.
If the first item in a list is a prepositional phrase, all items must be
prepositional phrases.
INCORRECT: He searched for the lost item everywhere: in the house, car, and at
the office.
CORRECT: He searched for the lost item everywhere: in the house, in the car,
and at the office.

Rule 73 If a preposition is repeated once in a list, it must be


repeated every time.

INCORRECT: The train stops in Stamford, Bridgeport, and in New Haven.


CORRECT: The train stops in Stamford, in Bridgeport, and in New
Haven.
CORRECT: The train stops in Stamford, [...] Bridgeport, and [...] New
Haven.
The same goes for the function word “to” in infinitives. If it is repeated
once, it must be repeated every time.
INCORRECT: His dream is to graduate from MIT, develop an original theory,
and to win a Nobel prize.
CORRECT: His dream is to graduate from MIT, to develop an original
theory, and to win a Nobel prize.
CORRECT: His dream is to graduate from MIT, [...] develop an original
theory, and [...] win a Nobel prize.

© The MBA Center 233


Total PrepKit for the GMAT® Step 7.1 Lesson

Rule 74 In the structures “both X and Y,” “either X or Y,” and


“neither X nor Y,” X and Y must be parallel.

INCORRECT:Architects today must consider both how much a material


costs and its durability.
CORRECT: Architects today must consider both a material’s cost and
its durability.
CORRECT: Architects today must consider both how much a
material costs and how long it will last.

INCORRECT:The speaker will go either by train or fly to the conference in


Philadelphia.
CORRECT: The speaker will go either by train or by plane to the
conference in Philadelphia.
CORRECT: The speaker will either go by train or fly to the conference
in Philadelphia.

Rule 75 In the structures “not X but Y” and “not only X but also Y,”
X and Y must be parallel.

INCORRECT:The restaurant’s success is due not to the quality of food but


where it is located.
CORRECT: The restaurant’s success is due not to the quality of food
but to its location.

INCORRECT:Thomas Edison was not only a brilliant inventor, but he was


also a shrewd businessman.
CORRECT: Thomas Edison was not only a brilliant inventor but also
a shrewd businessman.

Rule 76 When you use “be” to equate or compare two elements, the
elements must be parallel.
A noun can be likened to a noun:
CORRECT: Vanilla is more popular than chocolate.
A gerund can be compared to a gerund:
CORRECT: Seeing is believing.
An infinitive can be compared to an infinitive:
CORRECT: It is better to give than to receive.
Avoid mixing grammatical forms in a comparison:
INCORRECT:Having a spouse with whom one can openly discuss failure
and self-doubt is to be happily married.
CORRECT: To have a spouse with whom one can openly discuss failure
and self-doubt is to be happily married.

INCORRECT:To learn a foreign language means discovering another way to


categorize reality.
CORRECT: Learning a foreign language means discovering another
way to categorize reality.
Do not use a clause beginning with “when” to define a noun:
INCORRECT: The novel’s saddest event is when the heroine dies.
CORRECT: The novel’s saddest event is the death of the heroine.

234 © The MBA Center


Grammar and Style Review

EXERCISE 8 (PARALLELISM)
Directions: Rewrite the following sentences to correct any parallelism errors. (Note
that some sentences may be correct as written.)

1. To be a celebrity means having no privacy.

2. The meeting is not scheduled for today but tomorrow.

3. The dieter resolved neither to eat too much nor too little.

4. He is not only responsible for the design, but also the execution.

5. The turning point in the plot is when the protagonist disappears.

6. A teacher’s job is to inspire creativity as well as imparting knowledge.

7. The artist is neither naturally talented nor is she independently


wealthy.

8. The two biggest events in his life were when he got married and when
his first child was born.

9. By the time Baldwin arrived to help, his colleagues had already laid out
the site, cleared the land, and began to excavate.

10. The changeover to the metric system did not succeed because people
could not become accustomed to using meters instead of feet, grams
instead of ounces, or to using liters instead of quarts.

© The MBA Center 235


Total PrepKit for the GMAT® Step 7.1 Lesson

9. IDIOMS
Rule 77 “From” goes with “to,” and “between” goes with “and.”
Do not use “between” with “to.”
INCORRECT: Between 1980 to 1990, the population of Centerville grew by
50 percent.
CORRECT: From 1980 to 1990, the population of Centerville grew by
50 percent.
CORRECT: Between 1980 and 1990, the population of Centerville grew
by 50 percent.
No preposition is needed with a hyphen to indicate a range of dates.
INCORRECT: The symposium will take place in the nation’s capital between
August 11–16.
CORRECT: The symposium will take place in the nation’s capital [...]
August 11–16.

Rule 78 “Either” goes with “or,” and “neither” goes with “nor.”
Do not use “neither” with “or.”
INCORRECT: Neither profit warnings or gloomy unemployment news
deterred investors.
CORRECT: Neither profit warnings nor gloomy unemployment news
deterred investors.
Do not use “either” with “nor.”
INCORRECT: The young scholar did not intend either to contradict his
mentor’s theses nor even to question them.
CORRECT: The young scholar did not intend either to contradict his
mentor’s theses or even to question them.

Rule 79 “Not only” is always followed by “but also.”


A common mistake is to follow “not only” with a mere “but.” Do not forget
the “also.”
INCORRECT: Alaska is not only the biggest American state in area, but the
smallest in population.
CORRECT: Alaska is not only the biggest American state in area, but
also the smallest in population.

Rule 80 “Different” is normally followed by “from.”


In casual English it is acceptable to say or write “different than” before a
clause. But in SWAE it is always “different from.”
INCORRECT: College life is different than most new students expect.
CORRECT: College life is different from what most new students expect.
It is true that in Britain one hears and reads “different to,” but in the U.S. it
is “different from.”
In any case, the phrase “different than” is almost never correct. Remember
that “than” is used only with comparatives — words with “more” in front
of them or with “-er” attached at the end. “Different than” can be correct
only when it is “more different than”:
CORRECT: She knew serving in the Senate would be different from serving
in the House of Representatives, but it turned out to be more
different than she expected.

236 © The MBA Center


Grammar and Style Review

Rule 81 “Lay” is transitive. “Lie” is intransitive.


Use “lie” when there is no direct object.
INCORRECT: She felt ill and decided to go lay down for a while.
CORRECT: She felt ill and decided to go lie down for a while.
What makes “lay” and “lie” so difficult to distinguish is that the word “lay”
is also the past tense of “lie.” (Now isn’t that confusing!) The simple past
and past participle forms for “lie” are “lay” and “lain.” The simple past and
past participle forms for “lay” are “laid” and “laid.”
INCORRECT: He went to the beach and laid in sun all day.
CORRECT: He went to the beach and lay in sun all day.
INCORRECT: He has lain all his cards on the table.
CORRECT: He has laid all his cards on the table.

Rule 82 “Raise” is transitive. “Rise” is intransitive.


Use “rise” when there is no direct object.
INCORRECT: If the embargo is not lifted, prices will continue to raise.
CORRECT: If the embargo is not lifted, prices will continue to rise.
The past forms for “rise” are “rose” and “risen.” The past forms of “raise” are
“raised” and “raised.”
INCORRECT: Because of the embargo, fuel companies have risen prices.
CORRECT: Because of the embargo, fuel companies have raised prices.

Rule 83 Use the idiomatic preposition after a verb.


The vast majority of verb + preposition idioms are no problem. A native
speaker knows perfectly well not to say “participate to the event” or
“approve at the choice.” But even smart native speakers have trouble with
a few, including “prefer + to,” “prevent + from,” “prohibit + from,” and
“credit + with.”
“prefer to”
Do not write “prefer over” or “prefer as opposed to.” It is “prefer to.”
INCORRECT: Our customers prefer chocolate over vanilla.
CORRECT: Our customers prefer chocolate to vanilla.
“prevent from”
You do not “prevent” someone “to do” something. You “prevent” someone
“from doing” something.
INCORRECT: The fence was erected to prevent cattle to escape.
CORRECT: The fence was erected to prevent cattle from escaping.
“prohibit from”
You do not “prohibit” someone “to do” something. You “prohibit”
someone “from doing” something.
INCORRECT: A new law prohibits one to use a mobile phone while driving.
CORRECT: A new law prohibits one from using a mobile phone while
driving.
“credit with”
You do not “credit” someone “as having done” something. You “credit”
someone “with” something.
INCORRECT: Schoenberg is credited as having invented the dodecaphonic
system of musical composition.
CORRECT: Schoenberg is credited with the invention of the
dodecaphonic system of musical composition.

© The MBA Center 237


Total PrepKit for the GMAT® Step 7.1 Lesson

Some verbs take different prepositions for different meanings.


“differ with” and “differ from”
When people disagree, they “differ with” each other. When things are not
alike, they “differ from” each other.
INCORRECT:The son differs from his mother on the merits of movie
musicals.
CORRECT: The son differs with his mother on the merits of movie
musicals.
INCORRECT:The son differs with his mother in that he is creative and
talented.
CORRECT: The son differs from his mother in that he is creative and
talented.
“compare to” and “compare with”
To “compare to” is to point out the similarities. To “compare with” is to
point out the differences.
CORRECT: Shakespeare asked his lover, “Shall I compare thee to a
summer’s day?”
CORRECT: When we compare this year’s numbers with last year’s, we see
that revenues are up and expenses are down.
Do not use “compare to” when the comparison is about differences.
INCORRECT: Attendance is up, compared to last year.
CORRECT: Attendance is up, compared with last year.

Rule 84 Use “like” to compare nouns. Use “as” to compare clauses.


In comparisons, “like” is a preposition, and “as” is a conjunction. What
follows a preposition is a noun, and what follows a conjunction is a clause
with a conjugated verb.
INCORRECT:As many working parents, Jones usually rises before dawn and
retires well after midnight.
CORRECT: Like many working parents, Jones usually rises before dawn
and retires well after midnight.
INCORRECT: This peach is firm but juicy, like a ripe peach should be.
CORRECT: This peach is firm but juicy, as a ripe peach should be.
“As” as a preposition means “in the capacity of.”
CORRECT: As a friend and financial advisor, he recommended selling.

Rule 85 Use “such as” to introduce a list of examples. Do not use


“like.”
There is a difference between “such as” and “like.” If you say you went to a
party and saw “people such as Bill Clinton and Madonna,” then Bill Clinton
and Madonna were there. But if you say you went to a party and saw
“people like Bill Clinton and Madonna,” then Bill Clinton and Madonna
were probably not there. You just saw people that were “like” them.
INCORRECT:Nineteenth-century Russian writers produced great epic novels
like “Crime and Punishment” and “War and Peace.”
CORRECT: Nineteenth-century Russian writers produced great epic novels
such as “Crime and Punishment” and “War and Peace.”

Rule 86 The correct expressions are: “with regard to,” “in regard
to,” and “as regards.”
In the expressions “with regard to” and “in regard to,” the word “regard” is

238 © The MBA Center


Grammar and Style Review

a noun. In “as regards,” the word “regards” is a verb. In SWAE, “with regards
to” and “in regards to” are unidiomatic.
INCORRECT:The architectural firm has filed several complaints with
regards to city zoning laws.
CORRECT: The architectural firm has filed several complaints with
regard to city zoning laws.
All of these expressions are stuffy and wordy. Usually a simple “concerning”
or “about” is enough.
CORRECT: The architectural firm has filed several complaints about city
zoning laws.

Rule 87 Use “between” with two items. Use “among” with three or
more.

INCORRECT:The search committee has narrowed it down to a choice


between Jefferson, Kilmer, and Nardo.
CORRECT: The search committee has narrowed it down to a choice
among Jefferson, Kilmer, and Nardo.

Rule 88 When referring to an alternative, “whether” is better than


“if.”
“If” is common in everyday English, but “whether” is preferred in SWAE.
INCORRECT:Scholars have been unable to determine if the frescoes were
painted by Giotto.
CORRECT: Scholars have been unable to determine whether the frescoes
were painted by Giotto.

Rule 89 Be careful with commonly confused words.


Here are a few sets of words that even smart people sometimes confuse.
“affect” and “effect”
“Affect” is usually a verb, meaning “influence.” The noun form of that
meaning is “effect.” In other words, to “affect” means to “have an effect.”
INCORRECT:Farmers wonder how the winter drought will effect next
summer’s crops.
CORRECT: Farmers wonder how the winter drought will affect next
summer’s crops.
CORRECT: Farmers wonder what effect the winter drought will have on
next summer’s crops.
As a verb, “effect” means “do” or “cause.”
CORRECT: It is always difficult to effect fundamental change.
As a noun, “affect” means “consciousness of emotion.” Unless you are
writing about psychology, you will probably never need to use “affect” as a
noun.

“alternately” and “alternatively”


Use “alternately” to describe an action that switches back and forth. Use
“alternatively” when you are suggesting another option.
CORRECT: The weather was unsettled, alternately sunny and rainy.
CORRECT: The company could declare bankruptcy. Alternatively, it
could restructure, consolidate, and perhaps emerge from this
troubled period.

© The MBA Center 239


Total PrepKit for the GMAT® Step 7.1 Lesson

Do not confuse the words.


INCORRECT: Because of the scholarship offer, she will probably go to
Princeton; alternately, she could take out a loan and go to
Harvard.
CORRECT: Because of the scholarship offer, she will probably go to
Princeton; alternatively, she could take out a loan and go
to Harvard.

“appraise” and “apprise”


“Appraise” means “estimate, judge the value of.” “Apprise” means
“inform.”
CORRECT: Before deciding whether to move, they had their house
appraised.
CORRECT: This notice is to apprise our faithful customers of a change in
opening hours.
Do not confuse the words.
INCORRECT: He asked his friend to keep him appraised of any further
developments.
CORRECT: He asked his friend to keep him apprised of any further
developments.

“assure,” “ensure,” and “insure”


“Assure” means “promise.”
CORRECT: The clerk assured her that the manager would see her shortly.
“Ensure” means “make certain.”
CORRECT: Proper planning will ensure success.
“Insure” means “guarantee legally or financially.”
CORRECT: Our firm is insured against theft.

“continually” and “continuously”


Something that happens repeatedly can be said to happen “continually.”
Something that happens without interruption can be said to happen
“continuously.”
CORRECT: Reporters continually ask questions about private matters.
CORRECT: The factory never closes because the furnaces must burn
continuously.
Do not confuse the words.
INCORRECT: It snowed continually for 24 hours.
CORRECT: It snowed continuously for 24 hours.

“discreet” and “discrete”


These are two different words. They are pronounced the same but have
distinct meanings. “Discreet” means “prudent.”
CORRECT: The officer observed the suspect from a discreet distance.
“Discrete” means “separate, distinct.”
CORRECT: The manager broke the project down into discrete tasks.

“farther” and “further”


“Farther” refers to a greater distance. “Further” refers to a continuation.
CORRECT: He ran farther than he thought he could.
CORRECT: They decided to discuss the matter further over dinner.

240 © The MBA Center


Grammar and Style Review

“imply” and “infer”


To “imply” is to suggest; to “infer” is to read into. The speaker or writer
implies. The listener or reader infers.
CORRECT: She did not say so directly, but she implied that she would be
available to help.
CORRECT: The reader can infer from the passage that Raphael and
Michelangelo were contemporaries.
Do not use “infer” when you mean “imply.”
INCORRECT: Although he does not say so explicitly, the writer of this
editorial infers that capital punishment ought to be abolished.
CORRECT: Although he does not say so explicitly, the writer of this
editorial implies that capital punishment ought to be
abolished.
“incredible” and “incredulous”
“Incredible” means “unbelievable.” “Incredulous” means “unbelieving,
skeptical.”
CORRECT: After he told an incredible story about having been abducted
by aliens, his listeners were incredulous.
In formal writing, do not use “incredible” to mean “excellent.”
INCORRECT: All were sated after the incredible meal.
CORRECT: All were sated after the excellent meal.
A story or a fact can never be “incredulous.” Only people can be
“incredulous.”
INCORRECT: The film is said to be based on fact, but I found it incredulous.
CORRECT: The film is said to be based on fact, but I found it incredible.

Rule 90 Be careful with commonly misused words.


“anticipate”
To “anticipate” is to “act in advance of, prepare for.”
CORRECT: Residents anticipated potentially damaging winds by boarding
up windows.
Do not use “anticipate” to mean “expect.”
INCORRECT: We do not anticipate any further delays.
CORRECT: We do not expect any further delays.
“anxious”
To be “anxious” is to be “worried.”
CORRECT: The patient was anxious about the results of the blood test.
Do not use “anxious” to mean “eager, excited.”
INCORRECT: After waiting months for approval, the team was anxious to
get started.
CORRECT: After waiting months for approval, the team was eager to get
started.
“could care less”
For some reason the trite expression “couldn’t care less” is often shortened
to the illogical “could care less.”
INCORRECT: More apathetic than ever, voters could care less who wins the
election.
CORRECT: More apathetic than ever, voters could not care less who wins
the election.

© The MBA Center 241


Total PrepKit for the GMAT® Step 7.1 Lesson

You can get away with the expression in speech, but do not write “could
care less” unless that is really what you mean.
CORRECT: I do not care enormously about the Academy Awards, but I
could care less.
“due to”
Use “due to” only after the verb “be.”
CORRECT: This morning’s departure delays are due to inclement weather.
Do not use “due to” at the beginning of a sentence or anywhere else to
mean “because of.”
INCORRECT:Due to circumstances beyond our control, the event has been
canceled.
CORRECT: Because of circumstances beyond our control, the event has
been canceled.
“epitome”
The “epitome” of something is the most ordinary, typical example.
CORRECT: The epitome of the suburban commuter, he spends four hours
a day in heavy traffic.
It is a common mistake to use “epitome” to mean “extreme example.”
INCORRECT: He is the epitome of a baseball fan: nobody knows more about
the game than he.
CORRECT: He is the extreme example of a baseball fan: nobody
knows more about the game than he.
“ironic”
An event that is the opposite of what one would expect is “ironic.”
CORRECT: It is ironic that such a sinner would become such an
accomplished religious painter.
Do not use “ironic” to mean “a curious coincidence.”
INCORRECT: It is ironic that Verdi and Wagner, the antipodes of nineteenth-
century opera composers, were born in the same year.
CORRECT: It is a curious coincidence that Verdi and Wagner, the
antipodes of nineteenth-century opera composers, were born in
the same year.
“literally”
Taken literally, “literally” means “actually,” or “exactly as the words say.”
CORRECT: He took the boss’s advice literally and jumped in a lake.
Do not use “literally” in a weak attempt at emphasis. In particular, do not
use “literally” when you really mean “figuratively.” The best solution is
usually just to delete it.
INCORRECT: Congressional leaders literally held a gun to the President’s
head.
CORRECT: Congressional leaders [...] held a gun to the President’s head.

“peruse”
Contrary to popular usage, “peruse” means “read thoroughly.” Do not use
“peruse” to mean “glance over, skim, read superficially.”
INCORRECT: The department head did not have time to read the report
thoroughly, so she just perused it.
CORRECT: The department head did not have time to peruse the report,
so she just skimmed it.

242 © The MBA Center


Grammar and Style Review

EXERCISE 9 (IDIOMS)
Directions: Rewrite the following sentences to correct any idiom errors. (Note that
some sentences may be correct as written.)

1. There were something between twenty to thirty thousand people in


attendance.

2. The candidate is not inclined either to debate his opponent face to face
nor even to respond to questions about the accusations.

3. Fletcher is not only the youngest member of the team, but, with three
university degrees, he is the most educated.

4. The world is a totally different place now than it was before the advent
of the personal computer.

5. The children were anxious to start eating, but they knew they had to
wait until everyone was served.

6. The director of human resources uses quaint Southern colloquialisms


like “fixin’ to.”

7. Just like a child does, he uses his finger to point at the words as he
reads them.

8. Her elocution coach taught her to pronounce her r’s like they are
pronounced in Britain.

9. The experience of war in the trenches had a profound affect on the


young English poets.

10. The jury must first decide if the defendant is telling the truth.

© The MBA Center 243


Total PrepKit for the GMAT® Step 7.1 Lesson

10. STYLE
Rule 91 Avoid redundancy.
It is bad form to repeat ideas unnecessarily.
INCORRECT: We are ready for any unforeseen event that may or may not
occur.
CORRECT: We are ready for any unforeseen event [...].

Some common examples of redundancy are simply forbidden in SWAE.


Editors and essay graders shriek at the sight of the following notorious
bugbears:
reason is because equally as
reason why hardly never
return back each and every
repeat again in any way, shape, or form
INCORRECT: The reason the business failed was because of mis-
management.
CORRECT: The business failed because of mismanagement

Here are some more examples of redundancies to be avoided:


advance planning five in number
consensus of opinion PIN number
safe haven ATM machine
blue-colored GMAT test

Rule 92 Omit unnecessary words.


Avoid wordiness. If you can delete a word or series of words without
changing or obscuring the meaning, do so. Do not, for example, write
“considered as” or “considered to be” when “considered” will do.
INCORRECT: He is considered to be a leading expert on Etruscan art.
CORRECT: He is considered [...] a leading expert on Etruscan art.
And why write “whether or not” when “whether” means the same thing?
INCORRECT: The committee will decide whether or not to call witnesses.
CORRECT: The committee will decide whether [...] to call witnesses.
Avoid flabby expressions.
as of yet
at this point in time
due to the fact that
in spite of the fact that
of a ____ nature
in a _____ manner

INCORRECT: In spite of the fact that the director is more than eighty years
of age, the film which he recently directed can been said to be
fresh as well as fast, and it is funny also.
CORRECT: Although the director is more than eighty years old, his film is
fresh, fast, and funny.

Rule 93 Do not omit necessary words.


Economy is a virtue in SWAE. You want to use as few words as possible to
express your meaning. If a word is unnecessary — if the sentence has the
same meaning without it — delete it. But do not omit words that are
grammatically or logically necessary.

244 © The MBA Center


Grammar and Style Review

INCORRECT: This year’s event should be as big if not bigger than last year’s.
CORRECT: This year’s event should be as big as, if not bigger than, last
year’s.
BETTER: This year’s event should be at least as big as last year’s.
INCORRECT: This team has not yet and never will surrender.
CORRECT: This team has not yet surrendered and never will surrender.

Rule 94 Avoid slang and colloquialisms.


You probably know better than to write something like “he ain’t got none”
or “That kid is awesome.” Slang obviously has no place in SWAE. The
dividing line between formal and casual is a fuzzy one. The following words
are usually too casual for SWAE:
pretty (as in “a pretty tough question”) a lot
real (as in “a real important job”) nice
awful or awfully stupid
INCORRECT: Ten thousand dollars is an awful lot of money.
CORRECT: Ten thousand dollars is a great deal of money.
INCORRECT: This company is a pretty nice place to work.
CORRECT: This company is a rather pleasant place to work.

Rule 95 Be careful with jargon and trendy expressions.


Conservative editors and English teachers despise fashionable business-
speak. They cringe at words like “prioritize,” “proactive,” and “outsource.”
But these words are so widely used and understood that they have to be
accepted now. Use one of these words if it expresses your meaning precisely
and unambiguously, but be aware that some old-fashioned readers may
object. As always, consider your audience.
But a line has to be drawn somewhere. The following examples cross that
line and are unacceptable.
INCORRECT: The job was in-sourced because we had all the people we
needed.
CORRECT: We did the job in house because we had all the people we
needed.
INCORRECT: Because of budget cuts, the department had to dehire several
employees.
CORRECT: Because of budget cuts, the department had to lay off several
employees.
Conservatives particularly dislike the wanton use of nouns as verbs.
INCORRECT: We want to know how a tax cut would impact the economy.
CORRECT: We want to know what impact a tax cut would have on the
economy.

Rule 96 Prefer the active to the passive.


A passive sentence construction is one that turns an object into a subject.
“John ate the cake” is active. “The cake was eaten by John” is passive.
Sometimes the passive construction is appropriate, particularly when
the true subject is unknown.
CORRECT: Twelve paintings were stolen.
But when the true subject is known, it is usually better to make it the
grammatical subject as well.
INCORRECT: One thousand likely voters were interviewed by the pollsters.

© The MBA Center 245


Total PrepKit for the GMAT® Step 7.1 Lesson

CORRECT: The pollsters interviewed one thousand likely voters.


Do not write “able to be.”
INCORRECT: This problem is not able to be solved.
CORRECT: This problem cannot be solved.

Rule 97 Avoid weak intensifiers.


Writers often use words such as “very” or “really” in a vain effort to render
their writing more forceful or profound. Be sparing with “very.” And watch
out for “really,” “incredibly,” “totally,” and “absolutely.” These feeble
attempts to make bolster your words can sometimes lead to nonsense.
INCORRECT: The story was incredibly believable.
CORRECT: The story was [...] realistic.

Rule 98 Opt for the specific over the vague.


When you are tempted to use a vague word such as “interesting” or
“important,” consider whether there is a more specific word to use instead.
WEAK: That was a very interesting play.
STRONG: That was a thought-provoking play
Look at the following sentence from a White House aide’s testimony at a
Congressional hearing.
WEAK: I was provided with additional input that was radically
different from the truth. I assisted in furthering that version.
This speaker uses vague expressions intentionally. He wants to soften (if not
obscure) the truth, which is simply: “They lied, and I helped.”
Rule 99 Avoid illogical and meaningless sentences.
Here are several gems of illogical and meaningless nonsense that have been
attributed to various U.S. politicians:
NONSENSE: If we don’t succeed, we run the risk of failure.
NONSENSE: When people are out of work, unemployment results.
NONSENSE: More and more of our imports come from overseas.
NONSENSE: It isn’t pollution that’s harming the environment; it’s the
impurities in our air and the water that are doing it.

Rule 100 Avoid awkwardness.


Awkwardness is an ill-defined and subjective criticism. There are no hard-
and-fast rules for recognizing and eliminating awkwardness. But there is no
doubt that some sentences are awkward and need to be rewritten.
INCORRECT:Apart from the master bedroom, which has its own, all the
other bedrooms share a bathroom.
CORRECT: The master bedroom has its own bathroom; all the other
bedrooms share a bathroom.
Writers sometimes find themselves in quagmires of awkwardness in efforts
to avoid sexism. “He or she” and “his and her” may be politically correct,
but they are awkward. Try to rewrite the sentence in the gender-neutral
plural if you can.
INCORRECT:If a student wishes to change his or her area of concentration,
he or she must obtain written permission from his or her
academic advisor.
CORRECT: Students who wish to change areas of concentration must
obtain written permission from their academic advisors.

246 © The MBA Center


Grammar and Style Review

EXERCISE 10
Directions: Rewrite the following sentences to correct any style errors. (Note that
some sentences may be correct as written.)

1. Our nation must come together to unite.

2. When he goes to his country house for the weekend, he hardly never
returns back before Monday afternoon.

3. Due to the fact that it sold more than a million copies, the book was
considered to be a success.

4. The experiences are not able to be compared.

5. He says that he has not, does not, and never will eat broccoli.

6. According to her itinerary, the consultant will airplane it to New York


and from there train it to Boston.

7. Pricewise the possible new office is a bargain, but locationwise it is a


gamble.

8. The 1969 World Series will never be forgotten by the old baseball fan.

9. The judges, who were five in number, could not reach a consensus of
opinion on a winner.

10. The employee complained that the supervisor disrespected him.

© The MBA Center 247


Total PrepKit for the GMAT® Step 7.1 Lesson

ANSWERS TO EXERCISES

EXERCISE 1 (SENTENCE STRUCTURE) EXERCISE 3 (NOUN NUMBER)


1. The secretary arrived too late. The meeting had 1. Anyone requiring additional information should
already begun. consult the company's website.
2. The dangers of secondhand smoke have been 2. The United Nations has been debating a human
exaggerated; therefore, the government should rights resolution.
consider repealing some of the more restrictive 3. No error.
antismoking laws.
4. A solar eclipse is a phenomenon that is worth a
3. The reasons for the price adjustment are increasing journey to witness.
costs and decreasing demand.
5. The public is opposed to the proposed legislation.
4. English is not the most widely spoken language in the
world; nevertheless, it is the most practical language 6. This year’s promotional event attracted fewer people
for international business. than last year’s did.
5. No error. 7. On Sundays, when admission is free, the number of
people at the museum is astonishing.
6. Ask the librarian. He will know.
8. Everyone sang well and was heartily applauded.
7. No error
9. For the first time there were fewer species on the
8. Because it has not rained in two months — the worst
endangered list than in the previous year.
drought on record, reservoir levels are dangerously low.
10. No error.
9. Call Novak. He is the only one who can help us now.
10. No error.
EXERCISE 4 (VERB TENSE)
1. No error.
2. No error.
EXERCISE 2 (SUBJECT-VERB AGREEMENT) 3. If the team had started the project a month earlier,
they would be finished by now.
1. The only thing he values more than his career is his
wife and children. 4. If Americans ate less red meat, they would have lower
cholesterol levels.
2. The majority of the people of California support
recycling. 5. Financial experts recommend that one diversify one’s
investments.
3. In the last ten years, the number of professional sports
teams has doubled. 6. He resents the suggestion that he rewrite the paper.
4. No error. 7. No error.
5. No error. 8. The lecturer began with a quick plot synopsis for the
benefit of those who had not read the novel.
6. Not in several days has the mother or the children
been seen. 9. The civilization is believed to have flourished in the
fifteenth century.
7. No error.
10. Once the student passes the comprehensive oral
8. No less astonishing than the dimensions of the
examination, she will have satisfied all the requirements
surviving examples is their variety.
for the degree.
9. On the ground floor of the office building are a
newsstand, a coffee shop, and a restaurant.
10. Never before in the history of the state have
Montana’s vote-counting procedures received more
attention.

248 © The MBA Center


Grammar and Style Review

EXERCISE 5 (PRONOUNS) EXERCISE 7 (COMPARISONS)


1. No error. 1. Americans eat more corn than Europeans do.
2. No one will buy such an expensive product unless he or 2. The second of the author’s two published works sold
she really needs it. better.
3. The directors are looking for someone whom they can 3. The critic maintained that it was the worst film she
trust to abide by the rule. ever saw.
4. When one considers the candidate’s youth and 4. Unlike Los Angelenos, most New Yorkers use public
inexperience, one has to say that the victory was transportation.
unexpected. 5. Every year, more foreign tourists visit Paris than any
5. If the television viewer loses interest for just a few other European city.
seconds, he or she will switch channels. 6. The United States is the richest and most wasteful
6. It is sunny and hot today, but forecasters say it will rain country in the world.
tomorrow. 7. No error.
7. If one invests early and wisely, one will be able to enjoy 8. His job title is director of marketing, but his salary is
one’s retirement. still less than that of a secretary.
8. Because the parties have been unable to resolve the 9. Abraham Lincoln was perhaps one of the wisest men
dispute, a judge will have to do so. ever to serve as President.
9. After questioning the two suspects, the investigator 10. Choose either regular or express service: the former is
became convinced that it was they who committed the less expensive and the latter is faster.
crime.
10. No error.
EXERCISE 8 (PARALLELISM)
1. To be a celebrity means to have no privacy.
EXERCISE 6 (MODIFIERS)
2. The meeting is scheduled not for today but for
1. Driving soon became his favorite pastime. tomorrow.
2. With makeup and perfume, she tries to attract men. 3. The dieter resolved to eat neither too much nor too
3. Born and raised near the Mexican border, she easily little.
picked up Spanish. 4. He is responsible for not only the design, but also the
4. On the facade there is a sculpture that represents the execution.
rise of technology. 5. The turning point in the plot is the disappearance of
5. He hired a crew that has a truck and that is the protagonist.
accustomed to handling delicate antiques. 6. A teacher’s job is to inspire creativity as well as to
6. Deciding to get the story at any cost, the reporter impart knowledge.
could not be stopped. 7. The artist is neither naturally talented nor
7. The family devoured the whole pie, straight out of the independently wealthy.
oven and still piping hot. 8. The two biggest events in his life were his marriage
8. Even after receiving explicit instructions, the new and the birth of his first child.
employee did not know what to do. 9. By the time Baldwin arrived to help, his colleagues had
9. The supervisor ordered the assistant, dressed in a T- already laid out the site, cleared the land, and begun
shirt and jeans, to go home and change into more to excavate.
professional attire. 10. The changeover to the metric system did not succeed
10. Because they are no longer sworn enemies, relations because people could not become accustomed to using
between the United States and Russia have improved meters instead of feet, grams instead of ounces, or
considerably since the end of the Cold War. liters instead of quarts.

© The MBA Center 249


Total PrepKit for the GMAT® Step 7.1 Lesson

EXERCISE 9 (IDIOMS)
1. There were something between twenty and thirty
thousand people in attendance.
2. The candidate is not inclined either to debate his
opponent face to face or even to respond to questions
about the accusations.
3. Fletcher is not only the youngest member of the team,
but, with three university degrees, he is also the most
educated.
4. The world is a totally different place now from what it
was before the advent of the personal computer.
5. The children were eager to start eating, but they knew
they had to wait until everyone was served.
6. The director of human resources uses quaint Southern
colloquialisms such as “fixin’ to.”
7. Just like a child, he uses his finger to point at the words
as he reads them.
8. Her elocution coach taught her to pronounce her r’s as
they are pronounced in Britain.
9. The experience of war in the trenches had a profound
effect on the young English poets.
10. The jury must first decide whether the defendant is
telling the truth.

EXERCISE 10 (STYLE)
1. Our nation must come together.
2. When he goes to his country house for the weekend,
he hardly ever returns before Monday afternoon.
3. Because it sold more than a million copies, the book
was considered a success.
4. The experiences cannot be compared.
5. He says that he has not eaten, does not eat, and never
will eat broccoli.
6. According to her itinerary, the consultant will fly to
New York and from there go by train to Boston.
7. The price of the possible new office is a bargain, but
the location is a gamble.
8. The old baseball fan will never forget the 1969 World
Series.
9. The five judges could not reach a consensus on a
winner.
10. The employee complained that the supervisor showed
him disrespect.

250 © The MBA Center


Grammar Review

HOMEWORK/PRACTICE TEST
GRAMMAR REVIEW
ANSWER GRID

A B C D E
1 ! ! ! ! !
2 ! ! ! ! !
3 ! ! ! ! !
4 ! ! ! ! !
5 ! ! ! ! !
6 ! ! ! ! !
7 ! ! ! ! !
8 ! ! ! ! !
9 ! ! ! ! !
10 ! ! ! ! !
11 ! ! ! ! !
12 ! ! ! ! !
13 ! ! ! ! !
14 ! ! ! ! !
15 ! ! ! ! !

© The MBA Center 251


Total PrepKit for the GMAT® Step 7.2 Homework

Grammar Review – Homework Directions

End


When finished
Directions: In the following ques- reading
tions, each sentence has four underlined directions
words or phrases. The four underlined click on the
parts of the sentence are marked (A), icon below
(B), (C), and (D). Identify the one
underlined word or phrase that must be
changed in order for the sentence to be
correct. Dismiss
Directions

Test Section Answer


Time Help Confirm Next
Time
Quit Exit

252 © The MBA Center


Grammar Review

1. The Organization of American States, stretching 6. Because of the severe hailstorms and the repeated
A A B
from the North Pole to the Southern tip of flash floods that have struck the area, the police
C
Patagonia and including virtually every nation in demand that everyone should stay indoors.
B D
the Western Hemisphere, display a surprising ability
C
to confront important issues facing the Americas.
D 7. Although he was always an excellent student, he
A B
did not deserve getting the highest grade on the
C D
2. Dorothy, in addition to Jill and Craig, who also biology exam.
A
ran for office in the State Senate, were defeated in
B C D
the primaries. 8. Joe explained that if he arrives on time, and
A B
everything goes according to plan, he wants to
C
3. The Federal Government recently judged that go to the movies.
D
many than 10,000 Eskimos is sufficient to sustain
A B C D
their ancient culture and traditions.
9. The Acme Car Rental Company became world-
A
famous thanks to its slogan: "Our cars break
4. After finishing the grueling, six-hour ironman race, B C
A down less than do our competitors!”
Hercules was so thirsty that he ran up to the first D
B C
unsuspecting soul - who happened to be Apollo - to

ask for a water.


D 10. Joe asked his attorney for just two things on the
A B
eve of his 10- year sentence: a last meal of roast
C
5. On the Amazon river, although one must beware of duck and to call his beloved mother.
A B D
the man-eating piranha, you can safely travel
C
from beginning to end in under one month.
D 11. The police searched for the infamous Mr. McGoo
A B
in his office, the grocery store, and in his car,
C D
before giving up the futile petty larceny

investigation.

GO ON TO THE NEXT PAGE


© The MBA Center 253
Total PrepKit for the GMAT® Step 7.2 Homework

12. The second product built by Henry Ford's


A
company, the Model "A" was modeled from its
B C
predecessor, the simplistic Model "T."
D

13. While walking to Scarborough Fair, Osirus came


A
across a woman who asked him if it was necessary
B C
that he should go in that direction.
D

14. Scientists have determined that fewer of the small


A
number of not many tree frogs found in the
B C
Brazilian rainforest are alive today than ever.
D

15. Neil Armstrong, who was the first human being to


A
set foot on the Moon, was waIking only fifteen
B C
steps in the difficult lunar environment.
D

STOP
IF YOU FINISH BEFORE TIME IS EXPIRED
YOU MAY CHECK YOUR WORK
254 © The MBA Center
Grammar Review

GRAMMAR REVIEW HOMEWORK/PRACTICE TEST EXPLANATIONS FOR GRAMMAR REVIEW


ANSWER KEY HOMEWORK/PRACTICE TEST

1. C
2. C 1. (C) Subject/verb agreement error
3. A The subject is ”The Organization,” a singular
4. D collective noun which takes the plural verb, ”displays”
5. C instead of ”display.”
6. D
7. C
8. C 2. (C) Subject/verb agreement error
9. D The main subject is ”Dorothy,” which is singular
10. D and therefore requires the singular verb, ”was” instead
11. C of ”were.”
12. C
13. D
14. B 3. (A) Noun number error
15. C The correct expression for this noun number error
is ”more than”; ”many than” is not a correct
expression of a countable quantity in English.

4. (D) Noun number error


The non-count noun ”water” cannot take the
indefinite article ”a,” it must take the indefinite term
”some” instead.

5. (C) Pronoun error


Since the possessive pronoun ”one” is used in the
beginning of the sentence, ”one must beware,” then it
must be matched by ”one can safely ...” in the latter
part of the sentence.

6. (D) Verb tense error


After ”demand that,” you need the subjunctive. Do
not use the modal ”should” with the subjunctive.

7. (C) Verb tense error


Infinitive form is needed here. The gerund
”getting” must be changed to the infinitive form ”to
get.”

8. (C) Conditional error


In a conditional construction with the present
tense in the ”if” clause, you should use the future
tense in the following clause: ”he will want to go to the
movies.”

9. (D) Comparison error


The two terms of comparison here, cars and
competitors are not of the same type. To draw a
correct comparison, ”our competitors,” must be
changed to ”our competitors' cars.”

© The MBA Center 255


Total PrepKit for the GMAT® Step 7.2 Homework

10. (D) Parallelism error


All the items in a parallel list must be of the same
form: ”and to call ...” must be changed to ”and a call
to” in order to agree with the form of the first item in
the list, ”a last meal.”

11. (C) Parallelism error


All the items in a parallel list must be in the same
form: ”the grocery store” is not in the same forrn as the
other two items in the list, ”in his office” and ”in his
car,” and would need to be changed or eliminated to
make this sentence correct.

12. (C) Idiom error


The correct idiomatic phrasal verb in this sentence
should be ”modeled after” instead of ”modeled from.”

13. (D) Conditional error


After ”it was necessary that,” you need the
subjunctive. Do not use the modal ”should” with the
subjunctive.

14. (B) Redundancy error


Since ”a small number” and ”not many” have
roughly the same meaning, then to correct the
redundancy in this sentence, one of them must go.
Eliminate ”not many” to make this sentence correct.

15. (C) Verb tense error


For an action performed in the past and
completed, use the simple past tense. Change ”was
walking” to ”walked” to make this sentence correct.

256 © The MBA Center


Step 8
Grammar
Workshop

© The MBA Center


Total PrepKit for the GMAT® Step 8 Workshop

Grammar Review – Workshop Directions

End


When finished
Directions: In the following ques- reading
tions, each sentence has four underlined directions
words or phrases. The four underlined click on the
parts of the sentence are marked (A), icon below
(B), (C), and (D). Identify the one
underlined word or phrase that must be
changed in order for the sentence to be
correct. Dismiss
Directions

Test Section Answer


Time Help Confirm Next
Time
Quit Exit

258 © The MBA Center


Grammar Review

GMAT GRAMMAR WORKSHOP


6. Thinking she has not completed her forms on time,
TEST ONE: A
Karen is always concerned at the end of April when
1. The amount of people in the school district B
A tax are due although she doesn’t make
who signed up for Mr. Evergreen’s beginning swim C
B nearly enough money for the Internal Revenue
class corresponded to the Olympic boogie D
C Service to waste time to prosecute her if she does
boarding team that made it all the way to the
D happen to file late.
semi-finals last year.

7. Blake and Lori decided to start a band, as they both


2. The much aquatic resources recently discovered on A
A B C felt the desire to play instruments and
the Moon have been the subject of much debate.
D express their creativity through music, but they
B
couldn’t see how them would find a bass player
C
3. The new law in London, surely passing in the rest whom they could rely on.
A D
of England, dictates that it is illegal to obstruct foot
B
traffic on train platforms with public displays of
C D 8. Eddie, who is always late to work despite using five
affection. A
alarm clocks, realized that instead of taking the bus

every day and spending thirty minutes waiting for it


4. John asked me if I had heard the news today and, B C
A B to arrive, he decided to get his driver’s license and
as I knew that he was working on a thesis on D
C buy a car.
reactions to the popular media, I told him that

I have.
D 9. Since I never take sugar in my coffee, a habit I
A
have maintained since I was 20, I don’t think that
B
5. When living abroad and are looking for work, when I will go to Greece and am offered strong
A C
there is often a discrepancy between one’s abilities tea I will be tempted to sweeten the drink.
B C D
and the kind of job one must accept due to
D
competition and a reluctance to hire foreigners;

this is commonly referred to as the “immigrant’s

compromise”.

GO ON TO THE NEXT PAGE


© The MBA Center 259
Total PrepKit for the GMAT® Step 8 Workshop

10. The operation, although newer and more 14. My new neighbor, who reminds me of James, an
A
experimental than it of traditional techniques, to old English friend living in Los Angeles, has a
B
remove the tumor from the philosopher’s head kind heart, a patient ear, and an easy laugh very
A B
was unsuccessful and no more useful than the similar as those of James.
C C D
radiation which caused his hair to fall out.
D

15. Composed by tequila, coconut milk, and pine-


A
11. Fabrice, like Mark, is full of spirit and apple juice, the Pina Colada is very popular in
A B B C
energized with hope, both of which he learned places near Mexico, particularly in Southern
C
from his older brother, and he is motivated to California where the sun shines most of the time.
D D
complete each task well, a quality he inherited

from his mother.


16. Violet was ashamed with her parents when she
A B
was growing up, but after she met her boyfriend’s
12. Doing well in college requires study, dedication, C
A parents she changed her mind.
and knowing how to create meaning- D
B
ful relationships with professors; doing poorly in

college is often the result of too much partying,


17. The costume director of the Marseilles Opera
not enough concentration, and lacking sleep.
C D insisted, after seeing the scraps of cloth left over
A
from the last show’s needlework, that every actor
B
13. Justine, who studied art history for ten years, is an and actress should collect and donate the extra
A B C
expert on Medieval painting, Renaissance pieces of fabric to the local charity.
D
drawing, sculptures of the Pre-Raphaelite, and
C
period identification.
D 18. While it is likely that Mr. Schrempf might be the
A B
culprit, the detectives have had a difficult time

finding conclusive evidence against him.


C D

STOP
IF YOU FINISH BEFORE TIME IS EXPIRED
YOU MAY CHECK YOUR WORK
260 © The MBA Center
Grammar Review

GMAT GRAMMAR WORKSHOP


6. Frequent travelers have learned that it is
TEST TWO: A
easier and faster to board a commercial airplane if
1. The Michaelsons, after visiting their sick B
A they carry no more than two luggages.
grandmother in Iowa City, intends to travel to Italy C D
B
because their son Edward has a master’s degree in
C
Italian and wishes to continue his studies in 7. The many Indian owners of motel chains in the
D A B
Florence. United States expect to borrow from and only do
C
business to each other.
D
2. Is she really considering to go to Europe after she
A B C
graduates instead of inheriting her father’s business
D 8. The history of Latin America is slowly gaining
and carrying on the family name?
importance in North American schools like is
A B
evident in their curricula, which are now full of
3. Even though the man knew that he would C
A books and art by Latin American artists
probably not win a significant sum of money, he D
B and writers.
bought him a lottery ticket in the hope that just
C
once he would be able to call himself a winner.
D 9. The agency who created the marketing campaign
A B
for Nike products is, despite its success, still

4. All the women including the child was considered located in Portland, Oregon, the city where
A B C D
part of the demonstration which shook the it began.
C
foundation of Bender Hall and created a precedent

for all future demonstrations against war. 10. Unless Anne will put in the time to research her
D A B C
science project independently, her teacher will

5. When Virginia looked at the study on environmental never give her the grade she needs for her parents
A
problems in the Russian sector, she learned that to give her permission to go to the dance.
B D
there wasn’t many information to be collected from
C D
the old documents and asked her assistant to begin

a new analysis.

GO ON TO THE NEXT PAGE


© The MBA Center 261
Total PrepKit for the GMAT® Step 8 Workshop

11. Eddie and Susan will walk forever tonight should 16. Plato’s theories of justice, the forms, and his
A A B
they not to find a warm dry place to put the new theories of the political structure of the state are
B B C
baby; even a tree with thick branches would still studied today while some ideas of more
D
suffice for shelter from the driving rain. modern thinkers have already been forgotten.
C D

17. Kirsten, who is charming, intelligent, and open-


12. The volume of a cube is determined by its length, A
A B minded is so different to Mary, who is constantly
wide, and height multiplied together. B C
C D depressed, perpetually tired, and never willing to
D
go out and have a good time.

13. Jennifer’s dress, which has lace trim and a thick


A
black bodice, has more style than Emily’s: the
B C
two girls constantly compete for the title of
D
“Fairest at the Ball”.

14. By many numerous critics, Glenn Gould was


A
considered to be the most enigmatic concert
B
pianist of the twentieth century.
C D

15. Neutrinos they are so unpredictable that


A
physicists have difficulty measuring their precise
B C D
mass.

STOP
IF YOU FINISH BEFORE TIME IS EXPIRED
YOU MAY CHECK YOUR WORK
262 © The MBA Center
Grammar Review

GRAMMAR WORKSHOP EXPLANATIONS FOR GRAMMAR WORKSHOP


TEST ONE TEST ONE
ANSWER KEY
1. (A) noun number error
1. A Since “people” is a count noun it must take the
2. A phrase “number of” instead of “amount of,” which is
3. A used to modify only noncount nouns.
4. D
5. A
6. C 2. (A) noun number error
7. C The count noun “resources” must take the
8. D modifying phrase “many” instead of “much,” which is
9. C used to modify only noncount nouns.
10. B
11. D
12. D 3. (A) style error
13. C Change “surely passing” (passive) to “sure to pass”
14. C (active) to correct this sentence.
15. A
16. A
17. C 4. (D) verb tense error
18. B Part (B) correctly uses the past perfect — the
speaker’s hearing took place before John’s asking. Part
(D) should also use the past perfect. Change “have” to
“had” to correct the sentence.

5. (A) parallelism error


All of the parts in a list must be of the same
grammatical type or be expressed in the same form:
to make the correction here, change “are looking” to
“looking” to match the form of the first item in the list,
“living abroad.”

6. (C) subject/verb agreement error


The singular subject here, “tax,” must be changed
to the plural, “taxes,” to agree with the conjugated
plural verb “are” in order to correct this sentence.

7. (C) pronoun error


Change the object pronoun “them” to the subject
pronoun “they” to make the correction in this
sentence.

8. (D) pronoun error


It is redundant and ungrammatical to include the
subject pronoun ”he” when the sentence has already
a subject “Eddie.”

© The MBA Center 263


Total PrepKit for the GMAT® Step 8 Workshop

9. (C) verb tense error 17. (C) redundancy error


Only the present and past tenses are used after the After “insisited,” use the subjunctive, which is
connector word “when,” never the future tense. Since simply “collect.” It is redundant and ungrammatical to
the future tense is used here, “when I will go,” replace include the modal “should.”
“will go” with the present tense “go” to make this
sentence correct.
18. (B) redundancy error
Since “it is likely” and “might be” express the same
10. (B) comparison error idea, one of them is unnecessary: to make the
To correctly state a comparison here change the correction in this sentence change “might be” to “is.”
invalid form “than it of” to the valid comparison
phrase “than that of”: “newer...than that of...”

11. (D) parallelism error


All the parts in a list must be of the same
grammatical type or be expressed in the same form:
to make the correction here eliminate “he” from “he is
motivated” to match the form of the first item in the
list, “is full of spirit and energized with hope.”

12. (D) parallelism error


All the parts in a list must be of the same
grammatical type or be expressed in the same form:
to make the correction here change “lacking sleep” to
“lack of sleep” to match the form of the previous items
in the list.

13. (C) parallelism error


All the parts in a list must be of the same
grammatical type or be expressed in the same form:
to make the correction here change “sculptures of the
Pre-Raphaelite” to “Pre-Raphaelite sculptures” to match
the form of the previous items in the list.

14. (C) idiom error


To make the correction in this sentence change
“similar as” to the proper idiom of comparison “similar
to.”

15. (A) idiom error


To make the correction in this sentence change
“Composed by” to the proper prepositional idiom
“Composed of.”

16. (A) idiom error


To make the correction in this sentence change
“ashamed with” to the proper prepositional idiom
“ashamed of.”

264 © The MBA Center


Grammar Review

GRAMMAR WORKSHOP EXPLANATIONS FOR GRAMMAR WORKSHOP


TEST TWO TEST TWO
ANSWER KEY
1. (B) subject/verb agreement error
Since the subject, “The Michaelsons,” is plural, the
1. B conjugated verb which corresponds to it must also be
2. C plural: to make the correction in this sentence change
3. C the singular verb “intends” to the plural verb “intend.”
4. B
5. C
6. D 2. (C) verb tense error
7. D When the verb “consider” takes a verb as an object,
8. B it always takes the gerund, or -ing form of that verb:
9. A to make the correction in this sentence change the
10. B infinitive “to go” to the gerund “going.”
11. B
12. C
13. C 3. (C) pronoun error
14. A The personal pronoun “him” in answer choice (C)
15. A is unnecessary: to make the correction in this sentence
16. B eliminate “him” from “him a lottery ticket.”
17. B

4. (B) subject/verb agreement error


Since the third-person subject “...the women” is
plural, the conjugated verb which corresponds to it
must also be plural: to make the correction in this
sentence change the singular verb “was” to the plural
verb “were.”

5. (C) noun number error


Since “information” is a noncount noun in English,
it must be modified by the quantity word “much”
instead of “many,” which is used only for count nouns.

6. (D) noun number error


In order to specify one or more individual items
with a noncount noun such as “luggage,” the
appropriate form of the expression “a piece of” can be
used: to make the correction in this sentence change
“two luggages” to “two pieces of luggage.”

7. (D) idiom error


To make the correction in this sentence change “do
business to each other” to the proper reciprocal idiom
“do business with each other.”

8. (B) comparison error


To correctly state the comparison here between
two verb clauses change the invalid “like” to “as.”

© The MBA Center 265


Total PrepKit for the GMAT® Step 8 Workshop

9. (A) pronoun error


The pronoun “who” is only used to refer to people,
not things such as an advertising agency: to make the
correction here change “who” to “which.”

10. (B) verb tense error


The correct verb tense here after the time signature
“Unless” should be the present not the future tense: to
make the correction in this sentence change “will put”
to “puts.”

11. (B) verb tense error


Use the dictionary form after the modal “should.”
To correct the sentence, change “to find” to “find.”

12. (C) parallelism error


All the parts in a list must be of the same
grammatical type or be expressed in the same form:
to make the correction in this sentence change “wide”
to “width.”

13. (C) comparison error


To make a valid comparison between nouns such
as “Jennifer’s dress” and “Emily’s dress,” a verb must be
used after the comparison word “than”: to make the
correction here change “than Emily’s” to “than does
Emily’s.”

14. (A) redundancy error


Since “many” is similar in meaning to “numerous”
it is unnecessary and awkward in this sentence: to
make the correction, eliminate “many.”

15. (A) redundancy error


The pronoun “they” is unnecessary here since the
subject “Neutrinos” is placed next to the verb clause
“are so”: to make the correction in this sentence
eliminate “they.”

16. (B) parallelism error


All the parts in a list must be of the same
grammatical type or be expressed in the same form:
to make the correction in this sentence change “and
his theories” to “theories.”

17. (B) idiom error


To make the correction in this sentence change “so
different to” to the proper idiom “so different from.”

266 © The MBA Center


Step 9
Sentence
Correction

© The MBA Center


Total PrepKit for the GMAT® Step 9.1 Lesson

GENERAL OUTLINE

The objective of Sentence Correction questions is to test the rules of


English grammar and usage. The rules required to master these kinds of
questions seem intimidating, especially for non-native English speakers.
What makes the problem more complicated is that, unlike in the
Quantitative section, GMAC does not predetermine the content for these
questions. Fortunately, the number of grammar and usage rules for
English tested on the GMAT is reasonably limited, and you are never
required to correct punctuation or spelling. The lesson which follows
will discuss Sentence Correction question types in detail and give you
specific strategies for answering each one of them.

THE FOLLOWING TOPICS WILL BE COVERED IN THIS LESSON:

The Most Common Testing Points for Sentence Correction


Learning to Spot and Eliminate Wrong Answers
A Systematic Approach to Sentence Correction Questions
A Review of the Top 10 Mistakes in Sentence Correction

KEY TERMS

Statement: This is the original sentence located above the five answer
choices. It is either completely or partially underlined.

Answer choices: The five choices lettered (A) through (E) located below the
numbered sentence. They often appear in patterns such as the “3-2” and the
“2-2-1,” which are explained later in this lesson.

Same Error Answers: Answer choices that contain the same error as that in
the original sentence and change something else in the sentence to tempt
those who don’t identify the original mistake. There is rarely a set of answer
choices without at least one of these.

Almost Right Answers: Answer choices that correct the error in the original
sentence but introduce new errors elsewhere. There is rarely a set of answer
choices without at least two of these.

The Best Answer: The answer choice that is not only error-free, but is also
stylistically preferable. This is the one true answer to the question. There is
never a set of answer choices without one of these.

268 © The MBA Center


Sentence Correction

READY…

The Sentence Correction questions appear in the Verbal Section of the


GMAT and are the only grammar-based questions on the test. The Verbal
Section includes about 14 Sentence Correction questions mixed randomly
with Critical Reasoning and Reading Comprehension questions. Since it is
impossible for Reading Comprehension questions to appear at the
beginning of the section, it is very likely that among the first five questions
you will find between two and three Sentence Correction questions of
medium difficulty. As previously discussed, the first five questions are
especially important in determining your final score for the section. It is
therefore very important that you answer these early Sentence Correction
questions correctly because they will have a disproportionately large impact
on your final Verbal score. Although there are literally hundreds of
grammatical rules governing sentence structure and form in American
English, you should not be intimidated by Sentence Correction questions.
The GMAT tests your ability to recognize only a very limited number of
common grammatical and stylistic errors. In this lesson we will show you
how to quickly identify and repair those errors and zero in on the best
answer choice.

DIRECTIONS
On the CAT, you can click on the HELP button, and the directions for the
current question type will appear on the screen. Since you will need every
available minute to work on the questions, it is important that you become
familiar with the directions now so you will not waste time reading them
on test day. Below are the directions for the Sentence Correction questions.
Learn them now. They are always the same.

GMAT CAT – Section 4 : Verbal Directions

End
When finished
Directions: For each question below, part or reading
all of the sentence is underlined. Following the directions
sentences are five versions of the underlined part.
The first version repeats the original; the other
click on the
responses change the underlined part. Choose icon below
the best version of the sentence. If you think the
original is best, choose the first version.
Otherwise select one of the other responses.
These questions test your ability to identify
correct and effective writing. Select your response
Dismiss
based on the rules of standard written English. Directions
You should consider grammar, choice of words,
and sentence construction. Select the response
which most clearly and effectively expresses the
meaning intended in the original sentence. This
choice should avoid awkwardness, ambiguity,
redundancy, and grammatical error.
To review these directions for subsequent
questions of this type, click on HELP.

Test Section Answer


Time Help Confirm Next
Time
Quit Exit

© The MBA Center 269


Total PrepKit for the GMAT® Step 9.1 Lesson

Thus, according to Phineas, the Sentence Correction section tests your


ability to:
– Distinguish correct from incorrect English
– Avoid ambiguous and awkward expressions
– Eliminate unnecessarily wordy constructions

WHAT THE DIRECTIONS MEAN

The directions that the test makers give you do not offer much guidance
for answering Sentence Correction questions. You know that each sentence
will be completely or partially underlined, and that the first answer choice
(choice (A)) will always be the same as the original sentence. Certainly, you
are supposed to choose the best answer, but what the test makers mean by
this is not entirely clear. Note that the directions do not say “choose the
right answer.” This is because there may be more than one answer choice
that is grammatically correct. The second paragraph of the instructions
explains what the test makers believe the “best answer” to be, but there is
no mention of spelling, punctuation, or the range of grammar rules tested.
As it turns out, you don’t need to know how to spell, you don’t need to
know much about punctuation, and you will see later in this section that
the number of grammar rules tested is quite small.

270 © The MBA Center


Sentence Correction

AIM…

ANATOMY OF A SENTENCE CORRECTION QUESTION:


Sentence Correction questions are composed of a statement, which may
or may not contain a grammatical or stylistic error for you to identify, and
Check your work
five answer choices. The first answer choice is the same as the original Use the un-underlined part of
statement and the other four are different. Your job is to choose the best the sentence to your advan-
version of the statement from among the five answer choices. The “best” tage: it is the one part of the
answer choice is the one that eliminates grammatical mistakes and sentence you know is right.
expresses the meaning of the original statement in the most clear and
straightforward manner. Each statement will be either completely or
partially underlined.

Let’s look at a typical Sentence Correction statement:

STATEMENT

Although less impressive than the great masterpieces, contemporary


artists are more in touch with issues relevant to modern life.

Note that the un-underlined part is indisputable. You should use the un-
underlined part to help you figure out how the underlined part should be
corrected.

ANSWER CHOICES

(A) contemporary artists are more in touch with issues relevant


(B) contemporary artists have more relevance
(C) the work of contemporary artists are more in touch with issues relevant
(D) the work of contemporary artists are more relevant
(E) the work of contemporary artists is more relevant
Questions that are completely,
or almost completely,
If you read the initial statement you can identify the problem: a underlined tend to be the
modifier. The sentence compares masterpieces (works of art) to artists most difficult because there is
(persons who create works in art). Such a comparison is nonsensical. You are nothing correct with which to
looking for an answer choice that fixes the mistake. (Don’t worry, we’ll compare the incorrect under-
lined part.
discuss modifiers later.)
After scanning the answer choices (never read the first answer choice: it
is always exactly the same as the initial statement) you can see that the
second proposal (choice (B)) has been changed, but still doesn’t fix the
mistake.
The third proposal (choice (C)) fixes the initial mistake but introduces a
new mistake: faulty subject/verb agreement between “the work” and “are.”
The fourth proposal (choice(D)) changes what was awkward in choice
(C) – “in touch with issues relevant” – but keeps the same mistake: faulty
subject/verb agreement between “the work” and “are.”
The last proposal (choice (E)) is the best one. It fixes the modifier mistake
and has correct subject/verb agreement.

© The MBA Center 271


Total PrepKit for the GMAT® Step 9.1 Lesson

Standardized tests = standardized mistakes

In Step 1 of this manual we saw that Phineas has an agenda. His goal in
giving the GMAT is to tell business schools which candidates have average
abilities, less than average, more than average, unusually high, etc. He must
therefore create wrong answer choices that tempt test takers. Specifically,
there must be some choices that are not completely wrong and yet have
something attractive to tempt some test takers. There are some questions
that are almost right, so that competent test takers will get them right half
the time and get them wrong the other half. Only the most competent test
takers get all, or nearly all, the questions right. We call the technique of
producing wrong answers according to the principles of standardization the
Looking in vain
Wrong Answer Factory.
Sometimes there is no error in
the statement. In fact, you
should expect (A) to be There are generally three types of answer choices for Sentence
correct as often as any other Correction questions:
choice – about once in five
questions.
Same Error Answers: Answer choices that repeat the error from the
initial statement, but change something else in the answer choice. Most of
the time, you will see at least one same error answer choice. This is apparent
in choice (B).

Almost Right Answers: Answer choices (in this case (C) and (D)) that fix
the error from the original sentence, but introduce new errors.

Best Answers: Answer choices that are grammatically correct,


straightforward, and clearly express the meaning of the initial sentence.
There will never be errors in spelling or punctuation.

Understanding the Wrong Answer Factory is a sure way of finding the


best answer. Once you have identified Same Error and Almost Right
Answers, using the Process of Error Identification to find the Best Answer is
easy.

Many questions are made


difficult by the addition of ANSWER CHOICE PATTERNS
distracting words. By identi-
fying a key word, phrase, or
clause, you will spot the “3-2” - You can quickly see a “3-2” pattern in the answer choices. In this
testing patterns and focus on example, there are two choices that compare great masterpieces to
the initial errors, making “contemporary artists” and three choices that compare great masterpieces to
elimination easier.
“the work of contemporary artists.” You don’t have to zero in on the right
choice immediately. Instead, look for patterns and eliminate answer choices
with the same mistakes.

“2-2-1” - Sometimes you see a “2-2-1” pattern, in which two choices have
one expression, another two choices have a different expression, and a fifth
choice is completely different from all the others. These questions tend to
be a little harder, but after recognizing the “2-2-1” pattern you can begin to
eliminate any answer choices that contain the same mistake.

272 © The MBA Center


Sentence Correction

THE MBA CENTER APPROACH

Knowing that the number of question types is limited and that all the
questions are written according to the Wrong Answer Factory principles, we
have designed an approach to Sentence Correction appropriate for each of
the question types.

Step 1: Read the initial statement and identify the type of problem

The least advised approach to Sentence Correction would be to read the


answer choices instead of the question to determine the type of problem.
First look at the original sentence to determine the problem (if any – always
check). Once you do, you’ll know what error to focus on in the answer
choices, and can use the Process of Error Identification to eliminate wrong
answer choices.

DRILL 1
DIRECTIONS:
FOR EACH SENTENCE BELOW IDENTIFY THE TYPE OF ERROR (IF ANY):

1. The Secretary of State’s strategic advisor, together with her staff of


three top aides, are accompanying her on a diplomatic tour of capitals
in the Middle East.

Type of problem: ________________________________________________

2. Unlike previous hypotheses concerning climate changes, paleon-


tological circles have introduced a new theory for the extinction of
the dinosaurs in the form of a lethal comet.

Type of problem: ________________________________________________

3. The French are renowned for their eating habits, not only for the
quality and preparation of their food, but also for the refined
manner in which they eat it.

Type of problem: ________________________________________________

4. While some of the blood samples were contaminated today,


rendering them useless for further experiments, the scientists
already finished their research.

Type of problem: ________________________________________________


Statistically, 20% of the time
the first answer choice is
5. Whereas warriors in centuries past counted among their nations’ correct. No error, or answer
heroes, so too do today’s sports stars, such as Michael Jordan and choice (A), is more common
Ronaldo, embody the physical and competitive aspirations of their than you might think. How to
make sure? Even if you are
countrymen.
right and think there is no
error to identify, scan for
Type of problem: ________________________________________________ differences among answer
choices to make certain there
is nothing wrong in the initial
statement.

© The MBA Center 273


Total PrepKit for the GMAT® Step 9.1 Lesson

Step 2: Do not reread the first answer choice

What is the difference between the underlined part and answer choice (A)?

Economists are hard pressed to explain why, occasionally, out of


many years of steady economic growth emerge the twin problems of
unemployment and large income gaps.

Answer choice (A): emerge the twin problems of unemployment and large
income gaps.

Notice that the first answer choice, (A), is always the same as the
underlined part in the initial statement. Keep in mind as you go through
this lesson that you always have choice (A) as an option for best answer. To
gain time, if you’re not satisfied with the initial statement, move directly
down to answer choice (B) and begin comparing all the answer choices.

Step 3: Identify the Same Error Answers

Identifying Same Error Answer choices is the first step in the Process of
Error Identification. Same Error Answer choices are the easiest to spot, so
they’re a natural place to start. To spot Same Error Answers, find the answer
choices that repeat the error from the initial statement.

DRILL 2
DIRECTIONS: FOR THE QUESTIONS BELOW, DETERMINE THE SAME
ERROR ANSWERS IN EACH SET OF ANSWER CHOICES.

Score Value: 500

01:15 GMAT CAT – Section 4 : Verbal 1 of 41


The Secretary of State’s strategic advisor, together with her


staff of three top aides, are accompanying her on a diplomatic
tour of capitals in the Middle East.

! (A) together with her staff of three top aides, are


! (B) as well as her staff of three top aides, is
! (C) also her staff of three top aides, are
! (D) bringing along with her staff of three top aides, are
! (E) additionally with her staff of three top aides, is

Test Section Answer


Time Help Confirm Next
Time
Quit Exit

274 © The MBA Center


Sentence Correction

Score Value: 600

01:13 GMAT CAT – Section 4 : Verbal 2 of 41


Unlike previous hypotheses concerning climate changes,
paleontological circles have introduced a new theory for the
extinction of the dinosaurs in the form of a lethal comet.

! (A) paleontological circles have introduced a new theory for the


extinction of the dinosaurs in the form of a lethal comet
! (B) paleontologists are examining a new theory for the extinction
of the dinosaurs, one involving a lethal comet
! (C) a new theory for the extinction of the dinosaurs has made its
way into paleontological circles, this time citing a lethal comet
as the cause of the extinction
! (D) a new theory citing a lethal comet as the cause of the
extinction of the dinosaurs has made its way into
paleontological circles
! (E) the introduction of a new theory, citing a lethal comet as the
cause of dinosaur extinction, has made its way into
paleontological circles


Test Section Answer
Time Help Confirm Next
Time
Quit Exit

Score Value: 650

01:11 GMAT CAT – Section 4 : Verbal 3 of 41


The French are renowned for their eating habits, not only for
the quality and preparation of their food, but also for the refined
manner in which they eat it.

! (A) the quality and preparation of their food, but also for the
refined manner in which they eat it
! (B) the quality and preparation of their food, but by the refined
manner in which they eat it
! (C) the quality and preparation of their food, but also the refined
manner in which they eat it
! (D) their food’s quality and preparation, but for the refined manner
in which they go about eating it
! (E) the quality and preparation of their food, and the refined
manner in which they eat it

Test Section Answer


Time Help Confirm Next
Time
Quit Exit

© The MBA Center 275


Total PrepKit for the GMAT® Step 9.1 Lesson

Score Value: 650

01:09 GMAT CAT – Section 4 : Verbal 4 of 41


While some of the blood samples were contaminated today,
rendering them useless for further experiments, the scientists
already finished their research.

! (A) While some of the blood samples were contaminated today,


rendering them useless for further experiments, the scientists
! (B) While some of the blood samples had been contaminated
today, rendering them useless for further experiments, the
scientists
! (C) Some of the blood samples had been contaminated today,
rendering them useless for further experiments; fortunately,
the scientists had
! (D) While some of the blood samples were contaminated today,
rendering them useless for further experiments, the scientists
had
! (E) Today some of the blood samples were contaminated,
rendering them useless for further experiments, but the
scientists


Test Section Answer
Time Help Confirm Next
Time
Quit Exit

Score Value: 700

01:07 GMAT CAT – Section 4 : Verbal 5 of 41


Whereas warriors in centuries past counted among their
nations’ heroes, so too do today’s sports stars, such as Michael
Jordan and Ronaldo, embody the physical and competitive aspi-
rations of their countrymen.

! (A) Whereas warriors in centuries past


! (B) Much like warriors in centuries past, who
! (C) Just like warriors in centuries past, who
! (D) Just as warriors in centuries past
! (E) Just as warriors in centuries past, who

Test Section Answer


Time Help Confirm Next
Time
Quit Exit

276 © The MBA Center


Sentence Correction

Step 4: Identify the Almost Right Answers

Once you have identified and eliminated all the Same Error Answers,
only Almost Right Answers and the best answer remain. Because they are
more difficult to spot, look for Almost Right Answers only after you’ve
located and eliminated all the Same Error Answers. At this point, compare
slight differences in wording between the remaining answer choices and
look for Almost Right Answers, which correct the original problem and add
an error to another part of the sentence to distract you. Spotting and
eliminating Almost Right Answer choices requires a bit more attention and
skill than for Same Error Answers. This stage of error identification is critical
and is where most mistakes are made.

DRILL 3
DIRECTIONS: IDENTIFY
THE ALMOST RIGHT ANSWERS AMONG THE
FOLLOWING ANSWER CHOICES.
Score Value: 550

01:05 GMAT CAT – Section 4 : Verbal 6 of 41


Some popular music lyrics written during the late 1960’s and
early 1970’s, including those of Bob Dylan and Creedence
Clearwater Revival, were revolutionary for their time, as it
eschewed romantic musings for declarations of political protest.

! (A) as it eschewed
! (B) as they eschewed
! (C) in its eschewance of
! (D) in that they eschewed
! (E) as they had eschewed

Test Section Answer


Time Help Confirm Next
Time
Quit Exit

Score Value: 600

00:03 GMAT CAT – Section 4 : Verbal 7 of 41


Ernest Hemingway, one of the most acclaimed American


writers of the twentieth century, has been credited for
revolutionizing modern English prose, which had become, in the
Victorian era, excessively verbose.

! (A) for revolutionizing


! (B) as revolutionizing
! (C) with revolutionizing
! (D) for the revolutionization of
! (E) with having had revolutionized

Test Section Answer


Time Help Confirm Next
Time
Quit Exit

© The MBA Center 277


Total PrepKit for the GMAT® Step 9.1 Lesson

Score Value: 510

01:01 GMAT CAT – Section 4 : Verbal 8 of 41


Before the introduction of the Miranda Rights, persons
suspected of having committed a crime was subject to search
and interrogation without the benefit of legal counsel.

! (A) was subject to


! (B) which were subjected to
! (C) were subjected by
! (D) being subject to
! (E) were subject to


Test Section Answer
Time Help Confirm Next
Time
Quit Exit

Score Value: 550

00:59 GMAT CAT – Section 4 : Verbal 9 of 41


The committee investigating the charges have found no
evidence of wrongdoing on the senator’s part, and will announce
their report next week.

! (A) have found no evidence of wrongdoing on the senator’s part,


and will announce their
! (B) have found no evidence of wrongdoing on the part of the
senator and will announce their
! (C) has found no evidence of wrongdoing on the senator’s part,
and will announce their
! (D) has found no evidence of wrongdoing on the senator’s part,
and as a result it will announce its
! (E) has found no evidence of wrongdoing on the senator’s part,
and will announce its

Test Section Answer


Time Help Confirm Next
Time
Quit Exit

278 © The MBA Center


Sentence Correction

Score Value: 610

00:57 GMAT CAT – Section 4 : Verbal 10 of 41


Having survived public disgrace and subsequent resignation
as President of the United States in the wake of the notorious
Watergate scandal, Richard Nixon’s image managed to alter from
political pariah to senior statesman in the latter years of his life.

! (A) Richard Nixon’s image managed to alter


! (B) Richard Nixon’s image altered
! (C) Richard Nixon managed to alter his image
! (D) Richard Nixon had managed an alteration of image
! (E) the alteration of Richard Nixon’s image


Test Section Answer
Time Help Confirm Next
Time
Quit Exit

Step 5: By the Process of Error Identification arrive at the right answer.

After completing all four steps, only the best answer should remain.
However, when you cannot determine the type of problem, the four-step
approach is not of much use.

WHAT IF I CANNOT IDENTIFY THE TYPE OF PROBLEM?

If you cannot identify the type of problem, compare the answer choices
for differences and similarities. Identify a key word or clause that appears in
all the sentences, and compare it with key words or clauses in the answer
choices. From this work you’ll be in a position to identify both the Same Error
Answers, in which the main error is the same as that of the original sentence;
and the Almost Right Answers, which correct the original error but introduce
new mistakes. Following these steps, and using the Process of Error
Identification, you can find the best answer, even if you cannot identify the
type of problem.

© The MBA Center 279


Total PrepKit for the GMAT® Step 9.1 Lesson

Now try one on your own!

DIRECTIONS: CHOOSE THE BEST VERSION OF THE SENTENCE FROM THE


ANSWER CHOICES BELOW.
Score Value: 710

00:55 GMAT CAT – Section 4 : Verbal 11 of 41


Before he wrote his masterpiece Notre-Dame de Paris,
otherwise known as The Hunchback of Notre Dame, Victor Hugo
saw a small sculpture along the northern wall of the Notre Dame
cathedral in Paris that inspired the character Quasimodo.

! (A) Before he wrote his masterpiece Notre-Dame de Paris,


otherwise known as The Hunchback of Notre Dame, Victor
Hugo saw a small sculpture along the northern wall of the Notre
Dame cathedral in Paris that inspired the character
Quasimodo.
! (B) Before writing his masterpiece Notre-Dame de Paris, otherwise
known as The Hunchback of Notre Dame, a small sculpture
along the northern wall of the Notre Dame cathedral in Paris
inspired Victor Hugo to create the character Quasimodo.
! (C) The character of Quasimodo, who appears in Victor Hugo’s
masterpiece Notre-Dame de Paris, otherwise known as The
Hunchback of Notre Dame, was inspired by a small sculpture
along the northern wall of the Notre Dame cathedral in Paris.
! (D) Quasimodo, the character appearing in Victor Hugo’s
masterpiece Notre-Dame de Paris, otherwise known as The
Hunchback of Notre Dame, had been an inspiration which
Victor Hugo had after seeing a small sculpture along the
northern wall of the Notre Dame cathedral in Paris.
! (E) The inspiration for Victor Hugo’s Quasimodo, the character in
his masterpiece Notre-Dame de Paris, otherwise known as The
Hunchback of Notre Dame, was inspired by a small sculpture
which appears along the northern wall of the Notre Dame
cathedral in Paris.


Test Section Answer
Time Help Confirm Next
Time
Quit Exit

280 © The MBA Center


Sentence Correction

STRATEGY AT WORK

Read the statement and answer choices below then turn the page for
practice with our five-step approach to find the best answer for Sentence
Correction questions. Score Value: 650

00:53 GMAT CAT – Section 4 : Verbal 12 of 41


A recent study from the Center for Disease Control has
concluded that much of the currently uncontrolled carcinogens
to which residents of northeastern states are exposed has not
been the major cause of increased breast cancer in the region.

! (A) much of the currently uncontrolled carcinogens to which


residents of northeastern states are exposed has
! (B) much of the currently uncontrolled carcinogens to which
residents of northeastern states are exposed to has
! (C) much of the carcinogens that are currently uncontrolled and
that residents of northeastern states are exposed to has
! (D) many of the carcinogens that are currently uncontrolled and
residents of the northeastern states are exposed to have
! (E) many of the currently uncontrolled carcinogens to which
residents of northeastern states are exposed have

Test Section Answer


Time Help Confirm Next
Time
Quit Exit

Step 1: Identify the problem in the statement.

What is the grammar problem in the statement?

Step 2: Do not reread the first choice

If you are not satisfied with the initial statement, go directly to the
second answer choice.

Step 3: Identify the Same Error Answers

On the grid below, cross out the Same Error Answer choices.

1 2 3 4 5 6
A
B
C
D
E

© The MBA Center 281


Total PrepKit for the GMAT® Step 9.1 Lesson

Score Value: 650

00:51 GMAT CAT – Section 4 : Verbal 12 of 41


A recent study from the Center for Disease Control has
concluded that much of the currently uncontrolled carcinogens
to which residents of northeastern states are exposed has not
been the major cause of increased breast cancer in the region.

! (A) much of the currently uncontrolled carcinogens to which


residents of northeastern states are exposed has
! (B) much of the currently uncontrolled carcinogens to which
residents of northeastern states are exposed to has
! (C) much of the carcinogens that are currently uncontrolled and
that residents of northeastern states are exposed to has
! (D) many of the carcinogens that are currently uncontrolled and
residents of the northeastern states are exposed to have
! (E) many of the currently uncontrolled carcinogens to which
residents of northeastern states are exposed have


Test Section Answer
Time Help Confirm Next
Time
Quit Exit

Step 4: Identify the Almost Right Answers.

On the grid below, cross out the Almost Right Answer choices.

1 2 3 4 5 6
A
B
C
D
E

Step 5: By elimination, you come to the best answer.

The best answer is ________________.

282 © The MBA Center


Sentence Correction

FIRE!

Let’s look at the specific errors tested on Sentence Correction. Remember, Rule: The subject of a verb
there are many, many grammar points that could be tested. In this section must agree in number and
we are going to look at the ten most common errors on Sentence Correction. person with that verb.
It is a simple rule, but leave it
to Phineas to find compli-
cations.
A. SUBJECT-VERB AGREEMENT ERROR

Rule: The main subject of the sentence must always agree in number
with the main verb of the sentence. Be careful: Phineas is fond of
using collective nouns as subjects that, though describing a plural
group, use a singular verb. (See the Grammar Review for a complete
explanation of this rule.)

DRILL 4
DIRECTIONS: FROM THE TWO CHOICES SELECT THE ONE THAT IS
BETTER.

1.
A) The National Rifle Association, seeking to defend the rights of law-
abiding citizens and sportsmen, are lobbying to preserve free access
to armor-piercing bullets.
or,
B) The National Rifle Association, seeking to defend the rights of law-
abiding citizens and sportsmen, is lobbying to preserve free access to
armor-piercing bullets.

The main subject is the “National Rifle Association,” which, although


made up of a plural number of politically active members, is a singular noun.

2.
A) After working on the computers for what seemed like days, the group
decided that the project was finished and that the members could
go home for the day.
or,
B) After working on the computers for what seemed like days, the group
decided that the project were finished and that the members could
go home for the day.

What is the subject? “Group” or “project”? Both are subjects with verbs
(group - decided) (project - was). The mistake is simple, despite the long and
difficult sentence. “Project” is a count noun, and, since it is singular, we must
use the verb “was.”

3.
A) The publisher is given to arguing with his writers, but in this case the
agreement, although 25 pages in length, has no room for negotiation.
or,
B) The publisher is given to arguing with his writers, but in this case the
agreement, although 25 pages in length, have no room for
negotiation.

© The MBA Center 283


Total PrepKit for the GMAT® Step 9.1 Lesson

The subject in question is “agreement.” Again, don’t let the extra infor-
mation clause (despite … long) confuse you. “Agreement” can be a count or
non-count noun; in this sentence it is count because of the article “the” and
must therefore take a singular verb, “has.”

4.
A) The legislative wing of the government, the group of politicians and
staff members charged with the writing and prosecution of all the
national laws, is prone to controversy.
or,
B) The legislative wing of the government, the group of politicians and
staff members charged with the writing and prosecution of all the
national laws, are prone to controversy.

Is the subject “wing” or “government”? Any expression following “of


the…” is a modifier. “Government” is modifying “wing.” (What kind of
“wing”? A government wing). “Wing” is a simple, non-count noun in the
singular form, therefore use the singular form of the verb to be: “is.”

5.
A) From all walks of life, students, teachers, professionals, and
performers alike must wait in line because employment, although
available through the government agency, is given based not on
experience, but on the applicant’s willingness to try new things.
or,
B) From all walks of life, students, teachers, professionals, and
performers alike must wait in line because employment, although
available through the government agency, are given based not on
experience, but on the applicant’s willingness to try new things.

What is the subject here? Phineas will always try to confuse you. It is
your job to break down the sentence into its critical parts. The subject here
is “employment,” a noncount noun that takes the singular verb, “is.”

6.
A) Even though he had driven around town all afternoon to get the
right papers, neither his passport nor the statements from his bank
were sufficient to prove his residency.
or,
B) Even though he had driven around town all afternoon to get the
right papers, neither the statements from his bank nor his passport
was sufficient to prove his residency.

Both are correct. Can you see why?


Fortunately, the test makers would never do this to you. If you are tested on
Subject/Verb Agreement with a “neither … nor” or an “either … or”
construction, your tendency is to use a plural verb anyway, right? So watch
out for two singular subjects.

284 © The MBA Center


Sentence Correction

EXAMPLE OF A SUBJECT-VERB AGREEMENT QUESTION


Score Value: 680

00:49 GMAT CAT – Section 4 : Verbal 13 of 41


Out of America’s fascination with all products low in fat have
grown a market for diet brands of foods and soft drinks that are
forcing marketing specialists to change the way they advertise
and package their food and beverage products.

! (A) products low in fat have grown a market for diet brands of
foods and soft drinks that are forcing
! (B) products low in fat has grown a market for diet brands of
foods and soft drinks that is forcing
! (C) products that are low in fat has grown a market for diet brands
of foods and soft drinks that force
! (D) low-fat products have grown a market for diet brands of foods
and soft drinks that are forcing
! (E) low-fat products has grown a market for diet brands of foods
and soft drinks that force

Test Section Answer


Time Help Confirm Next
Time
Quit Exit

If you can, determine the mistake in the original statement. When you
move down to the answer choices, remember always to skip over answer
(A). The original mistake concerns the correct subject, in this case the
inverted subject “market.” “Market” is a singular non-count noun and must
therefore have the singular verb, “has.” Therefore you can eliminate choices
(A) and (D), which both contain the plural verb “have.” What is the next
mistake? Notice the second verb in the underlined section changes with
each answer choice. Should it be “that is forcing,” “that force,” or “that are
forcing”? Again, establish the subject of this verb: who or what is performing
the action? “Market,” again. Since “market” is singular, you need a singular
verb. Eliminate answers (C) and (E).

Using the Wrong Answer Factory, you can easily eliminate answer
choices once you have determined the original mistake. If you can’t find a
mistake in the original statement, look directly at the answer choices. By
comparing what Phineas has or has not changed in the answer choices, you
can often find the mistake from the initial statement even if you don’t
know what type of problem it is.

© The MBA Center 285


Total PrepKit for the GMAT® Step 9.1 Lesson

B. MODIFIER PROBLEMS

Rule: A modifier will always describe the closest noun. Words or phrases
that modify should be as close as possible to the noun they
Modification Breakdown modify. Be careful: if the subject of the modifier is wrong, the
Rule: Words or phrases that sentence usually needs to be completely restructured.
modify another word should
be as close as possible to the
word they modify. DRILL 5
This is one of the hardest DIRECTIONS: FROM THE TWO CHOICES, SELECT THE ONE THAT IS
grammar rules tested on the BETTER.
GMAT.

1.
A) Having located the correct place where the digging and excavation
was going to occur, the shovels and rakes were put to work by the
laborers hired for the job.
or,
B) Having located the correct place where the digging and excavation
was going to occur, the laborers who were hired for the job put the
shovels and rakes to work.

Introductory verbal modifiers are difficult at first, but once you un-
derstand how to identify them, they are quite easy. In this sentence we have
an introductory modifier “Having located the place where the digging was going
to occur” with no subject. It will therefore modify the closest noun. Ask
yourself who or what is performing the action (who or what “has located”),
“shovels and rakes”? No, the “laborers.”

2.
A) Thinking about the project, although in a very thorough and
organized manner, Jack would have seen the problem sooner had he
just started working.
or,
B) Thinking about the project, although in a very thorough and
organized manner, the problem would have been seen sooner had
Jack just started working.

It is possible to have an extra information clause between the modifier


and the subject. In this sentence this clause (although … manner) is there to
distract you. Who or what is “thinking about the project”? Jack, of course.

3.
A) Agile and keen-witted, the introduction of the domestic cat to the
Australian continent has threatened the survival of a number of
usually predator-free small mammal species.
or,
B) Agile and keen-witted, the domestic cat, recently introduced to the
Australian continent, has threatened the survival of a number of
usually predator-free small mammal species.

Sometimes, with an adjectival introductory phrase the same rule for


modifiers applies. Ask yourself who or what is “agile and keen-witted,” the
“introduction”? No, “the domestic cat.”

286 © The MBA Center


Sentence Correction

4.
A) Bread and Wine is a fictionalized account of a small Italian village
at the beginning of the Fascist movement, which many literary
critics consider Silone’s greatest work.
or,
B) Bread and Wine, which many literary critics consider Silone’s
greatest work, is a fictionalized account of a small Italian village at
the beginning of the Fascist movement.

Who, that, and which are modifiers and must be as close to the words they
modify as possible. Did many “literary critics” consider the “Fascist movement”
to be “Silone’s greatest work,” or is it the book, “Bread and Wine”?

EXAMPLE OF A MODIFIER QUESTION


Score Value: 650

00:47 GMAT CAT – Section 4 : Verbal 14 of 41

Important because of the skeletal material discovered there,


archaeologists excavating the Choukoutien caves of northern
China also found significant information of the region’s
inhabitants.

(A) archaeologists excavating the Choukoutien caves of northern


China also found significant information of the region’s
inhabitants
(B) but also, archaeologists excavating the Choukoutien caves of
northern Chine also found significant information about the
region’s inhabitants
(C) the Choukoutien caves of northern China also contained
significant information about its inhabitants found by archaeo-
logists
(D) archaeologists’ excavations of the Choukoutien caves of
northern China found significant information of the region’s
inhabitants
(E) the Choukoutien caves of northern China also contained
significant information, found by archaeologists, about the
region’s inhabitants

Test Section Answer


Time Help Confirm Next
Time
Quit Exit

Identify the original mistake. A modifier will describe the closest noun.
What is “important because of the skeletal material discovered there”?
Archaeologists? No, it must be some place, not some person. Eliminate any
choices with the wrong subject, (A) and (B). What’s the next mistake?
Again, refer back to the original modifier. What is “important”? The “caves,”
not the “excavations.” Eliminate answer (D). We are left with answers (C)
and (E). (C) looks good, because it corrects the original mistake, but we can
eliminate it because it is ambiguous what was “found by archaeologists”
(information or inhabitants?). Also, we can eliminate choice (C) because
it uses the singular pronoun “its” to refer to the plural “caves”.

© The MBA Center 287


Total PrepKit for the GMAT® Step 9.1 Lesson

C. PROBLEMS WITH PRONOUNS

Rule: Every pronoun must have a clear antecedent, agreeing in number,


gender (for people), and case.

DRILL 6
DIRECTIONS: FROM THE TWO CHOICES, SELECT THE ONE THAT IS
BETTER.

1.
A) The position of the Earth in relation to the Sun is always changing,
as its axis tilt is not rigidly constant.
or,
B) The position of the Earth in relation to the Sun is always changing,
as the Earth’s axis tilt is not rigidly constant.

In the second clause, what does the possessive pronoun “its” refer to?
Each pronoun must have a clear antecedent; “position” is not the subject
that is “not rigidly constant”; therefore we must add the correct subject in the
second sentence.

2.
A) Peyton enjoys the company of many friends, especially Jim and
Jason, but he is especially careful not to consider any one friend
more important than another.
or,
B) Peyton enjoys the company of many friends, especially Jim and
Jason, but Peyton is especially careful not to consider any one friend
more important than another.

“He,” in first sentence, could refer to “Peyton,” “Jim,” or “Jason.” There is


no clear antecedent. Therefore we must rewrite the sentence, repeating the
subject, to avoid confusion.

288 © The MBA Center


Sentence Correction

EXAMPLE OF A PRONOUN QUESTION


Score Value: 590

00:45 GMAT CAT – Section 4 : Verbal 15 of 41


One result of the National Education Council’s recent study
of undergraduate life is the realization that in a student’s last
academic year, their concerns for job prospects often interfere
with their academic careers.

! (A) the realization that in a student’s last academic year, their


concerns for job prospects often interfere with their academic
careers
! (B) the realization that during students’ last academic year, their
concerns for job prospects often interfere with their academic
careers
! (C) to realize that in a student’s last academic year, his or her
concerns for job prospects often interfere with his or her
academic career
! (D) to realize that during a student’s last academic year, their
concerns for job prospects often interfere with their academic
careers
! (E) realizing that during students’ last academic year, their
concerns for job prospects often interfere with their academic
careers

Test Section Answer


Time Help Confirm Next
Time
Quit Exit

Whenever you see multiple pronouns in a sentence, you should be


suspicious. In this sentence there are two pronouns “their” that must refer
to a plural subject. What is the subject? “A student” is singular, therefore we
have a pronoun mistake. First scan the answer choices for those that repeat
the original mistake. Choices (A) and (D) can be eliminated. Choices (C)
and (E) fix the original mistake, but add new problems. (C) incorrectly
completes the construction “One result is...” with the infinitive “to realize,”
and (E) incorrectly completes it with the gerund “realizing.”

© The MBA Center 289


Total PrepKit for the GMAT® Step 9.1 Lesson

D. PROBLEMS WITH PARALLELISM

Rule: When making a list or a comparison, parallel concepts must be


expressed in parallel constructions.

DRILL 7
DIRECTIONS: OF THE TWO CHOICES, SELECT THE ONE THAT IS BETTER.

1.
A) On his vacations, Phil always enjoys looking at the museums and
the small streets, finding out-of-the-way cafes and restaurants,
although he sometimes gets lost, and buys original paintings.
or,
B) On his vacations, Phil always enjoys looking at the museums and
the small streets, finding out-of-the-way cafes and restaurants,
although he sometimes gets lost, and buying original paintings.

Parallel ideas must be expressed in parallel constructions. Here we have


three verbs that are expressing the same idea: what Phil enjoys “doing” on
his vacation. Be careful of the other verbs in the sentence; they are not part
of the parallel construction.

2.
A) William Blake is renowned not only for writing works of poetry, but
also for the drawings and paintings that accompanied them.
or,
B) William Blake is renowned not only for his works of poetry, but also
for the drawings and paintings that accompanied them.

The idiomatic expression “not only … but also” requires that the two
items listed maintain a parallel construction. In the first sentence above,
“writing works of poetry” is incorrectly paired with “the drawings and
paintings.”

3.
A) Thomas Jefferson’s interests ranged from the study of paleontology
to reading classical literature.
or,
B) Thomas Jefferson’s interests ranged from studying paleontology to
reading classical literature.

Here, the idiomatic phrase is “ranged from … to.” Again, verify parallel
construction between the two items.

Other expressions that require parallel construction include:


Either… or, neither… nor, prefer… to , both… and, between… and, from…
to, left … for.

290 © The MBA Center


Sentence Correction

4.
A) To say that the Middle Ages was a period of chaos and loss of
culture is to do a great disservice to the many great artists and
thinkers of the period.
or,
B) To say that the Middle Ages was a period of chaos and loss of
culture is doing a great disservice to the many great artists and
thinkers of the period.

What is the comparison here? Noticing the words “to say” ought to be a
big hint for you. In order to maintain parallel structure, we must repeat the
infinitive “to do” and avoid the gerund “doing”.

EXAMPLE OF A PARALLEL CONSTRUCTION QUESTION


Score Value: 700

00:43 GMAT CAT – Section 4 : Verbal 16 of 41

The wings of the dodo bird resembled a turkey’s, and so


were probably without flight.

(A) The wings of the dodo bird resembled a turkey’s


(B) The dodo bird’s wings resembled that of a turkey
(C) A dodo bird’s wings were similar to a turkey
(D) The dodo birds had wings that resembled a turkey
(E) The dodo birds had wings that resembled those of a turkey

Test Section Answer


Time Help Confirm Next
Time
Quit Exit

The original sentence is not a proper parallel construction, since it implies


that the wings of the dodo bird were without flight. Eliminate (A), (B) and (C).
The two parallel items are “had wings” and “were without flight” as these are
both qualities of the correct subject of the sentence, “the dodo birds”. Answer
choice (D) makes the incorrect comparison between “wings” and “a turkey”.
The correct answer is (E).

© The MBA Center 291


Total PrepKit for the GMAT® Step 9.1 Lesson

E. PROBLEMS WITH FAULTY COMPARISONS

Rule: In comparisons, the items being compared must be of a similar


nature. That is, people should be compared with people, furniture
with furniture, and actions with actions.

DRILL 8
DIRECTIONS: FROM THE TWO CHOICES, SELECT THE ONE THAT IS
BETTER.

1.
A) Even after the defeat of the Spanish Armada in 1588, Spain’s fleet
remained greater in size and mobility than England.
or,
B) Even after the defeat of the Spanish Armada in 1588, Spain’s fleet
remained greater in size and mobility than England’s.

When making comparisons, be careful that you are comparing similar


things. In the first example, the size of the “fleet” is larger than the size of
the country (“England” ). Wrong sentences of this kind are not necessarily
grammatically incorrect. You could say that “the parking lots in Texas are
larger than Rhode Island.” (Meaning the actual size of a parking lot is larger
than the actual size of Rhode Island… which is nearly true.)

2.
A) The rooms of the Louvre, without exception, are more ornate than
the MBA Center.
or,
B) The rooms of the Louvre, without exception, are more ornate than
those of the MBA Center.

You get the idea.

3.
A) Publishing journal articles and participating in the many lecture
circuits can be as important to a professor’s success as teaching
classes.
or,
B) Publishing journal articles and participating in the many lecture
circuits can be as important to a professor’s success as class.

What is the comparison? “Publishing” and “participating” are being


compared with “teaching,” not “class.” Make sure you are comparing similar
ideas.

292 © The MBA Center


Sentence Correction

4.
A) When I think of the French teacher, she pales in comparison to the
Spanish teacher’s beautiful spontaneity and passion for the small
things in life.
or,
B) When I think of the French teacher’s attitude, it pales in
comparison to the Spanish teacher’s beautiful spontaneity and
passion for the small things in life.

The comparison in this sentence is between the attitude of the French


teacher and the unstated attitude of the Spanish teacher. We can eliminate
the first sentence because it compares “teacher” to “spontaneity and passion.”

AN EXAMPLE OF A FAULTY COMPARISON QUESTION


Score Value: 680

00:41 GMAT CAT – Section 4 : Verbal 17 of 41


Safety belts, although known occasionally to cause injuries in
automobile accidents, pose less danger to drivers and
passengers than airbags.

! (A) Safety belts, although known occasionally to cause injuries in


automobile accidents, pose less danger to drivers and
passengers than airbags.
! (B) The risks posed by safety belts, which occasionally cause
injuries to drivers and passengers in automobile accidents,
are fewer than airbags.
! (C) Safety belts, although known occasionally to cause injuries in
automobile accidents, pose less danger to drivers and
passengers than do airbags.
! (D) Although safety belts occasionally cause injuries in
automobile accidents, they pose fewer risks to drivers and
passengers than that caused by airbags.
! (E) The risk of safety belts is less to drivers and passengers than
the risk of airbags, although they occasionally cause injuries in
automobile accidents.

Test Section Answer


Time Help Confirm Next
Time
Quit Exit

This sentence may sound right at first because people talk this way all
the time. (Phineas relies on the fact that you will think this sentence is
correct for that reason.) But, in fact, this sentence is ambiguous. It could be
interpreted to mean that safety belts pose less danger to drivers and
passengers than they do to airbags. Eliminate (A). All of the other answer
choices try to correct the faulty comparison, but only (C) manages it
directly and correctly, by inserting the “do” in the sentence as a verb to
accompany airbags. (C) should leap off the page for Correct. What is wrong
with the other choices? (B) compares the risks, rather than the safety belts,
to the airbags. (D) is wordy and includes a pronoun error (“that” instead of
“those”). (E) makes a quantitative error (“less” instead of “smaller” or “lesser”
– more on these soon), is wordy, and places the modifying phrase too far
from the subject.

© The MBA Center 293


Total PrepKit for the GMAT® Step 9.1 Lesson

F. PROBLEMS WITH IDIOMS

Rule: Idioms are simply phrases that denote fixed expressions of


language. The key here is to recognize the preferred usage of
Who makes the rules? idioms for the GMAT, not your personal preferred usage or that of
Rule: In GMAT-land, idioms your region of the world.
must be phrased exactly the
way Phineas wants them. Any
deviation and the sentence is And therein lies the rule: idioms are learned through usage, not through
incorrect. memorization. Below is a list of popular idioms on the GMAT.

approve/disapprove of ashamed of
based on between… .and
composed by/of concerned with
corrected by/about contrast to/with
credit with debate over
defined as different from
discourage from depend on
from… to further from
modeled after much as
not only… but also prefer… .to… .
refer to related to
regard as responsible for (something)
responsible to (a person) result in
result from composed of
consider

DRILL 9
DIRECTIONS: OF THE TWO CHOICES, SELECT THE ONE THAT IS BETTER.

1.
A) Research in astrophysics based on Einstein’s general theory of
relativity has yielded the concept of the black hole.
or,
B) Research in astrophysics based with Einstein’s general theory of
relativity has yielded the concept of the black hole.

2.
A) Regarded as priceless, the Crown Jewels are some of Britain’s most
heavily guarded treasures.
or,
B) Regarded for priceless, the Crown Jewels are some of Britain’s most
heavily guarded treasures.

3.
A) Visitors to California often prefer the compact, picturesque
cityscape of San Francisco over the sprawling, asphalt maze of
freeways that is Los Angeles.
or,
B) Visitors to California often prefer the compact, picturesque
cityscape of San Francisco to the sprawling, asphalt maze of
freeways that is Los Angeles.

294 © The MBA Center


Sentence Correction

4. When you see a long sentence


A) Ashamed by his job writing math questions, Chris decided to pursue with a very short underlined
part, there’s a good chance
his childhood dream of being an AAA manager in his hometown of your knowledge of idioms is
Houghton Lake, MI. being tested.
or,
B) Ashamed of his job writing math questions, Chris decided to pursue
his childhood dream of being an AAA manager in his hometown of
Houghton Lake, MI.

AN EXAMPLE OF AN IDIOM QUESTION


Score Value: 580

00:39 GMAT CAT – Section 4 : Verbal 18 of 41


A great rivalry existed between the New York Yankees, who
represented professionalism and success, in contrast with the
Brooklyn Dodgers, perennial underdogs who appealed to
diehard fans and working-class New Yorkers.

! (A) in contrast with the Brooklyn Dodgers


! (B) in contrast with the Brooklyn Dodgers fans
! (C) against the Brooklyn Dodgers
! (D) as opposed to the Brooklyn Dodgers
! (E) and the Brooklyn Dodgers

Test Section Answer


Time Help Confirm Next
Time
Quit Exit

The idiom in question here is “between … and.” Because the un-


underlined portion of the sentence cannot be altered, the underlined
portion must conform to it. Here, “between” is not underlined, thus the
sentence must follow with “and.”

© The MBA Center 295


Total PrepKit for the GMAT® Step 9.1 Lesson

G. PROBLEMS WITH TENSE

Rule: The verb tenses in English are relatively easy to use; however,
there are some simple tricks. Unless a sentence emphasizes
the sequential nature of events, tenses rely on either the verbs that
come before them or a changing time signal.

DRILL 10
DIRECTIONS: FROM THE TWO CHOICES, SELECT THE ONE THAT IS
BETTER.

1.
A) The long beginning phase of the Quaternary ice age has not been
closely subdivided, but it is given the general name Villafranchian
in Europe or Blancan in North America.
or,
B) The long beginning phase of the Quaternary ice age has not been
closely subdivided, but it has been given the general name
Villafranchian in Europe or Blancan in North America.

This is another example of parallel structure. The same verb form must
be used throughout the statement, in this case the present perfect tense. It
is not clear which action occurred first, therefore we use the present perfect
to refer to any action that occurred in the past but that is connected to the
moment of speaking.

2.
A) Before Hubert set up the wildly successful publishing branch of his
new company, he launched an independent software venture that
sought to introduce new material through the Internet.
or,
B) Before Hubert set up the wildly successful publishing branch of his
new company, he is launching an independent software venture
that sought to introduce new material through the Internet.

There are two events here: the one that occurred first must be put further
in the past than the more recent event. According to GMAT rules, the
“Before” in the sentence clarifies the chronology of events, so “launched”
does not need to be changed to “had launched.”

3.
A) The teacher, although given to expressing his beliefs, had to control
himself when he told Jack that it was imperative that he wrote more
clearly on his final paper.
or,
B) The teacher, although given to expressing his beliefs, had to control
himself when he told Jack that it was imperative that he write more
clearly on his final paper.

The phrase “it was imperative that” calls for the subjunctive, which is the
dictionary form “write.”

296 © The MBA Center


Sentence Correction

EXAMPLE OF A TENSE QUESTION


Score Value: 670

00:37 GMAT CAT – Section 4 : Verbal 19 of 41


Some doctors who were faulted last week by health officials
for improperly prescribing drugs not fully tested by the FDA were
improperly briefed by their hospitals on new medications.

! (A) Some doctors who were faulted last week by health officials for
improperly prescribing drugs not fully tested by the FDA were
! (B) Some doctors who were faulted last week by health officials for
prescribing drugs not fully tested by the FDA had been
! (C) Some doctors that health officials faulted last week for
prescribing improper drugs not fully tested by the FDA have
been
! (D) Last week health officials faulted some doctors for improperly
prescribing drugs not fully tested by the FDA but have been In this sentence, you can
! (E) Last week some of the doctors who were faulted for deduce that the improper
prescribing improperly drugs not fully tested by the FDA but prescriptions came after the
improper briefing; hence, the


had been
briefing occurred before the
prescriptions, and the action
should be further in the past.
Test Section Answer
Notice that among the
Time Help Confirm Next answer choices, only two of
Time
Quit Exit
them employ the past perfect
“had been”. Between those
two, which one is better?
The best answer is (B).
Is the order of events clear in the original sentence? Is it possible to
determine when the doctors were “improperly briefed”? No. Be suspicious of
the verbs immediately, since the last word in the underlined section is a
verb. Scan the answer choices quickly (just the last words) and you can
quickly eliminate answers because of verb problems. We need a certain
order of events, which means we need the past perfect tense. Answer choice
(B) has the past perfect, as does choice (E). (Eliminate (A), (C), and (D).) Of
the two, which one is better? Answer (E) has seriously altered the word order
so that the sentence has no meaning.

© The MBA Center 297


Total PrepKit for the GMAT® Step 9.1 Lesson

H. PROBLEMS WITH REDUNDANCY

Rule: The exact repetition of an idea (even if it uses different words)


already expressed in a sentence should be avoided.

DRILL 11
DIRECTIONS: OF THE TWO CHOICES, SELECT THE ONE THAT IS BETTER.

1.
The two expressions “at least” A) At least three hundred thousand people or more gathered in front of
and “or more” both indicate the White House in Washington, D.C., in 1968 to protest U.S.
minimum amount; the choice involvement in the Vietnam War.
of which one to remove is or,
the test makers', not yours.
B) At least three hundred thousand people gathered in front of the
White House in Washington, D.C., in 1968 to protest U.S.
involvement in the Vietnam War.

The expression “at least” has the same meaning as the expression “or
more”. Therefore one of them is redundant. Be careful. Taking either expres-
sion out will correct the sentence.

2.
A) While it is possible that the Trojan War depicted in Homer’s epic
poems might have actually occurred, it is unlikely that such a large,
well-coordinated expedition was motivated by a love triangle.
or,
B) While it is possible that the Trojan War depicted in Homer’s epic
poems actually occurred, it is unlikely that such a large, well-
coordinated expedition was motivated by a love triangle.

“While it is possible” and “might have” in this sentence are redundant.


Both are referring to the chance that an event occurred.

298 © The MBA Center


Sentence Correction

EXAMPLE OF A REDUNDANCY QUESTION


Score Value: 550

00:35 GMAT CAT – Section 4 : Verbal 20 of 41


The Napoleonic code requires that all voting citizens must
have a fixed address and at least three different kinds of
identification.

! (A) that all voting citizens must have


! (B) that all voting citizens should have
! (C) that all voting citizens that they must have
! (D) that all voting citizens have
! (E) of all voting citizens the having of


Test Section Answer
Time Help Confirm Next
Time
Quit Exit

Can you spot the redundancy? The phrase “requires that” calls for the
subjunctive. It is redundant and ungrammatical to include the modal
“must” or “should” with the subjunctive “have.” Eliminate (A) and (B).
Choice (C) adds an extra “that they.” Eliminate (C). Both (D) and (E) fix the
original mistake, but look what a mess (E) is.

© The MBA Center 299


Total PrepKit for the GMAT® Step 9.1 Lesson

I. PROBLEMS WITH COMPARISON/QUANTITY WORDS

Rule: Countable nouns must be modified by countable quantity words


(few, many, etc.), and noncountable nouns must be modified by
noncountable quantity words (little, much, etc.). Also, be careful
of comparisons between two items and comparisons among three
or more items.

DRILL 12
DIRECTIONS: OF THE TWO CHOICES, SELECT THE ONE THAT IS BETTER.

1.
A) Despite increased funds for political campaigning and voter
In comparisons between only
two items, only the expres-
registration reforms, fewer voters than ever before turned out for the
sions “less” and “more” can recent election.
be used. or,
In comparisons between three B) Despite increased funds for political campaigning and voter
or more items, only the
superlatives “least” and “most”
registration reforms, less voters than ever before turned out for the
can be used. recent election.

Is “voters” a count or noncount noun? Count, of course (one voter, 50


voters). Therefore we must use a count quantity word, like “few” or “many.”

2.
A) Of all the possible disasters that face the Indian elephant, the
possibility of continued decline in habitat size is perhaps the more
difficult to prevent.
or,
B) Of all the possible disasters that face the Indian elephant, the
possibility of continued decline in habitat size is perhaps the most
difficult to prevent.

What is being compared in this sentence? Two things or more than two
things? “Habitat size” is being compared to “all the possible disasters,” a
plural term. We must use the superlative, “most,” because we are comparing
three or more items.

300 © The MBA Center


Sentence Correction

EXAMPLE OF A COMPARISON/QUANTITY WORD QUESTION


Score Value: 550

00:33 GMAT CAT – Section 4 : Verbal 21 of 41

A recent study from the Center for Disease Control has


concluded that much of the currently uncontrolled carcinogens
to which residents of northeastern states are exposed has not
been the major cause of increased breast cancer in the region.

(A) much of the currently uncontrolled carcinogens to which


residents of northeastern states are exposed has
(B) much of the currently uncontrolled carcinogens that residents
of northeastern states are exposed to has
(C) much of the carcinogens that are currently uncontrolled and
that residents of northeastern states are exposed to has
(D) many of the carcinogens that are currently uncontrolled and
residents of northeastern states are exposed to have
(E) many of the currently uncontrolled carcinogens to which
residents of northeastern states are exposed have

Test Section Answer


Time Help Confirm Next
Time
Quit Exit

Again, here is a question where glancing at the first word of each answer
choice can lead you right to the question type. Should it be “much” or
“many”? That depends on whether the noun “carcinogens” is count or
noncount. In fact it is a count noun. (There are only a few noncount nouns
that end with “s”). Since it is a count noun, we can eliminate any answer
choice with noncount quantity words (choices (A), (B), and (C)). Now we
must decide between (D) and (E). (D) has changed the word order in the
sentence so drastically that the sentence no longer makes sense.

© The MBA Center 301


Total PrepKit for the GMAT® Step 9.1 Lesson

J. PROBLEMS WITH STYLE

Rule: The GMAT prefers a concise writing style that avoids the use of
unnecessary or ambiguous phrases. Beyond your own under-
standing of proper writing style, it is helpful to recognize certain
style errors that appear repeatedly and render an answer
choice incorrect.

DRILL 13
DIRECTIONS: OF THE TWO CHOICES, SELECT THE ONE THAT IS BETTER

1.
A) Often the most difficult question for employees working in a
company undergoing being merged with a larger corporation is
whether or not they renegotiate a collective bargaining agreement
with the new company directors.
or,
B) Often the most difficult question for employees working in a
company undergoing a merger with a larger corporation is whether
to renegotiate a collective bargaining agreement with the new
directors.

There are two style problems here. The first is “undergoing being merged”:
whenever possible (and it is always possible), avoid the inclusion of the
word “being.” Second, when given the choice between “whether” and
“whether or not,” go with the former, which carries the same meaning, but
fewer words.

2.
A) After the First World War, the Conference of Paris was working hard
to make a settlement to bring lasting peace to Europe.
or,
B) After the First World War, the Conference of Paris worked hard to
make a settlement to bring lasting peace to Europe.

302 © The MBA Center


Sentence Correction

EXAMPLE OF A STYLE PROBLEM


Score Value: 490

00:31 GMAT CAT – Section 4 : Verbal 22 of 41

Overcrowded and antiquated prison systems are often


resulting in convicted felons serving short sentences or none at
all.

! (A) are often resulting in convicted felons serving short sentences


! (B) often have as the result convicted felons serving short
sentences
! (C) often are resulting in the serving of short sentences by
convicted felons
! (D) result often in short sentences being served by prisoners
! (E) often result in convicted felons’ serving short sentences


Test Section Answer
Time Help Confirm Next
Time
Quit Exit

The first words of the underlined section are terrible; they “sound”
wrong. Eliminate them, this is a style problem. Notice that answer (C) is
using the same words although in a different order; sound any better? Nope,
eliminate it. Could we say “have as the result” in a simpler way? We think so;
eliminate answer (B). Only answers (D) and (E) are left. (D) has a simple
modifier problem because it separates “short sentences” and “none at all.”

© The MBA Center 303


Total PrepKit for the GMAT® Step 9.1 Lesson

SENTENCE CORRECTION LESSON


ANSWER KEY
DRILL 1
1. Type of problem:
Subject/Verb agreement
2. Type of problem: Modifier
3. No error
4. Type of problem: Verb Tense
5. Type of problem: Idiom (just as)

DRILL 2 DRILL 8
1. B 1. B
2. D 2. B
3. A 3. A
4. D 4. B
5. D

DRILL 3 DRILL 9
1. D 1. A
2. C 2. A
3. E 3. B
4. E 4. B
5. C

DRILL 4 DRILL 10
1. B 1. B
2. A 2. A
3. A 3. B
4. A
5. A
6. A, B

DRILL 5 DRILL 11
1. B 1. B
2. A 2. B
3. B
4. B

DRILL 6 DRILL 12
1. B 1. A
2. B 2. B

DRILL 7 DRILL 13
1. B 1. B
2. B 2. B
3. B
4. A

304 © The MBA Center


Sentence Correction

HOMEWORK/PRACTICE TEST
SENTENCE CORRECTION
ANSWER GRID

A B C D E
1 ! ! ! ! !
2 ! ! ! ! !
3 ! ! ! ! !
4 ! ! ! ! !
5 ! ! ! ! !
6 ! ! ! ! !
7 ! ! ! ! !
8 ! ! ! ! !
9 ! ! ! ! !
10 ! ! ! ! !
11 ! ! ! ! !
12 ! ! ! ! !
13 ! ! ! ! !
14 ! ! ! ! !
15 ! ! ! ! !
16 ! ! ! ! !
17 ! ! ! ! !
18 ! ! ! ! !
19 ! ! ! ! !
20 ! ! ! ! !
21 ! ! ! ! !
22 ! ! ! ! !

© The MBA Center 305


Total PrepKit for the GMAT® Step 9.2 Homework

Sentence Correction – Homework Directions

End


When finished
TIME - 25 MINUTES
reading
22 QUESTIONS
directions
Directions: For each question below, a part click on the
or all of the sentence is underlined. Following icon below
the sentence are five versions of the underlined
part. Response (A) repeats the original; the
other responses change the underlined part.
Choose the best version of the sentence. If you Dismiss
think the original is best, choose (A). Otherwise
select one of the other responses. Directions
These questions test your ability to identity
correct and effective writing. Select your
response based on the rules of standard
written English. You should consider grammar,
choice of words, and sentence construction.
Select the response which most clearly and
effectively expresses the meaning intended in
the original sentence. This choice should avoid
awkwardness, ambiguity, redundancy, and
grammatical error.

Test Section Answer


Time Help Confirm Next
Time
Quit Exit

306 © The MBA Center


Sentence Correction

1. In their research, distinguishing between Total (A) a danger that kills or wounds
Factor Productivity growth and capital investment is (B) as a danger killing or wounding
one of the most difficult tasks for economists. (C) killing or wounding
(D) as the deaths or wounds of
(A) is one of the most difficult tasks for economists (E) to kill or wound
(B) are the most difficult tasks for an economist
(C) is a most difficult task for the economist
(D) are the most difficult tasks for economists
(E) are economists’ most difficult tasks
5. While there is hardly unanimous agreement in
the field, some developmental psychologists have
found that much of the determinants of one’s future
2. Since last August, the pound has soared by 18% success in life is traced to the formative years of early
in trade-weighted terms at its highest level since childhood.
leaving Europe’s exchange-rate mechanism in
September 1992. (A) much of the determinants of one’s future
success in life is traced
(A) at (B) much of the many determinants of one’s future
(B) by success in life is traced
(C) for (C) much of the determinants of one’s future
(D) reached success are traced
(E) to (D) many of the determinants of one’s future
success in life can be traced
(E) many of the determinants of one’s future
success in life are to be traced
3. Huddie Leadbetter’s legendary performances on
the twelve-string guitar, Vincent Van Gogh’s landscape
paintings done in small rural towns in France, and
John Kennedy O’Toole’s A Confederacy of Dunces are 6. In the twelfth century students and masters
all examples of talented artists who only came into full migrated to the portion of the Left Bank known as the
recognition after their deaths. Latin Quarter, of which the name, though being
derived from Roman times, nevertheless continued to
(A) examples of talented artists who only came into be appropriate, because of all medieval scholarship
full recognition after their deaths being in Latin.
(B) examples of artistic works that came into full
recognition only after the deaths of the artists (A) though being derived from Roman times,
(C) examples of artistic works not fully recognized nevertheless continued to be appropriate,
when the artists were dead because of all medieval scholarship being in
(D) examples of artistic works that came into full Latin
recognition only after their deaths (B) though having derived from Roman times,
(E) artistic works which exemplify when artists do nevertheless had still been aptly named, for all
not come into full recognition until after their medieval scholarship was in Latin
deaths (C) having nonetheless derived from Roman times,
continued appropriately, for all medieval
scholarship had been in Latin
(D) derived from Roman times, nevertheless was
4. For residents of nations that have been involved still appropriate, as all medieval scholarship was
in wars in recent decades, the 110 million land mines in Latin
planted that have not yet been detonated remain a (E) having nonetheless derived from Roman times,
deadly menace, a danger that kills or wounds 30,000 had been named appropriately, as all medieval
people every year. scholarship was in Latin

GO ON TO THE NEXT PAGE


© The MBA Center 307
Total PrepKit for the GMAT® Step 9.2 Homework

7. In 1965 gold mining in Nevada accounted for (C) suggests that government efforts toward
forty-five percent of the gold produced in the United expanding wildlife preserves and stiffen
States; and sixty percent in 1995. penalties against poachers have been
successful, but that
(A) United States; and sixty percent in 1995 (D) suggests that government efforts to expand
(B) United States; in 1995 the figure was sixty wildlife preserves and stiffen penalties against
percent poachers were successful, but that
(C) United States; and in 1995 sixty percent (E) suggests that government efforts to expand
(D) United States; and sixty percent in 1995 was wildlife preserves and stiffen penalties against
the figure poachers have been successful, but
(E) United States that rose to sixty percent in 1995

10. What was as extraordinary as the numerous


8. Exit polls during the recent election show that sightings of comets in the last decade have been the
the numbers of people whose primary political concern insights gained by scientists through their study on
is the high level of crime grew by more than a twenty such diverse topics as the extinction of the dinosaurs
percent increase in the past four years and accounted and the age of the solar system.
for more than forty-two percent of the entire voting
community. (A) What was as extraordinary as the numerous
sightings of comets
(A) numbers of people whose primary political (B) The thing that was as remarkable as sighting
concern is the high level of crime grew by more many comets
than a twenty percent increase (C) No less extraordinary than the numerous
(B) numbers of people whose primary political sightings of comets
concern is the high level of crime grew more (D) Sighting numerous comets has been none the
than twenty percent less remarkable than
(C) numbers of people whose primary political (E) Sighting of numerous comets has been no less
concern is the high level of crime were raised by remarkable as
more than twenty percent
(D) number of people whose primary political
concern is the high level of crime increased by
more than twenty percent 11. Economists are hard-pressed to explain why,
(E) number of people whose primary political occasionally, out of even years of steady economic
concern is the high level of crime was raised by growth emerge the twin problems of unemployment
more than a twenty percent increase and large income gaps.

(A) emerge the twin problems of unemployment


and large income gaps
9. The decline in the Environmental Protection (B) emerges twin problems, those being unemploy-
Agency’s index of threatened species suggest that ment and an increasing income gap
government efforts to expand wildlife preserves and (C) emerge the twin problems, increasing income
stiffen penalties against poachers have been successful, gaps and unemployment
but that the continued decline of old-growth forests in (D) emerge those twin problems, large income
the Pacific Northwest indicates that in the coming gaps and unemployment
decades many plant and animal species are likely to be (E) emerges the twin problems of increasing
added to the EPA’s endangered species list. unemployment and income gaps

(A) suggest that government efforts to expand


wildlife preserves and stiffen penalties against
poachers have been successful, but that
(B) suggest that government efforts to expand
wildlife preserves and stiffen penalties against
poachers have been successful, but

GO ON TO THE NEXT PAGE


308 © The MBA Center
Sentence Correction

12. Eisuke Sakakibara’s recent book, “Beyond (B) Unlike physics or other sciences, which they
Capitalism,“ which extols the successes of the Japanese admit require a rigorous study of basic rules,
economy, has allowed him to advocate almost any many people are disinclined to recognize that
economic policy without fear of charges that he is fields in the humanities, such as literary criticism
acting contrary to Japan’s interests. and philosophy, also require a rigorous study of
basic rules.
(A) Eisuke Sakakibara’s recent book, “Beyond (C) Unlike physics or other sciences, fields in the
Capitalism,“ which extols the successes of the humanities, such as literary criticism and
Japanese economy, has allowed him philosophy, bring out a disinclination on the
(B) Eisuke Sakakibara’s recent book, “Beyond part of many people to recognize that they
Capitalism,“ because of its extolling of the require a rigorous study of basic rules.
successes of the Japanese economy, has made (D) Many people, willing to admit that physics and
him able other sciences require a rigorous study of basic
(C) Eisuke Sakakibara’s “Beyond Capitalism,“ the rules, are disinclined to recognize that fields in
recent book which extols the successes of the the humanities, such as literary criticism and
Japanese economy, makes it possible philosophy, require the same kind of rigorous
(D) Eisuke Sakakibara, whose recent book “Beyond study.
Capitalism“ extols the successes of the Japanese (E) Many people have a disinclination to recognize
economy, has allowed him that fields in the humanities, such as literary
(E) By extolling the successes of the Japanese criticism and philosophy, require a rigorous
economy in his recent book, “Beyond study of basic rules while willing to admit that
Capitalism,“ Eisuke Sakakibara has been able they need them for physics and other sciences.

13. Recent research in geophysics indicates that the 15. Lasers, widely used today in such diverse
severity of the destruction caused by an earthquake technologies as medical endoscopes and compact disc
depends to a great extent on if the crustal movement players, were originally considered by many people to
that leads to shaking unfolds quickly. be a scientific curiosity of little practical value.

(A) if the crustal movement that leads to shaking (A) to be a scientific curiosity of little practical value
unfolds quickly (B) as being a scientific curiosity and of little
(B) whether the crustal movement that leads to practical value
shaking unfolds quickly (C) as scientifically curious and of little practical
(C) whether or not the crustal movement that can value
lead to shaking will unfold quickly (D) scientific curiosities of little practical value
(D) its speed for moving the crust leading to (E) scientific curiosities and to be of little practical
shaking value
(E) the speed of its crustal movement leading to
shaking

14. Unlike physics or other sciences, there is a


disinclination on the part of many people to recognize
that fields in the humanities, such as literary criticism
and philosophy, require a rigorous study of basic rules.

(A) Unlike physics or other sciences, there is a


disinclination on the part of many people to
recognize that fields in the humanities, such as
literary criticism and philosophy, require a
rigorous study of basic rules.

GO ON TO THE NEXT PAGE


© The MBA Center 309
Total PrepKit for the GMAT® Step 9.2 Homework

16. Just as merchants in ancient Rome relied on 19. Spacecraft that fail to break out of the Earth’s
naval patrols to keep trade routes free of pirates, so do gravitational field either burn up completely in the
producers of intellectual property today depend on atmosphere, which poses no danger of colliding into
international trade organizations to prevent illegal the Earth, and break up into large components
reproduction of copyrighted material. capable of withstanding the intense heat and pressure
of their fall back to the Earth.
(A) Just as merchants in ancient Rome relied on na-
val patrols to keep trade routes free of pirates, so (A) which poses no danger of colliding into the
(B) Like merchants in ancient Rome relied on naval Earth, and break up
patrols to keep trade routes free of pirates, so (B) posing no danger of colliding into the Earth, or
(C) Merchants in ancient Rome relied on naval breaking up
patrols to keep trade routes free of pirates, and (C) without the posing of any danger of colliding
in a similar way into the Earth, or breaking up
(D) As with the reliance of merchants in ancient (D) posing no danger of colliding into the Earth, or
Rome on naval patrols to keep trade routes free break up
of pirates, so too (E) with the posing of no danger of colliding into
(E) Similar to ancient Rome where merchants relied the Earth, and break up
on naval patrols to keep trade routes free of
pirates, just so

20. Studies show that when there are several siblings


close in age, the oldest sibling often takes on parental
17. If the current rate at which fossil fuels and duties, such as supervising activities, taking the younger
natural gases are consumed worldwide continues into siblings to school and to pick them up afterwards, and
the next century, the world’s natural energy resources reprimanding them for dangerous behavior.
have been and will continue to decline, pushing up
prices and posing the risk of increased pollution. (A) taking the younger siblings to school and to
pick
(A) have been and will continue to decline (B) taking the younger siblings to school and
(B) have and will continue to decline picking
(C) will continue to decline, as they already did (C) to take the younger siblings to school and to
(D) will continue to decline, as they have already pick
(E) will continue to decline (D) to take the younger siblings to school and pick
(E) to take the younger siblings to school while
picking

18. One potential adverse effect of powerful halluci-


nogens such as LSD or acid is that not only do users
experience the immediate impact of disorientation, but
also they will encounter considerable suppression of
the ego for a period of up to three weeks additionally.

(A) but also they will encounter considerable


suppression of the ego for a period of up to
three weeks additionally
(B) but also there’ll be a three-week period of ego
suppression additionally
(C) but the ego becomes suppressed for a period
up to three weeks additionally
(D) but also they will encounter considerable
suppression of the ego for a period of up to
three weeks
(E) but the user’s ego is suppressed for a period of
up to three weeks more

GO ON TO THE NEXT PAGE


310 © The MBA Center
Sentence Correction

21. The artists who did the Altamira cave paintings


15,000 years ago employed flint tools to carve outlines
on the rock face, add color using brushes made of
twigs, feathers, tufts of hair or fur, and clumps of moss.

(A) add color using brushes made of twigs,


feathers, tufts of hair or
(B) and add color using twig brushes, feathers, tufts
of hair and
(C) and then added color using brushes made of
twigs, feathers, tufts of hair or
(D) using brushes to add color made of twigs,
feathers, tufts of hair or
(E) having added color using brushes made of
twigs, feathers, tufts of hair and

22. Poets’ Corner in Westminster Abbey


memorializes nearly six centuries of legendary English
authors with graves and plaques inscribed with such
celebrated names as Dylan Thomas, Samuel Johnson,
and Lord Byron.

(A) inscribed with such celebrated names as


(B) inscribing such celebrated names like
(C) inscribed with celebrated names like
(D) that are inscribed with names as
(E) with inscriptions of such celebrated names as

STOP
IF YOU FINISH BEFORE TIME IS EXPIRED
YOU MAY CHECK YOUR WORK

© The MBA Center 311


Total PrepKit for the GMAT® Step 9.2 Homework

SENTENCE CORRECTION
HOMEWORK/PRACTICE TEST
ANSWER KEY

1. A
2. E
3. B
4. C
5. D
6. D
7. B
8. D
9. E
10. C
11. A
12. E
13. B
14. D
15. D
16. A
17. E
18. D
19. D
20. B
21. C
22. A

312 © The MBA Center


Sentence Correction

EXPLANATIONS FOR 5. (D)


SENTENCE CORRECTION This is a quantity word question that also tests verb
tense usage. Since “determinants” is a countable
HOMEWORK/PRACTICE TEST noun (presumably there are a number of them that
can be counted) it requires the quantity word “many”
instead of “much,” which is only used with non-count
1. (A) nouns. So, eliminate any answer choices that retain
First notice the pronoun “their“ in the “much”: choices (A), (B), and (C). (E) corrects the
ununderlined part. This plural pronoun must have a quantity word problem but adds a verb tense error
plural antecedent, so any choice with the singular with the infinitive “to be traced” at the end of the
“economist“ is wrong (cross out (B) and (C)). Then, sentence. Only (D) changes “much” to “many” and
scanning the remaining choices, what main difference corrects the verb form from the infinitive “to be
do you notice? There is an “is“ in (A) and an “are“ in traced” to the modal “can be traced.”
(D) and (E). What is the subject of the verb? The
gerund “distinguishing,” which is singular (verbal
nouns are always singular). You need that “is.” 6. (D)
The phrase “because of all medieval scholarship being
in Latin“ in choice (A) is awkward. The present participle
2. (E) in the continuous aspect “being derived“ is also
If you only look at the underlined part and a few of awkward and doesn't make sense; the name is not still
the words that follow, this question may seem difficult. being derived. The past perfect “had been in Latin“ in
“At“ its highest level and “reached“ its highest level choice (C) implies that the Latin scholarship preceded
might sound okay. But which earlier word in the the appropriate naming of the area. Also the phrase
sentence determines the correct choice? “Soared.” “continued appropriately“ is not correct, as “continued“
“Soared to“ is an idiom and so (E) is the right answer. should not be used as a finite verb; “the name continued
to be appropriate“ is better. This whole question can be
done rather simply by eliminating the choices that
3. (B) don't have parallel verb tenses: (B) uses the past perfect
Scan the answer choices and compare. (A) says “had still been aptly named“ and the simple past “was in
“examples of talented artists,” (B), (C), and (D) say Latin.” (E) makes the same error. (D) is consistent in
“examples of artistic works,” and (E) says just “artistic tense, using the simple past for both clauses.
works.” What actually was mentioned in the
ununderlined part? “Performances,” “paintings“ and a
novel. These are works, not artists. So (A) is wrong. 7. (B)
The phrase “exemplify when“ in (E) is unidiomatic. The right answer must state what was “sixty
Now you have to compare (B), (C), and (D). (B) percent“ in 1995. In (A) and (C), is “sixty percent“ the
sounds okay and is the right answer. (C) doesn't make subject, the object, or what? After the semicolon, you
sense; you don't talk about when people were dead, as must have an independent clause. (A) and (C) are
presumably they still are dead. It should say “when the unclear and ungrammatical. In (D) there is a
artists were alive.” (D) may sound pretty good, but misplaced modifier; “in 1995“ should modify “the
notice the pronoun “their.” What is its antecedent? It figure,” not “sixty percent.” Choice (E) makes it sound
is supposed to replace “artists,” but “artists“ has not as though the United States rose to sixty percent. (B)
yet been used. It cannot replace the artists mentioned is clear, direct, and grammatical.
only in the possessive form (as in Van Gogh's, etc.)

8. (D)
4. (C) Choices (A), (B), and (C) all incorrectly use the
Choice (A) is wrong because it illogically implies plural “numbers.” In (A), the clause “grew by more
that the danger, and not the mines, kills or wounds than a twenty percent increase“ is redundant. In (E),
people. (B) makes the same error and unnecessarily “was raised by more than a twenty percent increase“ is
includes the word “as.” The present participles also redundant. (D) is the only choice that correctly
“killing“ and “wounding“ correctly modify the noun uses the singular “number“ and avoids redundancy.
phrase “land mines that... detonated.” (D) leaves a sen-
tence fragment, as the conjunction “as“ requires a
finite verb in the clause that follows. The infinitive 9. (E)
construction in (E) implies that the purpose of the A quick scan here reveals a 3-2 split between
mines is to “kill or wound 30,000 people every year“ “suggest“ and “suggests.” Which is correct? The
and, grammatically, should not follow the comma subject is “decline,” a singular noun, and so the
after “menace.” singular “suggests“ is required. Eliminate (A) and (B).

© The MBA Center 313


Total PrepKit for the GMAT® Step 9.2 Homework

(C) makes a parallelism error; “expanding“ and “Eisuke Sakakibara… has allowed him,” where it should
“stiffen“ are not parallel. (D) does not make this error, say “… the book has… allowed him.” (E) begins with a
as it correctly uses the parallel “expand“ and “stiffen,” modifying clause, followed by the correct subject of
but it incorrectly includes the word “that“ at the end. the modifier, Eisuke Sakakibara.
Rereading the sentence just for the grammatical
skeleton results in “the decline… suggests…. that the
continued decline… indicates that…“ There seems to 13. (B)
be one too many “thats.” This is a question of Notice that (A) begins with “if,” (B) and (C) begin
parallelism. The parallel structure is: “The decline in the with “whether,” and (D) and (E) use neither, but
EPA's index suggests that… but the continued decline of rather begin with “its speed“ and “the speed.” “If“
old-growth forest indicates that…” (E) correctly uses should never begin an indirect question. “If“ is only
parallel structure and has a verb (suggests) that agrees used for conditional statements. (B) and (C) are
with the subject (decline). similar, though (C) is longer and more awkward. The
“or not“ is redundant after “whether“ and the future
“will“ is also redundant. It is unclear in (D) and (E)
10. (C) what is “leading to shaking“ – is it the crust, the
Choices (A) and (B) do not have parallel tenses; moving, or the speed? “Speed for“ in (D) is also
they both use the past “was“ while the ununderlined unidiomatic. (B) correctly uses the word “whether“
part uses the present perfect “have been.” (D) and unambiguously indicates that crustal movement
incorrectly uses “none the less remarkable than“; the leads to shaking.
correct usage of the idiom for comparisons is “no less
remarkable than.” (E) makes the error of using “less…
as.” Only (C) avoids a tense error and uses the correct 14. (D)
idiom for the comparison. This looks like a nightmare question. The sentence
is long and completely underlined. Nevertheless, the
word “unlike“ at the beginning of the sentence tells us
11. (A) that there is a comparison involved, giving us a clue of
A quick look should reveal a 3-2 breakdown in where to begin. In (A), “Unlike physics or other sciences“
terms of verb number; (A), (C), and (D) use the plural should be compared to other academic fields. It
“emerge,” and (B) and (E) use the singular “emerges.” cannot be followed by “there is a disinclination…” In
What is the subject? It is a bit tricky here, as the (B), “physics or other sciences“ are erroneously
subject follows the verb. The subject is “twin prob- compared to “many people.” (C) corrects the faulty
lems.” Since it is plural, eliminate (B) and (E). (C) and comparison, but it uses the awkward phrase “brings
(D), besides being stylistically weak, are ambiguous. In out a disinclination on the part of many people“ and
(C), it is unclear whether the problems are 1) an ambiguously uses the pronoun “they.” (Does “they“
increasing income gap and 2) unemployment, or 1) refer to “fields in the humanities“ or “many people“?
an increasing income gap and 2) increasing un- Does it matter?) (D) is clear and idiomatic, avoiding
employment. In (D), does “large” modify only ambiguous pronouns and faulty comparisons. (E) has
“income gaps,” or “unemployment” as well. (B) is the many flaws, including the ambiguous uses of “they“
only choice with the correct verb number, a lack of and “them.” In the phrase “they need them,” who
ambiguity, and a clear style. needs what? “Many people have a disinclination“ is also
Note that the ambiguity in (C) and (D) is quite minor. unidiomatic; “many people are disinclined“ is better.
Phineas likes to give you choices in which the main error is
style, but to avoid being accused of giving subjective
questions (for, after all, style generally is a subjective 15. (D)
issue), they (or should we say “it“?) include very minor (D) is the only choice that uses the correct idiom of
grammatical faults as well. consider + adjective. Incorrect are “consider as“ and
“consider to be“ used in (A), (B), (C), and (E).The other
choices include other errors, too. (E), for example, is
12. (E) inconsistent. It uses “to be“ in front of “of little practical
This is a question of modifiers and pronouns. (A) value“ but not in front of “scientific curiosities.” (A) and
uses the pronoun “him.” What does “him“ replace? (B) use the singular “a scientific curiosity“ to describe
“Eisuke Sakakibara“ has not been mentioned, except in the plural “lasers.”
the possessive form, which doesn't count, as it is really
“Eisuke Sakakibara's recent book“ that has been
mentioned. (B) makes the same error and uses the 16. (A)
awkward phrase “because of its extolling of the This is a question of idiom. For comparisons, “just
successes.” (C) is unclear because it doesn't specify who as“ must be complemented by “so.” (B) is wrong
can advocate economic policy. (D) illogically states because “like” is a preposition and cannot introduce

314 © The MBA Center


Sentence Correction

an independent clause, that is, a clause with a subject 20. (B)


and a verb. (C) implies that the way producers of This is a case of parallel construction, probably
intellectual property depend on trade organizations is easier than question 19. There is a list, beginning with
similar to the way merchants in Rome relied on naval “supervising,” and so we need “taking“ and “picking.”
patrols. The statement does not compare how the two (B) is the only choice that provides these.
groups are (or were) reliant, but simply the fact that in
both cases, there is (or was) a reliance. In general the
GMAT doesn't like the phrase “in a similar way.” (D) 21. (C)
makes the error of comparing “the reliance of Since these paintings were made 15,000 years
merchants“ with “producers of intellectual property.” (E) ago, we should use the past tense to describe what
also makes a faulty comparison, comparing “ancient the artists did. Therefore eliminate (A) and (B), as
Rome“ to “producers of intellectual property.” “add“ is the present tense. In (D), it sounds like the
color, not the brushes, were “made of twigs, feathers,
…“ (E) illogically implies that they added color before
17. (E) incising the outlines. Even if you like to draw in the
This is a verb tense question. “have been and will colors before making the outlines, the word “added“
continue to decline“ is faulty, because “have been“ indicates that the color came after the outline,
requires the present participle “declining.” In (B), conflicting with the implication of the past participle
“have“ requires the past participle “declined.” (C) and “having added.”
(D) are redundant; something that continues to do
something must have already been doing it. (E) is
simple, clear, and correct. 22. (A)
How would you know to choose (A) if you weren’t
sure it contained an error? By comparing answer
18. (D) choices and eliminating those that have obvious errors
Any time you see “not only” in a Sentence or don’t sound “right” when you read them to
Correction question you should be on alert for a “but yourself. Since on this difficult question there is no real
also” to follow it. A quick scan of the choices reveals a pattern to spot (only (A) and (C) begin with the
“3-2” split between 3 choices that have a “but also” participle “inscribed”) among the choices, you must
and 2 that don’t. Right away, eliminate the two that examine each and every one looking for differences
don’t: choices (C) and (E). Since the best answer and similarities. (B), eliminate because the continuous
should be stylistically as well as grammatically correct, aspect of “inscribing” doesn’t match the simple
a redundancy error such as “but also … additionally” present verb, and the comparative phrase “such …
(also and additionally share a very similar meaning) in like” doesn’t exist in standard written English. (C) is a
choices (A) and (C) is unacceptable. You can eliminate classic Almost Right Answer: it retains the correct
these without further evaluation. Only (D) corrects the participle “inscribed” from (A) but adds a new mistake
redundancy error and correctly uses the idiom “not with another nonexistent comparative “with … like.”
only … but also,” one of Phineas’ favorites. (D) starts off OK with the formal but not necessarily
incorrect “that are inscribed,” but again an error is
added with the imaginary (and incorrect) “with …as.”
19. (D) (E) is simply wordy and awkward when inserted into
This question is a bit tricky. There are the issues of the original statement. “With graves and plaques with
parallel construction and modifiers. The word “which“ inscriptions…” is not an improvement over the original
in choice (A) grammatically modifies the preceding statement. Even though it’s close, eliminate.
word, “atmosphere.” But this is illogical, as it should be
the spacecraft, not the atmosphere, “which poses no
danger of colliding into the Earth.” In (B), the participle
“posing,” unlike the relative pronoun “which,” can
modify a word further back, in this case “spacecraft.”
The problem with (B) is that “breaking up“ should be
parallel with “burn up,” not with “colliding.” The
“spacecraft… either burn up… or break up.” Therefore
“breaking up“ in (B) is wrong. (C) makes the same
error. (E) says “and break up,” whereas “or“ is required
following the word “either.” In (C), “without the posing
of any danger“ is also awkward. (D) is the only choice
that correctly follows “either burn up“ with the parallel
“or break up.”

© The MBA Center 315


© The MBA Center
Step 10
Sentence Correction
Workshop

© The MBA Center


Total PrepKit for the GMAT® Step 10 Workshop

Sentence Correction – Workshop Directions

End
When finished
TIME: 50 MINUTES reading
30 QUESTIONS
directions
Directions: In each problem below, either
click on the
part or all of the sentence is underlined. icon below
Following the sentence are five versions of
the underlined part. Answer choice (A)
repeats the original; the other answer choices
vary. If you think that the original phrasing is Dismiss
best, choose (A). If you think one of the other
answer choices is best, select that choice.
Directions

This section tests your ability to identify


correct and effective writing. Select your
response based on the rules of standard
written English. You should consider
grammar, choice of words, and sentence
construction. Select the response which most
clearly and effectively expresses the meaning
intended in the original sentence. This choice
should avoid awkwardness, ambiguity,
redundancy, and grammatical error.

Test Section Answer


Quit Exit Time
Time
Help Confirm Next

318 © The MBA Center


Sentence Correction

EASY
4. Of all the efforts to prevent Napoleon's 1812
campaign into Russia, the destruction of crops, stores
1. The decline in levels of educational achievement of grain, and livestock by the retreating Russian army
among grade-school students, whether the result of was perhaps the more effective.
decreasing emphasis on homework or of inadequate
public school systems, have been a disturbing national (A) the destruction of crops, stores of grain, and
trend. livestock by the retreating Russian army was
perhaps the more effective
(A) have been a disturbing national trend (B) the destruction of crops, stores of grain, and
(B) have caused a national disturbance livestock by the retreating Russian army was
(C) has been a cause of national disturbance perhaps the most effective
(D) has been a disturbing national trend (C) in the destruction of crops, stores of grain and
(E) has been disturbing the national trend livestock by the retreating Russian army was
perhaps the most effective
(D) destroying crops, stores of grain and livestock
by the retreating Russian army was perhaps the
2. While some vacationers prefer destinations more effective
offering luxury and convenience, others choose more (E) the most effective on the part of the Russian
remote locations where little news or services exist. army being perhaps the destruction of crops,
stores of grain and livestock
(A) little news or services exist
(B) little news or services exists
(C) few services and little news exists
(D) there is little news or services available 5. We know less about prehistoric peoples that left
(E) there are few services and little available news their legacy only in stones, bones, and pottery, than we
do about the Sumerians, who were the world's first
writers and historians.

3. In a surprising turnaround, the City Council (A) prehistoric peoples that left their
announced that it would overturn the decision to (B) prehistoric peoples, who left their
impose a weekday curfew of 10:00 p.m. on minors. (C) prehistoric peoples and their
(D) prehistoric peoples leaving its
(A) it would overturn the decision to impose a (E) prehistoric peoples leaving their
weekday curfew
(B) it would overturn the impose of a weekday
curfew
(C) it would overturn to impose a weekday curfew 6. In industrialized nations, recent generations of
(D) they would overturn the imposition of a curfew parents tend to have less children than did their
on the weekdays ancestors.
(E) they would overturn the decision to impose a
weekday curfew (A) tend to have less children
(B) have a tendency to have fewer children
(C) tend to have a few less children
(D) tend to have fewer children
(E) not so many children

GO ON TO THE NEXT PAGE


© The MBA Center 319
Total PrepKit for the GMAT® Step 10 Workshop

MEDIUM
9. The challenges faced by many developing
African nations include the establishment of a stable,
democratic government, fighting the spread of AIDS,
7. Mastering both the technical complexities and and nurturing their domestic industries.
physical demands of the sport, Dan Gable’s wrestling
career, which culminated in an Olympic gold medal, (A) the establishment of a stable, democratic
has distinguished him as one of America’s all-time government, fighting the spread of AIDS, and
greatest wrestlers. nurturing their domestic industries
(B) establishing a stable, democratic government,
(A) Dan Gable’s wrestling career, which culminated fighting the spread of AIDS, and nurturing their
in an Olympic gold medal, has distinguished domestic industries
him as one of America’s all-time greatest (C) the establishment of a stable, democratic
wrestlers government, the fight against the spread of
(B) Dan Gable’s wrestling career, which has AIDS, and nurturing their domestic industries
distinguished him as one of America’s all-time (D) to establish a stable, democratic government,
greatest wrestlers, culminated in an Olympic fight the spread of AIDS, and nurture their
gold medal domestic industries
(C) Dan Gable, one of America’s all-time greatest (E) the establishment of a government which is
wrestlers, culminated his career by winning an both stable and democratic, the fight against
Olympic gold medal the spread of AIDS, and to nurture their
(D) Dan Gable culminated his career with an domestic industries
Olympic gold medal, one of America’s all-time
greatest wrestlers
(E) the Olympic gold medal won by Dan Gable
distinguished him as one of America’s all-time 10. In an attempt to wage its war on drugs at all
greatest wrestlers levels, the Drug Enforcement Agency has focused its
efforts both on the small-time dealer, who is subject to
mandatory prison sentences for drug-related offenses,
and on the leaders of international trafficking rings,
8. The hospital planning commission has begun to who are now pursued even in their own countries.
institute new guidelines that require of health care
employees the wearing of face masks and gloves. (A) both on the small-time dealer, who is subject to
mandatory prison sentences for drug-related
(A) that require of health care employees the offenses, and on the leaders of international
wearing of face masks and gloves trafficking rings, who are now pursued even
(B) that require health care employees wearing (B) both on the small-time dealer, who is subject to
face masks and gloves mandatory prison sentences for drug-related
(C) that require health care employees to wear face offenses, and pursuing the leaders of
masks and gloves international trafficking rings
(D) requiring face masks and gloves being worn by (C) both on the small-time dealer, who is subject to
health care employees mandatory prison sentences for drug-related
(E) to require of health care employees that they offenses, and on the leaders of international
wear face masks and gloves trafficking rings, the pursuit taking place even
(D) both on the small-time dealer, being subject to
mandatory prison sentences for drug-related
offenses, and on the leaders of international
trafficking rings, being now pursued even
(E) both on the small-time dealer, who is subjected
to mandatory prison sentences for drug-related
offenses, and on trafficking by leaders of
international trafficking rings, who are now
pursued even

GO ON TO THE NEXT PAGE


320 © The MBA Center
Sentence Correction

11. What was as extraordinary as Beethoven’s 13. Medical researchers, alarmed by the
musical development and impressive productivity has appearance of a lethal new strain of influenza, are
been his influence on successive generations of devoting their efforts to understanding and to develop
classical composers such as Robert Schumann, Anton a vaccine against it.
Bruckner, and Gustav Mahler.
(A) are devoting their efforts to understanding and
(A) What was as extraordinary as Beethoven’s to develop
musical development and impressive produc- (B) are devoting their efforts in order to understand
tivity and develop
(B) The thing that was as extraordinary as (C) are devoting their efforts first to understanding
Beethoven’s musical development and impres- it, then to developing
sive productivity (D) are devoting their efforts to the understanding
(C) No less extraordinary than Beethoven’s musical of and the development of
development and impressive productivity (E) is devoting their efforts to understand it, then
(D) Extraordinary musical development, for develop
Beethoven, and impressive productivity
(E) Development of Beethoven’s extraordinary
music has been no less impressive as his pro-
ductivity 14. According to studies by the Commerce
Department, families, which are richer than ever
before, is holding much less in cash than they were
only ten years ago.
12. Stephen Hawking's theories concerning the
concept of the black hole suggest that the universe (A) which are richer than ever before, is holding
began from a singular point, and that it will continue much less in cash than they were
to expand. (B) which are richer than ever before, are holding
much less in cash than they were
(A) suggest that the universe began from a singular (C) which are richer than ever before, are holding
point, and that it will continue to expand much less in cash than
(B) suggests that the universe began from a (D) which are richer than ever before, are holding
singular point, and that it will continue to much fewer in cash than
expand (E) which are richer than ever before, are holding
(C) suggests that the universe began from a fewer cash than they were
singular point, and that expansion has occurred
and will continue
(D) suggest that the universe began from a singular
point, but that it has and will continue to
expand
(E) suggest a singular point as having started the
universe, but expansion will continue

GO ON TO THE NEXT PAGE


© The MBA Center 321
Total PrepKit for the GMAT® Step 10 Workshop

DIFFICULT
15. The Rosetta Stone, one of archeology's most
significant and valuable finds, are on display in the
British Museum.
17. Less expensive than a long-distance telephone
(A) are on display in the British Museum call and swifter and more convenient than a letter, an
(B) are displayed in the British Museum e-mail recipient may send and receive messages
(C) is on display in the British Museum virtually instantly from around the world.
(D) is in the British Museum, on display
(E) is a display in the British Museum (A) an e-mail recipient may send and receive
messages virtually instantly from around the
world
(B) an e-mail recipient may send and receive
16. To create a fantastic new invention and messages around the world at a fraction of the
successfully marketing it are two very different tasks. cost and with infinitely greater speed than was
previously possible
(A) successfully marketing it are two very different (C) an e-mail address allows its owner to send
tasks messages and receive them as well virtually
(B) to market it successfully are two very different instantaneously from around the world
tasks (D) e-mail enables messages to be sent and
(C) the successful marketing of it is two very received virtually instantly around the world
different tasks (E) e-mail allows the sending and receiving of
(D) to market it with success is a very different task written messages to be achieved instantly
(E) having marketed it successfully are different around the world
tasks

18. Not only are the Chengjiang fossils from


southwest China older than the Burgess fossils from
the Canadian Rockies by more than one hundred
million years, yet they’re not so deformed and are less
difficult preparing for analysis.

(A) yet they’re not so deformed and are less


difficult preparing for analysis
(B) but they are also less deformed and less difficult
to prepare for analysis
(C) they are not as deformed and are less difficult in
preparation for analysis
(D) but that they are less deformed and less difficult
to prepare for analysis
(E) yet they are not as deformed, and preparing
them for analysis is less difficult

GO ON TO THE NEXT PAGE


322 © The MBA Center
Sentence Correction

19. The defenders against foreign invaders, the 21. Unlike previous hypotheses concerning climate
bone marrow creates white blood cells, also known as changes, paleontological circles have introduced a
lymphocytes. new theory for the extinction of the dinosaurs in the
form of a lethal comet.
(A) The defenders against foreign invaders, the
bone marrow creates white blood cells, also (A) paleontological circles have introduced a new
known as lymphocytes. theory for the extinction of the dinosaurs in the
(B) Defending the body against foreign invaders, form of a lethal comet
white blood cells are created in the bone (B) paleontologists are examining a new theory for
marrow, also known as lymphocytes. the extinction of the dinosaurs, one involving a
(C) Created in the bone marrow, defense against lethal comet
foreign invaders is provided by white blood (C) a new theory for the extinction of the dinosaurs
cells, also known as lymphocytes. has made its way into paleontological circles,
(D) Created in the bone marrow, white blood cells, this time citing a lethal comet as the cause of
also known as lymphocytes, defend the body extinction
against foreign invaders. (D) a new theory citing a lethal comet as the cause
(E) The bone marrow creates white blood cells, the of extinction of the dinosaurs has made its way
defenders against foreign invaders, also known into paleontological circles
as lymphocytes. (E) the introduction of a new theory, citing a lethal
comet as the cause of dinosaur extinction, has
made its way into paleontological circles

20. Responding to the public’s growing concern for


maintaining a healthy, balanced diet, food manufac-
turers are marketing products that have less fat and 22. Given the choice between higher tax rates,
calories than did their predecessors. which would fund extensive social programs, or lower
tax rates, resulting in reduced resources for the socially
(A) less fat and calories than and economically disadvantaged, the American public,
(B) less fat and less calories than judging from voting patterns, seems to opt for the
(C) less fat and fewer calories than latter.
(D) fewer fat and calories than
(E) less fat and fewer calories compared to (A) which would fund extensive social programs, or
lower tax rates, resulting in
(B) funding extensive social programs, or lower tax
rates, resulting in
(C) which would fund extensive social programs, as
opposed to lower tax rates, which result in
(D) which would fund extensive social programs,
and lower tax rates, which result in
(E) which would fund extensive social programs,
and lower tax rates, the result being

GO ON TO THE NEXT PAGE


© The MBA Center 323
Total PrepKit for the GMAT® Step 10 Workshop

23. Although their marriage was a tumultuous one, 25. Depending on the source, the legend of King
F. Scott Fitzgerald and Zelda maintained a regular Arthur and the Knights of the Round Table take place
correspondence in which they confided their most either in the French region of Brittany or in parts of
intimate thoughts only to one another. England.

(A) to one another (A) take place either in the French region of
(B) one with the other Brittany or in
(C) one with another (B) take place in either the French region of
(D) each other Brittany or in
(E) to each other (C) takes place in either the French region of
Brittany or in
(D) takes place either in the French region of
Brittany or in
24. While some of the blood samples were (E) either take place in the French region of
contaminated today, rendering them useless for Brittany or
further experiments, the scientists already finished
their research.

(A) While some of the blood samples were 26. While some environmental litigators believe
contaminated today, rendering them useless for that environmental law should be a requirement in
further experiments, the scientists every law school curriculum, others maintain that
(B) While some of the blood samples had been students should study environmental law only if it
contaminated today, rendering them useless for would be offered as a separately elective course.
further experiments, the scientists
(C) Some of the blood samples had been (A) only if it would be offered as a separately
contaminated today, rendering them useless for elective course
further experiments; fortunately, the scientists (B) if it would only be offered as an elective course,
had separately
(D) Though some of the blood samples were (C) if it was offered only as a course elected
contaminated today, rendering them useless for separately
further experiments, the scientists had (D) if it would be offered only as a course separated
(E) Today some of the blood samples were and elective
contaminated, rendering them useless for (E) only if it is offered as a separate, elective course
further experiments, but the scientists

GO ON TO THE NEXT PAGE


324 © The MBA Center
Sentence Correction

27. Director Martin Scorsese and actor Robert De 29. While it is probable that a single mother
Niro have worked together on numerous films, from language may very well have spawned all the world's
Taxi Driver, a disturbing portrait of one man's languages, determining its specific origins has proven
alienation and insanity, and Raging Bull, a difficult for linguists.
biographical account of boxer Jake La Motta.
(A) may very well have spawned all
(A) and Raging Bull, a biographical account of (B) may very well to have spawned all
boxer Jake La Motta (C) is a valid possibility for having spawned all
(B) up to Raging Bull, a biographical account of (D) was spawning all
boxer Jake La Motta (E) spawned all
(C) in addition to Raging Bull, boxer Jake La Motta
biographical account
(D) to Raging Bull, a biographical account of boxer
Jake La Motta 30. While opinions vary, William Shakespeare is
(E) to Raging Bull, a biographical account considered by many literary scholars the greatest
recounting the life of boxer Jake La Motta English writer of all time.

(A) is considered by many literary scholars the


greatest English writer of all time
28. An MBA, while undoubtedly an essential degree (B) is considered by many literary scholars to be
for qualification for more satisfying, higher-paying the greatest English writer of all time
jobs, remain an expensive and time-consuming (C) is considered by many literary scholars as being
endeavor. the greatest English writer of all time
(D) is the greatest English writer of all time, as
(A) remain an expensive and time-consuming considered by many literary scholars
endeavor (E) is considered as the greatest English writer of all
(B) remain an endeavor both expensive and time- time by many literary scholars
consuming
(C) remains an expensive and time-consuming
endeavor
(D) remains an expensive endeavor which is time-
consuming as well
(E) is remaining an endeavor as expensive as it is
time-consuming

STOP
IF YOU FINISH BEFORE TIME IS EXPIRED
YOU MAY CHECK YOUR WORK

© The MBA Center 325


Total PrepKit for the GMAT® Step 10 Workshop

SENTENCE CORRECTION WORKSHOP


ANSWER KEY

1. D
2. E
3. A
4. B
5. B
6. D
7. C
8. C
9. B
10. A
11. C
12. A
13. C
14. B
15. C
16. B
17. D
18. B
19. D
20. C
21. D
22. D
23. E
24. D
25. D
26. E
27. D
28. C
29. E
30. A

326 © The MBA Center


Step 11
Critical Reasoning

© The MBA Center


Total PrepKit for the GMAT® Step 11.1 Lesson

GENERAL OUTLINE

About one-third of the questions in the Verbal Section of the GMAT


CAT are Critical Reasoning questions. These questions evaluate your
ability to reason effectively in the areas of argument construction and
argument evaluation. In this lesson, we’ll introduce the most common
types of Critical Reasoning questions and show you specific strategies for
answering each one of them.

THE FOLLOWING TOPICS WILL BE COVERED IN THIS LESSON

Anatomy of a Critical Reasoning Question


The Different Types of Inferences
The MBA Center Approach to Critical Reasoning Questions
The Eight Critical Reasoning Question Types

KEY TERMS

Argument: Central to every Critical Reasoning question is the argument.


An argument is an ordered line of reasoning composed of premises,
assumptions, and a conclusion. Understanding the elements of an argument
is essential to performing well in this section.

Premise: Each Critical Reasoning argument contains at least one premise.


Premises are pieces of factual information that provide evidence used to
support the conclusion of the argument. For the purposes of Critical Reasoning
arguments, premises are facts not subject to dispute.

Conclusion: The conclusion is the endpoint of the line of reasoning of an


argument. Think of it as the result of the argument. The line of reasoning
leading to a conclusion is often where errors in logic are made.

Assumption: Assumptions are unstated facts and logical connections in an


argument. In order for the conclusion of an argument to be true, the
assumptions upon which that argument is based must also be true.

328 © The MBA Center


Critical Reasoning

READY…

Critical Reasoning questions test your ability to use basic logic to analyze
and critique arguments made up of premises and conclusions. While they
are a large part of your Verbal score, the approximately 14 Critical
Reasoning questions in the Verbal Section test grammatical and verbal
abilities much less than do the Reading Comprehension or the Sentence
Correction questions. Rational thinking, or common sense, plus a
competent display of the basic rules of logic are what these questions test.
Indeed these types of questions are also part of the Analytical Section in the
Graduate Record Examination (GRE), which is notoriously abstract and
difficult. Critical Reasoning and Sentence Correction questions are the most
important types of questions to learn and understand for the Verbal part of
the GMAT. Critical Reasoning questions do not outnumber Reading
Comprehension questions, but they do have a greater influence on your
score. Remember too that your Verbal score depends heavily on the
accuracy of your answers to the first five questions or so. (See “What Is This
Thing Called the GMAT?” to review adaptive scoring.) Also, you are more
likely to find Critical Reasoning questions at the beginning of the Verbal
Section than the longer, more-involved Reading Comprehension passages.
So, stay alert in this section, practice, and get to know these unique
question types, which count for so much (you’ll soon see) on the GMAT. To
begin the exam informed and with confidence is an excellent strategy.
One more note: official test directions for this section state that no
knowledge of the terminology and conventions of formal logic is
presupposed. That may be true, but Critical Reasoning questions are not
simple or familiar to most students and do require some introductory
knowledge of logic in order to understand the structure of formal
arguments and the question types.

© The MBA Center 329


Total PrepKit for the GMAT® Step 11.1 Lesson

DIRECTIONS

Before starting the section, or by clicking on the Help button while


answering a Critical Reasoning question, the following directions will
appear:

GMAT CAT – Section 4: Verbal Directions

End
When finished
Directions: Choose the best response for reading
each question in this section. directions
click on the
To review these directions for subsequent icon below
questions of this type, click on HELP.

Dismiss
Directions

Test Section Answer


Time Help Confirm Next
Time
Quit Exit

DIRECTIONS TRANSLATED FOR YOU

Each Critical Reasoning problem consists of a short, usually flawed,


argument, a question about the argument, and five answer choices. No
outside knowledge is necessary. Each question can be answered solely on
the basis of the information contained in the argument.

330 © The MBA Center


Critical Reasoning

Score Value: 550

01:15 GMAT CAT – Section 4: Verbal 1 of 41


In years past, professional baseball players lifted weights less but
were also injured less often during games. Obviously, the more an
athlete lifts weights, the higher the likelihood of injury.

The conclusion above presupposes which of the following?

! (A) The increase in baseball injuries is due to a factor other than


weightlifting.
! (B) The activities of baseball players represent those of athletes
as a group.
! (C) Most baseball injuries today result from too much weight-
lifting.
! (D) There is no proven correlation between how much athletes
lift weights and how likely they are to be affected by injury.
! (E) Weightlifting has always been common practice for
professional athletes.

Test Section Answer



Time Help Confirm Next
Time
Quit Exit

ANATOMY OF A CRITICAL REASONING QUESTION

Every Critical Reasoning question is made of three parts: an argument,


a question, and answer choices.

The Argument
In years past, professional baseball players lifted weights less but were
also injured less often during games. Obviously, the more an athlete
lifts weights, the higher the likelihood of injury.

The Question
The conclusion above presupposes which of the following?

The Answer choices


(A) The increase in baseball injuries is due to a factor other than weightlifting.
(B) The activities of baseball players represent those of athletes as a group.
(C) Most baseball injuries today result from too much weightlifting.
(D) There is no proven correlation between how much athletes lift weights and
how likely they are to be affected by injury.
(E) Weightlifting has always been common practice for professional athletes.

The Argument: Each argument contains three main parts: premise,


conclusion, and assumption. Take a look.

1. A premise or several premises. Here the premise is “In years past,


professional baseball players lifted weights less and were injured less
often during games.”
A premise is a piece of information; it is factual, and for the
purposes of the question, is not arguable. It is upon the premise(s)
that an argument’s conclusion depends.

© The MBA Center 331


Total PrepKit for the GMAT® Step 11.1 Lesson

2. A conclusion. The conclusion constitutes the main point or opinion


Although Critical Reasoning is
a part of your Verbal score, it and is usually placed at the end of an argument – though on the
is more a test of analytic GMAT it might also appear at the beginning or in the middle.
abilities than it is of verbal Conclusions rely upon their factual premises for support but are
abilities. You can find exactly
themselves open to question and critique – your job, in fact, is to
the same kind of questions on
other tests, such as the GRE, analyze their various flaws.
where they are in the A conclusion is sometimes easy to spot by noticing introduction
"Analytic," and not "Verbal," words such as “thus,” “therefore,” “obviously,” or “clearly then.” But,
section.
the GMAT is a sophisticated test, and in general trigger words like
these are rarely found. What is a conclusion then? A statement or
sentence that cannot stand on its own.

3. An assumption. An unstated premise or idea that must be true for an


argument to be valid. As these Critical Reasoning questions are
constructed, an assumption, though unstated (does not appear in the
text), is in fact part – the unstated part – of an argument. Some
examples soon to come will illustrate this for you.

4. Some arguments also include a fourth element: the inference. You


might think of an inference as a reasonable guess or estimation
based on the facts, or premises, given to you. It is the result of the
type of reasoning that Sherlock Holmes was so proud of – deduction.
The most important concept here is that inferences on the GMAT are
obvious, undeniable truths which the argument provides but are not
directly stated. Get to know inferences very well for they appear in one
form or another throughout the Verbal Section.

One more note: though questions that test your ability to understand
and recognize an assumption are not rare in this section, they’re not to be
feared either. Once you’ve devoted the time to know them well, in class and
in practice on your own, you’ll see assumptions for what they are:
standardized, or uniform, and therefore knowable.

The Questions: Eight different question types have been identified in


this section and each type of question requires a specific
approach. Because of the variety of available question types, you
must concentrate on understanding the nature of each question. Stick
closely to the text and determine precisely what you’re being asked to
do. You might be asked to identify assumptions, to supply a
conclusion, to strengthen, or provide support for, an argument, or
to choose an answer choice that weakens, or does not provide
support for, an argument. More details and question types to come.

The Answer Choices: Unlike in Sentence Correction, no answer choice


patterns are found in Critical Reasoning. The content of the answer
choices is never the same – it varies according to the argument and
question type. Answer choices must always be read very carefully
and you must read each and every one. On more difficult
questions a close comparison (after Process of Error Identification)
between two possible answer choices is sometimes necessary,
indeed unavoidable. As the language used is often dense, formal,
and unfamiliar, get in the habit of patient, thorough evaluations of
Critical Reasoning answer choices, questions, and arguments.

332 © The MBA Center


Critical Reasoning

DRILL 1
Directions: In the exercise below there are six statements containing
assumptions. Your job is to identify and write down the assumption
contained in each statement. When you are finished, compare your answers
with those in the key below.

1- The Winavers must be planning to buy a new house, since they


visited the office of the local real estate agent yesterday.

2- John’s grade point average is below 3.2, so if he wants to enter


Graduate School he should apply as early as possible.

3- Unemployment in France is due to a lack of economic activity, so if


the government reduces the length of the work week, the
unemployment rate should decrease.

4- Professor Sully only gives good grades to European students. After all,
last semester he gave good grades to Hubert and Elio who are both
Europeans.

5- Anyone who wants to work as a strategy consultant wants to pursue


an MBA ranked in the top ten. Giles wants to apply to Wharton,
which is always ranked among the top ten MBA programs in the US.
Therefore, Giles wants to become a strategy consultant. Conclusions can appear at the
beginning (first sentence),
the middle (buried), or the
6- Many people who describe themselves as painters are not painters by end (last sentence) of an
profession, since painting is not their main source of income. argument. Concentrate your
efforts in this section on
learning to pinpoint a
conclusion—it does not stand
on its own, but must logically
follow from the meaning
(stated or implied) of a
statement or assertion.

Assumptions:

1- All people that visit the local real estate agent’s office want to buy
new houses.
2- Applying early to graduate school makes admission easier.
3- Diminishing legal work duration increases economic activity.
4- Hubert and Elio were the only students that received good grades.
5- All who apply to Wharton want to become strategy consultants.
6- To be a painter by profession one must have painting as one’s main
source of income.

© The MBA Center 333


Total PrepKit for the GMAT® Step 11.1 Lesson

AIM…

THE DIFFERENT TYPES OF ASSUMPTIONS

Once you've learned to find the conclusion and the premises of an


argument, you’ll improve your analytical skills by learning to take apart
arguments step by step, looking for their logical structure (premises,
conclusion) and their assumptions. If you learn to recognize the logical
flaws (the assumptions behind the inference) quickly and easily, you will
find that the actual questions asked on Critical Reasoning are, for the most
part, straightforward.

A) ANALOGY

An analogy argument is an argument in which the connection


between the premises is made by analogy. For instance, take a
look at the following argument:
Score Value:

One way to recognize


Analogy arguments is to pick 01:13 GMAT CAT – Section 4: Verbal 2 of 41
out a key term in the
conclusion that is not
included in the premises. In
the example here, “human Even though no actual cases have ever been documented, we
beings” is the new term in the know that human beings could transmit rabies to other human beings
conclusion. through a bite, since dogs can transmit rabies to all mammals
through a bite.

Premise: Dogs can transmit rabies through their bite.


Conclusion: Human beings can transmit rabies through their bite.

Test Section Answer


Time Help Confirm Next
Time
Quit Exit

Where is the logical gap in this argument? Once again, there is


something new that is introduced in the conclusion: the idea of
human beings. Where did this idea come from? It doesn't exist
anywhere in the premises since the premise is about dogs. How
can we draw any conclusion at all about human beings based on
what is true of dogs? Only if we assume that human beings are
similar to dogs, with regard to the potential of their bite to
transmit diseases like rabies.

This argument thus rests on the assumption that human beings


and dogs are analogous with regard to their potential to spread
diseases through their bite. In order for this argument to be
strengthened, evidence that supports this assumption (such as:
human beings and dogs both carry rabies in their saliva; human
beings and dogs can both transmit hepatitis through a bite, etc...)
is necessary. To weaken this argument, evidence that denies this
analogy between human beings and dogs (such as: human beings,
unlike dogs, do not carry rabies in their saliva; the teeth of human
beings do not penetrate as deeply as do those of dogs, etc...) needs
to be provided.

334 © The MBA Center


Critical Reasoning

B) THE PAST PREDICTS THE PRESENT/FUTURE

This sort of argument assumes that the way something happened


in the past will be analogous to how it will happen in the
present/future; or that the conditions in the past are similar
enough to predict an outcome for the present/future. Here is an
example:

01:11 GMAT CAT – Section 4: Verbal 3 of 41


When we hired Marizonian immigrants 10 years ago, the project
was a failure. They had problems speaking English and had
difficulties adjusting to American life, both of which made them
unreliable workers. For all these reasons, we should not have
Marizonian immigrants in our factory.

Conclusion: We should not hire Marizonian immigrants to work in


our factory.
Premise: When we tried doing this 10 years ago, it was a miserable
failure.
Premise: Their lack of knowledge of English and their difficulties in
adjusting to life in the U.S. made them unreliable workers.

Test Section Answer
Time Help Confirm Next
Time
Quit Exit

In this argument, the logical gap is a bit subtle. Note that the
conclusion is in the present tense, and is a statement about the
present: that we should not hire Marizonian workers today. On the
basis of what evidence? Ten years ago it didn't work. But how can
we draw a conclusion about what is the current situation based on
evidence from 10 years ago? Only if we believe that the conditions
are approximately the same today as 10 years ago, and that
therefore what happened 10 years ago will repeat itself again in the
present day. To strengthen this argument, give evidence to show
that the conditions are approximately the same today as 10 years
ago, and that therefore the past will accurately predict the present;
to weaken this argument, give evidence to show that today the
conditions are different, and that therefore the past may not
accurately predict the present or future.

© The MBA Center 335


Total PrepKit for the GMAT® Step 11.1 Lesson

C) CAUSAL ARGUMENTS

A causal argument is an argument that concludes that X is


responsible for (causes) Y, based on the evidence of a temporal
relation between X and Y – that X accompanies Y, or that X
precedes Y. For instance, we might say that because the doorbell rings
when you push the button, that therefore your pushing the button
causes the doorbell to ring. The problem, of course, is that not every
temporal relation implies a causal relation for it's also possible that two
things occur at the same time simply out of coincidence. If lightning
strikes just as I step out the door, this does not give me much reason
to believe that my stepping out the door caused the lightning. Thus, a
temporal relation (in which one development directly precedes
another, or in which two developments occur simultaneously) does
not by itself justify the conclusion that there is a causal relationship.

Therefore, if an argument includes only evidence of a temporal


relation and draws the conclusion that there is a causal
relationship, the author is making two assumptions: that the
relation is, in fact, a causal relation and not mere coincidence,
and that there is no other possible cause. (Remember this. It will be
important when we see weaken/strengthen questions.)

Here is one example.

01:11 GMAT CAT – Section 4: Verbal 4 of 41


The number of crimes committed in Los Angeles is highest during
school hours (between 8:00 a.m. and 3:00 p.m. on weekdays). On
any one given day, over 10,000 teenagers fail to attend school, and
are unaccounted for during these hours. Therefore, it is these
teenagers who are responsible for the high crime rate.

Premise: Crime is highest during school hours.


Premise: During school hours, many students skip school.
Conclusion: These students are responsible for the crime.

Phineas loves causal relation-


ships: the leap from a
temporal correlation in the Test Section Answer
premises to a causal correla- Quit Exit Time
Time
Help Confirm Next
tion in the conclusion is the
most common argument flaw
on the GMAT.
Here we are presented with events that occur at the same time:
during the same period (8:00 a.m. - 3:00 p.m. weekdays), two things
happen: a large number of crimes are committed, and a large
number of students skip school. But this could be a mere
coincidence of unrelated events. There is no evidence to show that
these two phenomena have a more substantial, that is, less
circumstantial relationship. There is no reason, based on the mere
fact that these two things happen together, to believe that one is
responsible for the other. Therefore, the conclusion rests on the
assumption that this is not a mere coincidence, and that teenagers
who skip school are, in fact, responsible for this crime. It is possible
to strengthen this argument by referring to further evidence that
teenagers who skip school commit crimes, and it is possible to
weaken this argument with evidence that this is a mere coincidence.

336 © The MBA Center


Critical Reasoning

D) WOULD THE PROPOSED SOLUTION SOLVE THE PROBLEM?

Sometimes an argument presents a problem and proposes a


solution to the problem. But it isn't always certain that the
proposed solution will, in fact, solve the problem. (In some cases,
the proposed solution might even make the problem worse!) In this
sort of case, the argument could be weakened by showing that the
proposed solution does not, in fact, solve the problem that it is
supposed to solve. This kind of problem is very close to a causal
argument because it depends, effectively, on the factors that cause the
problem in the first place. To respond to such a question does not
require a thorough investigation into the causes: all we need to show is
why the proposed solution might not work:

01:09 GMAT CAT – Section 4: Verbal 5 of 41


The number of crimes committed by adolescents in Los Angeles
is highest during those hours when they should be in school
(between 8:00 a.m. and 3:00 p.m. on weekdays). If we want to stop
teenage crime, we should take action to ensure that teenagers do not
skip school.

Premise: Teenage crime is highest during school hours, when Some Critical Reasoning
teenagers are absent from school. arguments look eerily similar
Conclusion: Teenage crime will be stopped by making sure that to Problem Solving word
teenagers attend their classes. problems. Historically, GMATs
averaged 2 to 3 math-heavy

Critical Reasoning questions


per section.
Test Section Answer
Time Help Confirm Next
Time
Quit Exit

The problem presented in this argument is that of teenage crime,


which happens mostly during school hours, by students who skip
school. The author then proposes that we stop teenage crime by
making sure that teenagers attend school. But do we know that this
solution will stop crime? Because we don't know what is responsible
for teenage crime, how do we know that making them go to school
will stop it? The thinking goes: if teenagers are in school, they
probably cannot at the same time commit crimes. But maybe, in the
absence of opportunities to commit crimes during school hours,
they will simply commit more crime after school, and the number
of crimes may remain unchanged. Thus the proposed solution may
not, in fact, solve the problem. We could weaken this argument by
giving any evidence to show that the number of teenage crimes
could – introducing an element of doubt – stay the same, even if
teenagers are forced to attend their classes.

© The MBA Center 337


Total PrepKit for the GMAT® Step 11.1 Lesson

Flawed arguments don’t


necessarily have false
E) EQUIVALENCE OF DATA (# = %)
conclusions; an argument is
considered flawed if readers A common type of argument is one that mixes real values and
are not provided with enough percentages or rates. Have a look at the following argument:
information to judge for
themselves, even if the
conclusion is accurate.

01:07 GMAT CAT – Section 4: Verbal 6 of 41


Because of a recent crackdown on school absences, the number
of crimes committed by teenagers during school hours (8:00 a.m.-
3:00 p.m. on weekdays) has dropped dramatically. It follows that a
higher percentage of teenage crime is committed outside of school
hours than during school hours.

Premise: The number of crimes committed during school hours has


dropped.
Conclusion: The percentage of crimes committed is lower during
school hours than outside of school hours.


Test Section Answer
Time Help Confirm Next
Time
Quit Exit

Once again, take careful notice of the different kinds of data


presented. The conclusion states that the percentage of crimes
during school hours is lower than the percentage outside of school
hours. The premise, however, states only that the number of
crimes during school hours has dropped. This does not necessarily
imply a change in the percentage. If, for instance, the number of
crimes outside school hours dropped as well, then the relative
percentage of crimes committed during and outside of school
hours might remain the same. We simply don't have the
information required to draw any conclusion at all about the
percentages of crimes committed during and outside of school
hours.

338 © The MBA Center


Critical Reasoning

F) PARTICULAR TO GENERAL

This sort of argument draws a conclusion about what is true for


each of the members of a class of things, based on what is true for
one specimen or member of that class. That is, it makes a
generalization based on what is true in a single, or particular,
case. In any such argument, it has been assumed that the
generalization is valid, and that what is true of one member of the
class is equally true of all of the other members of the class.

Here is a subtle example of an argument that contains, among


other things, a shift from the particular to the general:

01:05 GMAT CAT – Section 4: Verbal 7 of 41

John must be allergic to strawberries. He suffered an extreme


allergic reaction the other day after a meal of whipped cream and
strawberries. John's doctor said that John's allergic reaction was Although only one as-
certainly caused by something that John ate. John's mother certifies sumption is tested per ques-
tion, it’s good practice to
that he's not allergic to whipped cream. So, John must be allergic to
identify as many as you can in
strawberries. each of these sample ques-
tions: the more you do, the
Premise: John suffered a bad allergic reaction the other day after easier it gets.
eating whipped cream and strawberries.
Premise: The doctor said that the allergic reaction was definitely due
to something that John ate.
Premise: We know that John is not allergic to whipped cream,
Conclusion: So John must be allergic to strawberries.

Test Section Answer


Time Help Confirm Next
Time
Quit Exit

This argument relies on a couple of major assumptions. First,


there is the causal assumption that it really was the strawberries
that caused the allergic reaction and not something else. Even
though we know that it wasn't the whipped cream that caused his
reaction, there are still other possibilities: perhaps his reaction was
due to something else that John ate during the day. We could
therefore weaken this argument by citing some explanation for
John's allergic reaction other than strawberries. But even if we
know that it was the strawberries that caused his allergic reaction,
there is still another major assumption in this argument. Note that
the argument concludes that John is allergic to strawberries in
general, which means all strawberries. But all we know from the
premise is that John had an allergic reaction to these particular
strawberries. Thus the argument generalizes, and assumes that if
he's allergic to any one strawberry, then he's allergic to all of them.
We could thereby weaken this argument in two ways: by showing
that the particular strawberries that John ate are idiosyncratic in
some way (perhaps a pesticide was used on the strawberries, or
perhaps some chemical reaction occurred between the strawberries
and the whipped cream that created a substance to which John is
allergic), and that John's reaction to these strawberries may not
imply that he's allergic to all strawberries.

© The MBA Center 339


Total PrepKit for the GMAT® Step 11.1 Lesson

THE WRONG ANSWER FACTORY

We have analyzed the different types of inferences contained in the rea-


soning of formal arguments, and suggested how to strengthen or weaken
them. Equally important to scoring higher on this section is getting to know
how wrong answer choices are made. You’ll soon see that Phineas has set up
standardized mistakes to trick you and that the answer choices are not
written randomly.

The test taker's dilemma: Most people choose wrong answers for one of
two reasons:

1. They ignore slight but significant differences between answer choices.

2. They miss the flaw in the argument.

Let's return to Example 1:


The conclusion above presupposes which of the following?

Thus, we are looking for an assumption that must be true for the
argument to work.

Now, look back at the first argument:

In years past, professional baseball players lifted weights less but were
also injured less often during games. Obviously, the more an athlete
lifts weights, the higher the likelihood of injury.

Which part of the argument is the conclusion?


What is the flaw in the author's reasoning?

Answering these two questions before going to the answer choices can
help you to predict the answer (or something close to the answer).
Occasionally an argument will not be flawed, in which case you should go
directly from the argument to the answer choices.

Now let's look at the choices:

(A) The increase in baseball injuries is due to a factor other than weightlifting.
(B) The activities of baseball players represent those of athletes as a group.
(C) Most baseball injuries result from too much weightlifting.
(D) There is no proven correlation between how much athletes lift weights and
how likely they are to be affected by injury.
(E) Weightlifting has always been common practice for professional athletes.

How does Phineas design the wrong answer choices? As with


the other sections, he constructs answer choices that look
tempting but actually contain distorted or irrelevant information.
In Critical Reasoning, the choices that include irrelevant
information are outside the scope of the argument. Choice (E), for
example, is outside the scope of the argument.
Some choices are within the scope, but distort information in
the passage or stretch logic too far. Choice (C), for example, is
tempting, but the word "most" is a distortion. We do not know
how many baseball players are injured because they lift weights
too much, only that they lifted weight less in the past.

340 © The MBA Center


Critical Reasoning

THE MBA CENTER APPROACH TO CRITICAL REASONING QUESTIONS

Here is our method for coping with Critical Reasoning questions.

Step 1: Read the question first.


Reading the question tells you what type of information you are looking
for in the argument. Sometimes it will be an assumption, sometimes a
conclusion, an inference, or a weakness.

The most common types of question are:

- Which of the following can be most reasonably inferred from


the information above?

Translation: What is missing in the passage above?

- Which of the following is an assumption made in the passage


above?

Translation: Find the answer that supports the logic of the conclusion.

- Which of the following statements, if true, would be a valid


objection to the conclusion drawn above?

Translation: Choose the answer that weakens the conclusion of the


passage above.

Step 2: Read the argument.

Each time you read an argument you should identify the premise(s), the
conclusion, and the flaw (the assumption).

Example:

01:03 GMAT CAT – Section 4 :Verbal 8 of 41


Increased competition among certain grocery stores has resulted in


each store finding new methods of attracting customers. For example,
Val-U-Mart is offering a deal in which every time a customer purchases
an item at full retail price, he will receive a coupon entitling him to pur-
chase the same item at half the retail price the next time he comes to
the store. Therefore, Val-U-Mart’s profit will increase as customers are
drawn back into the store.

Premise : Increased competition among certain grocery stores has


resulted in each store finding new methods of attracting cus-
tomers.
Premise : For example, Val-U-Mart is offering a deal in which every
time a customer purchases an item at full retail price, he will
receive a coupon entitling him to purchase the same item at
half the retail price the next time he comes to the store.
Conclusion: Therefore, Val-U-Mart’s profit will increase as customers
are drawn back into the store.
Assumption: Val-U-Mart will be able to make a profit offering products
at half retail price.

Test Section Answer


Time Help Confirm Next
Time
Quit Exit

© The MBA Center 341


Total PrepKit for the GMAT® Step 11.1 Lesson

Some questions ask you to


Step 3: Paraphrase the argument using your own words.
complete the passage: for
these, the premise or
conclusion required for The idea is to gain time and to be able to compare the reasoning in the
completion will be located in argument with the answer choices. Phineas’ style is heavy, wordy, complex.
the answer choices instead of
The best way to understand the argument is to restate the main idea from
the argument.
the premises in the most simple manner using your own words.

DRILL 2
DIRECTIONS: PARAPHRASE THE STATEMENTS BELOW.

1. Most airline passengers don’t pay attention when flight attendants


give safety instructions before take-off. Unless airlines find a more
effective means of communicating to their passengers, most
passengers will not know what to do in case of an on-board
emergency.

Paraphrase: ______________________________________________________
_________________________________________________________________
_________________________________________________________________
_________________________________________________________________
_________________________________________________________________
_________________________________________________________________
_________________________________________________________________
_________________________________________________________________

2. In an effort to reduce daily commuter traffic entering a certain city,


the city government plans to raise the tolls on all major roads that
have access to the city. The government plans to use part of the
revenues generated by the tolls to modernize commuter rail systems.
However, critics object that under this plan drivers will unfairly pay
for a service that is beneficial only to those who ride trains.

Paraphrase:
_________________________________________________________________
_________________________________________________________________
_________________________________________________________________
_________________________________________________________________
_________________________________________________________________
_________________________________________________________________
_________________________________________________________________
_________________________________________________________________

342 © The MBA Center


Critical Reasoning

3. A rock-and-roll band recently won a lawsuit against a rap artist who


had ‘sampled’ the rock band’s song, using some of the music from the
song in one of his or her own. Subsequently, a law was passed
requiring rap musicians wishing to 'sample' to pay royalties
to the original artists. This will raise the cost of production for rap
albums, and therefore discourage the music industry from signing rap
acts.

Paraphrase:
_________________________________________________________________
_________________________________________________________________
_________________________________________________________________
_________________________________________________________________
_________________________________________________________________
_________________________________________________________________
_________________________________________________________________
_________________________________________________________________

4. A scientist planted two groups of plants under identical conditions of


light, temperature, humidity, and moisture. Every day he would play
sound effects of thunderstorms to one of the groups of plants and
sounds of city traffic to the other. The group to which he played
thunderstorms all died within a few weeks, but the other group
thrived during the experiment. He therefore concluded that the sound
of city traffic is more effective for helping plants grow than is the
sound of thunderstorms.

Paraphrase:
_________________________________________________________________
_________________________________________________________________
_________________________________________________________________
_________________________________________________________________
_________________________________________________________________
_________________________________________________________________
_________________________________________________________________
_________________________________________________________________

5. In normal conditions, increased competition among businesses leads


to lower prices. In midtown Manhattan, however, the prices of audio-
visual equipment are, on average, higher than those in most other
areas of America, despite the fact that there is a high concentration
of stores specializing in audio-visual merchandise.

Paraphrase:
_________________________________________________________________
_________________________________________________________________
_________________________________________________________________
_________________________________________________________________
_________________________________________________________________
_________________________________________________________________
_________________________________________________________________
_________________________________________________________________

© The MBA Center 343


Total PrepKit for the GMAT® Step 11.1 Lesson

Step 4: Predict the answer.

Making a prediction before going to the answer choices saves you from
confusion once you do start to compare answer choices. Remember, four
answer choices are designed to test your concentration in the face of
temptation. Take a moment between understanding the argument and
evaluating the answer choices to form an idea of what you think the best
answer choice should look like: it should be well-reasoned and strictly
relevant to the text.

Example:

Director of airport security: Because of a recent increase in terrorist


attacks, we have increased security measures at all terminals. We
now require that all passengers show two forms of identification
before boarding, and we x-ray and screen all carry-on luggage
thoroughly.

Which of the following, if true, most seriously weakens the


viability of the airport’s plan?

Make your predictions:

1. ______________________________________________________________
_________________________________________________________________
_________________________________________________________________
_________________________________________________________________
or

2. ______________________________________________________________
_________________________________________________________________
_________________________________________________________________
_________________________________________________________________
or

3. ______________________________________________________________
_________________________________________________________________
_________________________________________________________________
_________________________________________________________________

Now take a look at the answer choices and compare them with your pre-
dictions. Did you guess right? If not, how close to the right answer were you?

(A) Airport regulations had previously required passengers to show only one
form of identification.
(B) New government regulations state that all airport personnel must undergo
a thorough screening process before they start work.
(C) The rate of terrorist incidents has increased dramatically over the past
several years.
(D) Several recent terrorist incidents have resulted from explosives hidden in
luggage stored underneath the plane.
(E) The airport has never been victimized by a terrorist incident.

(D) is the best answer.

344 © The MBA Center


Critical Reasoning

If you are stuck on a question


Step 5: Use Process of Error Identification to eliminate the wrong and must guess, always
answers. choose the plain and simple
answer choice over the
The Process of Error Identification is the easiest way to find the “best complicated one.
answer.“ To narrow down the answer choices, you should identify:

- Answer choices outside the scope of the argument


- Answer choices that distort or exaggerate ideas from the argument

Score Value: 600

01:01 GMAT CAT – Section 4: Verbal 9 of 41

Ambi Inc. and Robin Corp. manufacture bargain stereo equipment.


Both companies use the same professional shipping service, and for
both companies shipping costs account for eight percent of the total
expenditures. Ambi Inc., in an effort to gain an advantage over Robin
Corp., plans to begin to ship their merchandise themselves.

Which of the following, if true, would most call into question


the viability of the plan of Ambi Inc.?

(A) Other shipping companies charge higher rates than the


company employed by Ambi Inc. and Robin Corp.
(B) The shipping company makes a profit of over 10 percent on
its dealings with Ambi Inc., but over 12 percent on its
dealings with Robin Corp.
(C) The shipping company used by both companies offers
special deals for bulk delivery. Every word is important in a
(D) Projections indicate that Ambi Inc.'s shipping costs under its Critical Reasoning argument.
new program would amount to eight percent of its total There are no details which are
expenditure, in addition to a substantial initial cash outlay. unimportant. This is the
(E) Robin Corp. ships materials farther than does Ambi Inc. principal difference between
close critical reading and
selective comprehensive
reading on the GMAT.
Test Section Answer
Time Help Confirm Next
Time
Quit Exit

© The MBA Center 345


Total PrepKit for the GMAT® Step 11.1 Lesson

FIRE!
We have seen that the answer choices and the arguments are written
according to standardized types. In this part of the lesson, we will show you
that each question type is standardized too and that there are 8 types of
questions for GMAT Critical Reasoning arguments. Each question type has
its own traps, and a specific strategy is required to ace each one of them.

A) DRAW A CONCLUSION

Some Critical Reasoning questions do not have a conclusion. For


this type of question you are asked to supply a possible
conclusion. The conclusion that you choose must closely and
logically follow from the premises in the argument.
Score Value: 510

00:49 GMAT CAT – Section 4: Verbal 10 of 41

A consumer’s advocacy group recently reported the results of a


study concerning the deregulation of the airline industry. Deregulation
has allowed the airlines greater flexibility in setting prices and
selecting routes, and has allowed for more mergers between airlines.
The report showed that prices have dropped on most flights without
a decrease in flight availability or safety. At the same time airline
profits are higher than ever.

If the results of the study are correct, which of the following


can be properly concluded about the effects of airline
deregulation?

(A) The airlines benefited from deregulation more than did the
consumers.
(B) Some airlines will soon go out of business because of
decreased revenue from lower ticket prices.
(C) The government will deregulate more industries following
the success of airline deregulation.
(D) Deregulation of airlines was financially successful both for
the airlines and for the consumers.
(E) Further decreases in ticket prices will start to erode the
Be wary of unjustified,
illogical, or incorrect compari- profits gained by airlines through deregulation.
sons in the answer choices
and choices that make
predictions about the future.
Test Section Answer
Time Help Confirm Next
Time
Quit Exit

Paraphrase the premises: Deregulation in the airline industry was


beneficial to both airlines and consumers, though we cannot say
which group benefited more.

346 © The MBA Center


Critical Reasoning

B) ASSUMPTION QUESTIONS
Understanding assumptions is crucial to doing well on Critical
Reasoning. Assumptions are unstated premises that are
necessarily true for the argument to work. Questionable
assumptions tend to make arguments weak, so finding the flaw in
an argument is often the same thing as finding a questionable
assumption.
Score Value: 650

00:47 GMAT CAT – Section 4: Verbal 11 of 41


Shortly before the elections in City C, many eligible voters polled
stated they objected to the mayor's plans for the police force. Yet the
mayor was elected by a 60 percent majority. So, at least 60 percent
of the voters supported the mayor's plans for the police force.

The conclusion of the argument above depends on which of


the following assumptions?

! (A) People who object to a politician's position will not vote for
that politician.
! (B) Some of the people in the poll have some involvement with
the police force.
! (C) Everyone who voted for the mayor did so primarily because
they didn’t agree with his plans for the police force.
! (D) Sixty percent of the voters elected the mayor because of his
plans for the health care system.
It is especially useful on
! (E) Most of the people polled did not vote in the election.
assumption questions to try
to predict the answer before

going to the answer choices.


Look for something in the
conclusion that goes further--
Test Section Answer stretches logic a bit more--
Quit Exit Time
Time
Help Confirm Next
than the evidence justifies.

Premise 1: Before the election many eligible voters stated they


objected to the mayor’s plans for the police force.
Premise 2: The mayor was elected by a sixty percent majority.
Conclusion: Sixty percent of voters supported the mayor’s plans for the
police force.

© The MBA Center 347


Total PrepKit for the GMAT® Step 11.1 Lesson

C) STRENGTHEN THE ARGUMENT


Strengthen-the-argument questions accept as true the built-in,
and more than likely flawed, assumption in the argument. For
this question type, look for an answer choice that fills a gap in
logic or provides further evidence to support the validity of the
conclusion.
Score Value: 590

00:45 GMAT CAT – Section 4: Verbal 12 of 41


Students at a certain university must take the equivalent of two
semesters of foreign language which can be done in a one-semester
intensive class or two semesters paced normally. An academic
investigative committee found that the students who take the
intensive class tend to achieve a higher level of proficiency than do
those in the two-semester class and therefore recommended that the
university eliminate the two-semester class.

Instituting the recommendation of the investigative committee


would be most likely to result in improved foreign language
proficiency of its students if which of the following were true?

! (A) The textbooks used in the intensive courses are not the
same as those used in the two-semester courses.
! (B) Students who chose the intensive courses generally did so
because they had some prior knowledge of the foreign
language.
! (C) There are no significant differences in the quality of students
taking the intensive courses versus those taking the two-
semester courses.
! (D) An intensive course in a foreign language makes it more
difficult for a student to learn vocabulary and practice
speaking.
Necessary vs. Sufficient ! (E) Students who take intensive mathematics courses tend to
The best answer to an get better grades than do those who take regular-paced
assumption question doesn’t mathematics courses. ➩
have to be sufficient to justify
the conclusion. It only has to
be necessary for the Test Section Answer
conclusion to work. Quit Exit Time
Time
Help Confirm Next

Premise 1: Students can take a 1-semester intensive or 2-semester


regular foreign language class.
Premise 2: Students in the 1-semester intensive class learn more
than do students in the 2-semester class.
Conclusion: The university should eliminate the 2-semester class (to
improve the foreign language proficiency of its students).

348 © The MBA Center


Critical Reasoning

D) WEAKEN THE ARGUMENT


Weaken-the-argument questions are by far the most common
question type in Critical Reasoning. For this type of question you
must invalidate – find evidence that does not support – the
conclusion. To ace this type of question, you must identify the
assumption in the argument, the weak spot, and find an answer
choice that does not restate, strengthen, or distort it.
Score Value: 600

00:43 GMAT CAT – Section 4: Verbal 13 of 41

In 1991, produce growers began using a new, inexpensive


pesticide, provoking many objections that they would damage both
the environment and the produce they were growing. However, the
fears have proven unfounded, as, through 1996, produce prices had
dropped and no ill effects had been reported.

Which of the following, if true, would be the strongest


objection to the argument above?

(A) Consumption of the produce declined from 1991 to 1993, but


rose sharply from 1994 to 1996.
(B) Several areas in which use of the pesticide was forbidden
have also experienced a drop in produce prices. Weaken-the-argument ques-
(C) The amount of produce grown in 1991 was larger than that tions almost always have
of 1996. at least one answer choice
that strengthens the argu-
(D) The time since the beginning of the use of the pesticide has
ment. Likewise, strengthen-
been too short to allow some of the predicted effects to the-argument questions
occur. almost always have at least
(E) Since 1992, new pesticides have been developed that one answer choice that
scientists agree are relatively risk-free. weakens the argument. In this
case, (E) is a choice that
comes close to strengthening
the argument (it doesn't
exactly strengthen it, but it
Test Section Answer supports the general idea that
Quit Exit Time
Time
Help Confirm Next pesticides can be risk-free).

Premise 1: Growers began using cheap, new pesticide in 1991.


Premise 2: Produce prices dropped.
Premise 3: No ill effects were reported (through 1996).
Conclusion: Fears that the produce and the environment would be
damaged are unfounded.

© The MBA Center 349


Total PrepKit for the GMAT® Step 11.1 Lesson

E) INFERENCE QUESTIONS
Inference questions ask you to infer something from the passage
other than an assumption or the conclusion. The correct answer
will contain an inference that can be logically and safely drawn
from the original argument. The best answer paraphrases words
and ideas from the text and contains an inference just one step in
logic away from the message of the text.

Score Value: 650

00:41 GMAT CAT – Section 4: Verbal 14 of 41


Being in debt does not necessarily restrict one's spending power.
If it did, people in debt would be unable to make large purchases and
would spend less money than people not in debt. In fact, when the
spending habits of people in debt are compared with those of people
who are not in debt, no such pattern emerges.

If the statements above are all true, which of the following


can be inferred?

! (A) Merchants make more money selling to people in debt than


they do to people not in debt.
! (B) People who are in debt do not necessarily stay that way for
prolonged periods of time.
! (C) If a graph were made of people's debts and their spending
habits, those who are most in debt would be demonstrated
as making the largest purchases.
! (D) Reliable comparisons between those in debt and those not
in debt are not available.
! (E) Going into debt often does not deter people from going into
greater debt.


Conclusion ≠ inference. An
inference is always true
whereas a conclusion is not
Test Section Answer
necessarily true.
Time Help Confirm Next
Time
Quit Exit

Note that this argument does not claim that people in debt spend more
money or make larger purchases than those not in debt. It claims only that
they don't spend less. Phineas will try to trick you into making hasty
assumptions or distortions. Can you identify which choices do this?

Premise 1: If debts restricted spending powers, people in debt


wouldn’t be able to make large purchases.
Premise 2: People in debt do make large purchases.
Conclusion: Debts don’t necessarily restrict spending power.

350 © The MBA Center


Critical Reasoning

F) EXPLAIN THE (APPARENT) CONTRADICTION


For this question type, you will be given an argument with two
statements that are apparently contradictory. Often the contra-
diction will be in the form of two trends that seem to oppose each
other. In order to ace this type of question, clearly identify the
apparent paradox. The best answer is the one that explains –
provides a logical link for – how the two apparently paradoxical
positions can be reconciled or both be true.
Score Value: 490

00:39 GMAT CAT – Section 4: Verbal 15 of 41 Try to predict a possible


answer before looking at the
choices.

Studies show that more and more Americans are removing meat
from their diets. Yet sales of meat increased in the United States last
year.

Which of the following, if true, could best explain the


simultaneous decrease in the number of Americans who eat
meat and increase in sales of meat?

(A) Last year, Americans imported more meat than they had in
the previous year.
(B) Those Americans who ate meat consumed more per person
than they had the previous year.
(C) The number of Americans who had stopped eating meat
during the previous year was smaller than the number of
Americans who started eating meat last year.
(D) The global meat industry grew in the previous year.
(E) The studies all used large samples that were representative If you find the answer you
of the general population. predicted, quickly scan the
rest of the choices to make
sure there is not something
better (this is not necessary
on math sections, but it is
useful for all the verbal
sections). All the answer
Test Section Answer
choices are in competition for
Time Help Confirm Next your attention.
Time
Quit Exit

Premise: More Americans are removing meat from their diets


(according to studies).
Premise: Sales of meat increased in the United States.
Conclusion (implied): There is a contradiction.

© The MBA Center 351


Total PrepKit for the GMAT® Step 11.1 Lesson

G) COMPLETE THE PASSAGE


Some questions ask you to complete a passage. For these types, the
question comes before the argument, and the end of the argument
is usually broken off in mid-sentence. The answer choice that best
completes the passage might provide a missing premise, state the
conclusion, or connect the premises to the conclusion.
Score Value: 680

00:37 GMAT CAT – Section 4: Verbal 16 of 41

Which of the following best completes the passage below?

At a recent meeting of a company's top management, most of the


managers present argued that since the black-and-white television
market as a whole was rapidly shrinking, the company should reduce
production levels for all models of black-and-white televisions. What
must be shown, of course, in order for the company not to lose profits
from hasty decision making, is that __________________________

(A) the market for color televisions is growing at a rate faster


than the market for black-and-white televisions is shrinking.
(B) none of the black-and-white televisions the company
currently sells match the technological standards of modern
color televisions.
(C) computer screens will eventually be capable of performing
all functions normally performed by televisions.
(D) none of the models of black-and-white televisions
manufactured by the company are popular enough to
sustain high profits at current production levels.
(E) the company has the engineers and production equipment
Don't make associations based necessary to mass produce state-of-the-art color televisions.
on your own knowledge that
are not made in the passage.
Color televisions and
computers have nothing to do
Test Section Answer
with this passage.
Time Help Confirm Next
Time
Quit Exit

Premise: The black-and-white television market is shrinking.


Conclusion (of the company management): We should reduce
production levels of all models of black-and-white
televisions.

352 © The MBA Center


Critical Reasoning

H) "EXCEPT" QUESTIONS.

An "Except" question differs from a strengthen/weaken question


in that all the wrong answer choices satisfy the question's
statement; only the correct answer choice does not. In other
words, if a question says “all the following strengthen the
conclusion EXCEPT,” then the correct answer choice simply fails
to strengthen rather than actively weaken. These questions can
also be phrased using “NOT” or “LEAST”. The best approach to an
EXCEPT question is Process of Error Identification.
Score Value: 550

00:35 GMAT CAT – Section 4: Verbal 17 of 41


Books put on prominent display in bookstores, either in the store
windows or near the cashiers, generally outsell the books on other
shelves. A publishing company, disappointed by the poor sales of a
new novel by a famous author, complained to a leading chain of
bookstores that the novel would have been a bestseller had it been
prominently displayed.
.
All of the following, if true, would support the publisher's
claim EXCEPT:

! (A) Many books which do not sell well on regular shelves


become popular after they are put on prominent display.
! (B) People who shop in bookstores often do not have specific
books in mind when they enter the store but buy books I’ll figure it out later
whose cover catches their eye. For strengthen- or weaken-
! (C) Managers of bookstores only put books on prominent the-argument questions
display that are already bestsellers. which have “excepts” in
! (D) Three of the author's recent books were bestsellers. them, mark a little “w” next to
! (E) The new novel received favorable reviews in newspapers each choice that weakens, an
“s” next to the choices that
and on radio upon its release.
strengthen, and an “n” next
to the choices that are neutral

or have no effect. This way


you don’t have to keep in
mind exactly what you are
Test Section Answer
looking for until you have
Time Help Confirm Next evaluated each choice.
Time
Quit Exit

Premise: Books on prominent display outsell other books.


Premise: A novel by a famous author was not put on display.
Premise: The novel sold at levels disappointing to its publishing
company.
Conclusion (of the publishing company): The novel would have been a
bestseller had it been put on prominent display.

© The MBA Center 353


Total PrepKit for the GMAT® Step 11.1 Lesson

SUMMARY
-There are about 14 Critical Reasoning items on the GMAT CAT.

- Each Critical Reasoning item consists of a short, usually flawed,


argument, a question about the argument, and five answer choices. The
section requires no outside knowledge. Each question can be answered
solely on the basis of the information in the argument.

- Each item is made of :

- An argument
- A premise or premises
- A conclusion
- An inference (not always)
- An assumption
- A question
- Five answer choices

- A premise is a piece of evidence, a conclusion is the main idea of the


argument and an interpretation of the premises, and an inference is an
unstated idea that follows logically from the reasoning of the premises.
Most arguments contain an assumption that makes them logically
unsound.

- The wrong answer choices are not written randomly. Therefore the
Process of Error Identification is the best way to find the “best answer.”

- Use the MBA Center Five-Step Approach

- Step 1: Read the question.


- Step 2: Read the argument.
- Step 3: Paraphrase the main idea using your own words.
- Step 4: Predict the possible answer.
- Step 5: Use Process of Error Identification to eliminate
wrong answer choices.

- There are 8 question types. A specific technique is required to ace each


one of them.

* Draw a conclusion * Inference

* Assumption * Explain the (apparent) contradiction

* Strengthen the argument * Complete the passage

* Weaken the argument * EXCEPT questions

354 © The MBA Center


Critical Reasoning

CRITICAL REASONING LESSON


ANSWER KEY
1. B
2-8. N/A
9. D
10. D
11. A
12. C
13. D
14. E
15. B
16. D
17. C

© The MBA Center 355


© The MBA Center
Critical Reasoning

HOMEWORK/PRACTICE TEST
CRITICAL REASONING
ANSWER GRID

A B C D E
1 ! ! ! ! !
2 ! ! ! ! !
3 ! ! ! ! !
4 ! ! ! ! !
5 ! ! ! ! !
6 ! ! ! ! !
7 ! ! ! ! !
8 ! ! ! ! !
9 ! ! ! ! !
10 ! ! ! ! !
11 ! ! ! ! !
12 ! ! ! ! !
13 ! ! ! ! !
14 ! ! ! ! !
15 ! ! ! ! !
16 ! ! ! ! !

© The MBA Center 357


Total PrepKit for the GMAT® Step 11.2 Homework

GMAT CAT — Section 4: Verbal Directions

End


When finished
TIME — 25 MINUTES reading
directions
16 QUESTIONS click on the
icon below
Directions: In this section, each
question follows a brief passage. Select
the best answer of the choices given.
Dismiss
Directions


Test Section Answer
Time Help Confirm Next
Time
Quit Exit

358 © The MBA Center


Critical Reasoning

1. A very popular, though relatively recent, theory 3. A beverage company recently hired a celebrity to
postulates that the extinction of the dinosaurs resulted introduce a new soft drink in radio and television
from climate changes caused by three large meteors advertisements. Sales of the new soft drink have exceeded
entering the Earth’s atmosphere near the end of the all expectations, and the beverage company has
Mesozoic Age. therefore decided to hire celebrities to endorse all its older,
well-known beverages. The company should therefore
Which of the following statements, if true, could be expect to see greatly increased profits in the future.
used in an argument against the theory?
The passage, in arguing that the company’s profits
(A) The dinosaurs were cold-blooded, meaning will rise greatly, assumes all of the following EXCEPT:
their body temperature was regulated by
external temperature. (A) The costs involved in the media campaigns will
(B) Archaeologists have located several large be significantly less than the increase in sales
craters, probably caused by falling meteors, that due to celebrity endorsements.
they have estimated were formed during the (B) Most of the customers buying the new soft
Mesozoic Age. drink did not stop buying other brands from the
(C) Comparisons between fossils found in different same company to do so.
areas of the planet indicate that the dinosaurs all (C) Celebrity endorsements will be more effective
became extinct within a short time frame. than the advertising methods currently
(D) Scientists have estimated that numerous species employed for the company’s better known
of flora intrinsic to the biosystem in which the beverages.
dinosaurs participated became extinct shortly (D) The sales of a well-known product can be
before the time at which the meteors are increased by a celebrity’s endorsement.
supposed to have struck the Earth. (E) All the older beverages sold by the company will
(E) In existence throughout the Mesozoic age, and be at least as popular as the new beverage once
long beyond it, were prehistoric mammals, who they are endorsed by celebrities.
were warm-blooded, meaning their body
temperature remained constant regardless of
external factors.

2. Company K claims that although the wholesale


price of its new product B is less than its manufacturing
cost per unit, the price is appropriate because people who
buy product B will also buy company K's expensive and
highly profitable product A. This could be most clearly
demonstrated by _________ .

Which of the following best completes the passage?

(A) showing that none of company K's competitors


manufacture products similar to product B
(B) the fact that sales of product A have been
steadily rising for the last four years
(C) examining the likelihood of customers who
already buy product A to buy product B
(D) by comparing the sales of product A in markets
where product B is available and markets where
product B is not available
(E) measuring the market share of product A
relative to its production costs in recent years.

GO ON TO THE NEXT PAGE


© The MBA Center 359
Total PrepKit for the GMAT® Step 11.2 Homework

4. An athlete, in order to excel in a sport, must have Questions 6-7 refer to the following:
a natural ability for that sport or spend a great deal of
time practicing. Thus if you are a college student and Senator #1: By taking people off welfare you are
your first priority is getting good grades, you should not cutting off their only means to support themselves.
do a sport unless you have a natural ability for that sport.
Senator #2: By taking people off welfare, we are both
The argument above assumes all of the following providing them with incentive to get jobs and change
EXCEPT: their lives, and saving money for the federal government,
enabling us to cut the deficit and improve the economy.
(A) College students should not do sports unless This will reduce poverty overall.
they want to excel in their sport.
(B) Time spent practicing a sport limits a college
student’s ability to get good grades.
(C) A college student cannot succeed in sports and 6. Which of the following, if true, best supports the
get good grades. second senator’s argument?
(D) Only college students who have a natural ability
for a sport can get good grades and do well in (A) There is a high unemployment rate.
their sport. (B) The majority of families on welfare include
(E) College students who spend a lot of time families with children too young to work.
practicing a sport will not get good grades. (C) Most recipients only receive welfare for brief
periods while they are between jobs.
(D) Welfare amounts to a relatively small per-
centage of the federal budget.
5. In an effort to combat sinking reading scores in its (E) Senator #2's plan is modeled on the welfare
public schools, Harville decided to fund the hiring of after- reduction plans of certain states that suc-
school tutors. However, in order to do this and stay cessfully reduced poverty in those states.
within its budget, the town will not be able to begin a
long-needed renovation of its park system, which the
students use for their physical education classes and
after-school sports leagues. 7. Which result of a serious reduction in welfare
expenditures would best indicate that the second
Which of the following best expresses the senator's reasoning was correct?
conclusion of the passage?
(A) A sudden lowering of the unemployment rate
(A) Because of budgetary constraints, Harville will (B) The initiation of programs by the federal
have to find a means of raising reading scores in government spurring businesses to hire former
its public schools that is less expensive than the welfare recipients
hiring of after school tutors. (C) A rise in the average salary for low-level,
(B) The quality of physical education in Harville will unskilled workers
decline as a result of the new after-school (D) A reduction in the number of public high school
tutoring system. students earning their diplomas
(C) Harville considers good reading scores in its (E) A sharp rise in inflation
public schools a greater priority than the
success of its school sports teams.
(D) Harville will not be able to fund the hiring of
after-school tutors and the renovation of its park
system and keep within its budget.
(E) Some students who used to participate in after-
school sports leagues in Harville will now take
part in after-school tutoring.

GO ON TO THE NEXT PAGE


360 © The MBA Center
Critical Reasoning

8. Excavations on the Island of Crete have revealed a (E) Most airline passengers, unless informed in
palace, built in a labyrinthine style, dating back almost advance of what do to in case of an emergency,
3,000 years. Many point to this as evidence that there is are likely to panic should there be an actual
an element of truth behind the Greek myth in which a emergency.
warrior from Athens named Theseus goes to Crete and
wanders a labyrinth until he kills the half-man, half-bull
called the Minotaur.
10. In a recent survey, Travel Company X found that
Which of the following, if true, most strongly 30% of its customers were not satisfied with the quality
supports the theory that there is an element of truth of service. Company X has lost many of its customers to
behind the myth? Company Y recently and has therefore decided to
increase its number of service representatives by 25
(A) Archaeologists have uncovered ruins from percent in an effort to regain customers lost to Company
Athens that date back to roughly the same Y.
period.
(B) The Minoan civilization, centered on Crete, is Which of the following, if it were discovered, would
universally agreed to have been extremely make Company X’s plan most likely to fail to regain
advanced; excavations of the palace dem- customers lost to Company Y?
onstrate that it may have had running water.
(C) Archaeologists theorize that the palace was (A) The poor quality of service at Company X is due
leveled by a natural disaster around 3,400 years to the fact that its customer service repre-
ago. sentatives are poorly trained and lack
(D) Documents indicate that people from kingdoms experience.
outside Crete, including Athens, often had (B) Company X has always charged less for the
reason to go to Crete, which was the center of same services than has Company Y.
power in the region at that time. (C) Not all of Company X’s customers responded to
(E) In most versions of the myth, Theseus is said to the survey.
have used a ball of thread to find his way (D) Company X has many loyal customers who are
through the labyrinth. satisfied with its services.
(E) Company X does most of its sales in package
deals, including airfare and hotels, while
Company Y does mostly “airfare only” sales.
9. Most airline passengers don’t pay attention when
flight attendants give safety instructions before takeoff.
Unless airlines find a more effective means of
communicating to their passengers, most passengers will
not know what to do in case of an onboard emergency.

Which of the following, if true, would most seriously


weaken the conclusion drawn above?

(A) Onboard emergencies are extremely rare, and


because of technological improvements are
likely to become even more rare.
(B) Most of the passengers who do not pay
attention are already familiar with airline safety
procedures, which are standardized on all
flights.
(C) In presenting an airplane’s safety features to
their passengers, flight attendants must follow a
script written by high-level flight engineers.
(D) In addition to the safety instructions given by
flight attendants, a card with all necessary safety
instructions is provided for each passenger.

GO ON TO THE NEXT PAGE


© The MBA Center 361
Total PrepKit for the GMAT® Step 11.2 Homework

11. From 1950 to 1975, the average number of (D) Lawn mower users generally find that they have
students graduating per year at a group of five the fewest accidents using older models all of
associated universities increased by 40 percent. During whose safety features they are already familiar
the same period, the number of students graduating with.
from these universities with honors degrees grew by only (E) A recent study on lawn mowers published in a
20 percent. consumers’ guide magazine reported that
many accidents caused by lawn mowers are
Which of the following statements about the period due to faulty safety equipment.
from 1950 to 1975 in these five universities is best
supported by the statements above?

(A) The criteria for selecting honors students 13. The introduction of plant or animal species to a
became more rigorous. region where the species do not exist naturally
(B) The faculty-to-student ratio declined, allowing sometimes has disastrous effects on the environment of
less time for advisors to encourage students to the region. Since countries X and Y are planning to
pursue honors degrees. eliminate all border controls for visitors traveling between
(C) The increase in enrollment resulted from lower the two countries, ecologists are worried that the
admissions standards. environments of the two countries will be at great risk.
(D) Students at the end of the period were more
career-oriented than those at the beginning. Which of the following points, if true, provides the
(E) The number of students graduating without strongest evidence that the ecologists’ fear concerning
honors increased by more than 40 percent. the environments of countries X and Y is unfounded?

(A) Both country X and country Y will continue to


enforce their border controls for visitors whose
12. A company that manufactures lawn mowers country of origin is neither of the two countries.
developed a new model for a lawn mower with safety (B) Country X and country Y are part of a large
features lacking in its earlier models still on the market. desert region whose plant and animal species
During the first year that the new model was sold, its exist throughout the entire desert.
sales lagged far behind the company’s older models. The (C) The elimination of border controls between
company management thus concluded that safety was countries X and Y will benefit the economies of
not the primary concern of its customers. both countries by increasing trade and
promoting transnational businesses.
Which of the following, if true, would most seriously (D) Government efforts to protect the environment
weaken the conclusion of the company management in country X in the last decade have successfully
in the passage above? resulted in the removal of a number of plant
and animal species from the endangered
(A) Other companies that manufacture lawn species list.
mowers developed similar safety features for (E) The border controls between the two countries
models that then sold very well. will be phased out gradually over a five-year
(B) The safety features included in the new model period.
did not result in a retail price higher than those
of the company’s earlier models.
(C) Many of the company’s lawn mowers are
bought by the municipal parks commission and
are used by professional gardeners to maintain
the city's park system.

GO ON TO THE NEXT PAGE


362 © The MBA Center
Critical Reasoning

14. A sporting goods company found that the water- (D) The less heavily-used ports are as convenient as
proof material it uses for manufacturing backpacks, ski ports that are centers of international trade for
bags, and tents has consistently resulted in low sales most private boat operators
when used to manufacture rain jackets. Tests, however, (E) Commercial boats are more likely to be involved
have revealed that the material is not only waterproof, in accidents than private boats.
but also lighter and less expensive than the material used
in more popular rain jackets.

Which of the following statements, if true, would 16. Television news programs often announce the
best reconcile the test results above with the poor sales results of medical studies indicating that a certain type of
of the rain jackets? food has been shown to be beneficial or harmful. Because
the news programs generally focus on one finding
(A) The company sells most of its products to out- relating to only one type of food, it is impossible for
of-state residents through mail-order catalogs. viewers not in the health profession to develop a coherent
(B) The company’s sales of backpacks and ski bags nutritional plan.
have increased by an average annual rate of 15
percent over the last three years. If all of the above statements are true, which of the
(C) People who buy rain jackets most often choose following can be properly inferred?
a jacket based on its fashion appeal.
(D) Some mountain climbers and hikers prefer (A) Nutritional experts focus their research primarily
material that is even lighter than that used by on the effects of a small number of foods.
the company in its rain jackets. (B) Television journalists are unable to relate specific
(E) The company has traditionally been more new medical findings to broad health issues.
successful in selling footwear than rain jackets. (C) Viewers of television news programs rely on the
programs for nutritional information.
(D) Information relating to only one type of food is
not useful to people who are not in the health
15. If the harbors around ports that are centers of profession.
international trade were restricted to commercial boats (E) Television news programs often distort scientific
and only those private boats equipped with advanced findings reported by nutritionists.
communication devices, most private boats would be
forced to use other ports that are less heavily used. Such
a reduction in the amount of private boat traffic would
lower the chances of collisions around the ports which are
centers of international trade.

The conclusion drawn in the first sentence is


dependent on which of the following assumptions?

(A) Most private boats that use ports that are


centers of international trade are not equipped
with advanced communication devices.
(B) Most of the less heavily used ports are unable to
handle big, commercial boats.
(C) The decline in the number of private boats used
at ports that are centers of international trade
would eventually result in a rise in the commer-
cial boat traffic at these ports.

STOP
IF YOU FINISH BEFORE TIME IS EXPIRED YOU MAY
CHECK YOUR WORK ON THIS SECTION ONLY
© The MBA Center 363
Total PrepKit for the GMAT® Step 11.2 Homework

CRITICAL REASONING HOMEWORK/PRACTICE TEST


ANSWER KEY

1. D
2. D
3. E
4. C
5. D
6. E
7. A
8. D
9. B
10. A
11. E
12. D
13. B
14. C
15. A
16. C

364 © The MBA Center


Critical Reasoning

EXPLANATIONS FOR 3. (E)


CRITICAL REASONING Premise: A celebrity introduced a new beverage product
HOMEWORK/PRACTICE TEST in advertisements.
Premise: Sales of the new product exceeded all expec-
tations.
Premise: Celebrities will endorse all of the company's
1. (D) beverages.
Premise: 3 large meteors entered the earth's atmos- Conclusion: The company's profits will increase greatly.
phere near the end of the Mesozoic era.
Premise: The meteors caused climate changes. This is an argument by analogy: what worked with
Conclusion: These climate changes caused the extinc- one product will supposedly work with others. The
tion of the dinosaurs. question asks for what is not assumed in the
argument, so four of the answer choices will be
The conclusion asserts a causal relationship. We assumptions that were made that, if true, strengthen
have to assume, first of all, that the dinosaurs became the argument that the two situations are analogous.
extinct at the end of the Mesozoic. Even then, though, The wrong choices will indicate that, indeed, the
the temporal relationship is not the same as a causal celebrity endorsements will be profitable. (E) is not
one. assumed, because it is not necessary that all the older
This is a weaken-the-argument question. It is beverages sell as well as the new one. The conclusion
necessary to find the answer choice which indicates that the company's profits will rise can be true even if
that climate changes from meteors did not cause the only some of the older beverages sell much better. The
extinction of the dinosaurs. (A) is irrelevant to the other choices are assumptions necessary to the
argument as it is stated; whether it is tangentially argument; if any of them were false, the argument
related to theories of the extinction of dinosaurs is would break down.
irrelevant here. (B) and (C) strengthen the argument.
(D) weakens the argument because it suggests an
alternate explanation for the extinction of the
dinosaurs – the flora which they needed became 4. (C)
extinct before the meteors landed. Providing Premise: To excel in a sport, you must have a natural
alternative explanations is a good way to weaken a ability or spend a lot of time practicing.
causal argument. The prehistoric mammals of (E) are Conclusion: College students who want good grades
outside the scope of the argument. should only do a sport if they have a natural
ability.

This question necessitates finding what is not


2. (D) assumed in the argument. (C) is not assumed,
Premise: The wholesale price of product B is less than its because, according to the argument, it is possible for
manufacturing cost per unit. a student to succeed in sports and get good grades (if
Premise: People who buy B will also buy the highly the student has a natural ability for the sport and
profitable A. doesn't need to spend much time practicing). The
Conclusion: Therefore product B is appropriately priced. other choices are assumptions that must be true for
the argument to work. By the way, if you felt that (E)
The wholesale price of new product B is less than is untrue because you think a student can spend time
the manufacturing cost. What is supposed to keep B practicing a sport and get good grades, you are basing
from losing money for Company K is that people who your answer on your own knowledge and not on the
buy product B, according to the premise, will also buy passage. Stick to the premises and the logic; stay
the profitable product A. B, then, must lead to in- within the scope of the argument.
creased sales of A. The best way to verify this is to
investigate whether sales of A increase in markets
where B is available, which, if demonstrated to be true,
would correlate purchases of the 2 products. (A) is
wrong because K's competitors are outside the scope
of the argument. So is the history of sales of A referred
to in (B). (C) is close, but it reverses the relationship –
it is necessary to find the likelihood of people who pur-
chase A to buy B, not the other way around. (E)
compares 2 details about product A that are not rele-
vant to the argument. (D) explicitly states the proper
method of investigating the potential profitability of B.

© The MBA Center 365


Total PrepKit for the GMAT® Step 11.2 Homework

5. (D) that at least part of his plan was working. (B) is a trap;
Conclusion: in order to fund after-school tutors and stay it can be interpreted as a positive result of cutting back
within its budget, Harville will not be able to on welfare, but it is not a sign that the senator's
begin a long-needed renovation of its park reasoning is correct. Unskilled workers and their
system. salaries, high school diplomas, and inflation,
Premises: everything else mentioned in (C), (D) and (E), respectively, are all
outside the scope.
Here you are looking for the conclusion. Remember
that "find the conclusion" questions are not necessarily
based on flawed arguments. Don’t waste your time 8. (D)
looking for flaws or assumptions. Premise: A labyrinthine palace from 3000 years ago was
(A) contradicts the premises – we expect that found on Crete
Harville will fund the tutors. (B) is a trap – it sounds Conclusion: There is some truth to the myth of the
good, but isn't necessarily true. Maybe Harville will Athenian Theseus going to Crete and killing the
find an inexpensive way to continue its physical Minotaur after wandering the labyrinth.
educational classes and maintain the quality. (C) is the
same – tempting, but goes too far. We don't know The conclusion asserts a link between the myth and
what the priorities of the town are, and, what's more, the discovered palace. All we really know, though, is
we don't know that the success of the sports teams is that the myth exists and the palace exists. We don't
related to renovating the parks. (E) is wrong because know whether they're related, i.e., whether the palace
there is no connection between the tutoring and found was actually the palace used by Theseus, or
sports teams in terms of participants. (D) is a good, even whether Theseus actually existed and went to
clean paraphrase of the conclusion. Crete. Notice the conclusion says there is "some" truth
behind the myth. We only need a choice that
strengthens the relationship between the conclusion
and the premise. (D) does this by pointing out that it
6 – 7. was common for Athenians to travel to Crete, thereby
Premise: Taking people off welfare provides incentive to supporting the possibility that someone went to Crete
find jobs and change their lives. at this time from Athens, perhaps finding his way
Premise: The federal government will save money through the labyrinth in the palace. None of the other
Premise: The deficit will be cut and the economy choices specifically relate to the myth.
improved.
Conclusion: Poverty will be reduced overall.

9. (B)
6. (E) Premise: Most airline passengers don't pay attention
Remember to read the question first. It tells you to during safety instructions.
focus on senator #2's argument, a "Does the proposed Conclusion: Most passengers will not know what to do
solution solve the problem?" argument. The premises in case of an emergency unless airlines find a
are really a series of predictions, and the conclusion is better way to communicate safety instructions.
just a broader prediction. Something could go wrong
in any of the steps, but keep in mind that the overall This argument suggests a solution (better means of
point of the plan is to reduce poverty. communication) for a supposed problem (most airline
In (A), a high unemployment rate doesn't show passengers don't pay attention during safety
that poverty will be reduced. In (B), the concept of instructions). The premise, though, says nothing
children too young to work is outside the argument's about emergencies. Maybe there's no problem at all
scope. (C) and (D) weakens senator #2's argument. and people already know what to do in case of an
(E), the right answer, shows that the plan is based on emergency. (B) asserts this and therefore weakens the
similar plans that have worked, therefore adding argument. (A) does not weaken the argument,
credibility to the plan. because the argument concerns what would happen if
there were an emergency; the likelihood of there
actually being an emergency is irrelevant. (C) is
7. (A) outside the scope; the people who write those
Again, we're trying to show that senator #2's annoying speeches are not part of the argument. (D)
proposed plan will solve the problem. We can do this may sound good, but it's definitely not as good as (B).
by showing that any one his predictions is valid. One The argument says a more effective means of
of the premises was that people would have more communication is necessary. The cards may not be
incentive to find jobs. Therefore a sudden lowering of effective at all (who reads them anyway)? (E)
the unemployment rate, or (A), would demonstrate strengthens instead of weakens the argument.

366 © The MBA Center


Critical Reasoning

10. (A) 12. (D)


Premise: 30% of Travel Company X's employees were Premise: A new lawn mower model with new safety
not happy with the service (according to a features was put on the market.
survey). Premise: In its first year of sales, the company's older
Premise: Company X has lost many of its customers to models sold far better than the new one.
Company Y recently. Conclusion (of the company management): Safety is
Premise: Company X will increase its number of service not the customers' main concern.
representatives.
Conclusion: Company X will regain customers from The conclusion establishes a causal relationship
Company Y. between the premises. The management tries to
explain why the new lawn mowers did not sell better.
Does the proposed solution solve the problem? To weaken a causal argument, the easiest thing to do
Look carefully at the premises. First of all there is a is to find another cause. (D) shows that even though
survey involved. That doesn't automatically invalidate the new lawn mowers have new safety features,
the data, but if the survey did not use a representative customers consider them less safe (because they are
sample, the conclusion would be invalid. The passage unfamiliar with them). This helps explain why the sales
doesn't state why customers were unhappy with the are lower than their old models and weakens the
service provided. Maybe there were not enough conclusion that safety is not the customers’ primary
service representatives (in which case the plan might concern.
help); maybe there were already enough but they
were incompetent, or ineffectual. (A) therefore is the
right answer. Notice that (C) is a trap. A survey doesn't
have to be filled out by everyone to be valid. 13. (B)
Only choose answers about samples to weaken Premise: New plant or animal species can have bad
arguments when the choice argues that the sample effects on a region's environment.
was not representative. Premise: Countries X and Y are planning to eliminate
border controls between the countries.
Conclusion (of ecologists): The environments of the two
countries are at great risk.
11. (E)
Premise: The number of students graduating from five The question asks which choice will weaken the
associated universities increased 40% from ecologists' "fear," or conclusion. The correct choice will
1950 to 1975. demonstrate that the environments of X and Y are not
Premise: The number of students graduating from these at risk. The threat to environments, according to the
universities with honors degrees grew by 20%. passage, is from new species. The argument assumes
Conclusion: none that species from X that don't exist in Y will be taken
across the border (or vice versa). (B) proves the
This question could be viewed as a resolve-the- assumption wrong, saying that the two countries have
contradiction question, in which two statistics need to the same environment and the same species; therefore
be resolved. The (apparent) contradiction is that the there is no threat and the ecologists' fear is unfounded.
number of students graduating overall increased more (A) is outside the scope because the argument
than the number of students graduating with honors doesn't concern countries other than X and Y. (C) is
degrees. The inference is pretty straightforward; the outside the scope because, though the increased trade
number of students graduating without honors might be good for the countries, the ecologists are
degrees must have increased by more than 40%. (E), worried about the environment, not the economy.
then, must be correct. (A) is a possible explanation for Ecologists might be happy to hear (D), but it does
the first premise, but it doesn't have to be true; the nothing to calm fears about the introduction of new
criteria could have stayed the same (or become even species. (E) perhaps postpones the fear for a while, but
less rigorous) while the students got worse. (B), in fact, it doesn't actually solve the problem.
is another possible explanation for the first premise,
but it also is not necessarily true. (C) is an explanation
for the second premise, but (C) doesn't have to be
true; the increase in enrollment (assuming enrollment
– and not just the number of students graduating –
increased) could be due to an increase in the number
of applications. (D) is outside the scope – we cannot
establish a connection between honors degrees (or the
lack of honors degrees) and career orientation.

© The MBA Center 367


Total PrepKit for the GMAT® Step 11.2 Homework

14. (C) (B) is irrelevant, because big commercial boats can


Premise: A company manufactures backpacks, ski bags, continue to use the busy ports. (C) is a prediction
and tents with a certain waterproof material. about the future that is not justified by the premises.
Premise: When the company uses this material for rain (D) doesn't have to be true; the passage does not state
jackets, the jackets sell poorly. the policy will be convenient for private boat
Premise: Tests show that the material is waterproof and operators. (E) refers only to the second sentence.
lighter and less expensive than the material in
more popular rain jackets.
Conclusion (implied): The premises seem to contradict
each other (the rain jackets should have sold 16. (C)
better). Premise: TV news programs announce the results of
medical studies about food.
This is a resolve-the-contradiction question. Why Premise: The programs usually focus on only one finding
did the rain jackets, whose material tested well, not about one food.
sell better? Look carefully. What did the company test? Conclusion: It is impossible for lay people to plan a good
It tested the fact that the material is waterproof, light, diet.
and inexpensive. What determines how well rain
jackets sell? There's no way to know! A choice which Impossible? What ever happened to people
explains that the popularity of rain jackets is reading, consulting doctors, learning from their
determined by something other than the material (or parents, listening to the radio? Since the question says
by an untested quality of the material) would solve the "If all of the above statements are true… ," it is not
problem. (C) does this by suggesting that it is a necessary to contradict the argument, even if you
jacket's fashion appeal, and not its material, that can personally disagree with the conclusion. The only way
determine its popularity. this conclusion would follow logically from the
(A) is outside the scope – in-state, out-of-state – premises is if people rely on TV news programs for
who cares? (B) focuses on backpacks and ski bags; it information to plan their diets. If not, it would simply
explains nothing about rain jackets. (D) is pretty close, be wrong to conclude that it's impossible to plan a
but it has two problems. First, the word "some" is good diet. (C), then, is a proper inference.
weak; "some" means one or more. Second, even if they (A) distorts the argument. TV programs focus on
prefer material lighter than that used by the company one type of food – the passage states nothing about
in its rain jackets, why should other rain jackets which the focus of nutritionists. (B) goes too far by claiming
are even heavier sell better? And as for (E), now really that the journalists are "unable" to do something – just
– "footwear"? That's way outside the scope. because they don't doesn't mean they can't. (D) also
goes too far. Even after accepting the fact that viewers
can't "develop a coherent nutritional plan," it should not
be assumed that information concerning only one type
15. (A) of food is useless. This choice would have been correct
Note that the question is only about the first had it said that the information is "of limited use." (E) is
sentence. You should therefore ignore the second a distortion. We know that the programs report about
sentence. one type of food item at a time. This doesn't mean
they "distort scientific findings." Watch out for choices
Premise: If harbors around ports that are centers of that might disparage professionals (even TV
international trade were restricted to journalists)!
commercial boats and only those private boats
equipped with advanced communication
devices…
Conclusion: then most private boats would be forced to
use other ports that are less heavily trafficked.

This argument goes from the particular to the


general. Why should most private boats be forced to
use other ports? The premise says that the busy ports
will be restricted to commercial boats and private
boats with advanced communication devices. Maybe
most private boats have advanced communication
devices. The conclusion is based on the assumption
that most private boats don't have these devices.
Therefore (A) is the answer.

368 © The MBA Center


Step 12
Critical Reasoning
Workshop

© The MBA Center


Total PrepKit for the GMAT® Step 12 Workshops

Critical Reasoning – Workshop Directions

End


When finished
TIME: 40 MINUTES reading
23 QUESTIONS directions
click on the
Directions: Choose the best response for icon below
each question in this section.

Dismiss
Directions


Test Section Answer
Time Help Confirm Next
Time
Quit Exit

370 © The MBA Center


Critical Reasoning

EASY on all bus lines, showing that some lines are


used by many more riders than others.
(E) The contract with the bus drivers union
1. Advanced Audio, a manufacturer of stereo stipulates that the city not lay off any drivers
components, is planning to expand its production because of reduced bus service.
in order to participate in the stereo sales boom
that has occurred over the last two years. The
company also plans to continue the heavy 3. A scientist planted two groups of plants under
marketing campaign that it commenced several identical conditions of light, temperature, humi-
years ago. dity, and moisture. Every day he would play sound
effects of thunderstorms to one of the groups of
Which of the following, if true, best weakens the plants and sounds of city traffic for the other. The
view that Advanced Audio can increase sales using the group to which he played thunderstorms all died
plan outlined above? within a few weeks, but the other group thrived
during the experiment. He therefore concluded that
(A) Advanced Audio's stereo components, with the sound of city traffic is more effective for helping
those of three other companies, accounted for plants grow than is the sound of thunderstorms.
over half of the industry's increase in sales
last year. Which of the following, if true, would most
(B) Advanced Audio manufactures stereo parts for seriously weaken the scientist's conclusion?
other companies in addition to those sold
under its own name. (A) The scientist put different varieties of plants in
(C) Although Advanced Audio lowered its retail each group.
price last year, its market share also fell. (B) The light affecting the plants changed
(D) Advanced Audio's products are among the according to the time of day.
most state-of-the-art in the industry, which (C) The plants in the group for which he played
makes them among the most expensive. city traffic sounds died several weeks after the
(E) Advanced Audio's products are sold more in experiment.
authorized dealerships than in discount stores. (D) The plants were all purchased at the same time.
(E) The plants in the group for which he played
city traffic sounds required more water than the
2. A city's public transportation board has decided to scientist actually gave them.
cut costs by reducing the frequency of its bus
service from an average of eight minutes to an
average of ten minutes between buses. The board 4. The town council recently voted to rename 11th
announced that it can do so without seriously Street "Shane O'Connor Street," after the famous
reducing the quality of service. poet and novelist who once lived there. Supporters
of the plan argued that the change would
Which of the following statements, if true, would promote community pride and togetherness.
most strengthen the validity of the board's
announcement? The council members' plan is based on the
assumption that
(A) The less frequent use of the buses will lower
maintenance costs, resulting in savings that can (A) Shane O'Connor's books are required reading
be used for much-needed repairs of the city's in the town's public schools
pedestrian bridges. (B) Most of the residents of 11th Street favor the
(B) At rush hour, congestion in the city slows bus name change
service by thirty percent. (C) There are no other famous writers who have
(C) Because of a robust economy in the city, lived on the street
passenger patronage has increased substan- (D) Some of the residents of the town are proud of
tially in recent years. the fact that O'Connor once lived there
(D) The public transportation board has recently (E) O'Connor mentioned the town numerous
gathered detailed data on the levels of ridership times in his work

GO ON TO THE NEXT PAGE


© The MBA Center 371
Total PrepKit for the GMAT® Step 12 Workshops

5. A popular Internet service provider changed its 7. In an effort to alleviate the problem of traffic
billing system, charging customers per each congestion in City R, the municipal authorities
connection to the system rather than per total authorized roadwork which would increase the
hours connected. According to company number of lanes on several major highways.
representatives, under the new system, customers However, when the new highways were opened,
will spend more time connected to the Internet the level of congestion actually increased.
while being billed the same or smaller amounts.
Which of the following, if true, best explains why
Which of the following statements, if true, would the level of congestion did not decrease when the new
most strengthen the conclusion of the company highways were opened?
representatives?
(A) People drive faster on highways than on other
(A) Customers will connect to the service less types of roads.
frequently and spend more time connected to (B) Highways do not have traffic lights as other
the service each time they do. types of roads do.
(B) The change in the billing system will attract (C) The highway patrol police increased their
new customers resulting in increased profits for efforts to catch drivers who were speeding.
the company. (D) A person's inclination to drive depends in part
(C) By spending more time connected to the on his or her perception of space.
Internet customers will be able to take (E) The new lanes enabled journey times for many
advantage of services that previously would trips to be substantially reduced.
have been too expensive.
(D) The popularity of other Internet service
providers relies on their having billing systems
similar to the one this compay is adopting.
(E) The company's employees, all of whom have
free unlimited Internet access, support the
change in billing.

6. A film critic: Since he moved from the South End


to the North End of the city, Willy Marcus has
totally rejected the lower-class subjects of his
greatest films. He has only made films about the
wealthy and elite. Therefore, in order to get back
in touch with his roots, Marcus should move back
to the South End before making more movies.

If the film critic's conclusion above is accurate,


which one of the following statements can be most
reasonably inferred?

(A) Most filmgoers prefer Marcus’s early films.


(B) The lower-class subjects of Marcus’s films bear a
relation to the South End.
(C) The South End is populated by people with
more money than those living in the North End.
(D) Marcus’s films about upper-class society fail to
make valuable insights about the society they
portray.
(E) Marcus comes from an impoverished back-
ground.

GO ON TO THE NEXT PAGE


372 © The MBA Center
Critical Reasoning

MEDIUM 10. In response to years of increasing congestion at


airport X, the government decided to redistribute
8. During a recent earthquake over half of a city's landing slots. Henceforth, all international flights
residential housing was destroyed. Many feared arriving from continent A would be rerouted to
that there would be a great housing shortage and nearby airport Y; all flights arriving from continent
that many people would be homeless. However, B would continue to land at airport X. Several
just six weeks after the disaster, the rate of airlines opposed this measure on the grounds that
homelessness was virtually the same. Very few it would result in lost business.
people had left the city either. In fact, the only
things that changed were the addresses of those Which of the following, if true, justifies the airlines'
people whose homes had been destroyed and the conclusion?
rent they were paying, which was substantially
higher for equivalent accommodation. (A) The airlines' customers prefer less congested
airports.
Which one of the following can be inferred from the (B) It takes five minutes more flying time to reach
passage? the second airport.
(C) There are fewer runways, and thus less capacity,
(A) The quality of people's lives declined after the at airport Y.
earthquake. (D) Airport Y is located in a region with better
(B) The new accommodation that people found transport links to the final destinations of many
was higher in quality than their old homes travelers from continent B.
which had been destroyed. (E) Many customers traveling between continent A
(C) Many other types of buildings apart from and continent B choose certain airlines because
residences were also destroyed in the of the easy flight connections they offer at
earthquake. airport X.
(D) Increasing rents allow the market to quickly
supply more housing when it is needed,
such as after a major disaster like an 11. The admissions officers at a certain university use
earthquake. standardized test scores as the main factor in
(E) After the earthquake many new laborers moved deciding whether to admit an applicant. One year
to the city in the hope of getting lucrative work the average test scores of all applicants went up.
for new construction. The admissions officers concluded that the
difficulty of the test had declined that year and
raised the score required for admission.
9. The owners of gambling casinos are keen to attract
inexperienced poker players because, on average, The admissions officers assumed which one of the
these people lose money to the casino, which following in reaching their conclusion?
increases the casino's profits. This is because the
average inexperienced player does not have (A) Fewer applicants reached the standard required
sufficient skill at the game to win. for admission.
(B) The overall applicant population maintains a
Which one of the following can be inferred from the constant ability from year to year.
above argument? (C) Less intelligence was required to score highly
on the test this year.
(A) There is always an element of chance when (D) The test is becoming less and less relevant to
playing poker. the admissions process.
(B) The probability of winning a game of poker (E) Some questions on the test had become
increases with experience. harder.
(C) Casinos make extremely large profits.
(D) Inexperienced players lose more money than
they expect to when playing poker at casinos.
(E) All games played at casinos involve an element
of risk.

GO ON TO THE NEXT PAGE


© The MBA Center 373
Total PrepKit for the GMAT® Step 12 Workshops

12. It has often been hypothesized that global oil 14. In the past, companies in the paper production
consumption, which increases every year, will industry have always opposed restrictions on
deplete the supply of oil, with catastrophic results cutting down trees in forests because they
for the global economy. However, these claims perceived that this would impede their ability to
never stand up to scrutiny, as the volume of oil in earn profits. The government recently proposed to
reserves around the world has remained constant. ban the cutting of trees in all national forests. The
paper production companies were in favor of this
Which of the following, if true, best resolves the ban. Many industry observers subsequently
apparent paradox? concluded that the paper production industry was
no longer interested in protecting its ability to
(A) The actual annual consumption of oil is below earn profits.
that which many experts estimate.
(B) The cost of operating oil refineries has steadily Which one of the following is an assumption made
decreased over time. by the industry observers?
(C) The consumption of oil has greatly increased in
the past 50 years. (A) The profits of paper production companies will
(D) It is the policy of all major oil producers to not decrease after the ban comes into
locate new reserves at a rate consistent with effect.
that at which old reserves are depleted. (B) The companies concerned only manufacture
(E) The number of oil-producing countries has paper products.
been steadily declining. (C) Maximizing profits is the only objective of the
paper production companies.
(D) The legislation will take effect immediately.
13. Kobayashi coffee has more caffeine than Marlowe (E) Paper production companies always act in their
Select coffee. But since Chula Vista coffee has interest.
more caffeine than Valentino coffee, it follows that
Kobayashi coffee has more caffeine than Valentino
coffee.

Any of the following, if introduced into the


argument as an additional premise, makes the
argument above logically correct EXCEPT

(A) Marlowe Select coffee has more caffeine than


Valentino coffee.
(B) Marlowe Select coffee has more caffeine than
Chula Vista coffee.
(C) Marlowe Select and Chula Vista coffees have
the same amount of caffeine.
(D) Kobayashi and Chula Vista coffees have the
same amount of caffeine.
(E) Chula Vista coffee has more caffeine than
Kobayashi coffee.

GO ON TO THE NEXT PAGE


374 © The MBA Center
Critical Reasoning

15. Japan has the highest rate of savings in the (D) The hospital, under private ownership, will
developed world. Research indicates that the charge low-income residents more for health
average family savings of the middle 40 percent of care than it did when it was publicly run.
the population declined by 9.9 percent from 1989 (E) The staff of the hospital will remain unchanged
to 1996. At the same time, the average family after the transition of the hospital to private
savings of the richest 15 percent of the country ownership.
increased by 12.7 percent. Statistics imply that
this decline in savings is due to recession and
inflation in the early 1990's and that there has DIFFICULT
been a significant redistribution of wealth from
middle-class families to affluent families. 17. Which of the following best completes the
passage?
Which of the conclusions below can be properly
drawn from the information above? Critics of Country A's trade policy with Country Z
contend that Country A's low tariffs are
(A) Affluent families are less susceptible to responsible for its large trade deficit with Country
economic downturns than are middle-class Z. Government officials, however, argue that
families, and maintain a high rate of savings there is a trade deficit with Country Z because low
under such conditions. labor costs in Country Z allow its companies to
(B) Average savings in Japan have declined by 9.9 manufacture goods cheaply. The officials also
percent from 1989 to 1996. claim that economic competition from Country Z
(C) Middle-class families can sustain customary is responsible for better prices for Country A's
rates of saving even when prices rise and consumers. Therefore, they say, the most logical
economic momentum slows. way to lower the trade deficit without hurting
(D) A return to normal economic conditions will Country A's consumers is to ________.
reverse the redistribution of wealth from
affluent families to middle-class families.
(E) Spending levels for affluent families declined (A) raise the tariffs on goods imported from
from 1989 to 1996. Country Z
(B) encourage businesses in Country A to reduce
their labor costs
16. A town plans to turn the public hospital over to a (C) increase taxes on all goods not manufactured in
private organization in order to maintain its Country A
financial viability. While no one questions the (D) improve the products manufactured by
ability of the private organization to provide Country A's companies and market them
quality health care, such a change will make the heavily in Country A
hospital unaffordable for the town's low-income (E) subsidize all of Country A's companies that
residents. manage to maintain their prices at the level of
the goods produced by Country Z
If the above statements are all true, which of the
following inferences can most reasonably be drawn?

(A) State regulations require that government aid


for medical costs in public hospitals be made
available to all citizens on the basis of need.
(B) The wealthier residents in the town have
health-care plans that cover all their medical
costs.
(C) Without federal aid, most people cannot afford
the high costs associated with specialized
medical procedures.

GO ON TO THE NEXT PAGE


© The MBA Center 375
Total PrepKit for the GMAT® Step 12 Workshops

18. As the popularity of American professional sports (C) Some motorcyclists have fought the law using
and sports merchandise grows worldwide, the the argument that not wearing a helmet is a
demand for paraphernalia - caps, jackets, etc. - tenet of their religious beliefs.
marketed with the logos of American sports teams (D) Automobile owners have had to pay higher
will continue to grow as well. By necessity an insurance rates because of the need to pay for
increasing quantity of this merchandise will be the injuries or deaths of motorcycle riders who
manufactured. However, it is doubtful that this were not wearing helmets.
increase will provide new jobs for American factory (E) Many automobile drivers and passengers have
workers. suffered injuries in collisions with motorcycles,
the drivers of which were not wearing helmets.
Which of the following, if true, would most
strengthen the conclusion drawn above?
Questions 20 and 21:
(A) Competition among various U.S. manufacturers
of sports team–related equipment insures that Scientist A: If carbon dioxide emissions continue to
retail prices of this merchandise will stay low. rise, this will lead to an increase in the
(B) Professional sports teams in other countries are concentration of carbon dioxide in the
starting to model their marketing plans and atmosphere. If there is an increase in the
production of merchandise on those of concentration of carbon dioxide in the
American sports teams. atmosphere, more heat from the Sun will be
(C) Companies that manufacture merchandise trapped, and the temperature of the atmosphere
relating to American sports teams recently will rise.
established production plants in foreign
countries in preparation for a popularity boom Scientist B: That theory is incorrect because if the
of their product on the foreign market. concentration of carbon dioxide in the atmosphere
(D) Increased attendance at American professional increases, the metabolism of plants which
sporting events has led to an increase in sales of consume carbon dioxide and convert it into
sports merchandise at those events. oxygen will increase.
(E) Past increases in demand for products related
to American sports teams have led to increased
popularity of sports retail equipment overall. 20. Scientist B responds to Scientist A's argument by

(A) accusing Scientist A of misrepresenting the facts


19. One argument of opponents of laws that require (B) challenging Scientist A's ability to form a
drivers and passengers of motorcycles to wear scientific hypothesis
safety helmets is that not wearing a helmet (C) showing that the circumstances which produce
endangers no one but the rider, and that it is a a certain effect in Scientist A's theory will also
right of all citizens to have the option of putting produce a second effect which will negate the
themselves in danger as long as they do not first effect
endanger others. Therefore, critics conclude that (D) showing that the concentration of carbon
helmet laws unnecessarily restrict riders' rights. dioxide in the atmosphere is not the only factor
that influences atmospheric temperatures
Which of the following, if true, most seriously (E) showing that the Earth's temperature will never
weakens the conclusion of the critics above? change because it is naturally self-regulating

(A) Many motorcycle riders have suffered serious


injury in accidents that occurred when they
were not wearing helmets.
(B) Proponents of the law liken helmets to seatbelts
in automobiles, which most states require by
law to be worn by automobile drivers.

GO ON TO THE NEXT PAGE


376 © The MBA Center
Critical Reasoning

21. If Scientist B's objections are true, Scientist A's 23. What could the government do to lower
argument could still be valid if it could be shown that demand and solve the problem of shortages in the
health care system?
(A) plants can consume other atmosphere-
warming chemicals (A) Reduce the amount of health care services
(B) the erosion of the Earth's ozone layer will have offered to the population
a warming effect on the Earth's atmosphere (B) Increase the number of new doctors trained
(C) scientist B lacks formal scientific qualifications each year
(D) scientist A is an acknowledged expert in the (C) Impose a charge on citizens every time they use
field of global-warming research the health care system
(E) the rate of increase of the amount of carbon (D) Consolidate the provision of health care
dioxide in the atmosphere is greater than the services into large medical centers rather than
rate of increase of carbon dioxide absorption by a greater number of small ones
plants (E) Expand the range of treatments available under
the national health care system

Questions 22 and 23

The government of a certain country decided to


provide a system of national health care which
was free at the point of use for citizens of the
country. Before the creation of the system, all
citizens had to pay for their health care out of
their own pocket. After it was first created, the
system worked well. After a generation, however,
the demand placed on the system by the
population increased dramatically and shortages
started to occur even though the population and
amount of health care services offered remained
constant.

22. Which one of the following does most to


explain the rise in demand on the national health care
system?

(A) As soon as a service is provided for free, the


demand for it rises.
(B) The quality of health care provided declined
after the national system was created.
(C) Private health care providers outside the
national system increased the amount of
business they did every year after the creation
of the national health care system.
(D) People’s propensity to use health care services is
higher if they have never had to pay for such
services.
(E) Doctors’ salaries increased after the national
health care system was created.

STOP
IF YOU FINISH BEFORE TIME IS EXPIRED
YOU MAY CHECK YOUR WORK

© The MBA Center 377


Total PrepKit for the GMAT® Step 12 Workshops

CRITICAL REASONING WORKSHOP


ANSWER KEY

1. C
2. A
3. A
4. D
5. A
6. B
7. E
8. D
9. B
10. E
11. B
12. D
13. E
14. E
15. A
16. D
17. B
18. C
19. D
20. C
21. E
22. D
23. C

378 © The MBA Center


Step 13
Analytical Writing
Assessment

© The MBA Center


Total PrepKit for the GMAT® Step 13.1 Lesson

GENERAL OUTLINE

The Analytical Writing Assessment is a one-hour section, which


takes place before you begin the two main test sections of the
computer-adaptive GMAT. You are allowed 30 minutes for each of
the two essays you are asked to write. For all test takers, but
especially those from educational systems outside of North
America, these essays are an opportunity to demonstrate
proficiency in the standards of written English and abilities in
academic writing. This lesson introduces the two types of
Analytical Writing Assessment questions and shows you specific
strategies for writing effectively about them.

THE FOLLOWING TOPICS WILL BE COVERED IN THIS LESSON:


Presentation of Analysis of an Issue and
Analysis of an Argument Question Types
The MBA Center Approach to
Analytical Writing Assessment Questions
Sample Analytical Writing Assessment Essays
Brief Essay Writing Review

380 © The MBA Center


Analytical Writing Assessment

READY…
The Analytical Writing Assessment was added to the GMAT in October
1994, about 40 years after the first GMAT was given. It is scored
independently of the multiple choice section, and a photocopy of the essays
is sent to all the business schools that receive your score reports. The AWA
consists of two 30-minute essays (not necessarily in this order):

1. ANALYSIS OF AN ISSUE

You will be presented with a short, usually broad, and (somewhat)


controversial topic. Your task is to analyze the topic and develop an
argumentative position using examples in an essay of about 500 words.
Though the test makers do not specify the length, our experience shows
that short essays get poor scores.

2. ANALYSIS OF AN ARGUMENT

For this assignment you will be given a short argument to analyze.


Phineas writes these by adapting a Critical Reasoning question; the result AWA in a nutshell:
Two 30-minute essays:
therefore looks eerily similar to a CR problem. Your task is NOT to agree or 1) Analysis of an Argument
disagree with the author, but to evaluate and criticize the argument. 2) Analysis of an Issue
Argument essays can be a bit shorter than Issue essays because they don't
require detailed evidence.

The major challenges of this exercise are timing, typing (for the CAT),
and staying focused on the relatively short passages you read before the
essay.

Why would the GMAC introduce the AWA? The GMAC, remember,
is made up of an assembly and a board consisting mostly of MBA admission
officers. The admission officers requested the essays. Since the number of
non-native English speakers applying to MBA programs has increased
significantly in the last several years, the essays give the admission officers
an extra opportunity to assess the verbal skills of their applicants. Some
observers, more cynical than we might think, suggest that the admission
officers suspect that some applicants pay their friends (or teachers) to write
their application essays for them. The AWA provides a writing sample that
the officers can be sure was actually written by the applicant. Do the schools
really read these essays? Sometimes, yes. But no one expects them to be as
clearly written as your application essays (which are undoubtedly prepared
months in advance, leaving ample time to proofread and reflect), though a
reasonably lucid and well-written response to the AWA questions can help
your admission chances.

What does the AWA test? It tests your ability to write a clear, well-
organized essay, with good grammar and relevant examples, on a vague,
boring topic, early in the morning, in a short period of time.

© The MBA Center 381


Total PrepKit for the GMAT® Step 13.1 Lesson

Your essay will be scored from 0 to 6. Your AWA score has nothing to do
with your overall GMAT score. You could, in fact, get every multiple-choice
question right and practically ignore the AWA, and score 800 on the GMAT
and 0 on the AWA. Note, however, that you must write something on the
AWA. Faliure to write essays will result in no reportable scores.

THE SCORING SYSTEM

6: Outstanding
5: Strong
4: Adequate
3: Limited
2: Seriously flawed
1: Fundamentally deficient
0: Completely illegible, on a topic obviously unrelated to the
question, or non-verbal response

Of course the essays are not graded by computers, even on the CAT. They
are graded by official graders, who read piles of essays in intense grading
sessions, devoting, on average, about two minutes per question. For a more
detailed explanation of grading criteria, consult the Official Guide to GMAT
Review or The GMAT Bulletin.

HERE IS AN EXAMPLE OF EACH QUESTION TYPE:

00:29 GMAT CAT Analytical Writing 1

Analysis of an Issue:

“Some people claim that space exploration should not be funded


by the government, arguing that it is too costly and provides little
benefit for most of the population. Others disagree, claiming that
fundamental scientific knowledge is valuable in and of itself, and that,
additionally, knowledge gained through space exploration is often
later used by industry and research scientists for purposes not
foreseen by those who planned the exploration.”

Analysis of an Issue questions Which do you find more convincing, the argument that space
often cover some of these
exploration should not be funded by the government or the
subjects and topics: Personal
achievement/Personal suc- response to it? Explain your point of view using relevant
cess; General business, United examples, based on your experience, observations, or readings.
States; Issues of ethics and
leadership in business and
management; Topical issues
involving Government and
Business, United States. Test Section Answer
Time Help Confirm Next
Time
Quit Exit

382 © The MBA Center


Analytical Writing Assessment

00:29 GMAT CAT : Analytical Writing 2


Analysis of an Argument:

“It has become almost impossible to eat a balanced, healthy


diet. Television news programs are constantly providing new,
sometimes contradictory information about what is healthy and
what is not. The average consumer cannot make informed
decisions from this kind of scanty information and is generally
overwhelmed upon entering increasingly larger supermarkets
with more and more brands to choose from.”

Discuss how logically convincing you find this


argument. In explaining your point of view, be sure to
analyze the line of reasoning and the use of evidence in
Phineas often writes Analysis
the argument. Also discuss what, if anything, would make of Argument questions as if
this argument more sound and persuasive, or would help they came from sources like
you to better evaluate its conclusion. these: Speeches, by corporate


spokespersons/government
officials; Newspapers, News-
Answer
letters, Memorandums, Busi-
Test Section
ness proposals, and Trade
Time Help Confirm Next
Time
Quit Exit
publications.

© The MBA Center 383


Total PrepKit for the GMAT® Step 13.1 Lesson

AIM…
To prepare for the AWA, it is important to develop an organizational
plan and to review some grammar rules and style techniques that can
improve your score.

A) ANALYSIS OF AN ISSUE

The graders are looking for several key elements in your essay:

Development that explores the issue from different perspectives


Organized support for your position with relevant examples
Varied sentence structure
Good command of grammar

There are three key steps necessary before writing.

Step one: Identify the issue. Properly identifying the issue is, of
course, imperative. Pay close attention to the wording used
You have 30 minutes for each
essay. Try allocating your time in the question.
this way:
10 minutes (8-10) Step two: Consider at least one reason for each side of the issue. The
Brainstorm and organize
graders want to see that you understand the complexity of
15 minutes (15 - 18)
Write the argument.
5 minutes (2 - 5)
Proofread for large errors Step three: Choose a position and list several supporting reasons.

Let's try this with an example.

00:29 GMAT CAT : Analytical Writing 1

Analysis of an Issue:

“Some people claim that space exploration should not be funded


by the government, arguing that it is too costly and provides little
benefit for most of the population. Others disagree, claiming that
fundamental scientific knowledge is valuable in and of itself, and that,
additionally, knowledge gained through space exploration is often
later used by industry and research scientists for purposes not
foreseen by those who planned the exploration.”

Which do you find more convincing, the argument that space


exploration should not be funded by the government, or the
response to it? Explain your point of view using relevant
examples, based on your experience, observations, or readings.

Test Section Answer


Time Help Confirm Next
Time
Quit Exit

Step one: The issue will be clearer if you rephrase it as a yes/no


question. In this case, you should think, "Should space
exploration be funded by the government?"

384 © The MBA Center


Analytical Writing Assessment

Step two: Yes, because it is important for a country to advance


scientifically and technologically.
or No, because there are more pressing needs the government
should address such as education and health care. (Of
course, you can use other reasons.)
Step three: The government should fund space exploration because …
1. space technology has practical, short-term benefits
(communication satellites, weather forecasting, etc.)
2. it provides jobs for scientists and engineers
3. the knowledge gained is of philosophical and scientific
interest, and can not be learned elsewhere
or
The government should not fund space exploration because …
1. the astronomical (ha ha) expenses could be used for more
important things
2. a lot of the benefits go to private companies which ought to
make the investment themselves
3. there is a danger to astronauts

Because the instructions are always the same (even though the
topics themselves change), you can develop a model essay in
advance. Here is a model for Analysis of an Issue:

The issue of (restatement of issue) is complex and


controversial. While (reason for one point of view),
some people argue that (reason for other point of
view). Both perspectives are important, but, overall, the
advantages of (your position) outweigh the drawbacks.
(Your position) is important because (first reason). This
is evident (support for first reason).
Secondly, (second reason). This can be illustrated by
(support for second reason).
In addition, (third reason). For example, (support for
third reason).
Some people might object that (support for other
position). However, this objection can be countered by
(reason against objection). Use the template method to
prepare your essays in
While the issue of (restatement of issue) must be
advance. Test it. Try it out for
considered carefully, the reasons of (brief restatement yourself, changing it to suit
of reasons) provide convincing support that (your your style.
position) is appropriate.

B) ANALYSIS OF AN ARGUMENT
The planning stage of the Argument essay differs from that of the Issue
essay. There is less brainstorming and more analysis.

A GOOD ANALYSIS OF AN ARGUMENT ESSAY SHOULD HAVE

– a development that identifies all the aspects of the argument


(the conclusion and the premises)
– an examination of the argument’s logic (the assumptions
which connect the premises and the conclusion)
– a discussion of what would strengthen the argument
– varied sentence structure
– a good command of grammar

© The MBA Center 385


Total PrepKit for the GMAT® Step 13.1 Lesson

In planning your essay before you begin writing, you should follow two
key steps:

Step one: Identify the conclusion of the argument.

Step two: Compare the conclusion with the evidence, listing any
questionable assumptions.

Let's look at an example.

00:29 GMAT CAT: Analytical Writing 1


“It has become almost impossible to eat a balanced, healthy diet.
Television news programs are constantly providing new, sometimes
contradictory information about what is healthy and what is not. The
average consumer cannot make informed decisions from this kind of
scanty information and is generally overwhelmed upon entering
increasingly larger supermarkets with more and more brands to
choose from.”

Discuss how logically convincing you find this argument. In


explaining your point of view, be sure to analyze the line of
reasoning and the use of evidence in the argument. Also
discuss what, if anything, would make this argument more
sound and persuasive, or would help you to better evaluate its
conclusion.


Test Section Answer
Time Help Confirm Next
Time
Quit Exit

Step one: The conclusion is the author's main point. If you aren't
sure, try paraphrasing the argument: the author believes
"X" because of "Y." "X" is the conclusion. Here, the author
believes "it's impossible to eat a good diet" (conclusion)
because "TV news programs keep announcing different
things" (evidence) and "supermarkets have lots of different
brands" (more evidence).

Step two: What must be true for this argument to work? That is,
what does the author assume?
The author assumes…
most people watch TV news programs
most people rely on TV news programs for diet information
most people shop in supermarkets
the wide selection of brands makes it very difficult to find
healthy food
(You might have found some other assumptions or
expressed these differently)

Think about the assumptions above. If any of the above are


false, the author's argument is weakened. If, for example, most
people plan their diets based on recommendations from their
doctors, the author's conclusion loses all of its validity. If the wider

386 © The MBA Center


Analytical Writing Assessment

selection of brands makes it easier to find healthy food, and not


harder, the argument is also weakened.

Note: All arguments given to evaluate are seriously flawed.

MODEL FOR ANALYSIS OF AN ARGUMENT


You can, of course, modify this template or make your own.

The argument presented is not convincing. The author


concludes (conclusion) based on (evidence). There are
several questionable assumptions, resulting in a logical
gap between the premises and conclusion.

A major assumption made in this argument is that


(first assumption). This assumption may very well be
wrong, as (why it may be wrong).

The validity of the conclusion also depends on (second


assumption), a key concept omitted in the argument.
For if (opposite of second assumption) were true, it is
likely that (opposite of conclusion).

Finally, even if the two assumptions discussed above


were true, the argument would be unconvincing
because, without providing any evidence, it requires ‘Argument’ questions might
that (third assumption). sound familiar, suspiciously
familiar – very much like
Critical Reasoning arguments,
In order to make the argument more persuasive, the no? Ask yourself the same
author would have to demonstrate that the questions: What is the
assumptions discussed above are, in fact, true. Thus, conclusion? and What are the
evidence supporting (first assumption), (second flaws?
assumption), and (third assumption) would tighten the
link between the premises and conclusion, making the
reasoning more sound. Without any discussion of these
ideas, however, it is impossible to evaluate the
conclusion.

BRIEF ESSAY-WRITING REVIEW


The following are some guidelines for effective writing. None of the
following hints are related to the rules of grammar tested by GMAT
Sentence Correction questions, and they are not included in the Grammar
Review. However, all of the rules explained in the Grammar Review should
also be adhered to for the AWA essay.

The guidelines which follow are intended to help you improve your AWA
essay but are by no means a comprehensive review of the rules for effective
essay-writing.

© The MBA Center 387


Total PrepKit for the GMAT® Step 13.1 Lesson

PUNCTUATION
Here are a couple of important points to remember.

– Avoid comma splices


A common mistake is to link what should be two separate sentences with
a comma (,). Here's an example: "Research into this disease is important,
governments should support it." There are two ways to remedy this error:
"Research into this disease is important and governments should support it," and
"Research into this disease is important, and governments should."

– Avoid most forms of punctuation, period.


Commas and periods should be all that are necessary to the essay. Of
course, if there is a rhetorical question included, it must be followed by a
question mark.
If you’re not sure about
If you have been utilizing semicolons all your life and you are 100%
whether to punctuate or not
to punctuate, don’t. certain that you've mastered the difficult art, go ahead and use them.
Otherwise, pretend they don't exist. Many good essays are reduced in
effectiveness by a proliferation of unnecessary semicolons (written by
people who have semicolon cancer).

– Colons can also be somewhat tricky. If you insist on using them, remem-
ber this: colons are abbreviated equal signs. What is on one side of them must
be roughly equal to what is one the other side of them (within the sentence).

– Parentheses, although are fun to play with, have no place in a


short, formal essay, in which every sentence should be relevant and should
contribute equally.

388 © The MBA Center


Analytical Writing Assessment

SOME ELEMENTS OF STYLE

Logical links

For clarity in your essays, employ transitions to signify emphasis or


contrast, or simply to provide the essay with a sense of fluidity. Here are
some useful expressions.

– Giving examples:
… is demonstrated by…
… is illustrated by …
… is typified by …
For example …
One instance of this …
… as in the case of …

– Emphasis
In fact …
Obviously …
It is clear that …
Definitely …
Pertinently …

– Contrast
Conversely …
Regardless …
On the contrary …
While … is true, it can also be argued …
Although … another viewpoint is that …
Despite this situation …
In contrast to …
… is in opposition to … Good ideas can easily be lost
without logical links. Graders
like ‘em because they make
– Causal relationships explicit the relationships
Therefore … among your ideas. So, link it
Thus … up.
Consequently …

SENTENCE VARIETY

Diversify your phrasing. Avoid writing more than two sentences in a


row of the same length. Avoid writing more than two sentences in
a row that are patterned on the subject-verb-modifier formula.
Avoid writing more than two sentences in a row that begin with
the same word. (You see what we mean?)

Don't: Jake ran to the post office to mail his letter. Jake arrived at
the post office at 5:05. Jake found the post office closed. Jake
went to another post office not far away. Jake tried again to
mail his letter. Jake found that post office closed too.

Do: Jake ran to the post office to mail his letter. At 5:05 he
arrived, found the post office closed, and decided to try
again at a different post office. Jake went to another post
office. It too was closed.

© The MBA Center 389


Total PrepKit for the GMAT® Step 13.1 Lesson

VOCABULARY
While use of formal language (specialized vocabulary used more
in writing than in speech; such as specific business terms used in
business documents) can improve your essay, it is more important
to use language you know how to use. Try to strike a balance
between sophisticated and showy.

Most importantly, be specific. Avoid vague words. Before you write


"people," think to yourself "What kind of people?" There's probably a more
specific word you could use, whether it's "homeowners," "consumers,"
"students," "test takers," etc. Apply the same process for "things," "places," and
other words that encompass broad categories.

Here are some verbs that are generally overused and vague:

– To do
It is rare that the verb "to do" is necessary. In formal writing you can
almost always replace it with a more specific verb. Instead of:
"Governments need to do something about this problem," try
"Governments need to remedy this problem." Instead of "College
graduates do great work for the world," try "College graduates
perform important actions in society."

– To have
"To have," of course, means "to possess, to own". Avoid phrases such
as "The argument has three parts." There might be three parts in the
argument, but does the argument possess those parts, the way you
possess, for example, the new Madonna CD? Would you write "The
argument owns three parts"? or "Three parts belong to the argu-
ment"? No, not unless you are writing metaphorically, and metaphorical
writing should be avoided on the AWA. Try, "The argument is
composed of three parts," or "The argument can be divided into
three parts."

VAGUE SPECIFIC
give provide
show, point out demonstrate, depict, illustrate
get receive, become

– Avoid the passive voice of verbs.

Instead of "The discovery was made by scientists in 1934” … write


"Scientists made the discovery in 1934 …"

Instead of "The voters' opinions are demonstrated by the results of the


survey," write "The results of the survey demonstrate the
voters’ opinions."

– Avoid colloquial words such as: really, pretty (as in “The example is
pretty important”), and lots of.

– Avoid contractions.

– Never state in many words what you could just as effectively state in
a few.

390 © The MBA Center


Analytical Writing Assessment

FIRE!

ANALYSIS OF AN ISSUE

This is an example of a "2," or "seriously flawed" essay. Look for


problems of content, style, grammar, and organization as you
read this essay. Think about how it could be improved and
compare your ideas to the critical comments that follow.

Space exploration is definitely a good idea.


Not just now, also for the future. There's already
5 billion people on the planet. We put people in
a biosphere, why not in space?

5 The moon may turn out to be uninhabitable,


but Mars is pretty similar to the atmosphere on
earth. And the Russians, while having problems,
have kept people in a space station for a long
time. If we didn't fund space exploration, we
10 wouldn't have any of these things.

It's important to learn about space. If Galileo


didn't make a telescope, we would still think the
earth went around the sun! Over the centuries
we have learned a lot about space and science,
15 and now that we have the technology, we really
should continue. There are lots of advantages
for space research. Economic, scientific, even
health advantages.
Thank you.

Content:
Notice that the writer of this essay misses the point that the
issue is government funding of space travel rather than space
travel itself. The author makes basically one point: that space
exploration will benefit humanity if it is judged possible for people
to live in space, thus relieving the earth of its population problem.
This point is valid, and the references to the world's over-
population problem and the biosphere experiments would support
it well if their relevance had been explained. A good essay would
include at least one other benefit and also address some of the
opposing arguments.

Organization:
There seems to be a vague attempt here to fit this essay into a
introduction-body-conclusion format, but it crumbles. There is also
no development of the author's main point: it is rather a stream-of-
consciousness ramble of somewhat related facts and ideas.

Grammar:
The essay is littered with mistakes. The problem is that not only
are there grammatical errors, but also the errors interfere with the
essay's intended (one guesses) meaning. For example, the sentence
fragment that concludes the passage might actually contain a valid
argument (that space research could produce benefits in various
fields). Its unintelligibility, however, prevents its contributing to
the essay.

© The MBA Center 391


Total PrepKit for the GMAT® Step 13.1 Lesson

Style:
Note that this essay does contain an admirable variety of
sentence lengths. Stylistic problems, though, pervade. The
vocabulary is limited. Changing a few words to strengthen the
transitions between the ideas (the "but" in line 6 would be more
effective as a "however"; the "and" in line 7 could be a "furthermore";
etc.) would improve the style of the essay. Furthermore, too much
colloquial language is used: "lots of," "we really should," etc.

This is an example of a "4," or "adequate" essay. Again, look for


problems of content, style, grammar, and organization as you
read. Think about how it succeeds, how it could be improved, and
compare your ideas to the critical comments that follow the
essay.

I am strongly in favor of space exploration


and government funding thereof. I believe that it
is an essential aspect of humanity that we are
explorers, and that the money, though there is a
5 great deal of it necessary, could not be better
spent than on our journeys into the nether
regions of our knowledge!
What distinguishes humankind from the
bestial elements of the earth? Our ability to
10 think, to reason, and above all, to imagine,
does. Humankind has been granted the gift of
intelligence. What a shame it would be to waste
it. The frontier of space is the last remaining
border of our knowledge! It is there to be
15 conquered.
There are more mundane reasons, too, to
explore space, like, for example, the short-term
benefits. These include helping unemployment
levels stay down by providing jobs to everyone
20 associated with the space programs, and aiding
meteorology by placing satellites in orbit around
the earth. Moreso, however, it seems that is our
responsibility to expand the limits of our
knowledge!
25 I know what you are going to say, that the
money should be spent on social initiatives such
as schools, feeding the poor, cleaning the
environment. These are valid ways to spend
money, but not as essential as stretching the
30 possibilities for knowledge!
Although government funding of the space
program is a sensitive issue, I think it is clear
that, if governments can afford to have
humanity explore the universe, it is our
35 responsibility to do so.

Content:
This essay manages to focus on the issue of government
funding and presents two sides of the argument, clearly choosing
one side and acknowledging the other. The author's main point
here, the quasi-mystical belief that space, as a final frontier,
represents some sort of metaphysical barrier to be breached, is

392 © The MBA Center


Analytical Writing Assessment

supported, though only slightly, by mention of the difference


between human beings and other animals, and that there is a great
deal of further exploration possible in space. The author laudably
adds two other reasons for why space research is valuable,
although the benefit of keeping people in work seems of only
marginal relevance to the issue.

Organization:
This essay is fairly well-organized. The first paragraph states the
author's position while referring to the main flaw in his or her
position. A more detailed description of the opposing position
would have strengthened this paragraph. Each ensuing paragraph
has a clear individual focus, although the “knowledge” tirade
somehow manages to show up everywhere (it shouldn't).

Grammar:
There are numerous minor grammatical mistakes in this essay,
none of which interferes with its meaning. The “we,” for example,
in line 3 does not have a grammatically correct antecedent,
although it is obviously meant to refer to “humanity.”

Style:
The essay contains well-crafted transitions between ideas: Flaubert did it, maybe you can
"moreso," "however," "although," etc. The rhetoric, unfortunately, is a too... but not on the AWA:
tad overblown. It is advisable to shy away from phrases such as "the Resist dramatic flourishes and
nether regions of knowledge." The word choice is also occasionally focus on specifics – it is very
easy to lose sight of the facts
suspect ("cleaning the environment"? Really.) Furthermore, the when you try to write
phrase "I know what you are going to say," is a poor attempt at a dramatically.
rhetorical device.

This is an example of a "6," or "outstanding" essay. Notice its


strengths and weaknesses and compare your ideas to the critical
comments that follow the essay.

The issue of government funding of space


exploration is complex and controversial. The
costs required to fund a major space program
are enormous, and some might argue that the
5 investment would be made more sensibly and
productively in education, housing for the poor,
or other programs with concrete social benefits.
There are many long-term advantages to space
exploration, however, that justify the high costs.
10 Both perspectives on the issue are valid, but,
overall, the advantages of government funding
of space exploration outweigh the disad-
vantages.
Space exploration and research are impor-
15 tant because they offer practical technological
advances. For example, weather satellites and
communications technology would not have
reached nearly their advanced stages were it not
for the information acquired by space programs.
20 Perhaps with continued research further
advances in these or other fields will develop.
Secondly, because scientists are certain of
relatively little about the universe in comparison

© The MBA Center 393


Total PrepKit for the GMAT® Step 13.1 Lesson

with, for example, human anatomy, space


25 exploration is a field wide open for scientific
thought. Such research not only encourages
intellectual thinking and new research, but also
provides a format in which theoretical physicists
can test abstract hypotheses.
30 Furthermore, the knowledge gained through
exploration of space is of immense scientific
and, potentially, philosophical interest, and
could not be learned elsewhere! Continuing this
research is the sole method by which the human
35 race can continue to learn about it's place in the
universe.
Naturally, the objections that can be raised
about space exploration, particularly concerning
its financial costs, are substantial. It is true that
40 money is badly needed elsewhere in society.
However, it seems likely, though unfortunate,
that there will never be a time that funds won't
be required to improve the lot of the destitute.
The time is right for governments to capitalize
45 on what public support there is for space
exploration, and for space research programs to
build on what progress has been made.
While the funding of space exploration is an
issue that must be considered carefully, the
50 scientific and technological advances that may
be within the grasp of our research,
compounded with the short-term benefits,
provide ample support that governments should
continue to support the space program.

Content:
Both sides of the issues are addressed. One is chosen. The
author supports his or her position with three reasons: that space
research has practical technological advances, that it encourages
scientific thought, and that there are no other available methods
by which this information will become available. The counter-
argument is acknowledged and, not dismissed, but refuted. Notice
that the grounds for this refutation are perhaps not perfectly
sound, but they are elucidated as thoroughly as possible
considering the time constraints.

Organization:
First of all, note the structure of the first paragraph. The issue
and its complexities are clearly identified. The author then
presents the stance to be taken in the ensuing paragraphs.

Grammar:
The essay contains exactly one grammatical error. (Can you
spot it?) Obviously, it doesn't affect the essay.

Style:
Note the preponderance of transitional or emphatic phrases em-
ployed: “however,” “For example,” “Secondly,” “not only … but also,”
“furthermore,” “naturally,” etc. The essay thus reads fluidly and easi-
ly. Note also that the essay is not stylistically flawless – the phrase,
“weather satellites and communications technology would not have

394 © The MBA Center


Analytical Writing Assessment

reached nearly their advanced stages,” is slightly clumsy. (The sentence


could have been written: “the tremendous progress in meteorological
and communications technology might not have been achieved.”) An
essay does not have to be perfect to be scored a 6!

ANALYSIS OF AN ARGUMENT
The following is an example of an essay that was scored a “2.”
Look for problems of content, organization, grammar, and style.
Think about how it could be improved and compare your own
analysis to the comments that follow.

This argument isn’t really convincing. It may


be true and it may be false, but the “author”
doesn’t really give so much information. What
do supermarkets have to do with this? What if
5 you eat in restaurants or shop in others kinds of
stores? One could then still have a good diet.

The author also fails to mention exercise.


Even if you don’t eat that well, if you exercise a
lot you might still be in good health. The author
10 talks about TV (I don’t watch TV) but he should
also mention radio and newspapers and if a
news program gives contradictory information,
it maybe isn’t there fault. Maybe it is the
doctors, not the journalists, who keep changing
15 they’re mind.

The point about their being lots of different


brands is a good one. What with all the hassles
of modern life, who has the time to study each
brand and decides what’s healthy and what’s
20 not. And all the fast food restaurants.
Everywhere you go there’s a McDonald's or
Burger King. How can you have a very healthy,
good diet under these circumstances?

If the author discussed other kinds of places


25 where you get food, and talked about doctors
not just TV, he would have a pretty good point.
As it stands, I disagree with the argument.

Content:
The essay fails to stay focused on dissecting the argument. The
issue of exercise is not presented in the argument; neither is the
issue of whose fault it is for the misleading health information on
television news programs. They are therefore irrelevant. Another
salient error is the author's inclusion of his or her opinion of the
argument. This is never appropriate.
There are some positive points in this essay. Note the
references, incoherent though they are, to flaws in the argument –
specifically, the assumption that all consumers receive
information from television, and the assumption that everyone
purchases food from supermarkets. Another positive point is the
author's inclusion, in the final paragraph, of a proposal to
strengthen this argument.

© The MBA Center 395


Total PrepKit for the GMAT® Step 13.1 Lesson

Organization:
As in the case of the previous “2” essay, here there are hints of
a introduction-body-conclusion format, but hints just aren't good
enough. The individual paragraphs show no coherence.

Grammar:
There are many grammatical errors. Among the most salient are
the conflation of “their,” “they're” and “there,” and repeated
pronoun errors.

Style:
The essay is a mess stylistically. The preponderance of rhetorical
questions lends this piece an absurdist quality. Notice the
parentheses. (You shouldn't use parentheses.) The use of the first-
person pronoun should be avoided. In this instance what could
have been a sound critique of the argument, the observation that
not everyone watches television, is obscured by the style. The
word choice is poor. Note that the phrase “give so much
information,” should be “provide enough information.” There are also
a strange set of unnecessary quotations marks around the word
“author.”

The following is an essay that was scored a “4.” Notice its


relative strengths and weaknesses. How would you improve it?

“It has become almost impossible” to eat a


balanced diet states the author, yet without
providing ample backup. There are some sound
ideas presented here. Unfortunately they are
5 marred by a lack of rationality and a tendency to
overstate the case. The writer doesn't give
enough solid examples or provide enough
reasons why this is so.
One of the author's biggest points is that the
10 size of supermarkets prevent consumers from
being able to locate healthful food. While this
may be true to an extent, this line of reasoning,
in order to justify the author's viewpoint, needs
strengthening; this could be accomplished by,
15 for example, citing specific food types and the
brands of that ilk that are available. Another
way to bolster this argument might be to show
statistics including where people shopped and
what they buy.
20 Although the authors' second point, about
how the television news media people don't
know what they are talking about half the time,
makes sense, again, it would be productive to
include specific examples. This could be done
25 through mentioning key individuals who
propagate the myths of health that permeate
society, or by showing that television influences
our lives. Again, statistics showing how many
people watch television and exactly for what
30 they use it for would support the author's
argument.

396 © The MBA Center


Analytical Writing Assessment

What the author mainly fails to do in this


piece of writing, though, is restrict their
commentary to those to whom it actually
35 applies: people who shop in the supermarkets
and watch so much TV that they get all their
information there. If the passage had stayed
focused on them instead of generalizing, it
would have been successful. As it stands, it fails
40 to convince me that “it is impossible too eat a
balanced, healthy diet.”

Content:
This essay successfully identifies the flaws in the argument and
offers solutions which could strengthen the argument. The
proposed solutions in the main body of the essay, however, are not
the most direct methods to strengthen the argument. Only in the
concluding passage does the author include the clearest remedy,
re-focusing the argument on those who actually receive their
nutrition information from television and shop only in
supermarkets.

Organization:
The essay presents an overall view of the argument in the
introductory paragraph, and then proceeds to break down the
features of the argument paragraph by paragraph.

Grammar:
There are several minor grammatical or spelling errors, none of
which interfere with the author's intended meaning.

Style:
There are numerous stylistic problems here. First of all, the
author's prejudices repeatedly color his or her analysis. The phrase
“the television news media don't know what they're talking about half
the time,” could contribute more effectively to the essay were it to
read “the television news media indiscriminately disseminate the latest
health findings,” or was written in another way with less opinion,
and more objectivity. The word choice is occasionally poor.
Instead of “biggest” (line 9), use “most important” or “major” (the
points are all the same size). Also, note the semicolon in line 14. It
is actually used correctly, but is unnecessary since there is no need
to connect those two sentences, which would be fine independent
of one another.

This is an example of an essay that was scored a “6.” Look for a


successful analysis of the argument, and the use of style,
grammar, and organization. Why was this essay given the top
score?

The author's conclusion that it's almost


impossible to eat a balanced, healthy diet,
depends heavily on two principles: misleading
information provided by television news
5 programs, and the increasingly wide selection of
brands available at supermarkets. That these
points are the main factors contributing to the
average diet is disputable, and consequently, so

© The MBA Center 397


Total PrepKit for the GMAT® Step 13.1 Lesson

is the author's position. The argument rests on


10 numerous questionable assumptions and vague,
incomplete evidence.
The premise concerning the new, “sometimes
contradictory” information provided on TV news
programs may be valid. This evidence is only
15 relevant, however, if it is true that average
consumers watch television news programs,
and, moreover, use the information to plan their
diets. Clearly, if people are unaware of the
information provided, the contradictions in the
20 information won't affect their eating habits.
Even if it is accepted that consumers watch
news programs, the premise of the author's
argument is flawed, as it rests on the
assumption that viewers actually rely on these
25 programs to determine their diets. It is likely that
people use other, readily available, sources of
information, such as nutritionists, doctors,
literature, for this purpose. The conclusion that it
is “almost impossible” to eat a healthy diet is
30 therefore completely unfounded.
The author's second premise, pertaining to
supermarkets, is debatable, and thus also results
in questionable assumptions. The author does
not explicate the assertion that there is a
35 correlation between having many brands to
choose from and the impossibility of dieting
healthily. Presumably, the intended contention is
that consumers, upon entering the markets, are
overwhelmed by the sheer volume, and
40 consequently are unable to make sound,
informed choices. This is certainly an unfounded
assumption. It is, in fact, arguable that the
greater choice makes it easier to find healthy
brands. The author fails to mention that in this
45 wide selection of brands comprises many
healthy food choices. It is entirely possible that
government regulations ensure a high quality of
food despite the large number of brands.
The author's statement about supermarkets,
50 furthermore, is only relevant to an analysis of
people who shop in supermarkets. Consequently,
yet another weakness in the argument is the
failure to account for alternative sources of food
products, such as specialized health food stores,
55 local markets, which may supply only healthy
food.
The argument would be more persuasive if it
addressed some of these questionable
assumptions. A validation of the stated
60 conclusion necessitates evidence that the
majority of consumers shop in supermarkets and
use no source of nutritional information other
than television news programs. Additionally, the
author should demonstrate that the diverse
65 selection of brands includes largely unhealthy
products. Another method by which the

398 © The MBA Center


Analytical Writing Assessment

argument could be strengthened would be a


limiting of conclusion to refer to people who
shop in supermarkets and rely on television for
70 diet information.

Content:
The essay analyzes the argument fairly thoroughly, pinpointing
the premises and the conclusion, and the flawed logic. It offers
several options by which the author could strengthen the
argument, including the most direct method – limiting the
conclusion to consumers who only shop in supermarkets and base
their purchasing decisions on information provided by the
television news.

Organization:
Notice the systematic analysis of the argument. Each paragraph
focuses on a specific aspect of the argument, and each contains an
individual introduction and conclusion.

Grammar:
There are several small grammatical errors. For instance note
that line 28 needs an “and” between “doctors” and “literature.”

Style:
The style is acceptable, but not flawless. There are some
awkward phrases, such as “A validation of the stated conclusion
necessitates evidence…” (line 59). Notice the effectiveness of the
transitional and emphatic phrases, such as “consequently,”
“however,” “therefore,” “clearly,” “furthermore,” etc.

© The MBA Center 399


Total PrepKit for the GMAT® Step 13.1 Lesson

FAMOUS LAST WORDS


Be sure you have read and understood both the essay-writing
review in this chapter and the Grammar Review in this book,
preferably before you have written any practice essays.

Prepare and memorize a template. All AWA essay topics can be


addressed using one of the templates which are included in this
chapter.

When you take your exam, spend a few moments preparing some
notes or an outline. Don't indulge in stream of consciousness
writing. Abstract poetry might be a hit in Lit. classes and at
cocktail parties, but on the GMAT it won't fly.

Try to leave a minute or two to proofread, just to make certain


you haven't made any egregious errors.

400 © The MBA Center


Analytical Writing Assessment

Analytical Writing Assessment – Homework Directions

End


When finished
ANALYSIS OF AN ISSUE
reading
TIME - 30 MINUTES
directions
Directions: Analyze the issue presented click on the
below and develop your position. There is no icon below
“correct” point of view. In presenting your posi-
tion, you should consider different perspectives.
Read the issue stated below and the
instructions that follow. You may make any Dismiss
notes in the empty space on the page that will
help you plan your essay. Directions


Test Section Answer
Time Help Confirm Next
Time
Quit Exit

ESSAY 1

Some educators argue that children who demonstrate particular aptitude for learning when they are young
should be placed in special programs where they are given the opportunity to advance more rapidly than they
might in normal classes. Many child psychologists, however, have publicly stated they do not support special
programs for gifted children. They claim that separating children by aptitude at an early age is unfair to those in
the slower classes, some of whom might also thrive in a more challenging environment.

Which do you find more convincing, the argument of the educators or that of the child psychologists? Explain
your point of view using relevant examples, based on your experience, observations, or readings.

STOP
IF YOU FINISH BEFORE TIME IS EXPIRED
YOU MAY CHECK YOUR WORK
© The MBA Center 401
Total PrepKit for the GMAT® Step 13.2 Homework

Analytical Writing Assessment – Homework Directions

End
When finished
ANALYSIS OF AN ARGUMENT
reading
TIME - 30 MINUTES
directions
Directions: Analyze the argument click on the
presented below. You should not develop your icon below
own position on the issue. Rather, you may
consider any assumptions made in the
argument, the ways in which the evidence is
used, how the argument’s conclusion might be Dismiss
strengthened, or any logical flaws you observe.
Read the argument and instructions that Directions
follow. You may make any notes in the empty
space on the page that will help you plan your
essay.

Test Section Answer


Time Help Confirm Next
Time
Quit Exit

ESSAY 2

Over the last five years, newspaper sales in Region X have declined by over 10%, while sales of weekly news
periodicals have increased slightly. This demonstrates that more and more people in the region have less time to
read a newspaper every day, and prefer to read a once-weekly summation of world events.

Discuss how logically convincing you find this argument. In explaining your point of view, be sure to analyze the
line of reasoning and the use of evidence in the argument. Also discuss what, if anything, would make this argument
more sound and persuasive or would help you better evaluate its conclusion.

STOP
IF YOU FINISH BEFORE TIME IS EXPIRED
YOU MAY CHECK YOUR WORK
402 © The MBA Center
Step 14
Analytical Writing
Assessment
Workshop

© The MBA Center


© The MBA Center
Analytical Writing Assessment

Analytical Writing Assessment – Workshop Directions

End


When finished
ANALYSIS OF AN ISSUE reading
TIME-30 MINUTES directions
click on the
Directions: Analyze the issue presented icon below
below and develop your position. There is no
“correct” point of view. In presenting your
position, you should consider different
perspectives.
Read the issue stated below and the Dismiss
instructions that follow. Directions


Test Section Answer
Time Help Confirm Next
Time
Quit Exit

ESSAY 1

“ I don’t think being well-rounded is particularly important; I would rather see people with a cutting edge on them.”

Discuss the extent to which you agree or disagree with the opinion expressed above. Support your point of view
with reasons and/or examples from your own experience, observations, or reading.

STOP
IF YOU FINISH BEFORE TIME IS EXPIRED
YOU MAY CHECK YOUR WORK

© The MBA Center 405


Total PrepKit for the GMAT® Step 14 Workshops

00:00 Analytial Writing Assessment – Workshop Directions

End
When finished
ANALYSIS OF AN ARGUMENT reading
TIME - 30 MINUTES directions
click on the
Directions: Analyze the argument icon below
presented below. You should not develop
your own position on the issue. Rather, you
may consider any assumptions made in the
argument, the ways in which the evidence is
used, how the argument’s conclusion might Dismiss
be strengthened, or any logical flaws you Directions
observe.
Read the argument and instructions that
follow.

Test Section Answer


Time Help Confirm Next
Time
Quit Exit

ESSAY 2

“ The country of Aviar could best solve its current trade deficit problem by lowering the price of parrots, its primary
export. If Aviar pursued this course of action, it would be able to compete more successfully for markets with other parrot-
exporting countries. Sales of Aviar’s parrots abroad would increase- an increase which would dramatically reduce Aviar’s
trade deficit.”

Discuss how logically convincing you find this argument. In explaining your point of view, be sure to analyze
the line of reasoning and the use of evidence in the argument. Also discuss what, if anything, would make this
argument more sound and persuasive, or would help you better evaluate its conclusion.

STOP
IF YOU FINISH BEFORE TIME IS EXPIRED
YOU MAY CHECK YOUR WORK

406 © The MBA Center


Step 15
Reading
Comprehension

© The MBA Center


Total PrepKit for the GMAT® Step 15.1 Lesson

GENERAL OUTLINE

Slightly more than one-third of the questions in the Verbal Section


of the GMAT CAT are Reading Comprehension questions. These
questions test your ability to understand and analyze information
presented in written form. This lesson introduces the most common
Reading Comprehension question types and provides strategies and
specific reading theories for answering each one of them. In addition,
in this lesson you will learn reading techniques that will help you
improve your reading efficiency and comprehension.

THE FOLLOWING TOPICS WILL BE COVERED IN THIS LESSON:


The MBA Center Reading Theories for
Reading Comprehension Passages
The Three Kinds of Reading Comprehension Answer Choices
The Four Groups of Reading Comprehension Question Types
The MBA Center Approach to Reading Comprehension
Four Drills in Reading Comprehension

KEY TERMS

Passage: Reading Comprehension passages are generally between 250


and 450 words in length and relate to a variety of subjects, including
literature, philosophy, law, the social sciences, and the physical sciences.

Trigger words: Words in a passage which identify the conclusion, idea


transitions, changes in points of view, evidence of authorial opinion, and
introductions to examples.

Scope: This concept describes what is relevant or irrelevant in a Reading


Comprehension passage. Any information, ideas, or assumptions not
directly related to what is stated or implied in a passage are considered
irrelevant and therefore out of scope. Understanding scope improves
your ability to answer Reading Comprehension questions quickly and
accurately.

408 © The MBA Center


Reading Comprehension

READY…
The Reading Comprehension passages and questions are part of the
Verbal Section of the GMAT CAT. The questions appear randomly during
the Verbal Section in groups of 5 to 7. There are between 14 and 18 Reading
Comprehension questions in the section divided among 4 passages (one of
which is experimental) of varying lengths. The Reading Comprehension
questions are the only ones on the CAT that are not adaptive. This means
that once you are given a passage to read you can expect a set of 5 to 7
questions that will not vary in score value. Because the questions on the
Reading Comprehension passages are not adaptive, you will not see them at
the beginning of the Verbal Section, since the first 7 questions or so of either
section determine your testing level. In short, because this section is not
scored adaptively, lower and higher numerical score values are not assigned
to each question, although there is still a range of difficulty from easy to
difficult for any set of questions.

The types of passages you will find address business, science, or social
science topics. In the interest of test-fairness, so that one student will not
have an advantage over another, the 4 passages you see should each cover
one of the three subject areas listed above. The length of the passages ranges
from 250 to 450 words written according to a dense, academic style, what
we call GMAT-style. Reading the passages and answering the questions on
the CAT test is no easy task: staring at the monitor, not making notations
on the monitor, and using the mouse to scroll up and down are all
disadvantages of computerization.

The passages and questions in Reading Comprehension require a strange


combination of speed, critical reasoning, and less-than-complete
understanding. Because the CAT does not allow you to look ahead at
different passages or questions before responding to them, an awareness of
your strengths and weaknesses regarding subject topics, reading, and
vocabulary skills will help you invest your time more effectively in two
ways. One, you can spend more time on subjects you grasp well and, two,
you can use the MBA Center strategy and guessing tips to answer questions
about passages you find particularly confusing or difficult.

On the GMAT CAT, the


passages you are given come
from magazines, newspapers,
books, and academic journals.
Phineas rewrites them in his
incomparable style to make
them difficult to understand.

© The MBA Center 409


Total PrepKit for the GMAT® Step 15.1 Lesson

DIRECTIONS

If you click on the Help Button on your screen the day of the test, the
following directions will appear. Learn them now and save yourself the time
and trouble on the day of the exam.

GMAT CAT – Section 4 : Verbal Directions

End
When finished
Directions: This set of questions are reading
based on your understanding of a passage. directions
Select the best answer following the passage click on the
on the basis of what is stated or implied in the icon below
passage.

Dismiss
Directions

Test Section Answer


Time Help Confirm Next
Time
Quit Exit

As usual, the directions don't provide a great deal of information. Just


remember that you get points from the questions, not from the passages.

THE GOOD NEWS


No outside knowledge of business, science, or social sciences is required
for this section. You are being tested on your ability to understand a text,
not on general knowledge. Any familiarity you do have with a particular
subject and vocabulary is a natural advantage for you in this section.

THE BAD NEWS

The Reading Comprehension passages are written in a dense,


long-winded, and confusing manner. You are rarely given a complete
enough presentation of a topic to grasp it fully. Nor are you given enough
Identify the main idea of a
passage first. Many questions
time.
refer to it directly or indirectly,
and it holds together the
network of supporting ideas WHAT ABOUT VOCABULARY?
which run through every
passage
GMAT-style is not an easy style to read and understand within the short
amount of time you are given. The grammatical structure of the sentences
is varied and complex and makes the ideas presented difficult to
understand, especially under the pressure of testing conditions. For
vocabulary and for grammar, you can only increase your understanding
with more and consistent practice. Nevertheless, you do not need to
understand every single word in a sentence or paragraph in order to answer
a question correctly. You can guess the meaning of most words by using
context clues to help you.

410 © The MBA Center


Reading Comprehension

THE CHALLENGE

The main challenge in Reading Comprehension is time: to understand


just enough of each passage to answer the set of questions in the very There are basically two
limited time allowed. Most students use their time to read each passage and structures used for Reading
each question for full comprehension. This is a normal reading habit, but it Comprehension passages:
has no place and no time in this section – there are better uses of your time. descriptive and argumen-
tative.
Since you score points only from correct answers and not from your reading
technique (no one will know how you read each passage), this lesson is
focused on reading passages with efficient but less than full comprehension,
while still being able to answer the most questions in the time given.

© The MBA Center 411


Total PrepKit for the GMAT® Step 15.1 Lesson

AIM…

1. GMAT READING THEORIES

THE MAKING OF A READING COMPREHENSION PASSAGE

Although Reading Comprehension passages represent a variety of


subjects (business, science, and social sciences), they are all constructed in
similar ways. Each passage is an adaptation of a full-length article which is
shortened and rewritten to conform to GMAT conventions. Remember:
standardized test = standardized content. In the rewritten GMAT passage:

Transitions and expositions are minimized to make the passages more


difficult to comprehend quickly.
The four most common
types of idea development Most of the important terms and concepts are not defined and require
are: refutation, illustration, you to derive their meaning from context in order to answer the
explanation, reconciliation. questions.

The opinion, tone, or argumentative position of the author is only


suggested, rarely stated, in each passage, so your ability to infer
meaning or information is very important.

The result is a passage containing the following features:

- A main idea at the beginning of the passage

- A theme by paragraph

- 1 to 2 key concepts
Don’t try to remember every
single detail. You do need to
remember the location of - Some examples and details
details in the passage (in
roughly the middle third of a - Some quotations from authorities on a given subject
passage) for quick reference
when asked a detail question.
- A few undefined phrases, terms, or concepts

- In argumentative passages, 2 or 3 competing theories.

With such a model, Phineas will use his standard question types and set
some traps in the answer choices.

Descriptive and argumentative passages are most often structured in the


following ways:

Descriptive passages Argumentative passages

1. A main idea is presented 1. A problem is presented


2. Supporting ideas 2. First position on the problem
3. Supporting details 3. Second position on the problem
4. Conclusion (not necessary) 4. Return to the first position,
or a third position is presented

412 © The MBA Center


Reading Comprehension

2. READING GMAT PASSAGES

Key 1: Look for the main idea. Ignore the facts.

Imagine that you’re playing a memory game and someone shows you a
satellite photo of Southern California, taken from space. You have a short
time to look at the photo and then it is removed from view. You are then
asked to make a list of the identifiable objects which you were able to
recognize before the photo was removed. What will you remember? The
largest objects which stand out in the photo, like baseball diamonds, the
Pacific coast line, or perhaps a large government building will stick in your
memory. You wouldn’t try to list the houses, cars, or trees (the little details)
because they are less significant. There is little chance that you would have
the memory to list every single object in the image.
The same technique applies to Reading Comprehension passages, which
are full of details, examples, facts, secondary ideas, and varieties of tone and
style – but should be reduced to one main idea. The reason is simple: if you
try to remember every single detail, you will not understand the main idea
or purpose of the passage. You’ll remember specifics but miss the general
sense of the passage, which is what counts.

Key 2: Do not read a passage entirely.

In these passages, the key sentences which carry the main idea from one
location to another, are usually at the beginning of each paragraph. The first
few sentences of the first paragraph usually give you the main idea of the
passage, and the first few sentences of the following paragraphs restate the
idea as it was developed in the passage. An exception is the last two or three
sentences of the final paragraph which contain the conclusion of the
passage. Do not skip these last sentences: they state the results of the line of
reasoning which has been developed throughout the passage, the line of
reasoning you should follow from beginning to end, tracing the progression
of just the main idea.

Key 3: Identify the trigger words.

Once you have a good, but less than complete, idea of the content of the
passage, it’s time to identify trigger words which give you clues to shifts in
points of view and take you directly to the overall meaning, or
primary purpose, of the passage.

Triggers words introducing:

New ideas Contrasts

In fact But
Similarly Although
For example However
Besides Not only...but also
Still Some...others
Therefore Yet
For instance Conversely
In contrast to
Either...or/Neither...nor

© The MBA Center 413


Total PrepKit for the GMAT® Step 15.1 Lesson

DRILL 1

The passage which follows the original below has been condensed to
approximately one-third of its complete length to highlight only key
sentences. After reading the condensed passage, answer the questions which
follow.

ORIGINAL PASSAGE

The world’s longest fence is the 3,307 miles of unbroken


wire mesh that comprise the "Dog Fence," which runs
from off the eastern coast of Australia, near Brisbane,
to the center of the continent's southern coast,
5 ending at a remote village called Yacka. Parts of it
almost a hundred years old, the fence is in place to
protect what has become the world's second-largest
sheep population and a wool industry that earns
Australia the equivalent of five billion U.S. dollars per
10 year. Employment opportunities generated by the
wool industry and maintained by its success provide
at least two jobs for every one hundred sheep and
are viewed as critical to a country in which over 10%
of the stable population is registered for
15 unemployment benefits. The fence protects the
sheep from dingoes, Australian wild dogs, cousins to
coyotes and jackals. Dingoes are indigenous to the
Outback regions, have never been effectively
domesticated, and hunt in packs like coyotes.
20 Voracious carnivores, dingoes have been the enemy
of Australian ranchers almost since the first European
settlers arrived, and, despite the "Dog Fence," still
manage to cause several millions of dollars’ worth of
damage to the industry each year. Efforts over the
25 last 50 years to control dingo populations – they are
rapid breeders – have been unsuccessful. That the
fence is necessary to the industry is demonstrated by
the fact that when damage occurs, such as when
1992 rains caused flooding that destroyed parts of
30 the fence in many areas, the punishment inflicted by
the dingoes becomes exponentially worse. The 1992
flood was one of the most severe in Australian history
and the government provided disaster relief for
thousands of inhabitants, many of them sheep-
35 ranchers, in the areas hardest hit by the rains.
The “Dog Fence," though, has stirred controversy and
protests from conservationists, most of which are
based around the notion that animal migration,
which the fence severely restricts for many species, is
40 an essential component of the ecology.
Conservationists have long been in conflict with
sheep ranchers who devote a large portion of their
annual revenues to retain lobbyists and attorneys to
protect their commercial interests. Some
45 environmentalists warn that the landscape has
become overgrazed inside of the fence. They argue
that there is overcompetition among species for the
scarce and diminishing nutriments available inside
the fence. This is partly due to another development

414 © The MBA Center


Reading Comprehension

50 which is a problem itself: The kangaroo population


inside the fence, consisting of red and gray
kangaroos and wallabies, has exploded, as the large
marsupials, natural prey for dingoes, have had no
enemies to keep their population in check. The most
55 recent government estimates point to a kangaroo
population increase of over 50% from 1973 to 1991.
The kangaroos now also threaten the sheep
population, competing with it for water and grass.
Ranchers have noted a corresponding decrease in
60 wool quality from sheep which are undernourished as
a consequence of this competition. As a result the
Australian government has had to cull 3,000
kangaroos per year from the environment. Obviously,
another solution to this problem should be found.

CONDENSED PASSAGE

The world’s longest fence is the 3,307 miles of unbroken


wire mesh that comprise the "Dog Fence," which runs
from off the eastern coast of Australia, near Brisbane,
to the center of the continent's southern coast,
ending at a remote village called Yacka ..................
……………………………………………………
……………………………………………………
……………………………………………………
……………………………………………………
The fence is in place to protect the wool industry and
what has become the world's second largest sheep
population.
The fence protects the sheep from dingoes, Australian
wild dogs, cousins to coyotes and jackals……………
……………………………………………………
……………………………………………………
That the fence is necessary to the industry is
demonstrated by the fact that when damage occurs,
such as when 1992 rains caused flooding that
destroyed parts of the fence in many areas, the
punishment inflicted by the dingoes becomes
exponentially worse.
……………………………………………………
The "Dog Fence," though, has stirred controversy and
protests from conservationists, most of which are
based around the notion that animal migration,
which the fence severely restricts for many species, is
an essential component of the ecology…………
……………………………………………………..
……………………………………………………..
The kangaroo population inside the fence, consisting of
red and gray kangaroos and wallabies, has exploded,
as the large marsupials, natural prey for dingoes,
have had no enemies to keep their population in
check. The kangaroos now also threaten the sheep
population, competing with it for water and grass
………………………………………………..
Obviously, another solution to this problem should be
found.

© The MBA Center 415


Total PrepKit for the GMAT® Step 15.1 Lesson

01:15 GMAT CAT – Section 4 : Verbal 1 of 41


1. The passage is primarily concerned with

! (A) the problem of overpopulation of large marsupials afflicting


certain areas of eastern Australia.
! (B) the conflict between environmentalists and ranchers over
sheep-grazing areas.
! (C) the challenges the "Dog Fence" places on animal migration.
! (D) the role of economics in determining human manipulation of
an ecological system.
! (E) the positive and negative effects of the "Dog Fence."


Test Section Answer
Time Help Confirm Next
Time
Quit Exit

01:13 GMAT CAT – Section 4 : Verbal 2 of 41


2. According to the passage, the erection of the "Dog Fence"
has contributed to all the following developments EXCEPT

! (A) a population boom among wallabies.


! (B) the success of the Australian wool industry.
! (C) areas where the high concentration of sheep and kangaroos
has caused overgrazing.
! (D) several million dollars worth of damage to the Australian
wool industry.
! (E) protests from environmentalists concerning Australia's
treatment of its wildlife.

Test Section Answer ➩


Time Help Confirm Next
Time
Quit Exit

01:11 GMAT CAT – Section 4 : Verbal 3 of 41


3. The author of the passage probably mentions the 1992


flood damage in order to

! (A) highlight the unreliability of the "Dog Fence".


! (B) convince the reader of the necessity of the wool trade to
Australia's economy.
! (C) give an example of how the "Dog Fence" manages to protect
the flocks of sheep.
! (D) demonstrate the potential damage to the wool industry
prevented by the "Dog Fence".
! (E) underline the savage nature of the dingo.

Test Section Answer


Time Help Confirm Next
Time
Quit Exit

416 © The MBA Center


Reading Comprehension

01:09 GMAT CAT – Section 4 : Verbal 4 of 41


4. According to the passage, the Australian government has
had to keep the kangaroo population in check because

! (A) overgrazing has damaged the soil in southeastern Australia.


! (B) they have caused damage to the "Dog Fence".
! (C) the rate at which the species is reproducing has accelerated.
! (D) there are now more kangaroos than sheep.
! (E) large marsupials use important resources necessary to the
wool industry.


Test Section Answer
Time Help Confirm Next
Time
Quit Exit

01:07 GMAT CAT – Section 4 : Verbal 5 of 41

5. The passage provides support for which of the following ➩


statements?

! (A) The "Dog Fence" has led to a steady deterioration of


Australia's environment.
! (B) Before the "Dog Fence" was built the dingoes preyed on
kangaroos.
! (C) Floods prevented Australia's wool industry from turning a profit
in 1992.
! (D) Uncontrolled overgrazing could endanger Australia's wildlife.
! (E) The "Dog Fence" was originally built by Australian ranchers to
prevent the flocks of sheep from leaving a certain area.

Test Section Answer


Time Help Confirm Next
Time
Quit Exit

01:05 GMAT CAT – Section 4 : Verbal 6 of 41


6. Which of the following solutions to the problem, if feasible,


would the author of the passage most likely support?

! (A) The gradual removal of the kangaroo population to another


region of Australia where there are fresh grazing areas.
! (B) The construction of another fence separating the sheep from
the large marsupials.
! (C) A dismantling of the fence in order to allow the dingoes to prey
on the kangaroos.
! (D) The use of pesticides to eradicate the dingo population.
! (E) A cessation of the Australian wool industry.

Test Section Answer


Time Help Confirm Next
Time
Quit Exit

© The MBA Center 417


Total PrepKit for the GMAT® Step 15.1 Lesson

DRILL 2
For the passage below you have between two and three minutes to get an
overview. Then determine the main idea, the main terms and concepts, the
contrasts or new ideas through trigger words, and identify the location of
details.

The politically charged atmosphere of Europe in the


1930’s drastically affected the production of art,
particularly in Germany and Russia, where
government intervention with artistic works not only
5 limited what was considered socially acceptable, but
also prompted many artists themselves to suddenly
demonstrate greater interest in politics than they had
previously. The result of this interaction was a
generation of European painters and sculptors who,
10 out of either necessity or impulse, became implicated
in the historical and political fabric of their time.
In Germany, some artists responded to the rise in power
of the Nazi Party with unmitigated pessimism. Raoul
Hynckes’ 1934 “The Sponge of Bitterness” depicts a
15 rotting skull and an exposed human brain sitting
amidst symbols of Nazi power, perfectly conveying
the despair felt by many Germans. Other artists were
swayed by the Nazi cause. Emil Nolde joined the
Nazi party in the 1920’s, but was castigated in the
20 1930’s and had his work displayed in the Munich
“Degenerative Art” exhibition which publicly
announced to the population exactly which artists
were not considered exemplary. Nolde was not
allowed to continue to paint, although he continued
25 to do so in secret. Otto Dix took another tack,
retreating to the country and painting innocuous,
and bland, landscapes, eschewing his more daring
work of the previous decade. Some artists banned
from working simply fled; many of those unable to
30 were eventually sent to concentration camps.
Restrictions and stigmatizations were simultaneously
occurring in Russia, where any art not falling under
the category “realism” had become unacceptable,
and any artists practicing unacceptable work put
35 themselves at risk. The abstract painter Malevitch’s
rather creative response to this was to antedate his
works to make it appear that he wasn’t countering
Communist dogma.
Artists were summarily forced to create work for the
40 propaganda machines being produced by the
dominant political parties of both countries. These
pictures, unswervingly heroic, are generally regarded
as absurdly pompous and vacuous. They are
unfortunately matched by much of the art of protest
45 produced elsewhere in Europe in the 1930’s, which to
the modern viewer usually appears, frankly, single-
minded.

418 © The MBA Center


Reading Comprehension

2. STANDARDIZED TESTS = STANDARDIZED MISTAKES

At this point, you must be familiar with our techniques and the famous
Wrong Answer Factory, the production line of seemingly right answer
choices that trick the average test taker. Does the Wrong Answer Factory
apply here?

Let’s have a look at question 1 of the ”Dog Fence” passage.


Use of Scope
Understanding scope helps
you recognize answer choices
01:03 GMAT CAT – Section 4 : Verbal 7 of 41
that add additional infor-
mation which is neither stated
nor implied in the text, or
The passage is primarily concerned with information which distorts
the truth or actual ideas ex-
(A) the problem of overpopulation of large marsupials afflicting pressed in the text.
certain areas of eastern Australia
(B) the conflict between environmentalists and ranchers over
sheep-grazing areas
(C) the challenges the "Dog Fence" places on animal migration
(D) the role of economics in determining human manipulation of
an ecological system
(E) the positive and negative effects of the "Dog Fence"

Test Section Answer


Time Help Confirm Next
Time
Quit Exit

If you look closely at the answer choices, you’ll notice that there are
three types:
The answers to primary
purpose/main idea questions
– Totally wrong answers. This type of answer introduces new are often found in the
information that is neither stated nor implied in the passage. This type of opening sentence. Ask your-
answer choice is easy to spot and eliminate because its contents don’t relate self, “Is this the topic that
remains the focus for the
to what you’ve read in the passage. This is the case with answers (B) and (D), entire passage?”, and double-
which are completely unrelated to the passage. check the passage to make
sure.
– Almost right answers. This type of answer has some information
directly from the passage and some information which is completely
irrelevant. The elements of truth it does have are greatly distorted. Read
every word of every answer choice carefully checking for zero-distortion.
Distortion is apparent in choices (A) and (C) which are related in some way
to the passage, and mentioned as potential dangers of the “Dog Fence” but
do not constitute the main concern of this passage.

-The Best Answer. Here (E) is the best answer because it is not out of
scope, does not distort the meaning, and does include the main concern of
the passage: evaluating the advantages (1st paragraph) and the disadvantages
(2nd paragraph) of the “Dog Fence.”

© The MBA Center 419


Total PrepKit for the GMAT® Step 15.1 Lesson

Sometimes understanding 3. SCOPE


just the main idea of a pas-
sage is enough to determine
the answer to detail questions In order to make best use of the Process of Error Identification and the
without returning to the text. principles of the Wrong Answer Factory, scope should be understood.
Anything outside the contents of the passage is outside its limits, or scope.
The scope of the passage begins with its opening words and ends with its
final sentence: this describes the region you are tested on. No outside
knowledge of the subjects you’ll read about is presumed, and this should tell
you something: you are expected to stick closely to the text itself, not to
your personal experiences, to answer these questions. The four answer
choices which must be written to distract you from the one best answer are
written with distorting scope in mind. Therefore, do not read outside the
text (even the inferences required are based closely on the text) and know
that four of five answer choices will have distortions of some kind.

Understanding the scope of the passage (what is relevant to it and what


Don’t approach the questions
is not) helps you interpret the answer choices to eliminate all that is neither
as enemies.
Many students fear questions stated nor implied, all that cannot be safely inferred from a passage.
and consider them as ene-
mies. But, questions can be Let’s look a question from the “Dog Fence” passage.
used to your advantage be-
cause many of them give you
clues on where to find infor-
mation in the passage. 01:01 GMAT CAT – Section 4 : Verbal 8 of 41

The author of the passage probably mentions the 1992 flood


damage in order to

(A) highlight the unreliability of the "Dog Fence"


(B) convince the reader of the necessity of the wool trade to
You need to go back to the
Australia's economy
passage for inference ques-
(C) give an example of how the "Dog Fence" manages to protect
tions. This is one reason why
it’s not worth reading the the flocks of sheep
passage carefully the first (D) demonstrate the potential damage to the wool industry
time. prevented by the "Dog Fence"
(E) underline the savage nature of the dingo

Test Section Answer


Time Help Confirm Next
Time
Quit Exit

If you look at the last sentence of the first paragraph, you can infer that
the event in 1992 indicates that the “Dog Fence” protects sheep from
dingoes and is therefore necessary to the wool industry, which needs sheep
alive and kicking. Let’s go to the answer choices:

Choice (A) is contrary to the example and distorts the truth.

Phineas is PC. Avoid any Choice (B) is completely out of the scope. The author tries to convince
answer choices that are us of the necessity of the sheep to the wool industry, not the necessity of
politically incorrect or could
be perceived as offensive to the wool industry to the economy. This is certainly true, but not directly
any minority group. relevant to the example.

Choice (C) seems very close to the truth and is a trick. You should
wonder, “Do they really give an example? Where is the example?” Of course
there is no example. Choice (C) should be avoided.

420 © The MBA Center


Reading Comprehension

Choice (E) is certainly true in the absolute, but is not relevant to the
primary purpose of the passage.

Choice (D) is definitely the best answer to the question. The author
mentions the 1992 flood example to show that without the Dog Fence,
sheep would be lost, which means fewer resources for the wool industry.

4. QUESTION TYPES

Up to now we’ve been discussing general information and showing you


general strategies. The existence of standardized types of questions has been
hinted at, and we now arrive at a pivotal moment in this lesson: your intro-
duction to the four groups of questions which accompany RC texts. In each
group, every type of question requires a specific approach combined with
our general strategy.

THE 4 STANDARDIZED GROUPS:

A) GENERAL QUESTIONS

These questions ask you for general information on the passage. They are
written in a number of ways. The four most common look like these:

What is the primary purpose of this passage?


What is the passage’s main idea?
What is the author’s tone or attitude?
Which of the following best describes the structure of the passage?

Here is the approach for general questions:

Step 1: Get an overview of the passage.


Step 2: Determine the general idea of the passage.
Step 3: Use scope to eliminate the wrong answers.
Step 4: Eliminate answer choices that are too vague.
Step 5: Eliminate answer choices that distort the truth.

B) SPECIFIC QUESTIONS

This type of question asks you to identify a specific detail or example in


the passage and find the best statement of its meaning from the answer
choices. Specific questions can be phrased like these:

According to the passage, which of the following is true?


The example of....in line 17 is included to demonstrate...
Information in the passage suggests that which of the following...
The author uses the term...in line 37 most probably in order to...

Here is how to approach specific questions:

Step1: Return to the passage if necessary.


Step2: Determine the information required.
Step3: Use scope to eliminate the wrong answers.
Step4: Eliminate the answers referring to the wrong part of the passage.
Step5: Eliminate the answer choices that rephrase information from the text
but from irrelevant parts of the passage.
Step 6: Make sure the right answer rephrases (that is, rewrites detail
information from the passage in slightly different words) the correct
information as it appears in the passage.

© The MBA Center 421


Total PrepKit for the GMAT® Step 15.1 Lesson

C) INFERENCE QUESTIONS

This type of question asks you to make logical deductions based on the
information or argument in the passage. Inference questions are difficult
because they require a precise and thoughtful understanding of the passage.
They are often phrased like these:

Which of the following can be inferred from the passage…


The author mentions...(lines 31-33) most likely in order to...?
Which of the following can most reasonably be concluded on the basis
of information in the passage?
The passage suggests that the author believes which of the following?

Here is how to approach inference questions:

Step 1: Analyze and understand the logic of the question before going back to
the passage.
Step 2: Return to the passage to search for information, such as author’s point
of view.
Step 3: Eliminate answers that violate logic or common sense.
Step 4: Eliminate answers that distort logic or common sense.
Step 5: Use scope to eliminate remaining answer choices.

D) LOGIC-BASED QUESTIONS

In recent years, the test makers have introduced new question types
which come directly from the Critical Reasoning section. These are
logic-based questions, and you can now find strengthen, weaken, except, or
assumption questions mixed in with the other more common types of
questions in the Reading Comprehension section. Logic-based questions ask
you to evaluate a short argument based on something in the passage in
terms of the rules of logic, not in terms of your reading skills. To ace this
type of question, use the strategies in the Critical Reasoning lesson in this
book. Look closely at the AIM section for specific approaches to answering
logic-based questions. These are some of the most common forms of
logic-based questions:

Which of the following, if true, would most strengthen/weaken the


author’s assertion about...?
Which of the following would most logically be the topic of the
paragraph immediately following this passage?
Which of the following hypothetical situations best exemplifies x noted
in y location (lines 1-3)?

5. THE MBA CENTER APPROACH

Step 1: Get an overview of the passage using our reading theories.


Step 2: Determine the type of passage, the main idea, the structure, the
author’s attitude, and the location of details.
Step 3: On general questions a rapid overview of the passage is enough to
answer correctly.
Step 4: On detail or specific questions, go back to the passage to get precise
information (the line number is often contained in the question).
Step 5: Use scope to eliminate wrong answer choices.

422 © The MBA Center


Reading Comprehension

FIRE!
Now you are ready for some practice questions…

DRILL 3
DIRECTIONS: This exercise contains one passage followed by questions
on its content. Select the best response to each question based on your
understanding of the content of the passage.

In the early-twentieth century, influenza was


responsible for the deaths of millions of people
worldwide. In the spring of 1918, a pandemic of
influenza believed to have begun in the United States
5 spread quickly around the world killing more than 21
million people before it ended in 1919. The pandemic
of 1918–1919 is possibly the single most deadly
trauma ever survived by the human species.
Dubbed the Spanish flu—since Spain did not
10 participate in World War I, it was able to detail the
widespread illness of its population free of domestic
wartime censorship laws—this strain of influenza was
soon known to the rest of the world. After an initial
wave in the United States in the spring of 1918,
15 Spanish flu reappeared in a second, more intense
outbreak in August which struck hundreds of millions
of people. With no vaccine, doctors were unable to
effectively treat patients or immunize the general
public, and hundreds of thousands of patients died
20 before winter arrived. Young adults in particular were
among the victims in abnormally large numbers.
After a mild third wave struck, the pandemic subsided
in spring of 1919. By the time the pandemic finally
ended, more than 550,000 Americans had died, ten
25 times the number of American fatalities from World
War I.
From the United States, the disease spread quickly,
leaving virtually no part of the world untouched.
Regions which had never before known influenza
30 were particularly susceptible. For example, during
the last two months of 1918, 7,542 deaths were
recorded in Western Samoa, out of a total population
of 38,302. While the global mortality figure is
estimated to be in excess of 21 million, historians and
35 demographers have determined scores of unreported
deaths in Latin America, Africa, and Asia that
increased the actual mortality figure by several
million.
It is still unknown why Spanish flu proved to be
40 more lethal than other strains of influenza, or where
it originated. Medical researchers now know that
permanent immunity to influenza is impossible,
regardless of previous exposure, and that vaccines
are ineffective due to the virus’ brief incubation
45 period and its rapidity of contagion.

© The MBA Center 423


Total PrepKit for the GMAT® Step 15.1 Lesson

00:59 GMAT CAT – Section 4 : Verbal 9 of 41

The primary purpose of the passage is to

(A) evaluate the efficacy of disease prevention measures


(B) define the mortality assessment of a pandemic
(C) compare the similarities of a disease and a pandemic
(D) investigate the reasons for deaths by infectious diseases
(E) chronicle the history of a specific disease

Test Section Answer


Time Help Confirm Next
Time
Quit Exit

00:57 GMAT CAT – Section 4 : Verbal 10 of 41

According to the passage, the pandemic of 1918 - 1919 was


responsible for all of the following EXCEPT

(A) the deaths of young adults in higher amounts than earlier


contagions
(B) the development of a permanent vaccine for viral diseases
(C) more American fatalities than from World War I
(D) a record level of fatalities compared to previous well-
documented pandemics
(E) arriving first in the United States and then moving on to other
parts of the world

Test Section Answer


Time Help Confirm Next
Time
Quit Exit

424 © The MBA Center


Reading Comprehension

00:57 GMAT CAT – Section 4 : Verbal 11 of 41

The author of the passage most likely mentions the


Americans killed in World War I in order to

! (A) identify a weakness in the virulence of a contagious disease


! (B) estimate the percentage difference between American
mortality rates and worldwide mortality rates
! (C) accentuate the significance of the number of deaths in the
United States resulting from Spanish flu
! (D) provide data to support a previously established assumption
! (E) analyze the sociological impact of influenza on American
soldiers


Test Section Answer
Time Help Confirm Next
Time
Quit Exit

00:55 GMAT CAT – Section 4 : Verbal 12 of 41

According to the passage, influenza became known as ➩


Spanish flu because

! (A) censorship laws during wartime did not affect the country of
Spain which suffered greatly from the influenza pandemic
! (B) influenza afflicted Spanish citizens before it afflicted citizens
of other countries
! (C) the origin of influenza was discovered in 1919 to be from a
small region in Spain
! (D) a prominent immunological researcher from Spain was the
first to classify the properties of the disease
! (E) morbidity figures for Spain during 1918 - 1919 were higher
than anywhere else in the world

Test Section Answer


Time Help Confirm Next
Time
Quit Exit

© The MBA Center 425


Total PrepKit for the GMAT® Step 15.1 Lesson

00:53 GMAT CAT – Section 4 : Verbal 13 of 41


Which of the following can be properly inferred from the
passage?

! (A) Spanish flu led medical researchers to a permanent vaccine


for the disease.
! (B) Remote regions of the world have natural defenses against
viral outbreaks.
! (C) Self-immunity to influenza increased since 1919 as an
adaptive response to an environmental threat.
! (D) Human populations anywhere on earth can expect
reoccurrence of influenza and the possibility of widespread
contagion.
! (E) Prescription remedies are effective against the short


incubation period of influenza.

Test Section Answer


Time Help Confirm Next
Time
Quit Exit

00:51 GMAT CAT – Section 4 : Verbal 14 of 41


In the last paragraph, the author is primarily concerned with

! (A) correcting suppositional errors made by historians and


demographers in their mortality assessments
! (B) rebutting claims made by medical researchers about a known
cure for influenza
! (C) offering evidence to explain why the pandemic of 1918 - 1919
was so deadly
! (D) interpreting previously undiscovered mortality records from
Latin America, Africa, and Asia
! (E) summarizing the characteristics of influenza presented earlier
in the passage and offering updated information

Test Section Answer


Time Help Confirm Next
Time
Quit Exit

426 © The MBA Center


Reading Comprehension

DRILL 4
DIRECTIONS: This exercise contains one passage followed by questions
on its content. Select the best response to each question based on your
understanding of the content of the passage.

In order to develop a theory to explain galaxy


formation, physicists have focused their efforts
on neutrinos, weakly interacting elementary
particles that are abundant in the universe. By
5 establishing the average mass weights of
neutrinos, researchers hope to find the missing
piece of the galaxy formation puzzle. There are
currently two approaches to measure the mass
weights of neutrinos.
10 The first approach is based upon the idea of
neutrino oscillation, which hypothesizes that
neutrinos change their state unpredictably
during transmission in space. Using neutrino
sources from particle accelerators, reactors, the Sun,
15 and Earth’s atmosphere, physicists have captured,
observed, and measured neutrino oscillation in an
attempt to resolve the mystery of the apparent
shortage of neutrinos traveling from the Sun to Earth.
While early results were not conclusive enough to
20 gather widespread acceptance of neutrino oscillation
theory, a recent test at the Super-Kamionkande
laboratory in Japan has provided the most
verifiable statistical evidence to date to support
neutrino oscillation. Originating from cosmic rays
25 transmitted from outer space, neutrinos were
detected by photodetectors as they struck Earth’s
atmosphere. High-energy electrons, or muons,
emitted from the impact indicated that neutrino
oscillation occurs during every several hundred miles
30 of travel.
After evaluating thousands of muon emissions, physicists
were able to approximate an average mass weight
for neutrinos. Though more such tests must be
performed before theorists can cite a definitive body
35 of evidence, neutrino oscillation theory has become
widely accepted.
The second approach to measure neutrino mass
weights proposes the use of sophisticated satellite
technology to make cosmological measurements of
40 selected groups of neutrinos. The theory behind this
approach argues that since the universe is full of
neutrinos, their combined mass should be more than
sufficient to influence galaxy formation. Those in
favor of this approach intend to combine the most
45 accurate cosmological maps of galaxies with those of
the cosmic microwave background in order to draw
inferences about the mass weights of neutrinos above
a minimum level of 1eV. However, unlike neutrino
oscillation researchers, proponents of cosmological
50 measurements have yet to amass the data that
would support their theory of neutrino participation
in the formation of galaxies.

© The MBA Center 427


Total PrepKit for the GMAT® Step 15.1 Lesson

00:51 GMAT CAT – Section 4 : Verbal 15 of 41

In the passage, the author is primarily interested in

(A) describing an improved method for research which could


invalidate an established theory
(B) proposing an alternative to a failed research experiment
(C) analyzing the invalidity of data provided by a new scientific
test
(D) engaging in a debate about an accepted theory
(E) presenting a theory and explaining two scientific procedures
to verify that theory

Test Section Answer


Time Help Confirm Next
Time
Quit Exit

00:49 GMAT CAT – Section 4 : Verbal 16 of 41

The author of the passage would be most likely to agree with


which of the following statements about the cosmological mea-
surement approach?
(A) It is the only possible method to measure the mass of
neutrinos.
(B) It is not inferential enough to provide a statistical theory for
the mass weights of neutrinos.
(C) Its test results have been confirmed, and neutrino research
has been halted.
(D) For lack of data, it has yet to provide verifiable evidence to
support its claims.
(E) It is not a possible method to measure the mass weights of
neutrinos, although improved methods have been based on its
results.

Test Section Answer


Time Help Confirm Next
Time
Quit Exit

428 © The MBA Center


Reading Comprehension

00:47 GMAT CAT – Section 4 : Verbal 17 of 41


It can be inferred from the passage that cosmological
measurements taken from galaxy and cosmic microwave
background maps would be more useful to scientists if which of the
following were true?

! (A) The maps indicated neutrino mass weights below 1eV.


! (B) The maps were currently available to prove or disprove the
claims that neutrino mass weights above 1eV could be
determined from them.
! (C) The maps had far less specificity in reporting neutrino mass
differentiation.
! (D) The maps had indicated that heavier neutrinos reached Earth’s
atmosphere from the Sun.
! (E) The maps had been available for more than ten years, so recent
results could be compared with those from twenty years ago.


Test Section Answer
Time Help Confirm Next
Time
Quit Exit

00:45 GMAT CAT – Section 4 : Verbal 18 of 41


According to the passage, which of the following is true of the


theory of neutrino oscillation?

! (A) It provides an explanation for the abundance of neutrinos in


transmission from the Sun to the Earth.
! (B) It indicates that lighter neutrinos are more influential than
heavier neutrinos in galaxy formation.
! (C) It provides an explanantion for the lack of neutrinos in
transmission from the Sun to Earth.
! (D) It is the only theory to propose an approach to measure the
mass weights of neutrinos.
! (E) It explains neutrino oscillations at various times in the
formation of galaxies.

Test Section Answer


Time Help Confirm Next
Time
Quit Exit

© The MBA Center 429


Total PrepKit for the GMAT® Step 15.1 Lesson

00:43 GMAT CAT – Section 4 : Verbal 19 of 41

It can be inferred from the two approaches to neutrino


measurement that
(A) Different methods of testing have different methods of
interpretation which cannot be unified.
(B) More investment in satellite-imagery research must be
authorized before full-scale cosmological measurements can
be performed.
(C) Fewer photodetectors were used in prior research with the
same results as the Super-Kamionkande experiment.
(D) Accurate and verifiable measurements of neutrino mass
weights must be performed before theories proposing a
relationship between neutrinos and galaxy formation can be
confirmed.
(E) Similar neutrino experiments performed in the 1950’s made
use of measurement techniques which should be in use in
experiments today.

Test Section Answer


Time Help Confirm Next
Time
Quit Exit

00:41 GMAT CAT – Section 4 : Verbal 20 of 41

Which of the following statements about verifiable statistical


evidence is supported by information in the passage?

(A) A higher percentage of lighter neutrinos were found to


oscillate than were heavier neutrinos.
(B) Outcomes from the photodetector experiment were sorted in
three ways, which resulted in confusion about their
authenticity.
(C) A significant percentage of outcomes did not confirm neutrino
oscillation.
(D) Muon emissions from the photodetector experiment were
evaluated thousands of times, most likely in order to insure
the accuracy of the test results.
(E) By the 1950’s, unambiguous evidence existed to prove
neutrino oscillation.

Test Section Answer


Time Help Confirm Next
Time
Quit Exit

430 © The MBA Center


Reading Comprehension

IV. WHAT IF I DON'T WANT TO READ THE PASSAGE AT ALL?

Generally, completely ignoring the passage is ill-advised. Some


questions, however, can be answered without reading the passage by using
just critical reasoning abilities. The question below is an extrapolation of
this idea, not an actual Reading Comprehension question.

Try this one.

00:39 GMAT CAT – Section 4 : Verbal 21 of 41


It can be inferred from the passage that which of the following
would be one major difference in behavior between Manager X,
who uses intuition to reach his decisions, and Manager Y, who
uses only formal decision analysis?

! (A) Manager X analyzes first and then acts; Manager Y does not.
! (B) Manager X checks possible solutions to a problem by
systematic analysis; Manager Y does not.
! (C) Manager X takes action in order to arrive at the solution to a
problem; Manager Y does not.
! (D) Manager Y draws on years of hands-on experience in
creating a solution to a problem; Manager X does not.
! (E) Manager Y depends on day-to-day tactical maneuvering;
Manager X does not.

Test Section Answer


Time Help Confirm Next
Time
Quit Exit

© The MBA Center 431


Total PrepKit for the GMAT® Step 15.1 Lesson

SUMMARY

- There are from 14 to 18 questions covering 4 passages in the Reading


Comprehension section of the GMAT CAT.

- The Reading Comprehension questions are all worth the same number
of points, and this section is not adaptive.

- The Reading Comprehension passages are on the following subjects:


Science, Social Sciences, and Business. No outside knowledge is
required. You are simply asked to answer the questions based on
what is stated or implied.

- The main difficulty of this exercise is time, and your challenge is to


spend the bulk of your time understanding the passage and as
little time as possible reading the passage.

- The questions focus on only one third of the information in the


passage; the most important thing is to get the main idea in a two-
to three-minute overview.

- Remember that you earn points by answering the questions – not by


reading the passage.

- The most efficient tool you have to select the “best answer” is scope.
Understanding the scope of the passage will help you decide if an
answer choice adds information not originally in passage or if an
answer choice distorts information using a trick.

- There are four types of questions tested in Reading Comprehension


that are highly predictable. You have to anticipate these questions
when you read the passage. The four types of questions are:

- General
- Specific
- Inference
- Logic-based

432 © The MBA Center


Reading Comprehension

ANSWER KEYS

DRILL 1

1. E
2. D
3. D
4. E
5. B
6. A

DRILL 2
7. E
8. D

DRILL 3

9. E
10. B
11. C
12. A
13. D
14. E

DRILL 4
15. E
16. D
17. B
18. C
19. D
20. D

© The MBA Center 433


© The MBA Center
Reading Comprehension

HOMEWORK/PRACTICE TEST
READING COMPREHENSION
ANSWER GRID

A B C D E
1 ! ! ! ! !
2 ! ! ! ! !
3 ! ! ! ! !
4 ! ! ! ! !
5 ! ! ! ! !
6 ! ! ! ! !
7 ! ! ! ! !
8 ! ! ! ! !
9 ! ! ! ! !
10 ! ! ! ! !
11 ! ! ! ! !
12 ! ! ! ! !
13 ! ! ! ! !
14 ! ! ! ! !
15 ! ! ! ! !
16 ! ! ! ! !
17 ! ! ! ! !
18 ! ! ! ! !

© The MBA Center 435


Total PrepKit for the GMAT® Step 15.2 Homework

Reading Comprehension – Homework Directions

End


When finished
18 QUESTIONS reading
25 MINUTES directions
click on the
Directions: This section consists of icon below
passages followed by questions on their
content. Select the best response to each
question based on your understanding of the
content of the passage.
Dismiss
Directions


Test Section Answer
Time Help Confirm Next
Time
Quit Exit

436 © The MBA Center


Reading Comprehension

Many of New York City’s post-1890 laws 1. Which of the following best represents the main
governing tenement houses passed as a direct theme of the passage?
result of the campaign of Jacob Riis, a Danish-born
writer and photographer. Riis’ work, published in (A) Housing conditions in New York in the 1890’s
5 the New York Tribune and the Evening Sun, spurred reforms governing the construction of
affected the way the entire populace of New York new buildings.
City, at least the literate populace, viewed victims (B) Until Jacob Riis, no journalist had made an
of poverty. Previously, many newspaper readers attempt to adequately capture slum life for a
had assumed that the poor were so by choice and newspaper’s readership.
10 the unhygienic conditions in which they lived (C) Jacob Riis was a tireless social reformer and
resulted from moral failure rather than champion of the rights of the downtrodden.
economic reality. Slums had been viewed as (D) Jacob Riis was among the first to postulate that
either places to be shunned or, patronizingly, as “double-decker” housing was exponentially
exotic locales where intellectuals and tourists could more hazardous than single-building dwellings.
15 “slum” for the day before returning to their high- (E) The activities of Jacob Riis awakened many New
rent neighborhoods and rhapsodizing on the Yorkers to the horrendous living conditions of
enviable happiness of the simple lives of the the city’s poor.
poverty-stricken.
Riis changed the popular outlook by
20 sympathetically portraying the urban poor in his 2. The passage suggests which of the following
writings and photographs, and, furthermore, about the “intellectuals and tourists” mentioned in line
calling attention to the squalor and danger of 14?
tenement habitation. Among the hazards Riis
depicted were structural deficiencies, which (A) Their appreciation of the slums was based on
25 regularly caused the actual collapse of parts of the contentment of its denizens.
buildings, and conducivity to fires. Both of these (B) Their attitude toward the poor was not
problems resulted from construction with an predicated on a thorough examination of
emphasis on haste and using cheap materials. The factual or empirical evidence.
subjects of Riis’ work were also subject to disease, (C) Regular visits produced in them an intense
30 often produced by a negligible water supply and familiarity with the poor sections of New York.
close proximity to unsanitary activities such as (D) They envied the living conditions of the poor.
animal slaughtering and bone-boiling, and easily (E) They were fully aware of the hazardous
spread by the incredibly overcrowded conditions. circumstances in which poor people lived and
Riis was also responsible for exposing the considered these conditions reasons for the
35 differences between single tenements and what happiness of those living in the slums.
were known as “double-deckers,” which were
tenements consisting of two dwellings, front and
rear, in which all the problems afflicting tenement
life were greatly multiplied. A survey of an
40 area in lower Manhattan Riis cited in the 1890’s
gave the mortality rate at nearly 30 per thousand
for those living in single buildings and over 60 for
those in tenements with two structures, and an
equally drastic divergence in the infant mortality
45 rate: about 100 for the former, double that for the
latter.
Eventually, Riis took a more active role, arguing
for and actually supervising the demolition of
many of the culpable tenement houses, and
50 publicly supporting the licensing and rent control
of all lower-class housing.

GO ON TO THE NEXT PAGE


© The MBA Center 437
Total PrepKit for the GMAT® Step 15.2 Homework

3. From the information given in the passage, 5. Which of the following best describes the
which of the following can most likely be inferred structure of the second paragraph of the passage?
about Jacob Riis?
(A) A description of the methods used to promote
(A) His photographs of New York’s poor are of social reform, and some reasons those changes
particular historical interest. were necessary
(B) He was responsible for the prosecution of (B) A close analysis of a political problem and its
landlords caught violating laws concerning development
tenement ownership. (C) A tribute to a social reformer, followed by a
(C) His campaign to open the eyes of New Yorkers systematic appraisal of a social phenomenon
to the horrors of slum life was spurred by his (D) A portrayal of the use of popular media for
immigrant background. political purposes, followed by the results of
(D) He developed a close relationship with many such actions
New Yorkers who lived in squalid conditions. (E) A depiction of contrasting views of a social
(E) After establishing a foundation of support problem, one held by the general populace, the
among newspaper readers, he began to other by an individual crusader
become active politically, furthering the cause
of housing reform.
6. The author’s claim that Jacob Riis was directly
responsible for the reformation of New York’s housing
4. Which of the following is NOT mentioned in the laws would be most effectively strengthened if the
passage as a hazard of tenement houses of which Riis author
made the general populace aware?
(A) related the dates and provisions of the laws to
(A) The likelihood of fires Riis’ campaign
(B) The possibility of disease being spread due to (B) described the arguments for and against the
residents’ inability to maintain normal levels of passing of each law
hygienic care, resulting from the small amount (C) provided more statistics concerning the hazards
of water available of tenement living
(C) The unsanitary proximity to disease-ridden (D) explained in greater detail how readers of Riis’
animal carcasses work responded to their newfound sense of the
(D) The high rate of crime resulting from incredible dangers of tenement dwelling
overcrowding (E) gave specific examples of poor people living in
(E) The high rate of accidents caused by buildings New York tenements in the 1890s who were
constructed from inappropriate materials unable to relocate due to financial or social
constraints

GO ON TO THE NEXT PAGE


438 © The MBA Center
Reading Comprehension

Primatologists did not know of the existence of 55 societal rules are unusual among primates and
the bonobo, a species of great ape, until 1928, unique among great apes.
when researchers noticed that the primate long
known as the pygmy chimp was actually its own
5 species. The discovery of this close relative to man
and the ensuing study of its behavior have forced 7. The primary purpose of this passage is to
scientists to rethink some of their traditional beliefs
about male-female relationships in great ape (A) present two conflicting theories about male-
species. Violent acts committed by males have female relations in great apes
10 been witnessed in many ape species, such as (B) dispute early evidence that male great apes
infanticide by male gorillas, rape by male often engage in violent behavior
orangutans if their courtship is rebuffed by (C) explain the origin of bonobo behavior
females, and bloody territorial wars among rival (D) discuss the unusual nature of male-female
chimpanzee populations. Recent research, relations in bonobo society
15 however, shows that males do not partake in any (E) portray bonobo group behavior as more socially
of these types of behavior in bonobo society. advanced than that of other great apes
The reason for the relative tranquillity in
bonobo populations, according to scientists, is
that the punishment of transgressors, the keeping 8. The author implies that male-male violent
20 of peace, and the setting of behavioral limits are contests are limited in bonobo society because
enforced by females. Females will sound a distress
signal if, for example, a male tries to hoard fruit at (A) alpha males can sometimes be dominated by
a feeding site and prevent others from females in non-sexual settings
approaching. Males can sometimes dominate (B) female alliances ensure that transgressors are
25 females, but only those who rank lower than them punished
and a female just as easily dominates a lower- (C) female distress signals will alert other members
ranked male. This is in contrast to chimpanzee of the population
populations in which all females are subordinate (D) there is a plentiful food supply allowing for
to even the lowest ranked males. Bonobo females, stable group living
30 who are powerful enough to resist even the (E) the winner would be unable to secure breeding
strongest alpha male, are thus able to ensure that rights and thus has no motivation
aggressive misbehavior, either in sexual or non-
sexual settings, is not permitted. Even male-male
aggression is limited, though only indirectly due to 9. According to the passage, chimpanzees differ
35 the vigilance of females. Violent contests between from bonobos in that
males secure breeding rights in most ape species,
but as a dominant male bonobo cannot force an (A) female chimpanzees are always subordinate to
unwilling female to become a mating partner, males while male bonobos are always
there is little to be gained by such contests. subordinate to females
40 Scientists are uncertain how such female- (B) male chimpanzees forage in small groups and
enforced pacifism evolved, but the key may lie in bonobos forage individually
the bonobo food supply. Bonobos are adapted to (C) chimpanzees compete violently for both
both a diet of leaves and stems, plentiful on jungle territorial and mating rights, while bonobos
floors, and meat and ripe fruit, not always easily compete only for territorial rights
45 come by. Unlike chimpanzee communities, for (D) bonobo females use distress signals whereas
example, which split up into foraging parties of chimpanzee females do not
varying sizes to find meat and fruit, bonobo (E) females can outrank males among bonobos but
societies are able to form more or less permanent not among chimpanzees
groups, providing the chance for females to build
50 alliances, ensuring that males don’t abuse their
greater size. While female alliances were not
unknown in primate species before behavioral
studies of the bonobo, the absence of male
domination and the female enforcement of

GO ON TO THE NEXT PAGE


© The MBA Center 439
Total PrepKit for the GMAT® Step 15.2 Homework

The following text was written in 1997.


10. The author mentions gorilla infanticide in line With less than two years remaining before the
11 in order to introduction of the new European common
currency, dubbed the “euro,” businesses and
(A) contrast behavior of other great apes with the governments of the European Community are
relative tranquillity of bonobos 5 preparing for sweeping changes in the way that
(B) emphasize that gorillas are unusually violent for business will be conducted both nationally and
a great ape species internationally among the member states. During
(C) demonstrate that except in the case of the the three-year transition period, scheduled to last
bonobo, infanticide is common in most ape from 1999 to mid-2002, the euro will coexist with
species 10 national currencies; afterwards, the national
(D) suggest that some female gorillas cannot currencies will be phased out, leaving the euro as
properly care for their young the single European currency. The transition to the
(E) make a comparison between gorillas and common currency will be a difficult one, not only
orangutans due to its high cost to governments (which will
15 have to print millions of new banknotes) and to
businesses (which will need to completely revise
11. Which of the following situations most closely their accounting practices and cash handling
parallels the primatologists’ discovery of the bonobo systems) but also because many businesses will
species described in lines 1-5? find that they cannot compete in the new, more
20 open, European market. This openness, however,
(A) A judge makes a ruling making a certain action will be a boon for many business and particularly
legal for the first time. for consumers, who will benefit from a wider
(B) A team of astronomers devises a new method choice of products and services. Additionally, the
to measure the velocity of comets. new single currency will allow both businesses and
(C) Archeologists discover artifacts from a 25 individual consumers to save billions of dollars
previously unknown city. annually in transaction costs that are associated
(D) A doctor finds that what was previously with the current multi-currency system.
considered a variant of a common disease is One factor that has been largely ignored by
actually a separate disease. businesses and governments alike is the way that
(E) A stamp collector realizes that one of his more 30 consumers will react to this switch to the common
unusual stamps is a worthless counterfeit. currency, which will be imposed on them largely
without their input and, in many cases, without
their even understanding the reasons for this
12. The third paragraph, in relation to the rest of change. People may be resistant to accept the
the text, could most accurately be described as 35 euro for several reasons: an attachment to their
national currency as a symbol of their homeland,
(A) a discussion of the consequences of ideas the potential mistrust of a currency which is not
expressed in the second paragraph controlled by their own national bank, or a feeling
(B) a counter-theory to a theory put forth in the of complete disorientation when faced with the
first two paragraphs 40 conversion to a new standard. When the French
(C) a theoretical model which explains data from supermarket chain E. Leclerc decided, as an
the second paragraph experiment, to print several million dollars worth
(D) a proposed explanation for observations made of “euro” bills for use in its stores, it discovered
in the first two paragraphs that the majority of consumers were simply
(E) a reiteration of themes expressed earlier in the 45 confused by the new currency and preferred
passage paying with bank cards.

GO ON TO THE NEXT PAGE


440 © The MBA Center
Reading Comprehension

13. The passage is primarily concerned with 16. It can be inferred from the passage that the
author believes which of the following statements?
(A) the difficulties of changing from one currency to
another (A) The three-year transition period currently
(B) the three-year process of conversion to the euro scheduled for the conversion to the euro should
(C) the importance of a national currency for be considerably lengthened.
consumer confidence (B) The conversion to a single currency will have
(D) why the euro will be good for European some effect on competitive pressures on the
business European common market.
(E) the advantages and problems of the conversion (C) Many companies for whom the commission on
to the euro currency transactions is a substantial part of
their income will no longer be profitable after
the introduction of the euro.
14. The word "boon" used in line 21 is most accurately (D) If people educated themselves about the euro
described by which of the following? before its introduction in 1999, the transition
period would be made substantially easier.
(A) help (E) The number of new businesses that will be
(B) challenge created after the introduction of the euro will be
(C) mild difficulty greater than the number of businesses that will
(D) advantage be forced to cease their operations because of
(E) necessity the introduction of the euro.

15. It can be inferred from the second paragraph 17. Which of the following is NOT cited as a
that drawback to the conversion to the euro?

(A) having a currency controlled by a national bank (A) The need for businesses to substantially modify
gives some consumers a sense of security their accounting systems
(B) consumers often prefer paying with bank cards (B) The difficulties that European consumers may
than with cash have in adapting to the new currency
(C) many consumers do not yet know about the (C) The cost of printing and minting new euro bills
conversion to the euro in 1999 and coins
(D) The euro should not be introduced without a (D) The fact that bank cards will have to be adapted
concurrent public information campaign to operate with the new currency
(E) The introduction of the euro will entail less (E) The fact that many companies may have
national sovereignty for the member states of trouble surviving greater international compe-
the European Union tition

18. The author’s attitude or tone in regard to the


process of converting to the common currency is best
described as one of

(A) wholehearted support


(B) moderate criticism
(C) objective analysis
(D) indifference
(E) mild distaste

STOP
IF YOU FINISH BEFORE TIME IS EXPIRED
YOU MAY CHECK YOUR WORK

© The MBA Center 441


Total PrepKit for the GMAT® Step 15.2 Homework

READING COMPREHENSION
HOMEWORK/PRACTICE TEST
ANSWER KEY

1. E
2. B
3. E
4. D
5. A
6. A
7. D
8. E
9. E
10. A
11. D
12. D
13. E
14. D
15. A
16. B
17. D
18. B

442 © The MBA Center


Reading Comprehension

EXPLANATIONS FOR that the intellectuals' opinions that the lives of the
READING COMPREHENSION poor were “enviable” is probably inaccurate,
HOMEWORK/PRACTICE TEST according to the author. Thus the closest answer is
(B), which basically says that their attitudes were not
based on fact. (A) and (D) look at the intellectuals'
point of view, not the author's (remember, the
PASSAGE 1 question asks about what the passage suggests). (C)
misses the point that the intellectuals' visits were
The first sentence gives you an idea that this social superficial. (E), besides not being suggested by the
science passage is about the campaign of Jacob passage, could be considered offensive to the poor
Riis and its effect on government tenement and would never be credited by the GMAC.
housing laws. The rest of the first paragraph then
discusses how Riis' work affected people's views on
the poor.
Paragraph 2: Remember to pay attention to first and 3. (E)
last sentences. What did Riis do? He “changed the It is possible to infer (E) from the first sentence of
popular outlook.” How? “By sympathetically the passage and the last paragraph, which mention
portraying the urban poor in his writings and Riis's effect on the cause of housing reform. (B) is a
photographs.” The rest of the paragraph is details detail that we cannot infer – it is mere speculation;
on the poor conditions of tenements. landlords violating laws are not mentioned (if you
The last, short paragraph briefly alludes to Riis' later search the passage for “landlord” you come up empty).
activities. (C) is also pure speculation. The fact that Riis himself
That's enough to get out of a first read in 2 minutes. On was an immigrant does not tell us that his background
to the questions. provided his motivation. (D) is also a wild guess. Just
because the passage says that his photographs and
newspaper articles sympathetically portrayed the poor
doesn't mean he was close friends with those people.
1. (E) (A) is pretty close, but his photographs are just
The first question is a general one. The key word is mentioned briefly as, together with his writings,
“main” theme. The whole passage talks about Riis, so having “changed the popular outlook.” It goes too far to
(A), which doesn't even mention Riis, is out. (B) is a then infer that they are of particular historical interest.
sweeping generalization about the entire history of
journalism – there's no support in the passage for such
a strong statement. (C) might sound pretty good, but
it's too general – GMAT passages never have such 4. (D)
broad themes, and what's more, who's to say that Riis Crime is never mentioned in the passage.
was tireless? (D) on the other hand, is far too specific. Remember to go back to paragraph 2, which focused
While Riis did expose the dangers of “double decker” on the details of housing conditions.
housing, this choice is a detail mentioned in only one
paragraph. (E) covers all the paragraphs. The word
“activities” is broad enough to refer to his
photographs, journalism, and political or social 5. (A)
activities, while “the horrendous living conditions of the For an organization question like this, go back to
city's poor” is specific enough to stay in the scope of the paragraph, read the first sentence, then quickly
the passage. (Compare that to the broad language in skim the details. Predict an answer before looking at
(C): “the rights of the downtrodden.”) the choices. In this paragraph, we see that “Riis
changed the popular outlook” by doing some things.
Then there are details about housing conditions. So
the right answer should say something about bringing
2. (B) about change and why this was necessary. (A) is the
A question that gives you a line number is great, closest. (B) is wrong because there is not a political
but make sure you read a couple lines below and problem or a “development.” (C) is wrong because this
above the line cited to find the answer. Then try to is not a “tribute.” Watch out for making judgments
predict the answer before going to the choices. The about the author or the subject – you may admire Riis,
word “patronizingly,” a distinctly negative word, tells but that doesn't make this passage a tribute to him.
us that the author probably doesn't approve of the (D) is wrong because the paragraph doesn't talk about
intellectuals' attitudes. And because most of the results. (E) is wrong because there are no contrasting
passage is about how Riis opened people's eyes to the views.
terrible living conditions of the poor, we can conclude

© The MBA Center 443


Total PrepKit for the GMAT® Step 15.2 Homework

6. (A) aggression. (D) simply comes from the third


This is basically a Critical Reasoning question stuck paragraph. With detail and inference questions, you
into the Reading Comprehension section. The claim in usually need to locate the part of the passage that is
choice (A), made in the first sentence, is not strongly referred to – don't guess based on what just seems
backed up in the passage, primarily because, in order right.
to establish a link between Riis and the reformation of
housing laws, we need to know what the laws say and
when they were passed. Then we would be better able
to assess whether they are related to Riis's efforts. (C), 9. (E)
(D), and (E) are not related to laws, and so can't be The answer to this question is in paragraph 2. Note
right. (B) is not right because even if we knew the that the passage uses the words “this is in contrast to.”
arguments for and against the passing of the laws, it is Comparisons in GMAT passages almost always show
likely that we still wouldn't know whether it was Riis's up in the questions and are therefore important to
efforts that sparked the debate or affected the note when you skim the passage. You don't have to
outcome. remember the details of the comparison the first time,
just note that there is one and where it is in the
passage. (A) goes too far – male bonobos are not
always subordinate to females. (B) is tempting, but the
PASSAGE 2 passage does not state that bonobos forage
individually, only that they don't “split into foraging
parties of varying sizes.” (C) is outside the scope. (D) is
The broad topic of this passage is the bonobo, a species incorrect because the passage does not state whether
of great ape, but GMAT passages always have chimpanzee females use distress signals.
more specific topics. In this case, the first
paragraph leads us into the topic which is then
clearly stated in the first sentence of the second
paragraph. The topic is the female-enforced 10. (A)
pacifism in bonobo populations. The first sentence This is a logic question. Why does the author
of each paragraph reveals the structure. The first mention gorilla infanticide? Go back to the line
paragraph is a scientific discovery (that bonobos numbers and remember to read a little bit before and
are their own species), followed by some obser- after the lines the question asks about. The line before
vations. The second paragraph describes the (8) states that violent acts are committed in many ape
mechanism of bonobo tranquillity, and the third species. The sentence afterwards states that bonobo
suggests an explanation for the mechanism. males are an exception. Therefore the motivation
behind the mention of gorilla infanticide is to compare
violent acts in most great ape species with the tranquil
behavior of bonobos, or (A). (B) is a distortion--gorillas
7. (D) are not unusually violent among great apes, but
The unusual nature of male-female relations in typically violent. (C) is also a distortion. The passage
bonobo society is discussed throughout the first only mentions that gorillas commit infanticide. It says
passage. (A) does not even mention bonobos, and nothing about other ape species committing
there are no “conflicting theories” in the passage. (B) is infanticide. (D) is a value judgment with no basis in
wrong because the author did not write the passage the passage. (E) doesn't mention bonobos, the focus
to “dispute” something. (C) is a typical wrong answer of the entire passage, including this comparison.
to a primary purpose question, as it relates to only one
paragraph (the third). (E) is a value judgment that is
not justified by the passage (even if one happens to
agree with it). 11. (D)
Here it is necessary to draw a parallel between the
situation in the passage and a hypothetical one in the
answer choices. Go back to the lines to which the
8. (E) questions refer. The primatologists discover that what
You should remember to go back to the second they had thought was a kind of chimpanzee (a pygmy
paragraph for this, as that is where the author chimp) was actually its own species. (D) is the same
discusses the details of female-enforced pacifism. The idea but with doctors and a disease substituted for
right answer is a paraphrase of the last sentence. (A) is primatologists and bonobos. (A) is wrong because a
a distortion of a detail from another part of the ruling is arbitrary – it is not a discovery. There is no
paragraph. (B) is a true statement, according to the new method in the passage, making (B) wrong. The
passage, but does not specifically relate to male-male primatologists did not discover remains (or artifacts),

444 © The MBA Center


Reading Comprehension

nor were bonobos previously unknown. (E) sounds 13. (E)


like a value judgment – are bonobos “worthless The passage discusses both benefits and problems
counterfeits”? of the conversion to the euro, so (E) is the best choice.
(A) does not mention the euro, and the passage does
One way to tackle this type of question is to take not only focus on difficulties. (B) is too narrow. The
the choices and substitute in appropriate words from passage is not just focused on the 3 year conversion
the passage. In this case, using “primatologists” and process. (C) is a small detail, not the primary purpose,
“species,” (D) emerges as correct because “Primatolo- and it does not mention the euro. (D) ignores the
gists find that what was previously considered a variant benefits of the euro, and it ignores the consumer (the
of a common species is actually a separate species,” passage is not only focused on businesses).
describes exactly the discovery in lines 1-5.

14. (D)
12. (D) You don't have to know the meaning of the word
The first sentence of the paragraph gives it away: “boon” – you can figure out the answer from the
“the key may lie in…” The “may” indicates a proposed passage. “This openness… will be a boon… for
explanation. It is definitely not a discussion of the consumers, who will benefit…” Before looking at the
consequences of ideas from the second paragraph, so answer choices, you should have your own word in
eliminate (B); rather, it is a possible explanation, or a mind (based on the idea of benefiting). The closest
discussion of the causes. There is no “theoretical choice to the words “who will benefit” is (D),
model” in paragraph 3, nor is there any “data” in “advantage.” (A) is perhaps tempting, but “help” and
paragraph 2. These terms are too precise: out goes “advantage” are not the same. The consumers will
(C). (E) might sound tempting, but remember that benefit, but we don't know whether the euro will help
the question asks how the paragraph could “most them do anything. None of the other choices are
accurately” be described. A reiteration is not enough, positive, and so they cannot be right.
because there is an attempt to explain bonobo
behavior, even if the explanation isn't very thorough
or clear.
15. (A)
Note that the word “some” makes the choice more
attractive, as it is generally easier to defend or infer a
PASSAGE 3 statement about “some” things than about “most”
things or “all” things. The right answer refers
specifically to the line in the passage “the potential
As usual, you can quickly identify the broad topic: the mistrust of a currency not controlled by their own
“euro.” Also as usual, the passage has a more national bank…” Choice (B) is a distortion of the last
specific focus, though in this passage it is a bit sentence. People preferred bank cards in this case
difficult to spot. Look for key words and phrases to because they were confused by the new currency, not
make some order out of the passage, such as, in because they don't like cash. (C) is an exaggeration, or
paragraph 1, “transition period,” “afterwards,” distortion. People may not fully understand the euro,
“difficult one, not only due to … but also but that doesn't mean they don't know about it. The
because,” “however,” “additionally.” This shows “concurrent public information campaign” in (D) might
the structure and content. The paragraph is about sound like a good idea, but that doesn't mean it can
the introduction of the euro, the transition period, be inferred that the author deems it necessary. This
some possible results, why the introduction of the choice is a typical trap. Generally, in Reading
euro will be difficult, and why it might also be Comprehension stay away from choices that make an
beneficial (“however”). inference about what policy the author thinks should
be followed. (E) just isn't mentioned in the paragraph.
In paragraph 2, some key words are “one factor,”
“largely ignored,” “resistant… for several reasons,” and
“an experiment.” These indicate more problems with
the euro and a critical attitude on the part of the
author.

© The MBA Center 445


Total PrepKit for the GMAT® Step 15.2 Homework

16. (B)
This is a hard question because it provides no clue
of where to find the answer in the passage. The word
“some” in (B) makes it a more attractive choice (cf.
explanation quest. 15). In the first paragraph, the
author writes “many businesses will find that they
cannot compete in the new, more open European
market.” This is quite similar to (B). (A) is a distortion.
We cannot guess whether or not the author thinks
that the conversion period should be lengthened (at
least not on the GMAT, which is very picky about
staying within the scope). (C) is a prediction that isn't
justified – companies might become less profitable;
nowhere in the passage is it asserted that they won't
be profitable at all. (D) is a statement that sounds true,
but it is uncertain whether the author believes the
transition would be made “substantially” easier.
Moreover, the author seems to be sympathetic with
the consumer, and this choice sounds judgmental,
and many of the reasons the author cites for
difficulties people will face probably wouldn't change
even if people educated themselves about the euro.
(E) is too specific. Who would make such a precise
prediction about the future? Not a GMAT author.

17. (D)
Problems with bank cards are just not mentioned –
the only thing said about bank cards is that some
people used them instead of paying with cash
because they were confused by the euro. There is no
indication that bank cards will have to be adapted.

18. (B)
The author certainly does not have an attitude of
“wholehearted support,” as most of the passage
addresses problems with the transition to the euro.
(C), “Objective analysis,” might sound tempting, but
the second paragraph is clearly negative, as is part of
the first. The choice of words, such as “imposed on
them largely without their input” indicate that the
author is critical. “Indifference,” in (D), is wrong for the
same reason. “Mild distaste” is negative, but what
does “distaste” mean?

446 © The MBA Center


Step 16
Reading
Comprehension
Workshop

© The MBA Center


Total PrepKit for the GMAT® Step 16 Workshops

00:00 Reading Comprehension – Workshop Directions

End


When finished
TIME – 25 MINUTES reading
18 QUESTIONS directions
click on the
Directions: This section consists of icon below
passages followed by questions on their
content. Select the best response to each
question based on your understanding of the
content of the passage.
Dismiss
Directions


Test Section Answer
Time Help Confirm Next
Time
Quit Exit

448 © The MBA Center


Reading Comprehension

EASY
the Beat movement influenced the writings of
Baraka
The avant-garde movement of the 1950’s,
including the generation of poets and writers that
came to be known as the "Beats," was a 2. According to the passage, a contradiction of
movement that, in general, avoided specific Beat poetry is that
5 political commentary in favor of an emphasis on
intense self-expression. So it would seem ironic (A) while it attempted to maintain a distance from
that from this movement emerged one of the politics, its experimental form and content
most uncompromising and strident nationalistic expressed a dissatisfaction with the norms of
voices of African-American literature. Yet this American society
10 contradiction and others are precisely what (B) although it was a mainstream movement, it
contributed to the development of the original enabled African-American writers to find a
vision of Afrocentric radicalism expressed in suitable voice
the work of Amiri Baraka. (C) it influenced writers of diverse ethnic origins
That the Beats influenced Baraka is although its practitioners were mostly white
15 indisputable. Baraka recognized that their (D) it strove to re-create poetic form while adhering
experiments in form and structure and their to literary tradition
deviations from what had been the widely (E) it managed a widespread appeal despite
accepted subjects for poems placed them outside content that had been previously considered
the cultural mainstream and denoted a critique of unsuitable for poetry
20 the traditional depictions of American society as
inherently humanitarian and just. He saw in their
technique the possibility of creating a new rhetoric 3. The passage provides information to answer
that helped enable him to reject the common which of the following questions?
course of African-American poets who, in
25 attempts to chronicle the experience of an (A) How did Baraka’s poetic technique diverge
oppressed group, had nonetheless utilized from traditional approaches?
traditional poetic expression. Using the technical (B) How did Baraka reconcile his Afrocentric
model of the Beats, Baraka established his own inclinations with his immersion in a movement
poetic dialectic, one which allowed him to dominated by white poets?
30 demonstrate his repudiation of the notion that (C) How can an outspoken voice such as Amiri
African-American writing can express the main Baraka be considered, if only tangentially,
concerns of the African-American experience while associated with an ostensibly apolitical
simultaneously attempting to establish a movement?
complicity between a marginalized poet and a (D) What forces contributed to Amiri Baraka’s
35 mainstream readership. decision to eschew the traditions of African-
American poetry?
(E) How did the writings of Amiri Baraka differ from
1. The primary purpose of the passage is to those of contemporaries in the Beat movement
of the 1950’s?
(A) explain how Baraka manipulated the traditions
of western poetry in order to create a new form
that would express his revolutionary ideas
(B) demonstrate that, despite his ambivalent
relationship toward his literary forebears,
Baraka was firmly enmeshed in a tradition
(C) depict the influence of the Beat movement on
mainstream American culture and how that, in
turn, affected African-American poetry
(D) discuss the inherent contradictions in the
position of an African-American poet
(E) underline how, despite superficial dissimilarities,

GO ON TO THE NEXT PAGE


© The MBA Center 449
Total PrepKit for the GMAT® Step 16 Workshops

MEDIUM
4. The tone of the passage suggests that the author
most likely has what attitude toward Baraka’s For the past five years, the United States
predecessors in African-American poetry? government has spent over $2 billion per year to
fund the Human Genome Project (HGP), whose
(A) Their failure to establish a literary form distinct goal is to create a comprehensive "map" of every
from that of mainstream American poetry 5 human gene. The supporters of the HGP justify this
limited their ability to create an idiom relevant expenditure by pointing out the numerous
to revolutionary African-American politics successes that genetic research has had in
(B) They successfully documented the African- detecting and curing many serious diseases.
American experience in a poetic form accessible Martingale’s Syndrome is, in their view, a case in
to readers of various backgrounds. 10 point. Genetic researchers have discovered that this
(C) They wanted to impose the standards of debilitating disease is caused by a point mutation
traditional poetic forms on all practitioners of of a gene on the X chromosome, which leads to the
poetry. deformation of nerve cells in the brain. Since the
(D) They were marginalized by a mainstream early 1990’s those who might be carriers of the
literary public that failed to accept experiments 15 defective gene, but who do not suffer from the
in poetic content. disease, could have themselves tested in order to
(E) They altered the course of American poetry. avoid unknowingly passing the bad gene to a child
and risking that the child will contract the disease.
Recently, scientists have even been able to cure
5. It can be inferred from the passage that Baraka 20 Martingale’s Syndrome by injecting a "good" copy
of the gene into the fetus while it is still in the
(A) believed African-Americans should assimilate womb. Bolstered by their success against such
with American society diseases, scientists speculate that a substantial
(B) considered poetry the most effective vehicle for number of human illnesses have a genetic basis,
self-expression 25 and that once a complete genetic map is created,
(C) rejected the influence of his white counterparts they will be able to cure most of them by this same
in the avant-garde method of genetic manipulation.
(D) desired to create a poetic form with appeal to Certain critics, however, have appropriately begun
all readers regardless of their ethnic to question whether committing such a large
background 30 percentage of the U.S. research budget to the HGP
(E) thought traditional Western literary forms is the best thing to do. While admitting the
inadequate for expressing radical African- usefulness of mapping the human genome in
American attitudes order to detect and correct genetic anomalies,
these critics have pointed out that there are many
35 other factors involved in genetically-based diseases
6. The author uses the word "complicity" (line # 34) aside from the defective genes themselves, and that
in order to too much focus on mapping genes could prevent us
from finding potentially more useful ways of
(A) question African-American poets’ commitment treating such diseases. Often, the most significant
to their community 40 factor in contracting a genetically-based disease is
(B) emphasize the revolutionary aspect of Baraka’s not the defective gene, but the other factors that
work control whether the gene is active or inactive.
(C) demonstrate the direct correlation between If the gene is activated by any of a number of
Baraka and the Beats factors, ranging from hormonal to environmental
(D) critique the ability of traditional poetic form to 45 factors, the disease will manifest itself; if the gene
represent the African-American experience remains inactive, the disease will not. There may
(E) suggest that African-American poets conspired even be some genes which are defective in all
to appeal to the mainstream literary public humans, but which have been only activated in
certain people. In such a case, the discovery of the
50 factors which control the activation of this gene
would be a much more promising avenue of
research than the investigation of the gene itself.

GO ON TO THE NEXT PAGE


450 © The MBA Center
Reading Comprehension

7. This passage is primarily concerned with 10. Which of the following can be inferred from the
passage?
(A) the value of genetic research
(B) the potential benefits of the HGP relative to the (A) Most diseases will one day be curable by
money and time that it requires genetic manipulation.
(C) the number of diseases that might potentially (B) The effects of a defective gene can be
be cured by genetic manipulation counteracted by a good copy of the same
(D) what kinds of factors determine whether genes gene.
are active or inactive (C) With half of its current budget, the HGP could
(E) the soundness of the theoretical basis of the HGP still be completed in 5 years.
(D) The reasons that the budget for the HGP is so
high are essentially political and not scientific.
8. The critics of the HGP call into question (E) A complete genetic map is the most useful tool
we can create to help detect and cure
(A) whether the U.S. government should fund the genetically based diseases.
HGP
(B) whether or not the scientists who advocate
funding the HGP are acting in the best interests 11. According to the passage, which of the
of scientific research following is true of Martingale’s Syndrome?
(C) whether a cure for Martingale’s Syndrome
would have been found without genetic (A) Scientists have known about the disease only
research since the early 1990’s.
(D) whether the HGP is important enough to justify (B) Scientists have known that the disease was due
increased taxes to a point mutation on the X chromosome only
(E) whether some of the money and research effort since the early 1990’s.
spent on the HGP would be better spent on (C) Some people who carry the defective gene
other kinds of research which causes this disease do not suffer from
any of its symptoms.
(D) The discovery of the cause of Martingale’s
9. It can be inferred that the critics of the HGP Syndrome was one of the results of the HGP.
believe which of the following? (E) Martingale’s Syndrome is usually fatal.

(A) More money should be spent investigating


diseases that are not genetically based. 12. It can be inferred that the author’s attitude
(B) Mapping genes may not be the most useful toward the critics of the HGP is one of
way to fight genetically-based diseases.
(C) Environmental factors play a significant role in (A) general agreement
causing Martingale’s Syndrome. (B) amusement
(D) Even with a complete genetic map, we may still (C) mild impatience
not understand how to battle infectious (D) general disagreement
diseases. (E) objective indifference
(E) Many genetically-based diseases cannot be
cured by genetic manipulation.

GO ON TO THE NEXT PAGE


© The MBA Center 451
Total PrepKit for the GMAT® Step 16 Workshops

DIFFICULT
13. It can be inferred from the passage that the
author

Linguists currently classify the more than 5000 (A) disagrees with the analogy between linguistics
different languages of the world into 11 great and biology
language "families," for each of which, it is (B) thinks that there is no single mother language
supposed, there was at one time an original (C) is one of the linguists of the 1970’s who helped
5 language which gave birth to the other languages create the 11-family classification
in the family. This classification system, with its 11 (D) thinks that a single mother language is possible
original languages, was decided upon only in the (E) doubts the validity of the 11-family classification
1970’s, by using methods drawn from the scheme
biological sciences. Biologists can determine the
10 familial relationships among animals by analyzing
the similarities in their genetic structures; the more 14. If the analogy between the biological method
similar the genetic structures, the more closely cited in the passage and the method used in linguistics
related are the species, and the more recently they is a valid one, it can be inferred that
"branched off" from their common ancestor.
15 Similarly, by comparing the structures of several (A) any two languages in the same family will
languages, linguists have been able to determine sound more similar than any two languages not
the familial relationships between them. Of in the same family
particular importance to linguists in the 70’s were (B) the languages that split off more recently from
analyses of the grammatical structures of these the mother language will be more similar to it
20 languages, for phonetic similarities alone were than those which split off earlier
considered to be too unreliable to count as proof of (C) just as new species are created from time to
a common origin. time, so are new languages
In the last few years we have been witness to a (D) since biologists have discovered that all plant and
new effort to find a single original mother animal life came from a common origin, all
25 language. This ambitious project aims to push the human languages must have a common origin
analysis back one step further, and to show that as well
the basic tongues of the "great families" are all (E) linguists, just like biologists, often neglect
derived from one most original language. Since the the impact of their own observation on their
information about the grammatical structures of object of study
30 these ancient languages is scarce and vague, the
attempt to show their interrelations is being
carried out mostly on the basis of phonetic 15. The advocates of the single mother language
similarities between certain very basic words, such hypothesis assume which of the following?
as "mother," father," "water," and "eye". While the
35 single mother language may well have existed, this (A) Languages develop and change according to
scientific methodology leaves many unanswered the same fundamental principles as do animal
questions, particularly as to the reliability of these species.
data. First, because of the mixing of peoples and (B) The basic languages of all of the “great families”
cultures, one group of people can easily adopt a branched off of the single mother language at
40 foreign word for use in their own language. Thus a approximately the same time.
similar word might appear in two languages, (C) Phonetic similarities alone can, in some cases,
without being the result of any familial relation. be sufficient to determine the existence of a
Secondly, given the limited range of human relationship between two languages.
sounds, and the evolution of sound patterns in (D) It is impossible to analyze the grammar of the
45 human speech, by random chance alone a original languages of the “great families.”
substantial number of words will sound identical in (E) Linguistics is not as similar to biology as earlier
different languages. linguists believed.

GO ON TO THE NEXT PAGE


452 © The MBA Center
Reading Comprehension

16. The author’s attitude toward the single mother


language hypothesis can best be described as

(A) tentative acceptance


(B) mild surprise
(C) objective criticism
(D) simple denial
(E) indignation

17. It can be inferred that the linguists who classify


all the world’s languages into 11 "great families"

(A) assume that basic words like “mother” and


“eye” are more stable over time and are subject
to fewer phonetic changes than are words for
more abstract concepts
(B) can reconstruct the grammars of the mother
languages of the 11 “great families”
(C) believe that nobody ever actually spoke the
original languages of the “great families”
(D) believe that linguistic research should be
conducted just like biological research
(E) do not find the methods of those who advocate
the single language hypothesis to be sound

18. According to the passage, why do the linguists


who advocate the single mother language hypothesis
rely primarily on phonetic data to determine the
relations between languages?

(A) The phonetic analysis of words like "mother"


and "eye" provide themselves sufficient data to
establish clear relationships between languages.
(B) For the languages that are being compared by
these linguists, there are not enough data of
other kinds upon which their analysis could be
based.
(C) Phonetic data fit more closely with the
biological model than do other kinds of data.
(D) Phonetic data can be classified and compared
much more easily than other kinds of data.
(E) There are no other kinds of data which establish
relations with as much certainty as do phonetic
data.

STOP
IF YOU FINISH BEFORE TIME IS EXPIRED
YOU MAY CHECK YOUR WORK

© The MBA Center 453


Total PrepKit for the GMAT® Step 16 Workshops

READING COMPREHENSION WORKSHOP


ANSWER KEY

1. E
2. A
3. D
4. A
5. E
6. E
7. B
8. E
9. B
10. B
11. C
12. A
13. D
14. B
15. C
16. A
17. E
18. B

454 © The MBA Center


III. APPENDICES

© The MBA Center


© The MBA Center
Appendix 1
Before the GMAT

© The MBA Center


© The MBA Center
Before the GMAT

BEFORE THE GMAT

You're counting down to the GMAT. Before panicking or canceling your test appointment, or deciding that this
test is just too hard and you're going to try professional sports instead of business school, step back and get a little
perspective.

THE LAST WEEK

Before taking the GMAT, you should be thoroughly familiar with the test. You should know the timing and the
structure of the test, including the six multiple-choice question types and the two essay types. DO NOT CRAM! The
GMAT is not suited to intense, last-minute study sessions. There is not a great deal to memorize for the test; while
it is (arguably) inadvisable to cram for any test, you are more likely to succeed on a university history test, for
example, than on the GMAT, after staying up all night studying. You can do well on the history test by memorizing
all the facts in the appropriate chapters of the appropriate books. You cannot know in advance exactly what
questions you will see on the GMAT. The only way to be make significant progress is to learn techniques and to
use the practice examples, with some time lapse between the lessons and the practice tests. You must build your
GMAT instincts, so that by test-day you react quickly when you see, for example, a faulty comparison in a Sentence
Correction question.

If you have followed the suggested study plan in Chapter 2, your GMAT instincts are probably pretty sharp by
now. With one week to go, you should do a little self-assessment. Ask yourself which parts of the test are hardest
for you. Do some light review. Go back through the lessons, skimming the text, focusing on the key concepts (the
margin notes and chapter summaries should make this easy). Review some questions you worked through already.
As you re-do questions and check the answers, ask yourself certain questions:

If you got the question right, did you get it right for the right reason?
If you got the question wrong, was there a simple mistake that you can avoid the next time? Was there a clue
in the question indicating that there might be traps in the choices?
For Problem Solving questions, did you look for answers to eliminate before calculating?
For Data Sufficiency questions, did you look for shortcuts, such as setting up equations and comparing the
number of variables and the number of equations?
For Critical Reasoning, did you try to identify flaws in the argument and predict the answer?
For Sentence Correction, did you compare the choices, looking for patterns, instead of treating each choice like
an individual problem?
For Reading Comprehension, did you get a quick overview of the passage instead of trying to memorize every
detail?
For any question type, did you look for traps and use Process of Error Identification?

If the answer to any of these questions is "no," you need to review some basic techniques. But don't solve a ton
of new questions. Answering a large number of questions without analyzing your work will not help a great deal.

Don't feel that you have to have memorized every word in this book. You can work a little on bringing your
weaknesses up to par in the last week, but don't forget about your strengths. Remember that solid approaches to
every question type are more valuable than highly specific strategies that may only apply to one or two questions
(or none at all) on the day of the test.

© The MBA Center 459


Total PrepKit for the GMAT® Appendix 1

THE LAST TWO DAYS

With two days left before the GMAT, it is more important to get into a good mind-frame than to learn test-taking
techniques. Stop taking tests. Try to get at least two consecutive nights of good sleep. Minimize stress, especially
from non-GMAT-related activities. Avoid scheduling important meetings or business-related deadlines just before or
just after the exam. If you are able to take a half-day or even a full day off from work or school, do so. The actual
GMAT testing time for the CAT is three and a half hours, and including the tutorial, the paperwork, etc. you will
likely be at the test center for four and a half hours or more. The test is tiring and stressful, especially when you know
that this time, the score actually counts. What you do the two days before the exam can affect your endurance and
performance.

You might continue doing a few practice examples, but don't spend more than about twenty minutes at a time
on questions. The best last-minute preparation is relaxation. You might want to see a movie, go out for dinner, or
just watch TV the night before the exam.

Make sure you know exactly where your testing center is. Do not wait until the morning of the test to get
directions. Remember to bring at least two forms of identification, including a recent photograph, and your
admission ticket (there will not be any scalpers outside the test-site selling extra tickets). Bring several pens or pencils
(even for the CAT). Bring a watch. You have the option of seeing the time remaining on the computer screen, but
some people feel more comfortable using an old-fashioned wristwatch. Do not bring a calculator; the proctor will
just tell you to put it away. Bring a bottle of water or juice and a snack, and a newspaper or some light reading in
case you have to sit around before the exam begins.

THE MORNING OF THE TEST

Wake up with plenty of time before you are supposed to arrive at the test center. Eat a reasonable breakfast. Try
to do some activity that gets your brain into gear, such as reading the newspaper. Do a few GMAT problems (not
to learn new tricks, just to get warmed up). Do a short mental review. Think through some of the main techniques
that you've practiced, focusing on what you're good at. At this point, forget completely about the question types
that give you headaches. Think about some kind of celebratory or relaxing activity for when the exam is over. ("Boy,
when this is over I'm going to buy myself that Harley I've been eyeing!") Do not think about your score or interviews
with business school admission officers--you can worry about these things later.

Most of all, stay calm. Remember that you are prepared! A lot of other test takers are in a panic because they
don't know what they're getting into. You have seen all major question types. You know how the test writer thinks.
You will surprise yourself during the test by remembering how to solve problems that in the beginning of your
preparation which might have seemed nearly impossible.

THE TEST ITSELF

On the CAT, the first few questions count more than the others. Go a little slower on these questions. Overall,
you have nearly two minutes per question--just over two minutes for each math question, a little bit under two
minutes for each verbal question.

The questions always start out at medium difficulty. The difficulty level will change throughout the test. It is pretty
difficult to guess the difficulty level of questions during the test, especially for verbal questions. Whether you think
you're looking at an easy or a hard question, look out for traps. If you're convinced that you've seen several very
difficult questions in a row, and then you think you have a very easy question, you probably missed one of Phineas's
traps, i.e., the "easy" question only appears easy. The difficulty level doesn't jump a great deal from question to
question, though the change is more significant at the beginning and becomes gradually less significant as the test
progresses.

460 © The MBA Center


Before the GMAT

Don't obsess about time. You don't need to advance exactly every 2.02 minutes on math questions or every 1.82
minutes on verbal questions. You do, however, sometimes have to guess and count your losses. Don't get stubborn!
Never take more than three minutes or more to answer one question. Any extra time devoted to one question
means less time available for later questions.

If you find the clock on the screen distracting, you can turn it off by clicking on the clock icon. You can bring it
back and hide it again as often as you like. (Whee!)

Take the test actively. Try to predict answers on Critical Reasoning and Reading Comprehension questions. Look
for shortcuts on math questions. Scan and compare on Sentence Correction questions. Remember that you are in
control.

You can get an unofficial score immediately after the exam. You can also cancel your score immediately after the
test. The catch is that if you want to cancel your score, you must do so before seeing your score. In fact, if you cancel
your score, you (and the MBA programs) will never have any idea how you performed. Of course, once you find out
your score, you will never have the option to cancel it. You will not be able to retake the exam during the same
calendar month, whether you cancel or accept your score.

Good Luck!

© The MBA Center 461


© The MBA Center
Appendix 2
MBA Admissions

© The MBA Center


© The MBA Center
MBA Admissions

MBA ADMISSIONS

The MBA has never been more popular. Applications to business schools
worldwide are up over 50 percent from twenty years ago. Americans pioneered
the degree in the 1920’s, but today MBA programs are as global as Coca-Cola,
and almost as well marketed. Over 160,000 students (130,000 from the U.S.,
20,000 from Europe, and 10,000 from Asia) on three continents sought MBAs
in 1998. Popularity, however, is only part of the MBA story. As a mark of
distinction in the intensely competitive world of business, the MBA is coming
into its own. The blue-chips, the best of the best, employers like Hewlett-
Packard, Coca-Cola, and Merill Lynch, have a special fondness for the MBA.
Every year their recruiters sweep MBA graduates off their feet and into lucrative
positions in marketing, finance, consulting, and technical management. No
other business degree can set you apart or take you farther than the MBA.
MBA programs continue to evolve and adapt to the demands of employers.
Global business, in fact, has a very strong influence on what you’ll learn and
what you’ll do in business school. No matter how many billions employers have
sitting in the bank, they feel they pay a lot for their MBA recruits and aren’t shy
about telling schools what they like, what they don’t, and what they need. As
businesses everywhere and of every size strive to be the next Dell or Disney,
scooping up profits and making markets the world over, they want people to
help them do it: MBA people with international skills, experiences, languages,
and perspectives. So, when businesses talk, business schools listen, and what’s
being talked about now is wanting more and more from MBA graduates: more
experience (3 to 5 years), more specialized training (information technology),
more international exposure (an enormous competitive advantage), and more
dynamism (the entrepreneurial model is the plat du jour).
Between MBA programs and MBA employers is an understanding that
curricula must be continuously improved to stay relevant and competitive.
Competition improves the breed, and no one takes this more seriously than
business schools and businesses do. Quality schools make curriculum changes
yearly now which mirror the changes in the business world at large that we all
read about in the Wall Street Journal – like the shift from heavy manufacturing
to marketing and entertainment services.
So, business schools and employers have a nice cozy relationship but where
does that leave you? The more applications schools receive each year, the more
choices they have to fill each year’s MBA class, their pride and joy. Your research
then must be thorough and discriminating; your thinking, clear and long-term;
and your decisions prudent and well-informed. The competition for MBAs is
fierce – Stanford accepts less then one-fifth of those who apply – and
expectations run high, but you have more choices than ever before too.
The two-year MBA program used to be the only game in town, particularly
in the United States, but things have changed. At many of the over 1,500 MBA
programs worldwide, one-year and eighteen-month programs are springing up None of these guys did
like Burger King franchises to better serve an increasingly diverse student body; badly after getting their MBA:
a student body made up of more international students, students in their 30’s Donald Trump (Wharton)
and 40’s, female students, and executives. Warren Buffet (Columbia)
Alan Greenspan
(New York University)
Tatsuro Toyoda
1. THE DECISION TO PURSUE AN MBA (New York University)
Robert Lutz (Berkeley)
It’s a big decision to pursue an MBA and not an easy one to make. The
potential rewards are fantastic, but before you get there you’ve got to think
about things like interrupting your present career, a substantial debt load, loss
of salary, relocation, and significant commitments of both time and energy, to
not only getting in but also to staying in the school you covet. It’s no secret that
higher pay (between 60- and 90-thousand U.S. for graduates from quality

© The MBA Center 465


Total PrepKit for the GMAT® Appendix 2

Average job offers before schools) and job placement (quality schools place between 85 percent and 95
graduation
Stanford: 3.28 percent of their graduates within three months of graduation, and graduates
Harvard: 3.11 from the top Business Schools get an average of three job offers before
Wharton: 2.97 graduation) are universal advantages to an MBA, but the motives for pursuing
Columbia: 2.66 the degree are as diverse as the applicant pool. See if you recognize yourself in
Indiana: 2.37
Darden: 2.07 one of the profiles below.

THE MBA AS CAREER-ENHANCER

If you’re here you’ve had several years of work experience and are thinking
about bettering your chances in your present company for promotion and
salary hikes in the future by going back now for an MBA. You’ll improve your
specialized business and technical skills in a year or two, widen the range of
opportunities, and increase the level of responsibilities offered you in your
company when you return. No doubt you’re ambitious too, and though you
want to elevate your level of career achievement, pursuing an MBA is a personal
objective as well as a professional one.

THE MBA AS PROFESSIONAL CERTIFICATE

In some professions, such as finance, consulting, and operations


management, the MBA has become not just suggested for those who wish to
reach senior management status, but necessary to get to the fabled corridors of
power. Like a professor who must have a doctorate as the minimum
requirement for tenure, the lower- or middle-manager in these professions
needs an MBA even to be considered for higher posts, greater responsibility,
brighter glory. It’s extremely competitive for you if you’re here: even at your
present level of accomplishment, to move up and broaden your horizons you’ve
got to distinuguish yourself from ambitious colleagues with the same ideas.
Which means that in this case especially, the quality, reputation, and availability
of the latest, most up-to-date management theories and methods are very
important. Current emphases in quality U.S. programs are on self-direction, self-
assesment, teamwork or group work, and ethics in business. These are the
qualities judged essential for leadership at the senior level, and schools and
businesses have worked very closely together to turn these qualities into active
subjects in quality programs. Do some reading about any schools you have in
mind – look for evidence of close links between the school and businesses. You
want a school that’s as close to the market you want to be in as possible – it can
only benefit you in the end. One other thing: do some checking around your
company about tuition reimbursement, a great way to go if you can get it.

THE MBA AS CATALYST FOR CAREER CHANGE

If you’re here, chances are you want to make a break from your current
employer. Engineers, for instance, often find themselves trapped by work which
is purely technical and want to make a career change. In many cases their
company won’t sponsor them for an MBA – because they like engineers just the
way they are – so these engineers, and many others from different fields like
them, choose to go for an MBA both to quit their present job and enlarge their
possibilities for future jobs. How will MBA programs go about their business of
broadening you? By sharing with you a general management education, with
opportunities for specialization, and valuable hard skills, such as quantitative
analysis, in keeping with the most recent methodologies, employer demands,
and requests from general business. The MBA is a worthy tool for positioning –
placing yourself in the best path for the best job possible after graduation. If you

466 © The MBA Center


MBA Admissions

read The Economist and watch Business Today on CNN but feel generally out of
touch with the prevailing environment in management, then you’ve stumbled
upon the MBA at the right time in your life – a quality, contemporary MBA will
diversify, indeed enrich, your prospects far better than TV can.

THE MBA AS TRAINING GROUND FOR ENTREPRENEURS

Business school admissions people love to have present or future


entrepreneurs in their classrooms. They love that these dynamic, stimulating,
and creative folks are also wise enough to see the value in having a formal
business education whatever their entrepreneurial experience or success so far.
Schools now like enterprising ideals so much that they’ve introduced elements
of entrepreneirship into their programs, emphasizing characteristics such as
vision, action, and originality. Entrepreneurs like MBA programs for skills in
marketing and knowledge in finance – a practical education to go along with
their audacity. Corporate recruiters like very much the entrepreneurial model,
set of skills, and way of thinking; you might have heard many of them think of
themselves as management entrepreneurs now. Whether you’re an
entrepreneur now or want to find out if you’ve got what it takes to be one, do
your research--you don’t have to look far for a quality program with a
curriculum stressing entrepreneurial values. Even the most independent
entrepreneur can benefit from an MBA in three ways: one, networking
possibilites with other students; two, networking possibilities through the
alumni organization; and three, connections in the business world through the
program’s contacts. And one more item, data from ten or even fifteen years ago
shows that a sizeable number of MBA graduates did not end up in Fortune 500
companies – they ended up running small businesses instead.

THE MBA AS A PLACE TO GO

Three other types of MBA applicants we’ll roll into one here. What all three
of them have in common is only that they’re going for an MBA for none of the
reasons outlined above. They’re miscellaneous but they consistently show up in
the statistics derived from the applicant pool. Who are these citizens and what
do they want? Let’s take a look. A very small number of actual MBA students
pursue the degree principally as an academic challenge. These are the perpetual
students – if you’ve been around any college or university campus for long,
you’ve seen or heard of them. To these students, the immediate professional
reasons or benefits of the diploma are lost, but the test of one’s higher
intellectual capabilities is not – yep, you guessed it, they want the degree for the
degree itself. If this is your path in life, all the best....but, remember that the
MBA is a dedicated (and expensive) professional degree, and is rigorous enough
to make it difficult for even the hardiest professional student to justify.
The small piece of advice above applies to this next group as well. If you’re
here, you belong to an even smaller, even more rare type than the perpetual
student above – but some characteristics are shared. Who are we talking about
now? The younger student with little or no professional experience but with
excellent academic credentials who applies for business school admission.
Admissions officers advise all those in this category to think very carefully – and
to research very thoroughly – before applying for an MBA program. Further,
business schools and employers know precisely who they want in their program
or their organization, so in all except rare cases does the talented student or
intellectual who is not a professional, or who has no definite plans to be one,
gain admission or remain commited to an MBA program.
The third type of applicant is growing in number every year. If you’re here
you’ve had between seven and ten years of work experience and want an MBA

© The MBA Center 467


Total PrepKit for the GMAT® Appendix 2

to take a break from, or quit entirely, your present career. You not only want the
academic challenge and environment of business school, you also have personal
motives – often to reevaluate what you’re doing and to make a change. There
are quite a few reasons admissions officers cite for why applicants in this
category become excellent MBA students: they’re self-sufficient and believe in
change; they have strong clear ideas about who they are and what they want
What are your motivations? to change in their life, education, and career the next time around; their
According to the Association maturity and unique perspective are appreciated in the classroom; they’ve taken
of MBAs, people pursue an a huge step toward what is expected of top executives and managers today,
MBA for the following continuous learning.
reasons:
-Learning
-Internationalizing
their career 2. CHOOSING THE RIGHT SCHOOL
-Changing jobs
-Wanting a higher salary
Rankings are not the only method for choosing a business school. Using
rankings to choose your MBA school isn’t even recommended – if that’s all you’re
doing for research. Use rankings as a guide only, to what’s out there, to how
different schools are judged relative to the the national market, and to how
different schools are judged relative to regional markets. (United States only)
Magazines are only one small factor in a decision which is an important and highly
personal one. Choosing the right school for you should be an individual decision
which follows from close, thoughtful evaluation, and from thorough investigation.
To do it right means expense of time, of energy, of motivation, of frustration – of
searching your soul – to arrive at the first basic requirement of admissions:
knowing what you want. After thinking it through personally, think about it
methodically – it is a life-changing decision and an enormous financial investment.
Business schools expect, and implicitly require, investigation from their applicants;
they realize, they have lots of experience realizing, that it’s a difficult decision to
make. So ask all the questions you want of any school you’re considering – by
phone, by fax, by e-mail – and don’t hesitate, they expect you to. Consult any
sources of information you feel will help you but give yourself time to thoroughly
evaluate all the factors involved, such as reputation, location, job placement,
curriculum, and networking. Only you know: neither rankings, nor advice from
colleagues, friends, professors, or mentors can choose the right school for you.
Rankings
If you want to know more
about ranking, the following
magazines publish rankings. REPUTATION
They are: U.S. News and
World report (U.S. MBAs),
Business Week (U.S. MBAs), The reputation of a school is determined both by word of mouth and by
Capital (U.S. and European official rankings put together by independent researchers. There are a variety of
MBAs) sources for rankings in American and European publications. The rankings can
tell you one thing for sure about reputation: there’s a strong correlation
between the reputation of a school and the average starting salaries of that
school’s graduates. However, the rankings offer a perspective but not the whole
picture of what’s really going on and thought about in the world of business
schools – so use them as a guide, not for answers.
Everybody wants to go to the big schools, because they’re the best, right?
and because everybody – any employer on Earth – knows their reputation.
While this may be true, a school should suit the applicant well, especially at the
graduate level, in all ways, not only reputation. Reputation can be used, true,
but you must decide where you’re headed and in what field and where before
you consider how a school’s reputation might of benefit to you. Not every
reputation, for example, need be global for it to be good. Excellent schools exist
both in America and in Europe with solid, local reputations; these are regional
schools, like Babson in the U.S., which are not well-known nationally or
internationally yet have unquestionable reputations within their scope and close
ties to businesses in their area. So ask yourself the hard questions and keep in
mind what industry and in what location you’d prefer to end up in – many,

468 © The MBA Center


MBA Admissions

many lesser-known schools can provide quality and specialization. Two more
things to bear in mind. One, the quality and the reputation of a school are
judged by its facilities, its faculty, and its student body. Two, as the correlation
mentioned above implies, reputation is of considerable interest to employers
when recruiting, so remember that your MBA and the reputation of the school
it came from will follow you wherever you go in your career.

LOCATION
With the exception of the top, brand-name, Platinum American Express card
programs, the location of a school can make difference both to you personally
and to your potential employers somewhere down the line after graduation.
Smaller programs don’t carry much weight far from their regions, but they often
have valuable ties to local industries. Ask yourself: will the location affect the
quality of my business school experience? and will it affect my employability
after graduation? (Get yourself the answers too.) Think about long-term plans
as well – where do you want to settle, and is the school there or will it allow you
to get there? Take into consideration personal as well as practical concerns.
Urban, rural, or suburban enviroment? Do you like snow, rain, heat, the colors
of the fall? Keep in mind pragmatic issues like the time and costs involved in
commuting and travel – do you want to commute?, do you want to travel? This
is the time to consider your personal preferences in things like geography,
population, weather, economy (regular or extra-lucrative?) – you’ll spend time
and a lot of money wherever you go, so make sure it’s where you want to be.

PLACEMENT
An MBA program’s job placement record and practices should be of concern
to you. More than likely you’re not going to business school to be left
unemployed and drifting after you’re done. Statistics on placement are easily
available either from the schools themselves or from the shelf-loads of MBA
information guides. See the bookstore or library nearest you, but call, fax, or e-
mail the schools first for as much free information as you can get. Looking over
these figures, wherever they are from, keep in mind that not just statistics are
important; it is whether they tell you what you want to know. Average starting
salary statistics, for instance, are only a place to start: you’ll want to find out
whether recruiters prefer school experience or previous work experience when
determining your salary. Look for placement services such as a career library,
career counseling, career fairs, a job bank, off-site recruiting, and referrral
services for résumés to employers. Read closely for the kinds of jobs found by a
given school – jobs are usually listed by type of employer (manufacturing,
services, banking, etc.) and by functional area (finance, marketing, consulting,
etc.). Pay attention also to alumni accomplishments, they speak to the
reputation and soundness of the school. Your career is a long-term project, so
consider not just your first job offer but also where it might lead you.

CURRICULUM

The curriculum is the heart of the program. You’ll sit over coffee talking
management theory you learned by it, and the prestigious faculty earn their
bread and butter by teaching it. In American schools the curriculum is under
constant supervision and change, constantly responding to the needs of general
business and the marketplace. In this endeavor, the most advanced schools are
tightly linked to businesses and employers trying to hone their curriculum to its
best, most cutting edge. This link between schools, businesses, and recruiters is

© The MBA Center 469


Total PrepKit for the GMAT® Appendix 2

something you should be very aware of, for the results of it are found in nearly
every quality-school curriculum. Simply put, MBA directors now work to supply
what their business connections tell them they need – schools no longer are, or
want to be, insulated from the active and very real global business environment.
Schools feed businesses their polished graduates, and businesses give schools
their always-evolving assessment and their suggestions for improvement of the
strengths and weaknesses of a particular curriculum. Information-sharing, it is
sometimes called.
Teaching methods have changed in innovative programs. Their focus has
switched from a teaching model (you learn what we know) to a learning model
(you learn how to learn) after feedback from everyone involved – students,
teachers, and employers. Satisfaction among all three has been very high. The
learning environment is relevant for you in two ways. First, recruiting officers
from potential employers are very opinionated about – they know exactly
which – environments they prefer, so know that your learning environment can af-
fect how you’re employed in the end. In addition, the learning environment of
the school you choose should be suitable for your own personal needs, ends, and
personality; it should be in character both with you and what type of job you
want.
In programs that interest you, learn all you can about recent or on-going
curriculum improvements. Quality schools have made the student and how the
student learns their number 1 objective, so you should realize that advertised
faculty-to-student ratios don’t tell the complete and unabridged story of a
school’s learning environment. Look for curricula which highlight student-
teacher relationships, this is one of the improvements schools pride themselves
on – closing the gap between teachers and students. For example, there are
some two-year programs in which students see the same teachers for an entire
year. You may not want to get that close, but be aware of this changing
emphasis. Last, ask your target schools for the results of student exit-surveys
from the previous year’s graduating class. These can tell you a great deal if you
go through the trouble to get them and they are often published in some form
by the school.

NETWORKING

Networking is to business schools as makeup is to Ivana Trump: something


they are very, very serious about. Not long ago networking was heard as a term
tossed about by a few eager business school graduates; now it is much more than
a buzzword, it is a b-school way of life. Networking is a genuine element of
today’s MBA programs and a very commited purpose for their directors. Look
over almost any brochure published by almost any school, and you will see
the word littered throughout the pages.
What does it mean then? Networking means building relationships among
students, faculty, alumni, and the community; it is a very large part of the MBA
experience and can be very advantageous for you – think of jobs, advice, partner-
ships – in the long run. Think of it as neccessary and worthwhile whether you like
it, or are good at it, or not. Also seriously consider the network of the school – is
it established?, well-organized? Some schools are known for excellence in
networking, such as Thunderbird in Arizona, and others are not. In choosing a
school with an appropriate (for the type of job you want), effective, and diverse
network you can tremendously improve your long-term business and employ-
ment prospects. If you’ve read your Fortune and Forbes, you know that many a
successful partnership originated in business school – Duty Free Shops, Hewlett-
Packard, et al...

470 © The MBA Center


MBA Admissions

QUALITY OF LIFE

How do you want to live once at the school of your choice? Quality of life is
part of this picture too, though right now 10,000 other things probably seem
more important. Classroom experience is not the only one of importance in
school; your personal experience has its place too – particularly for full-time
students. So, quality of life or personal experience should be meditated upon
before choosing any of your target schools. The question is, what else does the
school and/or its surrounding area have that interests you personally? Activities,
community or cultural involvement, accomodations, sports, weather, cost of
living, and city size are all features to keep in mind. Whatever emphasis or
importance you place on quality of life, consider that business school is one to
two years of your life, is more than likely in a new city if not a new country, and
affects yourself, your family, friends, and others of significance to you.

3. EVALUATING YOUR CHANCES OF ACCEPTANCE

The admissions application and process is the school’s way of getting to


know you. Don’t assume that your undergraduate grades or your
undergraduate school aren’t good enough – you don’t know. Know that
admissions officers are regular folk who work all year round looking at scores of
applications – they’re professionals who know their job, and what they like to see
and they stress that they evaluate each applicant as an individual, not a set of
numbers. They use all the tools at their disposal – letters of recommendation,
essays, interviews – to get to know you as personally as they can.
Applicants for the MBA are a diverse and competitive group: you can’t
control your competition; however, you can control every aspect of your
presentation of yourself to admissions officers. Everything required of you in the
admissions process is your chance to present yourself in the manner you’d like
to be seen – it’s your ballgame until your admissions packet hits their doorstep,
so play the best game possible.
Prospective students are sized up by admissions officers in four ways: one,
according to intellectual, or academic, ability (scores and grades); two, by
looking at professional work experience (emphasizing the international) to
estimate professional potential; three, by judging the presentation of the
applicant (how were the essays, interviews, and recommendations?); and
four, by the personality, or personal qualities, of the applicant (what kind of
personality is there evidence of?). The order of evaluation and the order of
importance of these criteria vary from school to school. The process of
admissions for your MBA is a time to put your best work forward the first time
you apply – it’s first- impression time, and your golden opportunity to show
these overstressed creatures exactly the picture of yourself you want them to
see.
Beyond that there is no secret ‘right way’ or formula to get into business
school – not unless your family owns a very large corporation. If that’s not the
case, then knowing yourself, the program you want, what you want from your
MBA education, and submitting a compelling admissions application is your
best approach.

INTELLECTUAL ABILITY

We’ve mentioned this before, but it is important and applies here as well: the
MBA is a professional degree, and a difficult and demanding task, which
requires high intellectual stamina and acuity. The way admissions officers see
this demonstrated is by looking at your transcripts and so forth to see if you
have a solid history of accomplishment in the classroom. This is the part where

© The MBA Center 471


Total PrepKit for the GMAT® Appendix 2

your grades and scores come into play. Business schools want leaders, yes, but
they also want students who can keep up in the classroom – and you’ll have
some tough classrooms. Your undergraduate records help them to gauge your
intellectual aptitudes, your ability to study well at the graduate level. Some of you
already have graduate degrees of one sort or another, okay, this part isn’t a
challenge for you. But the scores, your GMAT, TOEFL, or both, are important for
everybody.
Of course the GMAT is the one they chiefly care about, and it is used to
appraise your basic academic skills and to allow comparison – a nice, neat
quantitative comparison – between competing applicants. The basic skills
judged are: math skills, primarily quantitative problem solving abilities; and
verbal skills, understanding and interpreting written material and basic writing
skills. Schools report a definite correlation between GMAT scores and class
performance during the first year of courses. Your test scores are used to some
extent by nearly every school and they’re not the only criteria looked at, but
they do give schools a sense of: can you study at the graduate level?

PROFESSIONAL POTENTIAL

You might ask yourself this question: who knew Donald Trump would end
up owning some casinos in Atlantic City? If you hazarded the guess, Wharton,
his business school, well maybe you’re right. What admissions officers would
love to do is to encourage the next Trump or Warren Buffet: what they’re
looking for from you is evidence of leadership – leadership is the stuff MBA
gradautes are made of. In this area, professional work experience – the quality
of it, what miracles you’ve performed while doing it, how much of it you’ve had
– are the most important of three factors; the other two are letters of
recommendation and personal essays. For quality, or the most selective schools,
full-time professional work experience is essential – they have plenty of
applicants with part-time experience, but very few of them become candidates.
Admissions officers say there is no more effective way of demonstrating your
leadership potential as a manager than by the fact of career success – it gets
them very excited about you. Show them your good side. For the most
favorable sketch of yourself, demonstrate clearly the successes you’ve had in
your career – don’t be shy. Also, tell them what contributions you’ll make to
your MBA class because of your success, and how you’ll make them. It comes
down to clearly presenting both your record of resposibility and leadership from
the past and your plan for resposibility and leadership in the future.

PERSONAL QUALITIES

Everyone knows that personality can take you a long, long way in life – look
how far it has taken Lee Iacocca. You just can’t keep a guy like that down. That’s
what these competitive schools want to see: something to show them that you
can take the blows, learn a lesson or two, and come back for more. We’re talking
about leadership, and business schools prize it as traders do bonus checks, that
is, highly – and very seriously. Why? Schools want MBA candidates who are fit
for responsibility, the sworn responsibilty of managing people and entire
organizations. They know from educating legions of CEO’s, COO’s, CFO’s, and
risk-takers like Trump and Lawrence Ellison that leadership is fundamental for
success in business.
In addition to a personality worthy of command, schools like to find in you
such personal qualities as communication skills, initiative, and motivation.
These are individual qualities, and they take their forms in each applicant in very
different ways. In presenting your own personal variety of ingenuity, leadership,
initiative, motivation, determination, creativity, and the like, it’s solely up to you

472 © The MBA Center


MBA Admissions

to think of examples which show you at your best and demonstrate the
principle involved. Individual qualities are what set you apart from your
competitors (nothing you can do about them anyway). Therefore, your
presentation of self – who you are, what you’ve done and why, and what you
want – makes as much difference in distinguishing you from another applicant
as your actual qualities do. Admissions officers don’t know you any better than
they know the other thousand like you who apply. They’ll work to surmise what
they can of you from letters of recommendation, your current professional
position, and any evidence of your personal interests – but in this case those are
the sketches, not the finished self-portrait. So use the tools you’re given, the
personal essays and the interview, to showcase your character and yourself. In
the application process, presentation is king.

4. FINANCING YOUR MBA

Did someone say they were handing out free MBA’s here? Not this year, pal,
you must not be from around here. Here, wherever you are and wherever you’re
going, an MBA will set you back a tidy sum. The amount varies from school to
school, country to country of course, but expect to pay from $10,000 to
$35,000 per year. What the MBA really is, at these prices, is an investment with
expected returns somewhere off in the long-term future. It may grieve you, but
probably won’t surprise you, to learn that the bill for tuition isn’t everything
you’ll be putting on your tab once in school. Think about accomodations,
transportation, personal expenses, and the odd textbook here and there to keep
the bookstores happy and prosperous. These non-tuition (indirect) costs are
typicaly just over one-half of the tuition (direct) costs; Harvard, for example, has
tuition costs of around $21,000 and non-tuition costs of about $13,500. If you
add up those two figures (you’re the MBA-hopefuls, you’ve probably already
done it), you can see the wisdom in figuring the total cost per year for each
school under consideration: tuition, room and board, books, computer, travel...
Any, or a combination of all of the following four methods could be used to
pay for your MBA.

FINANCIAL AID

Financial aid in one form or another is by far the most common way of
financing an MBA in the United States. If you’re an international student
planning to study in the U.S., however, very little financial aid is available to you;
on top of that, you must prove you have sufficient funds to cover the entire cost
of the program before you can be admitted to the program in question.
International students find their MBA money from: their government
(scholarships, grants, subsidies, etc.), their personal funds saved or borrowed for
the purpose, or from their families.
As a graduate student, you are considered independent: only your financial
status and resources are relevant, not those of your family. Further, financial aid
for graduate students in the form of grants or scholarships is very rare in the U.S.
Which means that most candidates take on sizeable debts in their pursuit of an
MBA, mainly in the form of loans, and count on high future earnings to pay
them off. It’s a vicious cycle, but other choices are few. If your loans are arranged
by your target school’s financial aid office, then they are very likely need-based –
by far the most common type of loan – though a very small percentage are
unsubsidized, or non-need-based which carry higher interest rates in any case.
The financial aid office of your intended school is the best resource for specific
and current information on what money is available, to whom, and how.

© The MBA Center 473


Total PrepKit for the GMAT® Appendix 2

BANK LOANS

Are you getting the idea that there’s no free money? If there is, there sure
isn’t enough of it. Which might explain why over 55 percent of all MBA
candidates finance their degree with bank loans. Many banks have loan
programs already set up for MBA students – you didn’t think you were the first
to need money for an MBA, did you? Ask your bank if they have something like
an MBA Loan or a Business Access Loan, or ask what they can do specifically for
MBA students. If you have a good relationship with a bank now, your path to
funds is going to be that much smoother. If you don’t, and you do think you’ll
have to borrow, make sure your credit history is accurate and up to date. Banks
are willing to loan money to MBA candidates who demonstrate a responsible
past and a high probablity for success in the future, they naturally want those
with the highest likelihood of paying off their loans. The future Wall Streeters,
the potential lions of the Bourse, the tigers of the Nikkei...

FINANCING FROM BUSINESS

Some of the best-known companies like Microsoft, Intel, Rolls-Royce, and


Dow Chemical have programs for financing an employee’s MBA. They know the
benefits of lifelong learning, they know the advantages to employing MBA
graduates, and they consider financing an MBA for a qualified individual a wise
and profitable investment. Tuition reimbursement programs are what they’re
generally called, and unless you already know the policy of your company, a
little checking (call the Human Resources office) can go a long way. Some
companies pay 100 percent of the costs of the MBA program, some pay for a
lower percentage, expecting you to cover the difference. These companies, rich
as they are, don’t cut you a check for $50,000 and send you on your way. They
want a little something from you in return: it’s usually an understanding that
after your MBA you’ll continue to work for them for at least another two or three
years. You may have been, or would like to be, hired by a company that allows
you to go off for an MBA with the stipulation that you begin work for them
immediately upon its completion. Either way, about 10 percent of all MBA
candidates go to school (most often in part-time programs) on the tuition
reimbursement plan sponsored by their company.

PERSONAL FINANCING

Financing an MBA through personal ways and means is sharply on the


increase: about 35 percent of all MBA students today finance the degree
themselves compared to about 20 percent only fifteen years ago. Maybe more
students are gifted stock-pickers, who knows, but in most cases the money
comes from three to five years of savings while in a profession. Contributions
from family are also part of this personal financing package, and in some lucky
cases, family contributions are the entire package. Schools figure that in these
cases it's a combination of savings, family contributions, and in more and more
cases, spousal support. If you can manage to pay for your own degree, great –
no bank or government loans to worry about. If you can’t manage on your own
resources, it is time to get creative and investigate the possibilites mentioned
above. Or ask your parents if they have any wealthy brothers or sisters that
you’ve never met – it’s a long shot, but you never know.

474 © The MBA Center


TOTAL PREPKIT FOR THE GMAT®

Copyright 2001-2007 by MBA Center Publications


MBA Prep ltd.
40 Woodford Avenue
Gants Hill, Ilford Sussex
IG2 6XQ
United Kingdom

Printed in France – March 2007


C L’Imprimerie
7, rue de Liège
95190 Goussainville
France

N° d’impression : NA-986/2007
Dépôt légal à parution

Written by:
Dr. Hubert Silly

Desktop Publisher:
Ilya Krupin

e-Learning:
Alex Nagorov
Albert Panyukov

ISBN: 978-2-916729-02-2

© The MBA Center


© The MBA Center

Das könnte Ihnen auch gefallen